You are on page 1of 343

1 Introduction to Set Theory

1.1 Sets and Its Elements


Definition:
A set is any collection of objects. The objects in the set are called elements or members of the set.
The set should be well-defined as it describes clearly its elements as to identify if an object does or
does not belong to a set. A set is usually denoted by a capital letter such as A, B, or C.

On the other hand, when 𝑥 is not contained in 𝐴, then we say that 𝑥 ∉ 𝐴.

Description of a Set
A set is enclosed with a pair of curly brackets. Capital letters of the English Alphabet are used to
denote a set. The following are used to describe a set:
A. Verbal Description
 Describing the elements in a set into words
Examples:
o The students of the graduating batch with a grade point average of not less than
90%.
o The employees of Company XYZ with a birth month of April.
B. Roster Method
 Listing of all elements in the set
 An ellipsis(… ) is used when there are too many elements and if a pattern is followed
Examples:
o 𝐴 = {𝑎, 𝑏, 𝑐, 𝑑, 𝑒, … }
o 𝐴 = {apple, orange, mango, guava}
C. Set-builder Notation
 The symbol | which means “such that” is used to define the elements of the set.
Examples:
o 𝐴 = {𝑥| 𝑥 is a letter of the English Alphabet}
o 𝐴 = {𝑥|𝑥 is an odd integer}
Kinds of Sets
A. Empty Set
 The set that has no elements
 It is denoted by Ø or {}
B. Finite Set
 The set which contains a finite number of elements
 Example: 𝐴 = {1,2,3,4,5}
C. Infinite Set
 The set which contains an infinite number of elements
 It starts with a few elements of the set then followed by an ellipsis (… ) which indicates that
the sequence continues indefinitely.
 Example: the set of integers denoted by ℤ = {… , −3, −2, −1,0,1,2,3, … }
D. Subset
 Given sets 𝐴 and 𝐵, set 𝐴 ⊂ 𝐵, read as “𝐴 is a subset of 𝐵”, when every element of set 𝐴
is contained in set 𝐵.

1
 In finding the number of subsets of a set, you’ll have to raise 2 to the power of the
number of elements in the set.
 Examples:
o 𝐴 = {1,2, }, 𝐵 = {1,2,3}. Since every element of set 𝐴 is contained in set 𝐵, then we
can say that 𝐴 ⊂ 𝐵.
o Find all the subsets of 𝐴 = {𝑟, 𝑠, 𝑡}. The subsets are:
∅, {𝑟}, {𝑠}, {𝑡}, {𝑟, 𝑠}, {𝑟, 𝑡}, {𝑠, 𝑡}, {𝑟, 𝑠, 𝑡}
o {a, b, c, d} has elements 𝑛 = 4 . Thus, we have 24 = 16 subsets
o { } has no element 𝑛 = 0. Thus we have 20 = 1 subset

E. Superset
 If 𝐴 is a subset of 𝐵, 𝐴 ⊆ 𝐵, then 𝐵 is the superset of 𝐴, 𝐵 ⊇ 𝐴.
 Example: 𝐴 = {1,2,3}, 𝐵 = {1}. Since the element of 𝐵 is contained in 𝐴, then 𝐴 ⊇ 𝐵.

F. Power Set
 The power set is the set of all subsets of the given set 𝑆.
 It is denoted by 𝑃(𝑆), where 𝑆 is the given set.
 Example: 𝑆 = {𝑎, 𝑏, 𝑐}, then 𝑃(𝑆) are all the subsets of set 𝑆. Since the set 𝑆 has 3
elements, then there are 23 = 8 subsets. Hence, 𝑃(𝑆) =
{∅, 𝑆, {𝑎}, {𝑏}, {𝑐}, {𝑎, 𝑏}, {𝑎, 𝑐}, {𝑏, 𝑐}}.
Elements vs. Subsets
Remember that given a set 𝐴 = {𝑎, 𝑏, 𝑐}, 𝒂 ∈ 𝑨 and {𝒂} ⊆ 𝑨.
Example:
Given 𝑋 = {1,2,3,4,5},
a.) Is 1 ∈ 𝑋? Yes, since 1 is an element of 𝑋 from the given.
b.) Is {1} ∈ 𝑋? No, since {1} is not contained in the set 𝑋.
c.) Is {1} ⊆ 𝑋? Yes, since {1} is contained in 𝑃(𝑋).
d.) Is 1 ⊆ 𝑋? No, since 1 is not in 𝑃(𝑋).
_______________________________________________________________________________
Lesson Exercise:
1. Which of the following is a set? (D) The set containing the integers
(A) The tall people in the room. from 1 to 10.
(B) The beautiful models of 2012.
(C) The graduating people of the 4. Which of the following is NOT a set?
batch. (A) ℕ = {1,2,3, … }
(D) The excellent people of the (B) ∅ = {1,2,3}
company. (C) The set of real numbers.
(D) The set of rational numbers.
2. Which of the following is a set?
(A) {}
(B) a, b, c, d 5. Which of the following is a subset of
(C) (Anne, Bert, Cole) set 𝐴 = {1,2,3, … }
(D) A = 1, 2, 3 (A) 𝐵 = {0,1,2}
(B) The set of negative integers
3. Which of the following is NOT a set? (C) The set of irrational numbers
(A) 𝐴 = {𝑥|𝑥 is a letter in the English (D) 𝐵 = {𝑥|𝑥 ∈ ℤ, and 3 ≤ 𝑥 ≤ 5}
alphabet}
(B) The short-haired employees in a 6. Given 𝐴 = {−1, −2, −3, … }, which of
company. the following is/are true?
(C) 𝐴 = {1,2,3, … ,100} i. Set 𝐴 is the set of all complex
numbers.
2
ii. 𝐴 = {𝑎|𝑎 ∈ ℤ} (B) 𝑃(𝑆) = {∅, {𝑚}, {𝑛}, {𝑜}}
iii. Set 𝐴 is the set of all negative (C) 𝑃(𝑆) =
integers. {∅, {𝑚}, {𝑛}, {𝑜}, {𝑚, 𝑛}, {𝑚, 𝑜}, {𝑛, 𝑜}}
iv. 𝐴 = {𝑦|𝑦 is any number} (D) 𝑃(𝑆) =
(A) iii only {∅, 𝑆, {𝑚}, {𝑛}, {𝑜}, {𝑚, 𝑛}, {𝑚, 𝑜}, {𝑛, 𝑜}}
(B) i and ii
(C) ii and iii 9. Which of the following is the superset
(D) ii, iii, iv of the set 𝐴 = {2,4,6,8, … }?
(A) 𝐵 = {1,2,3,4,5,6,7,8}
7. Given 𝐵 = {1,3,5,7}, which of the (B) 𝐵 = {𝑏|𝑏 ∈ ℤ}
following is/are true? (C) 𝐵 = {0,2,4,6,8,10}
i. Set 𝐵 is the set of odd (D) 𝐵 = {𝑏|𝑏 ∈ ℕ}
integers.
ii. Set 𝐵 contains all the real
numbers. 10. Given 𝑆 = {𝑠𝑡, 𝑢𝑣, 𝑤𝑥}, which of the
iii. 𝐵 = {𝑏|𝑏 is a whole number} following is/are true?
iv. B = {𝑏|𝑏 is an odd integer and i. {𝑤𝑥} ⊆ 𝑆
1 ≤ 𝑏 ≤ 7} ii. {𝑠𝑡} ∈ 𝑆
(A) i and ii iii. 𝑠𝑡 ⊆ 𝑆
(B) ii and iii iv. 𝑢𝑣 ∈ 𝑆
(C) iii only (A) i and ii
(D) iv only (B) i and iii
(C) i and iv
8. What is the power set of 𝑆 = {𝑚, 𝑛, 𝑜}? (D) iii and iv
(A) 𝑃(𝑆) = {∅, 𝑚, 𝑛, 𝑜}

1.2 Set Operations


Terms such as subsets and operations on sets are better understood with the use of Venn diagrams.
In this lesson, we will be discussing more about the different operations on sets and its
representations in Venn diagrams.

Two or more sets can be combined in more than one way to produce a different set. In order to do
this, we use different kinds of operations on sets. As we discuss the operations on sets, we represent
each operation in Venn diagrams in order to understand clearly these operations.
Definition:
Venn or Euler diagram was invented by John Venn. It is used as a graphical representation of sets
and their relationships.

In the diagram, it is assumed that there is a universal set that is the superset of all sets relating each
other. This is usually represented as a rectangle that encloses all the given sets which in turn is
represented by circles.
3
The Venn diagram can have shaded areas to represent relationships
between the sets given.
Shaded area represents the elements that are part of set A and also
part of set B.

The Operations on Sets

The following are the different kinds of operations on sets:


A. Disjoint Sets –These are two or more sets that have no
common elements.
Example:
A = {1, 2, 3, 4, 5}
B = {6, 7, 8, 9, 10}
Sets A and B have no common elements so
they are called disjoint sets.

B. Union – The union of sets A and B, denoted by A ∪ B, is the set which contain elements that
belong to either A or B or to both.
Example:
A = {1, 2, 3}
B = {3, 4, 5}
A ∪ B = {1, 2, 3, 4, 5}

C. Intersection – The intersection of sets A and B, denoted


by A ∩ B, is the set of all common elements of A and B.
Example:
A = {1, 2, 3}
B = {3, 4, 5}
A ∩ B = {3}

D. Complement – The complement of set A, written as A’, is


the set of all elements in the universal set, U, that are not
in A.
Example:
U = {x | x is a letter of the English alphabet}
A = {x | x is a vowel of the English alphabet}
A’ = {x | x is not a vowel of the English alphabet}

4
E. Difference – The set difference (or relative compliment)
of set B from set A, written as A – B, is the set of all
elements in A that are not in B.
Example:
A = {1, 2, 3}
B = {3, 4, 5}
A – B = {1, 2}
The Number of Elements in the Set
The number of elements in a set can be easily determined by counting it. But when you have to
count the total number of elements from a union of multiple sets without including redundant
elements, it is easier to use the Inclusion-Exclusion Principle. Let 𝑛 denote the number of
elements of the set.

 If 𝐴 and 𝐵 are any sets, then


𝐧(𝑨 ∪ 𝑩) = 𝐧(𝑨) + 𝐧(𝑩) − 𝐧(𝑨 ∩ 𝑩)
Example:
Ten students were surveyed on their preferred sport and it came out that 3 likes basketball,
5 likes tennis, and 2 likes both basketball and tennis. How many students did not like any of
the two sports?
Solution:
Using the formula above, we let: n(𝐴) − the number of students who likes basketball
n(𝐵) − the number of students who likes tennis
n(𝐴 ∩ 𝐵) − the number of students who like the two sports
Now, we have:
n(𝐴 ∪ 𝐵) = n(𝐴) + n(𝐵) − n(𝐴 ∩ 𝐵)
n(𝐴 ∪ 𝐵) = 3+5−2
n(𝐴 ∪ 𝐵) = 6
Thus, six students liked any of the two sports which mean that 10 − 6 = 4 students did not
like any of the two sports.
 If A, B and C are any sets, then
𝐧(𝑨 ∪ 𝑩 ∪ 𝑪) = 𝐧(𝑨) + 𝐧(𝑩) + 𝐧(𝑪)– 𝐧(𝑨 ∩ 𝑩)– 𝐧(𝑨 ∩ 𝑪)– 𝐧(𝑩 ∩ 𝑪) +
𝐧(𝑨 ∩ 𝑩 ∩ 𝑪)
Example:
Twenty kids were asked on their preference of ice cream flavors and the results are as
follows:

Flavor Preferred Number of Kids


Mango 8

5
Chocolate 9
Vanilla 5
Mango and Chocolate 3
Mango and Vanilla 4
Chocolate and Vanilla 2

If all twenty kids chose at least one flavor, how many kids chose all three flavors?
Solution:
Using the formula above, we let: n(𝐴) − number of kids who chose mango
n(𝐵) − number of kids who chose chocolate
n(𝐶) − number of kids who chose vanilla
n(𝐴 ∩ 𝐵) − number of kids who chose both mango and chocolate
n(𝐴 ∩ 𝐶) − number of kids who chose both mango and vanilla
n(𝐵 ∩ 𝐶) number of kids who chose both vanilla and chocolate

Now, we have:
n(𝐴 ∪ 𝐵 ∪ 𝐶) = n(𝐴) + n(𝐵) + n(𝐶)– n(𝐴 ∩ 𝐵)– n(𝐴 ∩ 𝐶)– n(𝐵 ∩ 𝐶) + n(𝐴 ∩ 𝐵 ∩ 𝐶)
20 = 8 + 9 + 5 − 3 − 4 − 2 + n(𝐴 ∩ 𝐵 ∩ 𝐶)
20 = 13 + n(𝐴 ∩ 𝐵 ∩ 𝐶)
20 − 13 = n(𝐴 ∩ 𝐵 ∩ 𝐶)
n(𝐴 ∩ 𝐵 ∩ 𝐶) = 7
Therefore, seven kids liked all the three flavors of ice cream.
 If A and B are disjoint sets, then
𝐧(𝑨 ∪ 𝑩) = 𝐧(𝑨) + 𝐧(𝑩)
Example:
A hundred people were asked about the subject that they like. Fifty-five chose Mathematics
while thirty-four chose Science. No one liked both Math and Science. How many people chose
at least one subject?
Solution:
Using the formula, we let: n(𝐴) − number of people who chose Math
n(𝐵) − number of people who chose Science
Now, we have:
n(𝐴 ∪ 𝐵) = n(𝐴) + n(𝐵)
n(𝐴 ∪ 𝐵) = 55 + 34
n(𝐴 ∪ 𝐵) = 89
Therefore, 89 people chose at least one subject.
________________________________________________________________________

6
Lesson Exercise:
Answer numbers 1-5 from the given below:
𝑈 = {1,2,3, … ,10}
A = {1,2,3,4,5}
B = {2,4,6,8,9}
C = {3,7,10}
1. What is 𝐴 ∪ 𝐶?
(A) 𝐴 ∪ 𝐶 = {1,2,3,4,5,7,9} 6. What are the elements of set 𝐴?
(B) 𝐴 ∪ 𝐶 = {3} (A) 𝐴 = {1,2}
(C) 𝐴 ∪ 𝐶 = {1,2,3,4,5,7,10} (B) 𝐴 = {1,2,7}
(D) 𝐴 ∪ 𝐶 = {1,3,5} (C) 𝐴 = {1,2,7,10}
(D) 𝐴 = {1,2,7,8,10}
2. What is 𝐴 ∩ 𝐵?
(A) 𝐴 ∩ 𝐵 = {1,2,3,4,5,6,7,8,9} 7. What is 𝐴 ∩ (𝐵 ∪ 𝐶)?
(B) 𝐴 ∩ 𝐵 = {2,4} (A) 𝐴 ∩ (𝐵 ∪ 𝐶) = {7,8}
(C) 𝐴 ∩ 𝐵 = {1,2,3,4,5,6,8,9} (B) 𝐴 ∩ (𝐵 ∪ 𝐶) = {7,8,10}
(D) 𝐴 ∩ 𝐵 = {1,3,5} (C) 𝐴 ∩ (𝐵 ∪ 𝐶) = {8,10}
3. What is 𝐶 ∩ (𝐴 ∪ 𝐵)? (D) None of the above
(A) 𝐶 ∩ (𝐴 ∪ 𝐵) = {1,2,3,9}
(B) 𝐶 ∩ (𝐴 ∪ 𝐵) = {3} 8. What is 𝐴 ∩ 𝐵 ∩ 𝐶?
(C) 𝐶 ∩ (𝐴 ∪ 𝐵) = {1,9} (A) 𝐴 ∩ 𝐵 ∩ 𝐶 = {8,10}
(D) 𝐶 ∩ (𝐴 ∪ 𝐵) = {3,7} (B) 𝐴 ∩ 𝐵 ∩ 𝐶 = {7,10}
(C) 𝐴 ∩ 𝐵 ∩ 𝐶 = {10}
4. What is 𝐴′? (D) 𝐴 ∩ 𝐵 ∩ 𝐶 = {9}
(A) 𝐴′ = Ø
(B) 𝐴′ = {6,7} 9. What is (𝐴 ∪ 𝐵)′ ?
(C) 𝐴′ = {6,7,8,9} (A) (𝐴 ∪ 𝐵)′ = {5,6,7,9,10}
(D) 𝐴′ = {6,7,8,9,10} (B) (𝐴 ∪ 𝐵)′ = {5,6}
(C) (𝐴 ∪ 𝐵)′ = {7,9,10}
5. What is 𝐴 − 𝐶? (D) (𝐴 ∪ 𝐵)′ = {5,6,9}
(A) 𝐴 − 𝐶 = Ø
(B) 𝐴 − 𝐶 = {1,2,4,5} 10. In a class of 30 students, 12 are in the
(C) 𝐴 − 𝐶 = {3} scrabble club, 13 are in the chess club,
(D) 𝐴 − 𝐶 = {1,2,3,4,5,7,10} and 5 are both in the chess and scrabble
club. How many students did not join in
any of the two clubs?
For numbers 6-9, use the given Venn diagram (A) 5
below. (B) 7
(C) 10
(D) 12
________________________________________________________________________

7
Chapter Exercise: ii. 𝑥⊆𝐶
iii. {𝑦} ⊆ 𝐶
1. Which of the following is a set?
(A) {𝑥|𝑥 is unknown} iv. 𝑧∈𝐶
(A) i and ii
(B) {𝑥|𝑥 are all tall people}
(B) i and iii
(C) {𝑥|𝑥 are all small things}
(C) ii and iii
(D) {𝑥|𝑥 is an irrational number}
(D) iii and iv
2. Which of the following is a subset of
𝐴 = {1, 1.5, 2, 2.5, … } 8. What is the power set of 𝑌 =
(A) {𝑥|𝑥 > 5} {𝑎𝑏, 𝑏𝑐}?
(B) The set of real numbers (A) 𝑃(𝑌) = {∅, 𝑎𝑏, 𝑏𝑐, 𝑎𝑏𝑐}
(C) 𝐵 = {3, 3.5, 4, 4.5} (B) 𝑃(𝑆) = {∅, 𝑌, 𝑎𝑏, 𝑏𝑐, 𝑎𝑏𝑐}
(D) The set of odd numbers (C) 𝑃(𝑆) = {∅, 𝑌, {𝑎𝑏}, {𝑏𝑐}}
(D) 𝑃(𝑆) = {∅, 𝑌, 𝑎𝑏, 𝑏𝑐}
For numbers 3-4, express the following sets 9. Which of the following is a subset of
in roster form: set 𝐴 = {−1, −2, −3, … }
3. The set ℕ2𝑘 of all even natural (A) 𝐵 = {0, −1, −2}
numbers. (B) The set of negative integers
(A) ℕ2𝑘 = {2,4,6} (C) The set of irrational numbers
(B) ℕ2𝑘 = {0,2,4} (D) 𝐵 = {𝑥|𝑥 ∈ ℤ, and 0 ≤ 𝑥 ≤ −5}
(C) ℕ2𝑘 = {2,4,6, … }
(D) ℕ2𝑘 = {2,4,6,8,10} 10. Which of the following is the
superset of the set 𝐴 =
4. The set 𝐵 of all letters in the word {−3, −2, −1,0,1,2, … }?
𝑚𝑎𝑡ℎ𝑒𝑚𝑎𝑡𝑖𝑐𝑠. (A) 𝐵 = {𝑏|𝑏 ∈ ℤ 𝑎𝑛𝑑 𝑏 ≥ −5}
(A) 𝐵 = {𝑚, 𝑎, 𝑡, ℎ, 𝑒, 𝑚, 𝑎, 𝑡, 𝑖, 𝑐, 𝑠, … } (B) 𝐵 = {𝑏|𝑏 ∈ ℤ− }
(B) 𝐵 = {𝑚, 𝑎, 𝑡, ℎ, 𝑒, 𝑡, 𝑖, 𝑐, 𝑠} (C) 𝐵 = {0,2,4,6,8,10}
(C) 𝐵 = {𝑎, 𝑐, 𝑒, ℎ, 𝑖, 𝑚, 𝑠, 𝑡} (D) 𝐵 = {𝑏|𝑏 ∈ ℝ− }
(D) 𝐵 = {𝑚, 𝑎, 𝑡, ℎ}
For numbers 11-14, refer on the given
For numbers 5-6, express the following sets below.
in set-builder notation: 𝐴 = {𝑎, 𝑏, 𝑐, 𝑑}
5. The set 𝑋 of all years with 367 days. 𝐵 = {𝑎, 𝑏, 𝑑, 𝑒}
(A) 𝑋 = {1991,1992,1993} 𝐶 = {𝑏, 𝑐, 𝑑, 𝑓}
(B) 𝑋 = {∅} 11. What is 𝐴 ∪ 𝐵 ∪ 𝐶?
(C) 𝑋 = {2014} (A) 𝐴 ∪ 𝐵 ∪ 𝐶 = {𝑎𝑏, 𝑐𝑑, 𝑒𝑓}
(D) 𝑋 = ∅ (B) 𝐴 ∪ 𝐵 ∪ 𝐶 = {𝑎, 𝑏, 𝑐, 𝑑, 𝑒, 𝑓}
(C) 𝐴 ∪ 𝐵 ∪ 𝐶 = {𝑎, 𝑏, 𝑐, 𝑑, 𝑒}
6. The set 𝑌 of all vowels in the word (D) 𝐴 ∪ 𝐵 ∪ 𝐶 = {𝑎, 𝑏, 𝑑, 𝑐}
𝑃ℎ𝑖𝑙𝑖𝑝𝑝𝑖𝑛𝑒𝑠.
(A) 𝑌 = {𝑖, 𝑒} 12. What is (𝐴 − 𝐶)′?
(B) 𝑌 = {𝑖, 𝑖, 𝑒} (A) (𝐴 − 𝐶)′ = {𝑎, 𝑒, 𝑓}
(C) 𝑌 = {𝑖, 𝑖, 𝑖, 𝑒} (B) (𝐴 − 𝐶)′ = {𝑏, 𝑐, 𝑑, 𝑒, 𝑓}
(D) 𝑌 = {𝑝, ℎ, 𝑙, 𝑛, 𝑠} (C) (𝐴 − 𝐶)′ = {𝑎}
(D) (𝐴 − 𝐶)′ = {𝑒, 𝑓}
7. Given 𝐶 = {𝑥, 𝑦, 𝑧}, which of the
following is/are true? 13. What is ((𝐴 ∪ 𝐵) ∩ 𝐶)′?
i. {𝑥} ∈ 𝐶

8

(A) ((𝐴 ∪ 𝐵) ∩ 𝐶) = {𝑎, 𝑒, 𝑓} (A) 𝐴 ∪ 𝐵 ∪ 𝐶
′ (B) (𝐴 ∪ 𝐵) ∩ 𝐶
(B) ((𝐴 ∪ 𝐵) ∩ 𝐶) = {𝑏, 𝑐, 𝑑}
′ (C) (𝐴 ∩ 𝐵) ∪ 𝐶
(C) ((𝐴 ∪ 𝐵) ∩ 𝐶) = {𝑎, 𝑏, 𝑒, 𝑓} (D) (𝐴 ∩ 𝐵) − 𝐶

(D) ((𝐴 ∪ 𝐵) ∩ 𝐶) = {𝑎, 𝑏, 𝑒}
18. What is the shaded region in the
14. What is 𝐵 ∩ 𝐴′? Venn diagram below?
(A) 𝑈 = {𝑒, 𝑓}
(B) 𝑈 = {𝑐, 𝑑, 𝑒}
(C) 𝑈 = {𝑒}
(D) 𝑈 = {𝑎, 𝑒}

Refer to the Venn diagram below for (A) (𝐴 − 𝐶)′


numbers 15-16. (B) 𝐴 − 𝐵 − 𝐶
(C) (𝐶 − 𝐴) − 𝐵
(D) (𝐵 − 𝐶) − 𝐴

Refer to the problem below for numbers 19-


20.

A survey of 100 students showed that 55


15. What are the elements of the set 𝐶? played scrabble, 30 played chess, 15 played
(A) 𝐶 = {𝑒, 𝑓, 𝑔, ℎ, 𝑖, 𝑗} chess and draughts, 25 played draughts and
(B) 𝐶 = {𝑓, ℎ, 𝑔} scrabble, 13 played scrabble and chess, and
(C) 𝐶 = {𝑖, 𝑗, 𝑒} 5 played all three games.
(D) 𝐶 = {𝑓, ℎ, 𝑗}
19. How many students played chess
16. What is 𝐴 ∪ 𝐶? only?
(A) 𝐴 ∪ 𝐶 = {𝑎, 𝑏, 𝑐, 𝑑, 𝑒, 𝑓, 𝑔, ℎ, 𝑖, 𝑗} (A) 0
(B) 𝐴 ∪ 𝐶 = {𝑎, 𝑏, 𝑑, 𝑒, 𝑓, 𝑔, ℎ, 𝑖, 𝑗} (B) 7
(C) 𝐴 ∪ 𝐶 = {𝑎, 𝑐, 𝑑, 𝑒, ℎ, 𝑖, 𝑗} (C) 11
(D) 𝐴 ∪ 𝐶 = {𝑎, 𝑏, 𝑐, 𝑑, 𝑒, 𝑓, 𝑗} (D) 8

17. What is the shaded region in the 20. How many students played draughts
Venn diagram below? only?
(A) 28
(B) 23
(C) 32
(D) 30

9
2 Real Numbers
2.1 Set of Real Numbers
The set of real numbers are composed of the following:
A. Natural Numbers ℕ
This set is also commonly known as the “Counting Numbers”
1, 2, 3, 4, 5, …

B. Whole Numbers ℕ0
These are the natural numbers together with the number zero.
0, 1, 2, 3, 4, …

C. Integers ℤ
This is the set of whole numbers including its negatives
… , −3, −2, −1, 0, 1, 2, 3, …

D. Rational Numbers ℚ
a
These are all numbers of the form , where a and b are integers (but b cannot be zero).
b

The decimals are either repeating or terminating.


… , −1.75, −1.333333, −1, −0.5, 0, 0.5, 1, 1.33333, 1.75, …

E. Irrational Numbers
This is the set of numbers that cannot be expressed in the ratio of integers and decimals that
do not repeat nor terminate.
√2 = 1.41421356 … , 𝜋 = 3.14159265 …

To clearly picture out the different kinds of real numbers, Figure 2.1 shows the subsets of the set
of real numbers enclosed in every circle and 2.2 shows a graphic tree of the distinction of the
subsets of real numbers.

10
Rational
Numbers

Integers 1
Whole 2
Numbers −1, −2, −3, …
Natural 0. 3̅
Numbers 0
1, 2, 3, … 1.75 Irrational
Numbers
3 √2
− 𝜋
4 1
√3

Figure 2.1. The Set of Real Numbers

Figure 2.2. The Distinction of the Real Numbers


The Real Number Line
The real number system can be visualized as a horizontal line that extends from a special point
called the origin in both directions towards infinity. (See Figure 2.3) Also associated with the line
is a unit of length. The origin corresponds to the number 0. A positive number 𝑥 corresponds to a
point 𝑥 units away from the origin to the right, and a negative number −𝑥 corresponds to a point
on the line 𝑥 units away from the origin to the left

11
Figure 2.3. The Real Number Line
____________________________________________________________________________

Lesson Exercise:
Identify which set each number belongs to.
1. 𝜋
(A) Rational Numbers 6. 3.5
(B) Irrational Numbers (A) Rational Numbers
(C) Integers (B) Irrational Numbers
(D) Whole Numbers (C) Integers
(D) Whole Numbers
2. √3.3
1
(A) Rational Numbers 7.
3
(B) Irrational Numbers (A) Rational Numbers
(C) Integers (B) Irrational Numbers
(D) Whole Numbers (C) Integers
(D) Whole Numbers
5
3. − 1
3 8. √
(A) Rational Numbers 2
(B) Irrational Numbers (A) Rational Numbers
(C) Integers (B) Irrational Numbers
(D) Whole Numbers (C) Integers
(D) Whole Numbers
4. 0
(A) Negative Integers 9. √25
(B) Irrational Numbers (A) Fractions
(C) Positive Integers (B) Irrational Numbers
(D) Whole Numbers (C) Negative Integers
(D) Whole Numbers
1
10. π
5. −10 2
(A) Rational Numbers
(A) Repeating Decimal
(B) Irrational Numbers
(B) Irrational Numbers
(C) Integers (C) Integers
(D) Whole Number
(D) Fraction

12
2.2 Inequality
Some mathematical statements involve the symbols <, >, ≥, or ≤. These symbols are referred to
as inequalities.

Definition:
A mathematical statement indicating that two algebraic expressions are not equal is called an
inequality.
If 𝑎 and 𝑏 are two numbers and 𝑎 is on the left of 𝑏 on the real number line, then 𝑎 is less than
b. This is written as 𝑎 < 𝑏.

𝑎 𝑏
𝒂<𝒃

Moreover, if 𝑎 is on the right of 𝑏 on the real number line, then 𝑎 is greater than 𝑏. This is written
as 𝑎 > 𝑏.

𝑏 𝑎
𝒂>𝒃
There are also inequality symbols for less than or equal to (≤) and greater than or equal to (≥).
Examples:
 7 ≤ 8, 7 is less than or equal to 8. This is true since 7 < 8.
 7 ≤ 7, 7 is less than or equal to 7. This is also true since 7 = 7.
_______________________________________________________________________________
Lesson Exercise:
1. Which of the following inequalities is 3. Which of the following inequalities is
true? false?
(A) 15 > 14 (A) 5 > −1
(B) −15 < −16 (B) −6 ≤ −8
(C) −2 > −1 (C) −2 > −3
(D) 0 ≤ −1 (D) 1 ≥ 1

2. Which of the following inequalities is 4. Which of the following inequalities is


true? false?
(A) 11 ≤ 11 (A) −3 ≤ −2
(B) 5 ≥ 5.000001 (B) 2 ≥ −3
(C) −6 > −4 (C) 5 > −4
(D) 10 > 10.000001 (D) 0 > 0

5. Fill in the blank: −15 ____ − 13


13
(A) > 8. 83____103
(B) = (A) >
(C) ≤ (B) =
(D) ≥ (C) ≤
(D) ≥
6. −10____12
(A) > 9. −52____ − 63
(B) = (A) >
(C) ≤ (B) =
(D) ≥ (C) ≤
(D) <
7. 240____ − 100
(A) > 10. 11____ − 11
(B) = (A) >
(C) ≤ (B) <
(D) < (C) ≤
(D) =

2.3 Absolute Value


The absolute value of any integer 𝑎, denoted by |𝑎|, is the distance of 𝑎 from zero on the
number line. It is always a positive number or a zero.

−𝑎, 𝑎 ≤ 0
The formal definition of absolute value is: |𝑎| = {
𝑎, 𝑎 > 0
Examples:
|5| = 5; |−1| = 1; |0| = 0
Two numbers with the same absolute value but on the opposite sides of zero are opposite
numbers, or opposites. This is also called additive inverse.
Examples:

The integers 2 and −2 have the same absolute value which is 2.

Lesson Exercise:
1. |5| = ____ (A) −8
(A) 0 1
(B)
1 8
(B) (C) 8
5
1
(C) −5 (D) −
8
(D) 5

2. |−8| = ____

14
2
3. |− | = ____
5
4 7. −|−10| = ____
(A) −
10
4 (A) 10
(B) (B) −10
20
2
(C) − (C) ±10
5
2 (D) None of the above
(D)
5

8. −|−134| = ____
4. −|−1.25| = ____ (A) 134
(A) −1.25 (B) −134
(B) 1.25 (C) ±134
(C) 1.00 (D) None of the above
(D) 0.25

9. −|4| = ____
5. |−11| = ____ (A) −4
(A) 11 (B) 4
(B) −11 (C) ±4
1
(C) − (D) None of the above
11
1
(D)
11
10. |−1| = ____
(A) 1
6. −|−20| = ____ (B) −1
(A) 20 (C) ±1
(B) ±20 (D) None of the above
(C) −20
(D) None of the above

2.4 Operations Involving Real Numbers


There are four operations involving real numbers. These are the following:
A. Addition of Real Numbers
Addition is the process of finding the 𝑠𝑢𝑚 of numbers. The numbers to be added are called
𝑎𝑑𝑑𝑒𝑛𝑑𝑠.

𝐴𝑑𝑑𝑖𝑛𝑔 𝑅𝑒𝑎𝑙 𝑁𝑢𝑚𝑏𝑒𝑟𝑠 𝑜𝑓 𝑡ℎ𝑒 𝑆𝑎𝑚𝑒 𝑆𝑖𝑔𝑛


To add real numbers with the same sign, directly add the numbers without regarding the
signs. Then copy the common sign of the numbers.
Example 1:
10 + 5 = +(10 + 5) = +15

Example 2:
−10 + (−5) = −(10 + 5) = −15

15
𝐴𝑑𝑑𝑖𝑛𝑔 𝑅𝑒𝑎𝑙 𝑁𝑢𝑚𝑏𝑒𝑟𝑠 𝑜𝑓 𝐷𝑖𝑓𝑓𝑒𝑟𝑒𝑛𝑡 𝑆𝑖𝑔𝑛𝑠
To add real numbers with different signs, subtract the smaller number from the larger
number. The sign of the larger number will be the sign of the difference.
Example 3:
−10 + 5 = −(10 − 5) = −5
Example 4:
10 + (−5) = +(10 − 5) = +5

Example 5:
−4 + 7 = +(7 − 4) = +3

B. Subtraction of Real Numbers


Subtraction is the process of finding the 𝑑𝑖𝑓𝑓𝑒𝑟𝑒𝑛𝑐𝑒 of numbers. The number that comes first
is the 𝑚𝑖𝑛𝑢𝑒𝑛𝑑 and the number that will be subtracted from the minuend is called the
𝑠𝑢𝑏𝑡𝑟𝑎ℎ𝑒𝑛𝑑, i.e.,
𝑚𝑖𝑛𝑢𝑒𝑛𝑑 − 𝑠𝑢𝑏𝑡𝑟𝑎ℎ𝑒𝑛𝑑 = 𝑑𝑖𝑓𝑓𝑒𝑟𝑒𝑛𝑐𝑒

𝑆𝑢𝑏𝑡𝑟𝑎𝑐𝑡𝑖𝑛𝑔 𝑅𝑒𝑎𝑙 𝑁𝑢𝑚𝑏𝑒𝑟𝑠 𝑜𝑓 𝑡ℎ𝑒 𝑆𝑎𝑚𝑒 𝑆𝑖𝑔𝑛


To subtract real numbers of the same sign,
i. Both positive:
Example 6:
+5 − (+6)
= +5 − 6 𝑇ℎ𝑒 − 𝑎𝑛𝑑 + 𝑠𝑖𝑔𝑛𝑠 𝑖𝑛 𝑏𝑒𝑡𝑤𝑒𝑒𝑛 − 5 𝑎𝑛𝑑 6 𝑖𝑠 𝑐ℎ𝑎𝑛𝑔𝑒𝑑 𝑡𝑜 − 𝑜𝑛𝑙𝑦.
= −(6 − 5) 𝐶𝑜𝑝𝑦 𝑡ℎ𝑒 𝑠𝑖𝑔𝑛 𝑜𝑓 𝑡ℎ𝑒 𝑙𝑎𝑟𝑔𝑒𝑟 𝑛𝑢𝑚𝑏𝑒𝑟,
6. 𝐺𝑒𝑡 𝑡ℎ𝑒 𝑑𝑖𝑓𝑓𝑒𝑟𝑒𝑛𝑐𝑒 𝑜𝑓 6 𝑎𝑛𝑑 5.
= −1 𝑆𝑖𝑚𝑝𝑙𝑖𝑓𝑦.

Example 7:
+11 − (+5)
= +11 − 5 𝑇ℎ𝑒 − 𝑎𝑛𝑑 + 𝑠𝑖𝑔𝑛𝑠 𝑖𝑛 𝑏𝑒𝑡𝑤𝑒𝑒𝑛 − 11 𝑎𝑛𝑑 5 𝑖𝑠 𝑐ℎ𝑎𝑛𝑔𝑒𝑑 𝑡𝑜 − 𝑜𝑛𝑙𝑦.
= +(11 − 5) 𝐶𝑜𝑝𝑦 𝑡ℎ𝑒 𝑠𝑖𝑔𝑛 𝑜𝑓 𝑡ℎ𝑒 𝑙𝑎𝑟𝑔𝑒𝑟 𝑛𝑢𝑚𝑏𝑒𝑟,
11. 𝐺𝑒𝑡 𝑡ℎ𝑒 𝑑𝑖𝑓𝑓𝑒𝑟𝑒𝑛𝑐𝑒 𝑜𝑓 11 𝑎𝑛𝑑 5.
= +6 𝑆𝑖𝑚𝑝𝑙𝑖𝑓𝑦.

ii. Both negative:


Example 8:
−10 − (−2)
= −10 + 2 𝑇ℎ𝑒 − 𝑎𝑛𝑑 − 𝑠𝑖𝑔𝑛𝑠 𝑖𝑛 𝑏𝑒𝑡𝑤𝑒𝑒𝑛 − 10 𝑎𝑛𝑑 − 2 𝑖𝑠 𝑐ℎ𝑎𝑛𝑔𝑒𝑑 𝑡𝑜 + 𝑜𝑛𝑙𝑦.
= −(10 − 2)
𝐶𝑜𝑝𝑦 𝑡ℎ𝑒 𝑠𝑖𝑔𝑛 𝑜𝑓 𝑡ℎ𝑒 𝑙𝑎𝑟𝑔𝑒𝑟 𝑛𝑢𝑚𝑏𝑒𝑟, 10. 𝐺𝑒𝑡 𝑡ℎ𝑒 𝑑𝑖𝑓𝑓𝑒𝑟𝑒𝑛𝑐𝑒 𝑜𝑓 10 𝑎𝑛𝑑 2.
= −8 𝑆𝑖𝑚𝑝𝑙𝑖𝑓𝑦.

16
Example 9:
−3 − (−6)
= −3 + 6 𝑇ℎ𝑒 − 𝑎𝑛𝑑 − 𝑠𝑖𝑔𝑛𝑠 𝑖𝑛 𝑏𝑒𝑡𝑤𝑒𝑒𝑛 − 3 𝑎𝑛𝑑 − 6 𝑖𝑠 𝑐ℎ𝑎𝑛𝑔𝑒𝑑 𝑡𝑜 + 𝑜𝑛𝑙𝑦.
= +(6 − 3) 𝐶𝑜𝑝𝑦 𝑡ℎ𝑒 𝑠𝑖𝑔𝑛 𝑜𝑓 𝑡ℎ𝑒 𝑙𝑎𝑟𝑔𝑒𝑟 𝑛𝑢𝑚𝑏𝑒𝑟,
6. 𝐺𝑒𝑡 𝑡ℎ𝑒 𝑑𝑖𝑓𝑓𝑒𝑟𝑒𝑛𝑐𝑒 𝑜𝑓 6 𝑎𝑛𝑑 3.
=3 𝑆𝑖𝑚𝑝𝑙𝑖𝑓𝑦.

𝑆𝑢𝑏𝑡𝑟𝑎𝑐𝑡𝑖𝑛𝑔 𝑅𝑒𝑎𝑙 𝑁𝑢𝑚𝑏𝑒𝑟𝑠 𝑜𝑓 𝐷𝑖𝑓𝑓𝑒𝑟𝑒𝑛𝑡 𝑆𝑖𝑔𝑛𝑠


Example 10:
+5 − (−6)
= +5 + 6 𝑇ℎ𝑒 − 𝑎𝑛𝑑 − 𝑠𝑖𝑔𝑛𝑠 𝑖𝑛 𝑏𝑒𝑡𝑤𝑒𝑒𝑛 + 5 𝑎𝑛𝑑 − 6 𝑖𝑠 𝑐ℎ𝑎𝑛𝑔𝑒𝑑 𝑡𝑜 + 𝑜𝑛𝑙𝑦.
= +(5 + 6) 𝐶𝑜𝑝𝑦 𝑡ℎ𝑒 𝑠𝑖𝑔𝑛 𝑜𝑓 𝑡ℎ𝑒 𝑙𝑎𝑟𝑔𝑒𝑟 𝑛𝑢𝑚𝑏𝑒𝑟, 6. 𝐺𝑒𝑡 𝑡ℎ𝑒 𝑠𝑢𝑚 𝑜𝑓 6 𝑎𝑛𝑑 5.
= +11 𝑆𝑖𝑚𝑝𝑙𝑖𝑓𝑦.

Example 11:
−7 − (+2)
= −7 − 2 𝑇ℎ𝑒 − 𝑎𝑛𝑑 − 𝑠𝑖𝑔𝑛𝑠 𝑖𝑛 𝑏𝑒𝑡𝑤𝑒𝑒𝑛 + 5 𝑎𝑛𝑑 − 6 𝑖𝑠 𝑐ℎ𝑎𝑛𝑔𝑒𝑑 𝑡𝑜 + 𝑜𝑛𝑙𝑦.
= −(7 + 2) 𝐶𝑜𝑝𝑦 𝑡ℎ𝑒 𝑠𝑖𝑔𝑛 𝑜𝑓 𝑡ℎ𝑒 𝑙𝑎𝑟𝑔𝑒𝑟 𝑛𝑢𝑚𝑏𝑒𝑟, 7. 𝐺𝑒𝑡 𝑡ℎ𝑒 𝑠𝑢𝑚 𝑜𝑓 7 𝑎𝑛𝑑 2.
= −9 𝑆𝑖𝑚𝑝𝑙𝑖𝑓𝑦.

C. Multiplication of Real Numbers


Multiplication is the process of finding the 𝑝𝑟𝑜𝑑𝑢𝑐𝑡 of two numbers.
The following are the symbols representing multiplication:
 1 × 2 = 10
 1 • 2 = 10
 (1)(2) = 10
 (1)2 = 10
 1(2) = 10

𝑀𝑢𝑙𝑡𝑖𝑝𝑙𝑦𝑖𝑛𝑔 𝑅𝑒𝑎𝑙 𝑁𝑢𝑚𝑏𝑒𝑟𝑠 𝑜𝑓 𝑡ℎ𝑒 𝑆𝑎𝑚𝑒 𝑆𝑖𝑔𝑛𝑠


In multiplying real numbers of the same signs, their product is always positive.
Example 12:
(12)(10) = 120

Example 13:
(−3)(−6) = 18

𝑀𝑢𝑙𝑡𝑖𝑝𝑙𝑦𝑖𝑛𝑔 𝑅𝑒𝑎𝑙 𝑁𝑢𝑚𝑏𝑒𝑟𝑠 𝑜𝑓 𝐷𝑖𝑓𝑓𝑒𝑟𝑒𝑛𝑡 𝑆𝑖𝑔𝑛𝑠


In multiplying real numbers of different signs, their product is always negative.

17
Example 14:
(12)(−10) = −120

Example 15:
(−3)(6) = −18

D. Division of Real Numbers


Division is the process of finding the quotient.

15 ÷ 3 = 5
quotient
dividend
divisor

The following are ways to represent division:


15
 = 5
3
 15 ÷ 3 = 5

𝐷𝑖𝑣𝑖𝑑𝑖𝑛𝑔 𝑅𝑒𝑎𝑙 𝑁𝑢𝑚𝑏𝑒𝑟𝑠 𝑜𝑓 𝑡ℎ𝑒 𝑆𝑎𝑚𝑒 𝑆𝑖𝑔𝑛𝑠


In dividing real numbers of the same signs, their quotient is always positive.
Example 16:
15
= 5
3

Example 17:
−15
= 5
−3

𝐷𝑖𝑣𝑖𝑑𝑖𝑛𝑔 𝑅𝑒𝑎𝑙 𝑁𝑢𝑚𝑏𝑒𝑟𝑠 𝑜𝑓 𝐷𝑖𝑓𝑓𝑒𝑟𝑒𝑛𝑡 𝑆𝑖𝑔𝑛𝑠


In dividing real numbers of different signs, their quotient is always negative.
Example 18:
−15
= −5
3

Example 19:
15
=−5
−3

18
𝑍𝑒𝑟𝑜 𝑎𝑛𝑑 𝑂𝑛𝑒 𝑖𝑛 𝐷𝑖𝑣𝑖𝑠𝑖𝑜𝑛
i. Zero divided by any number other than zero is 0.
0
= 0
𝑥
ii. Division by zero is not defined
𝑥
= undefined
0

iii. Any number other than zero divided by itself is 1


𝑥
= 1, 𝑥 ≠ 0
𝑥
iv. Any number divided by 1 is equal to the number.
𝑥
= 𝑥
1
______________________________________________________________________
Lesson Exercise:
1. (354) + (−252) = ____
(A) 102
6. Melanie’s internet bill is ₱990.00 for
(B) 112
the month of October. She paid
(C) 132
₱1,500 for her October bill and
(D) 152
received another bill for November
amounting to ₱1,020. How much
should she pay for her current bill if
2. (−1544) + (567) = ____
she has an excess payment last
(A) −954
October?
(B) −977
(A) ₱550
(C) −988
(B) ₱450
(D) −985 (C) ₱510
(D) ₱610
3. (2341) − 1725 = ____
7. Darren is an ice cream vendor. He
(A) 606
needs ₱1,500.00 in order to make 1
(B) 633
drum of ice cream. He sells one drum
(C) 616
of ice cream every day. On his first
(D) 622
day, he was able to earn ₱1,850.00.
He earned ₱1,940.00 on the second
day and on the third day, he earned
4. (−4138) − (237) = ____
₱1,680.00. How much was Darren’s
(A) −4355
net profit for three days?
(B) −4573
(A) ₱875
(C) −4375
(B) ₱587
(D) −4475
(C) ₱785
(D) ₱970
5. (155 + 172) − (254 − 173) +
(387 − 133) = ____ 8. In Indiana, one of the states in
America, the temperature (in Celsius)
(A) 450
was recorded for a week. The results
(B) 480
are as follows:
(C) 500
−1°, 3°, −5°, 8°, 4°, 0°, and − 2°
(D) 510

19
What is the average temperature in (D) 34 meters
Indiana for the entire week?
(A) 1.5℃ 10. One of the coldest place on Earth,
(B) 1℃ Antartica, has a recorded
(C) −1℃ temperature of −92 ℃ while one of
(D) 0.5℃ the hottest place on Earth, Ethiopia,
has a recorded temperature of 37 ℃.
9. Mt. Apo has an elevation of 2,954 Find the difference between the
meters while Mt. Pulag has an temperature of Antartica and
elevation of 2,922 meters. How much Ethiopia.
higher is Mt. Apo than Mt. Pulag? (A) 55
(A) 27 meters (B) 76
(B) 28 meters (C) 109
(C) 32 meters (D) 129

2.5 Order of Operations


In dealing with numerical expressions like 2 + 5 × 4 or 2 − 3 ÷ 3(4 + 1), we oftentimes get
confused on what operation to use. Sometimes, we even get different answers as we try to
evaluate these. In this lesson, we will learn how to evaluate and get a single value from the
numerical expressions.
Expressions like 3 − 5 × 5 and 25 ÷ 5 + 2 are called numerical expressions. These contain a
combination of numbers with operations such as addition, subtraction, multiplication, division, and
some with exponents.
In evaluating numerical expressions, the goal is to find its numerical value. And to avoid confusion,
mathematicians have agreed upon the following order of operations.
Simplify the expressions inside the grouping symbols or expressions inside
Step 1
parentheses.
Step 2 If there are any exponents, simplify them.
Step 3 Do the operation multiplication and/or division from left to right.
Step 4 Do the operation addition and/or subtraction from left to right.
Table 2.5.1
Numerical expressions have only one value.
Example 1:
Consider: 10 + 5 × 2

10 + 5 × 2 10 + 5 × 2
= 10 + 10 𝑀𝑢𝑙𝑡𝑖𝑝𝑙𝑦, 𝑎𝑛𝑑 𝑡ℎ𝑒𝑛 𝑎𝑑𝑑. = 15 × 2 𝐷𝑜 𝑛𝑜𝑡 𝑎𝑑𝑑 𝑡ℎ𝑒𝑛 𝑚𝑢𝑙𝑡𝑖𝑝𝑙𝑦.
= 20 = 20

Which is the correct value, 20 or 30? Following the order of operations given in Table 2.5.1, the
correct value of 10 + 5 × 2 is 20.
Example 2:
Consider: 24 ÷ 8 + 5 × 22

20
24 ÷ 8 + 5 × 22
= 24 ÷ 8 + 5 × 4 𝐸𝑣𝑎𝑙𝑢𝑎𝑡𝑒 𝑡ℎ𝑒 𝑒𝑥𝑝𝑜𝑛𝑒𝑛𝑡𝑠 𝑓𝑖𝑟𝑠𝑡.
= 24 ÷ 8 + 5 × 4 𝐷𝑖𝑣𝑖𝑑𝑒/𝑀𝑢𝑙𝑡𝑖𝑝𝑙𝑦 𝑓𝑟𝑜𝑚 𝑙𝑒𝑓𝑡 𝑡𝑜 𝑟𝑖𝑔ℎ𝑡.
= 3 + 20
= 23

Example 3:
Consider: 6(22 − 18) − 4 ∙ 5

6(22 − 18) − 4 ∙ 5
= 6(4) − 4 ∙ 5 𝐸𝑣𝑎𝑙𝑢𝑎𝑡𝑒 22 𝑎𝑛𝑑 18 𝑓𝑖𝑟𝑠𝑡.
= 24 − 4 ∙ 5 6(4)𝑚𝑒𝑎𝑛𝑠 6 × 4
= 24 − 20 4 ∙ 5 𝑚𝑒𝑎𝑛𝑠 4 𝑡𝑖𝑚𝑒𝑠 5
=4 𝑆𝑢𝑏𝑡𝑟𝑎𝑐𝑡 20 𝑓𝑟𝑜𝑚 24.
Example 4:
23−12÷4
Consider:
9−2×2

23 − 12 ÷ 4
9−2×2
= (23 − 12 ÷ 4) ÷ (9 − 2 × 2) 𝑅𝑒𝑤𝑟𝑖𝑡𝑒 𝑎𝑠 𝑎 𝑑𝑖𝑣𝑖𝑠𝑖𝑜𝑛 𝑒𝑥𝑝𝑟𝑒𝑠𝑠𝑖𝑜𝑛.
= (23 − 3) ÷ (9 − 4) 𝐸𝑣𝑎𝑙𝑢𝑎𝑡𝑒 12 ÷ 4 𝑓𝑖𝑟𝑠𝑡 𝑎𝑛𝑑 2 × 2.
= 20 ÷ 5 𝐸𝑣𝑎𝑙𝑢𝑎𝑡𝑒 23 − 3 𝑎𝑛𝑑 9 − 4.
=4 𝐷𝑖𝑣𝑖𝑑𝑒 20 𝑏𝑦 5.

______________________________________________________________________

Lesson Exercise:
1. Evaluate 3 + 4 × 5
(A) 35 4. Evaluate 5[3 − (12 ÷ 4) + 9]
(B) 24 (A) 40
(C) 21 (B) 45
(D) 23 (C) 50
(D) 55
2. Evaluate 20 ÷ 4 × 6
5
(A) 40−25
6
(B) 30 5. Evaluate
10÷2
(C) 45 (A) 1
(D) None of the above (B) 2
(C) 3
3. Evaluate 3(2 + 5) ÷ 7 (D) 4
(A) 1
(B) 2 6. Which expression has a value of 11?
(C) 3 (A) 2(3 − 4) + 32
(D) 4 (B) 15 ÷ 3 − 52 − 22

21
(C) 75 ÷ (2 + 3) − 4 defect in one of his polo shirt and one
(D) 5 − 2[−3(2 − 4)] pair of shoes and was able to get a
full refund of the defected items. How
7. Which expression has a value of much money did James have after
121? returning the defected items?
(A) −13 + 4(40 + 15 − 26) (A) ₱310.00
(B) 11(5 ∙ 2 + 1) (B) ₱577.00
(C) 12 ÷ 4 × 2 − 132 (C) ₱512.00
(D) 20 − 15 × 3 + 16 × 2 (D) ₱516.00

8. Which expression has a value of 18? 10. To generate income, high school
(A) (20 ÷ 2) − 10 × 3 students make 12 boxes of buko pie
(B) 2[2(6 − 3)] + 5 where each box contains 14 slices
(C) 27 ÷ 3 + (12 − 4) and sell each slice for ₱25.00. They
(D) (9 × 3) − 63 ÷ 7 also sell 8 dozens of munchkins and
sells each munchkin for ₱4.50. If the
9. James had ₱1,000.00 and withdrew students were able to sell all of the
₱500.00 from his bank account. He buko pie and munchkin, how much
bought a pair of socks for ₱85.00, 3 money were they able to generate?
polo shirts for ₱265.00 each, and 2 (A) ₱4,632.00
pairs of shoes for ₱308.00 each. (B) ₱3,560.00
After shopping, James discovered a (C) ₱4,514.00
(D) ₱3,156.00

2.6 Properties of Real Numbers


The six properties of real numbers are listed below.

A. Closure Property
If 𝑎 and 𝑏 are real numbers then (𝑎 + 𝑏), (𝑎 – 𝑏), and (𝑎 ∙ 𝑏) are real numbers. The
𝑎
form , where 𝑏 ≠ 0 is also a real number.
𝑏

Example 1:
Given 𝑎 = −6, 𝑏 = 9 and 𝑎, 𝑏 are both real numbers. Then,
 𝑎 + 𝑏 = −6 + 9 = 3 is a real number
 𝑎 − 𝑏 = −6 − 9 = −15 is a real number
 𝑎 ∙ 𝑏 = −6 ∙ 9 = −54 is a real number
𝑎 −6 −2
 = = is a real number
𝑏 9 3

B. Commutative Property
a.) Addition: 𝑎 + 𝑏 = 𝑏 + 𝑎
b.) Multiplication: 𝑎𝑏 = 𝑏𝑎
Example 2:
Given 𝑎 = 2 and 𝑏 = −3:

22
2 + (−3) = (−3) + 2 2(−3) = (−3)2
−1 = −1 −6 = −6

C. Associative Property
a.) Addition: (𝑎 + 𝑏) + 𝑐 = 𝑎 + (𝑏 + 𝑐)
b.) Multiplication: (𝑎𝑏)𝑐 = 𝑎(𝑏𝑐)
Example 3:
Given 𝑎 = 4, 𝑏 = 2 and 𝑐 = −1:
(4 + 2) + (−1) = 4 + (2 + (−1)) (4 ∙ 2)(−1) = 4(2 ∙ (−1))
6 + (−1) = 4+1 8(−1) = 4(−2)
5 = 5 −8 = −8

D. Distributive Property
𝑎(𝑏 + 𝑐) = 𝑎𝑏 + 𝑎𝑐 or (𝑏 + 𝑐)𝑎 = 𝑏𝑎 + 𝑐𝑎
Example 4:
Given 𝑎 = 2, 𝑏 = 4 and 𝑐 = 5:
2(4 + 5) = 2(4) + (5) (4 + 5)2 = (4)2 + (5)2
2(9) = 8 + 10 (9)2 = 8 + 10
18 = 18 18 = 18

E. Identity Property
a.) Addition: 𝑎 + 0 = 𝑎; 0 is the identity element for addition
b.) Multiplication: 𝑎(1) = 𝑎; 1 is the identity element for multiplication
Example 5:
Given 𝑎 = 10:
 10 + 0 = 10
 10(1) = 10

F. Inverse Property
a.) Addition: 𝑎 + (−𝑎) = 0
1
b.) Multiplication: 𝑎 ∙ = 1, 𝑎 ≠ 0
𝑎

Example 6:
Given 𝑎 = 5:
 5 + (−5) = 0
1
 5∙ =1
5

23
Lesson Exercise:
Identify the properties of numbers shown in (D) Inverse Property
the equations below:
6. 0.98 + (−0.98) = 0
1. 100 + 0 = 100
(A) Associative Property
(A) Distributive Property
(B) Commutative Property
(B) Commutative Property
(C) Distributive Property
(C) Identity Property
(D) Inverse Property
(D) Inverse Property
7. 3 (1) = 3
2. 12(5) = 5(12)
(A) Associative Property
(A) Distributive Property
(B) Commutative Property
(B) Commutative Property
(C) Identity Property
(C) Associative Property
(D) Inverse Property
(D) Inverse Property
8. (2 + 3) + 4 = 2 + (3 + 4)
3. (𝑝𝑞)𝑟 = 𝑝(𝑞𝑟)
(A) Associative Property
(A) Associative Property
(B) Commutative Property
(B) Commutative Property
(C) Identity Property
(C) Distributive Property
(D) Inverse Property
(D) Inverse Property
1
1 2 9. 10 ( ) = 1
4. ( ) ( ) = 1 10
2 1
(A) Associative Property
(A) Distributive Property
(B) Commutative Property
(B) Commutative Property
(C) Identity Property
(C) Identity Property
(D) Inverse Property
(D) Inverse Property
3 3 3 10. 0.25 + (−0.25) = 0
5. (𝑎+ 8) = 𝑎 + (8) (A) Associative Property
4 4 8
(A) Associative Property (B) Commutative Property
(B) Commutative Property (C) Identity Property
(C) Distributive Property (D) Inverse Property
______________________________________________________________________
Chapter Exercise:
1. Which set does 4𝜋 belong? (C) −5 > −1
(A) Rational Numbers (D) 2 ≤ −2
(B) Irrational Numbers
(C) Integers 1 3
3. |1 − 2 | = ____
(D) Whole Numbers 4 4
2
(A)
5
3
(B)
2. Which of the following inequalities is 2
1
true? (C)
4
(A) 10 > 15 3
(D)
5
(B) −10 > −11

24
1 2 3 1 3
( + )= +
4. A typical jeepney has a capacity of 22 2 3 4 3 8
passengers. From JY Mall, the (A) Inverse Property
jeepney was full of passengers. (B) Distributive Property
When it stopped in Ayala, 12 people (C) Commutative Property
got off while 5 people got on. In the (D) Associative Property
next stop in SM, 15 people got off
while 8 people got on. How many 11. Which set does 4.333 … belong?
passengers were left? (A) Rational Numbers
(A) 5 passengers (B) Irrational Numbers
(B) 7 passengers (C) Integers
(C) 10 passengers (D) Whole Numbers
(D) 8 passengers
3 8
12. (− ) ____ (− )
4 4
−101+19 352÷2 (A) =
5. ( )−( ) = ____ (B) <
2 4
(A) −3 (C) ≤
(B) 85 (D) ≥
(C) 3
(D) −85 13. Maria went to the wet market to buy
some fruits. She bought 8 apples for
P8 each, 12 pieces of mangoes
8
6. Which set does belong? which costs P14 each and 7 pieces
3
(A) Positive Integers of bananas which costs P2.50 each.
(B) Irrational Numbers If her money is P500, how much
(C) Negative Integers would be left after buying the fruits?
(D) Rational Numbers (A) P250.5
(B) P249.5
7. 4____ − 10 (C) P259.5
(A) = (D) P241.5
(B) <
14. Which of the following inequalities is
(C) ≤
true?
(D) ≥
(A) 12 ≤ 11
(B) 5.001 ≥ 5.000001
8. Evaluate 5[56 ÷ (7 ∙ 2)] − 42
(C) −5 > −3
(A) 4
(B) 8 (D) 9.01 < 9.0001
(C) 12
(D) 16 15. (52 + 3) ÷ (14 ÷ 2) + (−1 + 3 × 4) =
____
9. Which of the following inequalities is (A) 15
false? (B) 12
(A) 5.0 > 5.001 (C) −18
(B) −23 ≤ −8 (D) −20
2 2
(C) >
5 5 16. Which set does −24 belong?
(D) 1 ≥ 1 (A) Rational Numbers
(B) Irrational Numbers
10. What property is used in this (C) Fractions
equation: (D) Whole Numbers

25
17. −2 + (3 + 3) = (−2 + 3) + 3 312 −155
25. ( )( ) = ____
4 5
(A) Inverse Property
(A) 2418
(B) Distributive Property
(C) Commutative Property (B) −2418
(D) Associative Property (C) −2245
(D) −2345
18. −3____ − 5
(A) = 253−103 169÷13 30
(B) < 26. ( )( ) + ( ) = ____
10 13 6
(C) ≤ (A) 15
(D) ≥ (B) 20
(C) 25
19. |104 − 83| = ____ (D) 30
(A) 187
(B) −187 27. Which set does 15 belong?
(C) −21 (A) Negative Numbers
(D) 21 (B) Irrational Numbers
(C) Fractions
(D) Whole Numbers
20. |−105 + 100| = ____
(A) 5 12
28. Which set does belong?
2
(B) 205
(A) Zero
(C) −5
(B) Irrational Numbers
(D) −205 (C) Negative Integers
(D) Whole Numbers
21. Which set does −3 belong?
(A) Positive Integers 29. −|−56| = ____
(B) Irrational Numbers (A) 56
(C) Negative Integers
(B) −56
(D) Whole Numbers
(C) ±56
(D) None of the above
22. Which set does 10 belong?
(A) Positive Integers
(B) Irrational Numbers 30. −|32| + |−32| = ____
(C) Negative Integers (A) 64
(D) Zero (B) −32
(C) 0
23. |−11 + 12 − 8| = ____
(D) 32
(A) 7
(B) 8
31. Evaluate 10[9(2 + 4) − 6 ∙ 2]
(C) 9
(A) 300
(D) 10
(B) 400
(C) 450
(D) 420
24. |2.11 + 3.22 − 5.55| = ____
(A) 0.11
32. |5.8 − 2.3 − 1.4| = ____
(B) 0.22
(A) 1.1
(C) 0.33
(B) 1.5
(D) 0.44
(C) 2.1

26
(D) 2.5
39. (𝑥𝑦)𝑧 = 𝑥(𝑦𝑧)
33. Which expression has a value of 17?
(A) Inverse Property
(A) (20 ÷ 2) − 10 × 3
(B) Distributive Property
(B) 2[2(6 − 3)] + 5
(C) Commutative Property
(C) 27 ÷ 3 + (12 − 3) (D) Associative Property
(D) (9 × 3) − 63 ÷ 7

34. Which set does √5 belong? 40. (5 + 1)4 = 5(4) + 1(4)


(A) Rational Numbers (A) Inverse Property
(B) Irrational Numbers (B) Distributive Property
(C) Integers (C) Commutative Property
(D) Whole Numbers (D) Associative Property

35. Which of the following inequalities is


false? 41. −42____ − 36
3 5
(A) − ≤ − (A) =
2 2
(B) 2.5 ≥ −3.33 (B) >
5 4 (C) ≤
(C) ≥
10 12 (D) ≥
(D) 0 > −1.5

42. Baguio City has the following


36. −17____24 temperature for one week:
(A) = Friday 170
(B) <
(C) > Saturday 180
(D) ≥ Sunday 210
Monday 170
37. Mt. Apo, the tallest mountain in the Tuesday 180
Philippines has an elevation of 2956
meters. The next tallest mountain Wednesday 210
which is Mt. Dulang-Dulang has an Thursday 180
elevation of 18 meters less than Mt.
Apo. The third tallest mountain is Mt. What is the average temperature of
Pulag which is 16 meters less than Baguio City for the entire week?
Mt. Dulang-Dulang. What is the (Round off your answer to the
elevation of Mt. Pulag? nearest tenth)
(A) 2938 meters (A) 18.10
(B) 2922 meters (B) 18.30
(C) 2899 meters (C) 18.50
(D) 2890 meters (D) 18.60

38. 30____ − 15 43. −|0.33 − 3.33| = ____


(A) = (A) 3.33
(B) <
(B) −3.33
(C) ≤
(C) 3
(D) ≥
(D) −3

27
(D) −1332
3
44. Which set does belong?
4
(A) Rational Numbers 52. 7 + 0 = 7
(B) Irrational Numbers (A) Distributive Property
(C) Integers (B) Identity Property
(D) Whole Numbers (C) Inverse Property
45. −|54 − 12| = ____ (D) Associative Property
(A) 42
(B) 66
(C) −66 53. (2 + 5) + (−3) = 2 + (5 + (−3))
(D) −42 (A) Inverse Property
(B) Distributive Property
(C) Commutative Property
342 256 (D) Associative Property
46. (101 − 31 + 54) + − = ____
3 4 10−4
(A) 238 54. 5 − + 10 = ____
5−2
(B) 256 (A) 15
(C) 255 (B) −25
(D) 174 (C) 13
(D) −12
1
47. 2 ( ) = 1
2
(A) Inverse Property
55. (8 − 9 ÷ 3 × 3) − 15 = ____
(B) Distributive Property
(A) −8
(C) Commutative Property
(B) −16
(D) Associative Property
(C) 9
(D) 18
48. −2 + 2 = 0
(A) Inverse Property 56. 20 ÷ |10 − 23 | + (−5 ÷ 1 × 5) = ____
(B) Distributive Property (A) −15
(C) Commutative Property (B) −20
(D) Associative Property (C) −35
(D) 40
49. 10 ÷ [−2 − 2(1 + 3)] + 52 = ____
(A) -24
(B) 30 57. 5(3 + 2) = 15 + 10
(C) 36 (A) Distributive Property
(D) 24 (B) Identity Property
(C) Inverse Property
50. 10(5𝑥) = 5𝑥(10) (D) Associative Property
(A) Identity Property
(B) Distributive Property 58. A public elementary school has
(C) Commutative Property approximately 1250 students. On the
(D) Inverse Property first week of class, one-fifth of the
population was absent. On the
51. (1244 − 1568) + (−3568 + 2561) = second week, one-tenth of the
____ students were absent. And on the
1
(A) −1007 third week, of the population was
25
(B) −1324 absent. What is the average number
(C) −1331 of students who were absent for the

28
three weeks? (Round off your answer (C) 200
to the nearest whole number.) (D) 220
(A) 139 students
(B) 140 students 60. Evaluate 33 − 2{[5(11 − 3) + 5] ÷ 5}
(C) 141 students (A) 5
(D) 142 students (B) 10
(C) 15
59. Evaluate 9[(22 − 17) + 5(1 + 2)] (D) 20
(A) 150
(B) 180

3 Fundamentals of Algebra
3.1 Variables, Constants, and Exponents
Definition:
A variable is a symbol or letter of the alphabet used to represent unknown quantities.

Example 1: Let 𝑥 be an integer that lies between 6 and 10.5. What are the numbers that 𝑥
represents?
Solution: The integers between 6 and 10.5 are 7, 8, 9, and 10.

In Example 1, 𝑥 is called a variable and the set containing {7, 8, 9, and 10}, also called the
𝑟𝑒𝑝𝑙𝑎𝑐𝑒𝑚𝑒𝑛𝑡 𝑠𝑒𝑡, are the numbers that 𝑥 represents.

Definition:
A constant is a number or a symbol that contains a single value and does not contain any
variable.

Example 2: The number 8 is a constant since it has a single value. The same is true with √5
since it also has a unique value. Also, the Greek letter 𝜋 is widely known to have
a value of approximately 3.14. Thus, it is also called a constant.

Definition:
An exponent represents the number of times a number or symbol is multiplied to itself.

Example 3: Write the product of 𝑥 ∙ 𝑥 ∙ 𝑥 ∙ 𝑥 using exponents.


Solution: The variable 𝑥 is multiplied four times. In writing the product, 𝑥 will be the base
and the number of times 𝑥 is multiplied is the exponent. Thus, we have 𝑥 4 .
Laws of Exponents:

29
A. Product Rule – when multiplying powers with the same base, just add the exponents
and keep the common base.

𝒂𝒎 ∙ 𝒂𝒏 = 𝒂𝒎+𝒏 , for positive integers 𝑚 and 𝑛.


Example 4:
𝑥 2 ∙ 𝑥 3 = 𝑥 2+3 = 𝑥 5

B. Quotient Rule – when dividing powers with the same base, just subtract the exponents
and keep the common base.

𝒂𝒎
= 𝒂𝒎−𝒏 , if 𝑎 ≠ 0 and if 𝑚 > 𝑛.
𝒂𝒏
Example 5:
𝑥 10
= 𝑥 10−5 = 𝑥 5
𝑥5

C. Power Rule – when raising power to a power, just multiply the exponents and keep the
base.

(𝒂𝒎 )𝒏 = 𝒂𝒎𝒏 , for positive integers 𝑚 and 𝑛.


Example 6:
(𝑥 2 )3 = 𝑥 2 (3) = 𝑥 6

D. Product to a Power Rule – when raising product to a power, just raise each factor to
the power.

(𝒂𝒃)𝒎 = 𝒂𝒎 𝒃𝒎 , for positive integers 𝑚.


Example 7:
(𝑥 4 𝑦 2 )2 = 𝑥 4∙2 𝑦 2∙2 = 𝑥 8 𝑦 4
E. Quotient to a Power Rule – when raising a quotient to a power, raise the numerator
and the denominator to the power.

𝒂 𝒏 𝒂𝒏
( ) = , if 𝒃 ≠ 𝟎.
𝒃 𝒃𝒏
Example 8:
3
𝑥3 𝑥 3∙3 𝑥9
( 4 ) = 4∙3 = 12
𝑦 𝑦 𝑦

F. Zero Exponents – any number raised to zero power is equal to “1”.

𝒂𝟎 = 𝟏
Example 9:
(5𝑥 3 )0 = 1

G. Exponent of One - any number raised to the power of “1” is equal to itself.

𝒂𝟏 = 𝒂
Example 10:
(𝑥 2 )1 = 𝑥 2

30
H. Negative Exponent – When raising a number to negative power, get the reciprocal of
that number and change the sign of the exponent from negative to positive.
1 1
𝑎−𝑛 = or = 𝑎𝑛
𝑎𝑛 𝑎−𝑛

Example 11:
1 1
5−2 = 2 =
5 25

Example 12:
3
= 3𝑥 2
𝑥 −2

Lesson Exercise:
1. Give the replacement set of the given (D) 𝑇, where 𝑇 stands for the angle
5 sum of any triangle.
variables: 𝑥 is an integer between −
7
7
and 4. What is the product of 2 × 5 × 3 ×
3
(A) 𝑥 = {−1, 0, 1} 𝑔 × 𝑔 × 𝑔 × 𝑔 × ℎ × ℎ using
(B) 𝑥 = {0, 1, 2} exponents?
(C) 𝑥 = {−1, 0, 1, 2} (A) 15𝑔2 ℎ3
(D) 𝑥 = {0, 1, 2, 3} (B) 30𝑔3 ℎ2
(C) 10𝑔4 ℎ
(D) 30𝑔4 ℎ2
2. Which among the choices is a
constant?
(A) 𝑃, where 𝑃 stands for the 5. Use exponents to rewrite the
exchange rate of Philippine peso 𝑥∙𝑥∙𝑥∙𝑥∙𝑦∙𝑦
expression
to US dollar in one week 𝑧∙𝑧∙𝑧
𝑥𝑦
(B) 𝐻, where 𝐻 stands for the height (A)
𝑧
of the Eiffel Tower 𝑥 3𝑦2
(B)
(C) 𝑆, where 𝑆 stands for the number 𝑧2
𝑥 4𝑦2
of students in Cebu International (C) 3
𝑧
School for one decade 𝑥 3𝑦2
(D) 2
(D) 𝐶, where 𝐶 stands for the cost of 𝑧
one kilo of rice in five different
grocery stores 6. Which among the choices is TRUE?
(A) (−3)299 = 3299
3. Which among the choices is NOT a (B) (−2)1000 = −21000
constant? (C) (−1)250 = 1
(A) 𝐴, where 𝐴 stands for the age of (D) (−1)341 = 1
Dr. Jose Rizal when he died.
(B) 𝐶, where 𝐶 stands for the cost of
gasoline/litre for one month. 7. Simplify 𝑎3 𝑏 3 𝑐 2 𝑎2 𝑏 3 𝑐 4
(C) 𝑁, where 𝑁 stands for the (A) 𝑎5 𝑏 6 𝑐 6
number of days in a leap year. (B) 𝑎4 𝑏 5 𝑐 6
(C) 𝑎5 𝑏 6 𝑐 5

31
1
(D) 𝑎5 𝑏 5 𝑐 5 (A)
9
−1
8. Which among the choices is TRUE? (B)
5
𝑥 3 +𝑥 𝑥4 −1
(A) = 2 (C)
𝑥2 𝑥 9
−1
𝑎2 +𝑎4 1+𝑎2 (D)
(B) = 3 8
𝑎5 𝑎
𝑦 7 +𝑦 4 𝑦 3 +1
(C) =
𝑦4 𝑦
5
𝑧 +𝑧 5 10. Simplify (𝑥 3 )4 ∙ 𝑥 5 ∙ 𝑥 2
(D) =1
𝑧 10 (A) 𝑥 19
(B) 𝑥 20
(C) 𝑥 14
1 2
9. Simplify − ( ) (D) 𝑥 22
3

3.2 Algebraic Expression


Definition:
An algebraic expression is an expression that contains a variable, constant, or a combination
of both.
It is also composed of the following:

 Term  Each algebraic expression that is separated by a plus or minus sign


 Variable Term  A term together with a variable
 It is composed of the variable and the number together with this is
called a coefficient

Example 1:
Numerical Coefficients

2𝑥 2 + 11𝑥𝑦 − 3𝑦 + 4𝑥 + 13

Terms

In Example 1, the numerical coefficients are 2, 11, −3, 4, and 13. The terms are
2𝑥 2 , 11𝑥𝑦, −3𝑦, 4𝑥 and 13 while the variable terms are all the terms except 13.
Evaluating Algebraic Expressions
To evaluate algebraic expressions, substitute the given value of the variable to the algebraic
expression and simplify the resulting numerical expression.

Example 2: Evaluate 3𝑥 2 − 15 when 𝑥 = −2


Solution: 3(−2)2 − 15
= 3(4) − 15
= 12 − 15

32
= −3

−2𝑎2 𝑏3 𝑐 4
Example 3: Evaluate when 𝑎 = 2, 𝑏 = −1, and c = 3
3𝑏+2𝑐
−2(2)2 (−1)3 (3)4
Solution:
3(−1)+2(3)

−2(4)(−1)(81)
=
−3 + 6
8(81)
=
3
= 8(27)
= 216

Simplifying Algebraic Expressions


Like Terms – are terms in an algebraic expression that have the same variable parts. Constants
are also like terms.
Example 4:

Like terms (constant)

11𝑥 + 4𝑥 2 − 2 + 5𝑥 + 3

Like terms (with 𝑥)

To easily remember which are like terms in an algebraic expression during simplification, use
the commutative property to group them together.
From the example,
11𝑥 + 4𝑥 2 – 2 + 5𝑥 + 3 = 4𝑥 2 + 11𝑥 + 5𝑥 – 2 + 3
From then, combine the like terms using the distributive property, 𝑏𝑎 + 𝑐𝑎 = (𝑏 + 𝑐)𝑎 to add
the coefficients. Or simply, add or subtract the coefficients of like terms and the result would
retain the variable part.
4𝑥 2 + 11𝑥 + 5𝑥 – 2 + 3
= 4𝑥2 + (11 + 5)𝑥 + ( – 2 + 3)
= 4𝑥2 + 16𝑥 + 1

You may also use other properties of real numbers to easily simplify more complicated
expressions.
Example 5:
Simplify (𝑥 2 𝑦 3 + 3𝑥 3 𝑦 2 ) − 4𝑥 3 𝑦 2 using the Associative Property.
(𝑥 2 𝑦 3 + 3𝑥 3 𝑦 2 ) − 4𝑥 3 𝑦 2
= 𝑥 2 𝑦 3 + (3𝑥 3 𝑦 2 − 4𝑥 3 𝑦 2 )
= 𝑥2𝑦3 − 𝑥3𝑦2

33
Note: Evaluating algebraic expressions are easier when one simplifies the expression before
substituting the given values of the variable.
Example 6:
Simplify and evaluate 5𝑥 3 − 2𝑥 3 − 4𝑥𝑦 − 10 given 𝑥 = −1 and 𝑦 = 2
Step 1: Simplify the like terms in the given algebraic expression.
5𝑥 3 − 2𝑥 3 − 4𝑥𝑦 − 10 = 3𝑥 3 − 4𝑥𝑦 − 10
Step 2: Substitute the given values.
3(−1)3 − 4(−1)(2) − 10
= 3(−1) + 8 − 10
= −3 − 2
= −5

Lesson Exercise:
For numbers 1-5, simplify the given
algebraic expressions. 1 1
5. 2 ( ℎ − 𝑘) − 5ℎ + 10𝑘
1. 2𝑥 − 5[𝑥 − 3𝑥(2 − 𝑥)] + 3 3 2
−13
(A) −10𝑥 2 + 37𝑥 + 3 (A) ℎ+ 9𝑘
3
(B) −15𝑥 2 + 27𝑥 + 3 (B)
−15
ℎ+ 5𝑘
6
(C) 15𝑥 3 − 20𝑥 + 3
(C) 9ℎ − 10𝑘
(D) 13𝑥 2 + 37𝑥 + 3 −14
(D) ℎ + 7𝑘
3

2. 5𝑏 − 3𝑐 2 + 2𝑏𝑐 − 10𝑏 + 𝑐 2 − 3𝑏𝑐


(A) −5𝑏 − 2𝑐 2 − 𝑏𝑐 For numbers 6-10, simplify and evaluate the
(B) −3𝑏𝑐 + 2𝑐 2 − 9𝑏 given algebraic expressions.
(C) 5𝑏𝑐 − 10𝑏 + 2𝑐 2 6. 10𝑤 3 − 2𝑤 2 + (3𝑤 3 − 7𝑤 2 − 𝑤)
(D) 10𝑐 2 − 7𝑏𝑐 − 3𝑏 given 𝑤 = 2
(A) 54
(B) −72
3. 3𝑥 − 2(5𝑥 − 4) (C) 65
(A) 10𝑥 − 8 (D) 66
(B) −7𝑥 + 8
(C) 15𝑥 + 6
(D) 12𝑥 − 8 7. −10(−2𝑥 − 8) given 𝑥 = −2
(A) 30
(B) 40
4. 2𝑚 − 3𝑛 − 10𝑚𝑛 − 5(3𝑚 + 5𝑛 − (C) 45
𝑚𝑛) (D) 53
(A) 12𝑚 − 15𝑛 + 10𝑚𝑛
(B) −15𝑚 − 13𝑛 − 8𝑚𝑛
(C) −13𝑚 − 28𝑛 − 5𝑚𝑛 8. 5𝑥(3𝑦 − 5𝑦) − 3𝑦(2𝑥 − 7𝑥)
(D) −10𝑚 + 15𝑛 − 10𝑚 given 𝑥 = −1 and 𝑦 = 1

34
(A) −10
(B) −15 3 1
(C) −5 10. [5𝑞 − 10𝑞 2 (𝑞 − 3)] given 𝑞 = −
5 2
15
(D) 5 (A)
4
13
(B)
5
14
9. 8𝑎2 − 2𝑏 2 + 2𝑎2 − 3𝑎𝑏 + 4𝑏 2 (C)
3
given 𝑎 = 2 and 𝑏 = 1 16
(D)
(A) 66 3
(B) −104
(C) 36
(D) −56

3.3 Translating Verbal Expression to Algebraic Expression


The greatest challenge in Algebra is to understand and translate verbal expressions into algebraic
expression especially when given word problems. Table 3.3.1 shows mathematical symbols and
their meanings.

Symbol Meaning
+ add, plus, sum, more than, the total of,
increased by
− minus, subtract from, the difference of, take
away, less than, decreased by, the remainder
of, diminished by
⋅,(),𝑥 multiply, times, the product of, multiplied by
÷,/ divided by, the quotient of, over
√ the square root of

Table 3.3.1 Some mathematical symbols and their corresponding meanings.

A group of symbols which does not express a complete mathematical thought is called a
mathematical phrase.

English Phrases Mathematical Phrases


1. the sum of four and three 4+3
2. the square of nine (9)2
3. two times the difference between twenty 2(20 – 5)
and five

35
4. the quotient of twenty-four and twelve 24
12
5. eight is subtracted from 20 20 − 8
6. a certain number diminished by six 𝑥-6
7. twice a number increased by seven 2𝑥 + 7
8. the cube of a number 𝑥3
9. the square root of a number √𝑥
10. the square root of a number plus the √𝑥 + 𝑥
number itself
11. a certain number minus its square root 𝑥 − √𝑥
12. a certain number added to its square 𝑥 + 𝑥2
13. one-half the square of a number 1 2
𝑥
2

14. the product of the sum and difference of (𝑥 + 2)(𝑥 − 2)


a certain number and two
3
15. the difference between the cube root of √𝑥 − 𝑥 2
a number and its square

Table 3.3.2 Examples of some English phrases and their translation in the language of
mathematics.
Lesson Exercise:
Write an algebraic expression for the
phrases. 4. The square root of the product of a
number and its square
1. Thrice of 𝑥 decreased by 5
(A) 3𝑥 − 5 (A) 𝑥 2 √𝑥
(B) 5 − 3𝑥 (B) √𝑥(𝑥 2 )
(C) 5𝑥 − 3 (C) 𝑥√𝑥 2
(D) 3 − 5𝑥 (D) 𝑥 2 √𝑥 2

2. Five more than the sum of a number 5. A number plus 10 multiplied to the
and 11 quotient of 10 and thrice the number
(A) (𝑥 + 11) + 5 (A) (𝑥 + 10)10 ÷ 3𝑥
(B) (𝑥 − 11) + 5 (B) 3𝑥 ÷ 10(𝑥 + 10)
(C) 5 − (𝑥 + 11) (C) 3𝑥(𝑥 + 10) ÷ 10
(D) 5 − (𝑥 − 11) (D) (𝑥 + 10)(10 ÷ 3𝑥)

3. 𝑥 minus the quotient of 15 and 5 6. The square root of the product of 4


15
(A) − 𝑥 and 𝑥 2 is added to the cube root of
5
(B) 15(5) + 𝑥 the product of 9 and 𝑥 3 .
3
(C) 𝑥 −
15 (A) 4𝑥 2 + 9 √𝑥 3
5 3
15 (B) 4√𝑥 2 + 9√𝑥 3
(D) 𝑥 +
5

36
3
(C) √(4𝑥)2 + √(9𝑥)3 9. The sum of the square root of 𝑥 and
3
(D) √4𝑥 2 + √9𝑥 3 the quotient of 3 and 𝑥
(A) √𝑥 + 3𝑥
3
7. The square of the difference of twice (B) √𝑥 +
𝑥
of 𝑥 and 10 𝑥
(C) √𝑥 +
3
(A) √2𝑥 + 10 𝑥
(D) √𝑥 −
(B) (2𝑥 − 10)2 3
(C) (10 − 2𝑥)2
(D) (2𝑥)2 − 10 10. One-fourth of the cube of 𝑥
1
(A) 𝑥 3
4
8. The difference between the square 1
(B) √𝑥
of the product of 5 and 𝑥, and 3 4
13
(A) 3 − 5𝑥 2 (C) √𝑥
4
(B) 5𝑥 2 − 3 3 1
(C) (5𝑥)2 − 3 (D) √ 𝑥 3
4
(D) 3 − (5𝑥)2

Chapter Exercise:
1. Give the replacement set of the (C) {0, 1, 2, 3, 4 ,5 ,6}
given variables: 𝑥 is an integer (D) {−1, 0, 1, 2, 3, 4, 5, 6}
12 18
between − and inclusive.
3 6
5. The sum of 5 and thrice of 𝑥
(A) {−3, −2, −1, 0, 1, 2, }
(A) 5 − 3𝑥
(B) {−4, −3, −2, −1, 0, 1, 2}
(B) 3𝑥 − 5
(C) {−4, −3, −2, −1, 0, 1, 2, 3}
(C) 5 + 3𝑥
(D) {−3, −2, −1, 0, 1, 2, 3}
(D) 3 + 5𝑥
2. The product of the sum and
difference of 𝑥 and 8
6. (3𝑏 2 − 2𝑏 + 5) − (5𝑏 2 − 4𝑏 + 10) =
(A) (𝑥 − 8)(𝑥 − 8)
__________
(B) (𝑥 + 8)(𝑥 − 8)
(A) 3𝑏 2 − 3𝑏 + 5
(C) (𝑥 + 8)(𝑥 + 8)
(B) 5𝑏 2 − 2𝑏 − 5
(D) (𝑥 + 8 + 𝑥 − 8)
(C) 2𝑏 2 − 2𝑏 − 5
(D) −2𝑏 2 + 2𝑏 − 5
3. 10(𝑎 + 2) + 3𝑎(5 − 2𝑎) = __________ Answer: d
(A) 3𝑎2 − 10𝑎 + 15
(B) −6𝑎2 + 25𝑎 + 20
(C) −15𝑎2 − 10𝑎 + 20 7. Which among the choices is a
(D) 10𝑎2 − 15𝑎 + 15 constant?
(A) 𝐴, where 𝐴 stands for the
allowance of John every week
4. Give the replacement set of the which is ₱1,000.00.
given variables: 𝑦 is a whole number (B) 𝐵, where 𝐵 stands for the branch
between −1 and .
20 managers’ salary in five different
3
restaurants in Cebu
(A) {1, 2, 3 ,4 ,5}
(B) {1, 2, 3, 4 ,5 ,6 }

37
(C) 𝐶, where 𝐶 stands for the cost of Answer: b
a can of corned beef from 1990
to 2014.
13. Rewrite using equations: 𝑥 ∙ 𝑥 ∙ 𝑥 ∙ 𝑥 ∙
(D) 𝐷, where 𝐷 stands for the
𝑦∙𝑦∙𝑦∙𝑦∙𝑦∙𝑦∙𝑧∙𝑧
diameter of 3 different sizes of
(A) 𝑥 3 𝑦 4 𝑧 2
balloons.
(B) 4𝑥 ∙ 6𝑦 ∙ 2𝑧
(C) 𝑥 4 𝑦 6 𝑧 2
8. Ten more than the quotient of 𝑞 and
3 (D) 𝑥 4 𝑦 5 𝑧 2
𝑞
(A) 10 +
3 14. Five less than four times a number
𝑞
(B) 10 − (A) 5 − 4𝑥
3
𝑞 (B) 5𝑥 − 4
(C) 10 +
−3 (C) 4𝑥 − 5
3
(D) 10 − (D) 4𝑥 + 5
𝑞

15. −12[2𝑘 − 5(5𝑘 − 3)]


9. −3 + 15𝑥 − 10 + 10𝑥 − 5 − 3𝑥 = (A) 276𝑘 − 180
__________ (B) 110𝑘 2 − 350
(A) 53𝑥 − 10 (C) 267𝑘 − 382
(B) 20𝑥 − 16 (D) 380𝑘 − 177
(C) 22𝑥 − 18
(D) −24𝑥 + 20 16. Rewrite using exponents and
simplify:
3∙3∙5∙5∙5∙𝑘∙𝑘∙𝑚∙𝑚∙𝑚∙𝑚
10. Which among the choices is NOT a 3 ∙ 5 ∙ 5. 𝑘 ∙ 𝑚
constant? (A) 15𝑘𝑚2
(A) 𝑚, where 𝑚 is an integer from 1 3
(B) 𝑘𝑚3
5
to 10.
(C) 15𝑘𝑚3
(B) 𝑛, where 𝑛 is the area of a 5
triangle with base of 3 cm and (D) 𝑘𝑚2
3
height of 6 cm.
(C) 𝑜, where 𝑜 is the number of days 17. The cube of 𝑥 decreased by the
in a month on a leap year product of five and twice of 𝑥
(D) 𝑝, where 𝑝 is the angle measure (A) 5𝑥 2 − 𝑥 3
of a right triangle. (B) 𝑥 2 − 5𝑥 3
(C) 5𝑥 3 − 𝑥 2
11. Eight times the difference of twice a (D) 𝑥 3 − 5(2𝑥)
number and 7
(A) 8(2𝑥 − 7)
(B) 8(2𝑥) − 7 18. (2𝑘 + 3) − (5𝑘 − 4) + (2 − 2𝑘)
(C) (2𝑥 − 7) + 8 given 𝑘 = 3
(D) 8(7 − 2𝑥) (A) −6
(B) −8
(C) −9
12. 10{−3[5𝑝 − 3(10 − 𝑝)]} = __________ (D) −10
(A) 300𝑝 − 800
(B) −240𝑝 + 900
(C) −180𝑝 − 750 19. Use the rules of exponents to
(D) 200𝑝 − 760 simplify the expression:

38
2
2−5 ∙ 24 24. Solve for the expression: (5𝑥 2 −
( 3 −5 ) 2𝑥 + 5) − (10𝑥𝑦 + 5𝑥) + (𝑥𝑦 + 𝑥)
2 ∙2
given 𝑥 = 2 and 𝑦 = −2
(A) 512 (A) 36
1 (B) 48
(B)
32
(C) 49
(C) 2
(D) 52
(D) 4
25. Which of the following expressions
20. Three more than the sum of the
are equal?
square root of 𝑏 and 3 𝑥 5 +𝑥 4
(A) (√𝑏 + 3) + 3 (A) = 𝑥3 + 𝑥
𝑥2
𝑦2 1
(B) 3(√𝑏 + 3) (B) 3 5 =
𝑦 +𝑦 𝑦+𝑦 3
(C) 3(√𝑏) + 3 𝑧 5 +𝑧 3
(C) = 𝑧2 + 𝑧
(D) 3 + √𝑏 + 𝑏 𝑧3
𝑎5 𝑎3
(D) 2 3 =
𝑎 +𝑎 1+𝑎2

21. Solve for the expression: (10𝑎 + 26. A jeepney has a rate of 10kph slower
11𝑏 − 13𝑐) + (5 − 𝑎 + 𝑏 − 𝑐) − than a taxi cab. Write an algebraic
(2𝑏 + 3𝑐 − 5𝑎) given 𝑎 = 1, 𝑏 = 2, expression for the jeepney’s rate if
and 𝑐 = 3 the taxi cab has a rate of 𝑥 kph.
(A) 10 (A) 10 − 𝑥
(B) −12 (B) 𝑥 + 10
(C) 18 (C) 𝑥 − 10
(D) −16 (D) None of the above

22. Use the rules of exponents to 27. Solve for the expression:
simplify the expression: [𝑚 + 𝑛(𝑚 − 𝑛)] − (𝑚 − 𝑛) given 𝑚 =
2𝑥 6 𝑦 7 −2 and 𝑛 = 3
32𝑥 2 𝑦 4 (A) −12
1
(A) 𝑥 8 𝑦 11 (B) 18
16
1 6 7 (C) −30
(B) 𝑥 𝑦
16 (D) −24
1 4 3
(C) 𝑥 𝑦
16
1 2 4
(D) 𝑥 𝑦
16 28. Let 𝑥 be a negative number. Which
of the following is positive?
23. A worker is paid P8000 per month (A) 𝑥 37
and an additional P60 for every hour (B) −(𝑥)20
of overtime worked. Write an (C) 𝑥 15
algebraic expression for the salary of (D) 𝑥 48
the worker who works overtime for
𝑥 hours. 29. Write an algebraic expression on the
(A) 8000𝑥 + 60 sum of two consecutive odd
(B) 8000 + 60(30) integers.
(C) 8000 + 60𝑥 (A) 𝑥 + 𝑥 + 2
(D) 8000(60𝑥) (B) 𝑥 + 𝑥 + 1
(C) 2𝑥 + 3
(D) 𝑥 + 𝑥

39
30. Solve for the expression:
2𝑝 − 3𝑞 − (5 − 𝑝) + (𝑞 − 5)
given 𝑝 = 1 and 𝑞 = −1
(A) −5
(B) 8
(C) −10
(D) −12

4 Equations and Inequalities in One Variable


4.1 Equations and Properties of Equality
Definition:
Equations are mathematical statements that express equality between two algebraic
expressions. The symbol “ = ” denotes equality. Most equations contain variables and constants.
From the previous chapter, variables were defined as symbols, often letters, which stand for
numbers.
Example 1:
(a) 2 + 9 = 11
(b) 5𝑥 = 3
(c) 𝐴𝑥 + 𝐵𝑦 – 𝐶 = 0

Writing Sentences into Equations


In writing sentences into equations, it is important to look for the quantities and the relationship
between them. We follow two steps in writing sentences into equations.

Step 1: Identify your variables.


Identify and define the variables in the given statement. Then find out what the statement is
asking for.

Step 2: Translate and Combine.


The problem will be giving phrases that will aid us in the formulation of our equation. We
translate the given phrases in the problem to mathematical expressions and combine them to
form the equation.

Example 2: The sum of two numbers is 20. The second number is thrice the first number.
Solution: Step 1:
Let 𝑥 be the first number. The second number is thrice the first number thus
having 3𝑥 as our second number.
Step 2:
𝑥 + 3𝑥 = 20

40
Example 3: I am thrice as old as Eric. The difference of our ages is 10.
Solution: Step 1:
Let 𝑦 be the age of Eric. The person is thrice as old as Eric so we have 3𝑦 as his
age.
Step 2: 3𝑦 − 𝑦 = 10
Example 4: A spherical tank has a radius twice the height of Tony. What is the volume of the
tank?
Solution: Step 1:
Let ℎ be the height of Tony. The radius of the tank would be 2ℎ since it is twice the
height of Tony which is ℎ.
Step 2:
4
The formula for the volume of a sphere is 𝑉 = 𝜋𝑟 3 . Then replacing 𝑟 with 2ℎ, the
3
4
equation for the volume of the spherical tank is 𝑉 = 𝜋(2ℎ)3 .
3

Properties of Equality

Properties of Equality

For all real numbers 𝑥, 𝑥 = 𝑥.


Reflexive Property
A number is always equal to itself.
For all real number 𝑥 and 𝑦, if 𝑥 = 𝑦, then 𝑦 = 𝑥.
Symmetric Property
In equality, order does not matter.
For all real numbers 𝑥, 𝑦, and 𝑧, if 𝑥 = 𝑦 and 𝑦 = 𝑧, then 𝑥 = 𝑧.
Transitive Property If two numbers are equal to a number then they are equal to each
other.
For all real numbers 𝑥, 𝑦, and 𝑧, if 𝑥 = 𝑦, then 𝑥 + 𝑧 = 𝑦 + 𝑧.
Addition Property A number can be added to each side of the equation without
changing the solution of the equation.
For all real numbers 𝑥, 𝑦, and 𝑧, if 𝑥 = 𝑦, then 𝑥 − 𝑧 = 𝑦 − 𝑧.
Subtraction Property A number can be subtracted to each side of the equation without
changing the solution of the equation.
For all real numbers 𝑥, 𝑦, and 𝑧, if 𝑥 = 𝑦 and 𝑧 ≠ 0, then 𝑥𝑧 = 𝑦𝑧.
Multiplication
Property A nonzero can be multiplied to each side without changing the
solution of the equation.
𝑥 𝑦
For all real numbers 𝑥, 𝑦, and 𝑧, if 𝑥 = 𝑦 and 𝑧 ≠ 0, then = .
𝑧 𝑧
Division Property A nonzero can be divided to each side without changing the solution
of the equation.
Distributive Property For all real numbers 𝑥, 𝑦, and 𝑧, 𝑧(𝑥 + 𝑦) = 𝑥𝑧 + 𝑦𝑧.

41
For all real numbers 𝑥 and 𝑦, if 𝑥 = 𝑦, then 𝑦 can replace 𝑥 in any
Substitution Property
mathematical expression.

Table 4.1.1 Properties of Equality

Lesson Exercise:
From 1-5, write the appropriate equation
provided that
5. The distance travelled by a car with a
1. Five multiplied by 𝑦 is 200. speed of 20 mi/hr.
(A) 5𝑦 = 200 (A) 𝑑 = 20𝑡
(B) 5𝑥𝑦 < 200 (B) 𝑑 = 20
20
(C) 5 + 𝑦 = 200 (C) 𝑑 =
𝑡
(D) 5𝑦 < 200
(D) 𝑑 = 20𝑡 2
6. Given 2𝑎 = 6, Anna came up with
2. The perimeter of a rectangular 2𝑎 6
this statement = . What property
2 2
garden with a length of 10 meters and
was applied?
a width 𝑤.
(A) Addition Property of Equality
(A) 𝑃 = 10𝑙 + 𝑤
(B) Multiplication Property of Equality
(B) 𝑃 = 10 + 𝑤
(C) Subtraction Property of Equality
(C) 𝑃 = 2(10 + 𝑤)
(D) Division Property of Equality
(D) 𝑃 = 2(10𝑤)

7. What property is illustrated in this


3. The quotient of 𝑠 and 36 added to 1 statement? If 3𝑥 = 9, then 9 = 3𝑥.
gives 4.
𝑠 (A) Substitution Property of Equality
(A) =4 (B) Reflexive Property of Equality
36+1
𝑠 (C) Symmetric Property of Equality
(B) −1 < 4 (D) Addition Property of Equality
36

(C) 36𝑠 + 1 = 4
𝑠
(D) +1 =4 8. If 𝑧 = 7 and 5𝑧 + 6 = 0, then by
36
substitution property 5𝑧 is equal to?
(A) 12
4. The area of a circle with a radius 6.
(B) −2
(A) 𝐴 = 𝜋𝑟 3
(C) 35
(B) 𝐴 = 𝜋62
5
(D)
(C) 𝐴 = 𝜋𝑟 2 7

(D) 𝐴 = 𝜋63

42
2 10. By Reflexive Property of Equality,
9. If 8𝑡 + 7 = 𝑥 and 𝑥 = 4𝑠 + ,
3 286𝑡⁄ is equal to?
2
then 8𝑡 + 7 = 4𝑠 + . By what 𝑔ℎ
3
property of equality is the given (A) 286𝑡⁄𝑔ℎ
statement true?
(A) Addition Property of Equality (B) 𝑔ℎ

(B) Transitive Property of Equality (C) 286𝑡

(C) Reflexive Property of Equality (D) 0

(D) Distributive Property of Equality

4.2 Solving Equations of Different Forms


Equations of the form 𝒂𝒙 + 𝒃 = 𝒄
In order to solve equations of the form 𝑎𝑥 + 𝑏 = 𝑐, the goal is to manipulate the equation to get
the form 𝑣𝑎𝑟𝑖𝑎𝑏𝑙𝑒 = 𝑐𝑜𝑛𝑠𝑡𝑎𝑛𝑡. To do this, the properties of equality are used.
(a)
−3𝑎 + 40 = −2
−3𝑎 + 40 + (−40) = −2 + (−40) 𝐴𝑑𝑑 − 40 𝑡𝑜 𝑏𝑜𝑡ℎ 𝑠𝑖𝑑𝑒𝑠.
−3𝑎 −42 𝐷𝑖𝑣𝑖𝑑𝑒 𝑏𝑜𝑡ℎ 𝑠𝑖𝑑𝑒𝑠 𝑏𝑦 − 3.
=
−3 −3 𝑆𝑖𝑚𝑝𝑙𝑖𝑓𝑦.
𝑎 = 14

(b)
12 = 5𝑥 − 3
12 + 3 = 5𝑥 − 3 + 3 𝐴𝑑𝑑 3 𝑡𝑜 𝑏𝑜𝑡ℎ 𝑠𝑖𝑑𝑒𝑠.
15 = 5𝑥 𝑆𝑖𝑚𝑝𝑙𝑖𝑓𝑦.
15 5𝑥 𝐷𝑖𝑣𝑖𝑑𝑒 5 𝑡𝑜 𝑏𝑜𝑡ℎ 𝑠𝑖𝑑𝑒𝑠.
=
5 5
𝑆𝑖𝑚𝑝𝑙𝑖𝑓𝑦.
3 = 𝑥

Equations of the form 𝒂𝒙 + 𝒃 = 𝒄𝒙 + 𝒅


In order to solve equations of the form 𝑎𝑥 + 𝑏 = 𝑐𝑥 + 𝑑, the goal is to manipulate the equation to
get the form 𝑣𝑎𝑟𝑖𝑎𝑏𝑙𝑒𝑠 = 𝑐𝑜𝑛𝑠𝑡𝑎𝑛𝑡𝑠 and then simplify the constants and variables. To do this, the
properties of equality are still used.
(a)
3𝑥 + 5 = 2𝑥 − 6
3𝑥 + (−2𝑥) + 5 = 2𝑥 + (−2𝑥) − 6 𝐴𝑑𝑑 − 2𝑥 𝑡𝑜 𝑏𝑜𝑡ℎ 𝑠𝑖𝑑𝑒𝑠.
𝑥+5−5 = −6 − 5 𝑆𝑖𝑚𝑝𝑙𝑖𝑓𝑦. 𝑇ℎ𝑒𝑛, 𝑎𝑑𝑑 − 5 𝑡𝑜 𝑏𝑜𝑡ℎ 𝑠𝑖𝑑𝑒𝑠
𝑆𝑖𝑚𝑝𝑙𝑖𝑓𝑦.
𝑥 = −11

43
(b)
−10𝑟 − 2 = 4𝑟 − 7
−10𝑟 − 4𝑟 − 2 = 4𝑟 − 4𝑟 − 7
−14𝑟 − 2 + 2 = −7 + 2
−14𝑟 −5
=
−14 −14
5
𝑟 =
14

Equations containing grouping symbols


In order to solve equations containing grouping symbols, the distributive property of equality is
used.
(a)
−10(3𝑚 − 5) = 25
−30𝑚 + 50 = 25 𝐷𝑖𝑠𝑡𝑟𝑖𝑏𝑢𝑡𝑒 − 10 𝑡𝑜 𝑡ℎ𝑒 𝑏𝑖𝑛𝑜𝑚𝑖𝑎𝑙 (3𝑚 − 5).
−30𝑚 + 50 − 50 = 25 − 50 𝑆𝑖𝑚𝑝𝑙𝑖𝑓𝑦. 𝑇ℎ𝑒𝑛 𝑠𝑢𝑏𝑡𝑟𝑎𝑐𝑡 50 𝑡𝑜 𝑏𝑜𝑡ℎ 𝑠𝑖𝑑𝑒𝑠.
−30𝑚 −25 𝐷𝑖𝑣𝑖𝑑𝑒 𝑏𝑜𝑡ℎ 𝑠𝑖𝑑𝑒𝑠 𝑏𝑦 − 30.
=
−30 −30
5 𝑆𝑖𝑚𝑝𝑙𝑖𝑓𝑦.
𝑚 =
6

(b)
2𝑛 − 7(𝑛 − 1) =
3(𝑛 − 4)
2𝑛 − 7𝑛 + 7 =
3𝑛 − 12
−5𝑛 + 7 − 7 =
3𝑛 − 12 − 7
−5𝑛 − 3𝑛 =
3𝑛 − 3𝑛 − 19
−8𝑛 −19
=
−8 −8
19
𝑛 =
8
Equations containing fractions and decimals
In order to solve equations containing fractions, it is suggested to multiply both sides of the
equation by the least common denominator or LCD of these fractions.
(a)
1 2
2ℎ − = +ℎ
3 5
1 2
1 2 𝑀𝑢𝑙𝑡𝑖𝑝𝑙𝑦 𝑏𝑜𝑡ℎ 𝑠𝑖𝑑𝑒𝑠 𝑤𝑖𝑡ℎ 𝑡ℎ𝑒 𝐿𝐶𝐷 𝑜𝑓 𝑎𝑛𝑑 𝑤ℎ𝑖𝑐ℎ 𝑖𝑠 15.
(15) (2ℎ − ) = (15) ( + ℎ) 3 5
3 5 𝐷𝑖𝑠𝑡𝑟𝑖𝑏𝑢𝑡𝑒 15 𝑡𝑜 𝑒𝑎𝑐ℎ 𝑜𝑓 𝑡ℎ𝑒 𝑏𝑖𝑛𝑜𝑚𝑖𝑎𝑙𝑠.
30ℎ − 5 = 6 + 15ℎ 𝑆𝑢𝑏𝑡𝑟𝑎𝑐𝑡 15ℎ 𝑡𝑜 𝑏𝑜𝑡ℎ 𝑠𝑖𝑑𝑒𝑠.
30ℎ − 15ℎ − 5 = 6 + 15ℎ − 15ℎ 𝑆𝑖𝑚𝑝𝑙𝑖𝑓𝑦. 𝑇ℎ𝑒𝑛 𝑎𝑑𝑑 5 𝑡𝑜 𝑏𝑜𝑡ℎ 𝑠𝑖𝑑𝑒𝑠.
15ℎ − 5 + 5 = 6 + 5
15ℎ 11 𝑆𝑖𝑚𝑝𝑙𝑖𝑓𝑦. 𝑇ℎ𝑒𝑛 𝑑𝑖𝑣𝑖𝑑𝑒 𝑏𝑜𝑡ℎ 𝑠𝑖𝑑𝑒𝑠 𝑏𝑦 15.
=
15 15
11
𝑆𝑖𝑚𝑝𝑙𝑖𝑓𝑦.
ℎ =
15

(b)

44
2 4
(2𝑘 + 1) = (𝑘 − 3)
3 9
2 4
(9) [ (2𝑘 + 1)] = (9) [ (𝑘 − 3)]
3 9
6(2𝑘 + 1) = 4(𝑘 − 3)
12𝑘 + 6 = 4𝑘 − 12
12𝑘 − 4𝑘 = −12 − 6
8𝑘 = −18
8𝑘 18
= −
8 8
9
𝑘 = −
4

In order to solve equations containing decimals, it is suggested to multiply both sides by 101 = 10
if the number of decimal places is one, 102 = 100 if the number of decimal places is two, 103 =
1000if the number of decimal places is three, and so on. In general, if there are 𝑛 number of
decimal places, then multiply both sides of the equation by 10𝑛 .
(c)
0.25(5𝑥 − 2) = 3.25
100[0.25(5𝑥 − 2)] = 100(3.25) 𝑀𝑢𝑙𝑡𝑖𝑝𝑙𝑦 𝑏𝑜𝑡ℎ 𝑠𝑖𝑑𝑒𝑠 𝑤𝑖𝑡ℎ 102 = 100.
25(5𝑥−2) 325 𝑆𝑖𝑚𝑝𝑙𝑖𝑓𝑦. 𝐷𝑖𝑣𝑖𝑑𝑒 𝑏𝑜𝑡ℎ 𝑠𝑖𝑑𝑒𝑠 𝑏𝑦 25 𝑡𝑜 𝑠𝑖𝑚𝑝𝑙𝑖𝑓𝑦 𝑡ℎ𝑒 𝑓𝑟𝑎𝑐𝑡𝑖𝑜𝑛𝑠.
=
25 25 𝑆𝑖𝑚𝑝𝑙𝑖𝑓𝑦.
5𝑥 − 2 = 13 𝐴𝑑𝑑 2 𝑡𝑜 𝑏𝑜𝑡ℎ 𝑠𝑖𝑑𝑒𝑠.
5𝑥 − 2 + 2 = 13 + 2
5𝑥 15 𝑆𝑖𝑚𝑝𝑙𝑖𝑓𝑦. 𝑇ℎ𝑒𝑛 𝑑𝑖𝑣𝑖𝑑𝑒 𝑏𝑜𝑡ℎ 𝑠𝑖𝑑𝑒𝑠 𝑏𝑦 5.
=
5 5 𝑆𝑖𝑚𝑝𝑙𝑖𝑓𝑦.
𝑥 = 3

(6)
2(1.2 + 𝑥) − 3.2 = 4.15
100[2(1.2 + 𝑥) − 3.2] = 100(4.15)
200(1.2 + 𝑥) − 320 = 415
240 + 200𝑥 − 320 = 415
200𝑥 = 415 − 240 + 320
200𝑥 495
=
200 200
99
𝑥 =
40

Lesson Exercise:
Solve the following equations. 2. 5𝑠 − 2 = −17
1. −8𝑟 + 11 = 43 (A) 𝑠 = 25
(B) 𝑠 = −3
(A) 𝑟 = −4
(C) 𝑠 =5
(B) 𝑟 =7
(D) 𝑠 =3
(C) 𝑟 =4
(D) 𝑟 = −7
3. 3𝑎 − 2 + 10𝑎 − 4 = −2𝑎 − 6

45
(A) 𝑎 = 4 1
(A) 𝑦 =
12
(B) 𝑎 = −3 3
(C) 𝑎 = 0 (B) 𝑦 =
11
4
(D) 𝑎 = 1 (C) 𝑦 =
11
(D) None of the above
4. −18𝑏 + 3 + 5𝑏 + 5 = 7𝑏 − 2
𝑐−4 1 3𝑐−5
(A) 𝑏 = 2 8. ( ) = +
1 3 4 2
(B) 𝑏 = 11
2
1 (A) 𝑐 =
(C) 𝑏 = − 14
13
2
(D) 𝑏 = −2 (B) 𝑐 =
8
13
(C) 𝑐 =
14
(D) None of the above
5. 2[3𝑥 + 5(−2𝑥 − 1)] = 7𝑥 − 4
(A) 𝑥 = −
5 9. 2𝑧 − 3.2𝑧 + 4.15 = 5.25
4
3 13
(B) 𝑥 = − (A) 𝑧 =
4 54
(C) 𝑥 = −
2 (B) 𝑧 = 1.54
7 11
7 (C) 𝑧 =
(D) 𝑥 = 52
2 (D) None of the above
6. 5(3𝑧 − 1) + 2(1 − 𝑧) = 3𝑧 − 4
2
(A) 𝑧 = −
5 10. 𝑚 − 1.5(3 − 1.5𝑚) = 3
1
(B) 𝑧 = −
10 (A) 𝑚 = 3
3
(C) 𝑧 = − (B) 𝑚 = 1.5
7
(D) None of the above (C) 𝑚 = 4.5
(D) None of the above
2 4𝑦−3 2 𝑦−5
7. ( ) = ( )
5 3 15 3

4.3 Literal Equations


Definition:
Literal equations are equations that contain more than one variable. These equations can be
manipulated in order to solve for the value of one variable in terms of the other.
Examples:
(a)
Given the equation 3𝑥 + 2𝑦 = 12, solve for 𝑥. To do this, leave the variable 𝑥 on one side and
transfer all other variables and constants on the other side.
Solution:
𝑆𝑢𝑏𝑡𝑟𝑎𝑐𝑡 2𝑦 𝑡𝑜 𝑏𝑜𝑡ℎ 𝑠𝑖𝑑𝑒𝑠.
𝑆𝑖𝑚𝑝𝑙𝑖𝑓𝑦.
𝐷𝑖𝑣𝑖𝑑𝑒 𝑏𝑜𝑡ℎ 𝑠𝑖𝑑𝑒𝑠 𝑏𝑦 3.

46
𝑆𝑖𝑚𝑝𝑙𝑖𝑓𝑦.
(b)
Given the same equation in (a), solve for y.
Solution:
𝑆𝑢𝑏𝑡𝑟𝑎𝑐𝑡 3𝑥 𝑡𝑜 𝑏𝑜𝑡ℎ 𝑠𝑖𝑑𝑒𝑠.
𝑆𝑖𝑚𝑝𝑙𝑖𝑓𝑦.
𝐷𝑖𝑣𝑖𝑑𝑒 𝑏𝑜𝑡ℎ 𝑠𝑖𝑑𝑒𝑠 𝑏𝑦 2.
𝑆𝑖𝑚𝑝𝑙𝑖𝑓𝑦.
(c)
Given the equation 3𝑚𝑛 + 𝑚 = −5, solve for 𝑚.
Solution:
𝑈𝑠𝑒 𝑡ℎ𝑒 𝐷𝑖𝑠𝑡𝑖𝑏𝑢𝑡𝑖𝑣𝑒 𝑃𝑟𝑜𝑝𝑒𝑟𝑡𝑦.
𝐷𝑖𝑣𝑖𝑑𝑒 𝑏𝑜𝑡ℎ 𝑠𝑖𝑑𝑒𝑠 𝑏𝑦 (3𝑛 + 1) 𝑎𝑛𝑑 𝑠𝑖𝑚𝑝𝑙𝑖𝑓𝑦.

In the case that the variable to be solved is contained in both sides of the equation, use the
properties of equality to transfer all terms containing the variable on one side and use the
distributive property to extract the variable to be solved.
(d)
Given the equation 3𝑎𝑏 − 2 = 5𝑏 − 2𝑎, solve for 𝑎.
Solution:

𝑃𝑢𝑡 𝑎𝑙𝑙 𝑡𝑒𝑟𝑚𝑠 𝑐𝑜𝑛𝑡𝑎𝑖𝑛𝑖𝑛𝑔 𝑎 𝑜𝑛 𝑡ℎ𝑒 𝑙𝑒𝑓𝑡 𝑠𝑖𝑑𝑒.


𝑈𝑠𝑒 𝑡ℎ𝑒 𝐷𝑖𝑠𝑡𝑟𝑖𝑏𝑢𝑡𝑖𝑣𝑒 𝑃𝑟𝑜𝑝𝑒𝑟𝑡𝑦.
𝐷𝑖𝑣𝑖𝑑𝑒 𝑏𝑜𝑡ℎ 𝑠𝑖𝑑𝑒𝑠 𝑏𝑦 (3𝑏 + 2) 𝑎𝑛𝑑 𝑠𝑖𝑚𝑝𝑙𝑖𝑓𝑦.

Given the equation−2𝑟𝑠 + 5𝑠 − 2𝑟 = −10 − 5𝑟𝑠 + 2𝑠, solve for 𝑠.


Solution:

𝑃𝑢𝑡 𝑎𝑙𝑙 𝑡𝑒𝑟𝑚𝑠 𝑐𝑜𝑛𝑡𝑎𝑖𝑛𝑖𝑛𝑔 𝑠 𝑜𝑛 𝑡ℎ𝑒 𝑙𝑒𝑓𝑡 𝑠𝑖𝑑𝑒.

𝑈𝑠𝑒 𝑡ℎ𝑒 𝐷𝑖𝑠𝑡𝑟𝑖𝑏𝑢𝑡𝑖𝑣𝑒 𝑃𝑟𝑜𝑝𝑒𝑟𝑡𝑦.


𝐷𝑖𝑣𝑖𝑑𝑒 𝑏𝑜𝑡ℎ 𝑠𝑖𝑑𝑒𝑠 𝑏𝑦 3𝑟 + 3.

Lesson Exercise:

47
1. Given: 2𝑥𝑦 − 2𝑦 + 15𝑥 = −25, solve 6. Given: −5𝑦𝑧 − 18𝑧 = 13𝑧 − 11, solve
for 𝑦. for 𝑧.
5(2𝑥−3) 11
(A) 𝑦 = (A) 𝑧 =
2𝑥−2 −5𝑦−31
2(2𝑥−5) 11
(B) 𝑦 = (B) 𝑧 =
3𝑥−2 5𝑦−31
5(5+3𝑥) 11
(C) 𝑦 = (C) 𝑧 =
2(1−𝑥) 5𝑦+31
(D) None of the above (D) None of the above

2. Given: −8𝑚 + 3𝑛 − 15𝑝𝑚 = −3𝑚𝑛, 7. Given: 3𝑑 − 2𝑒 = 11𝑓 − 2𝑒𝑓 + 15,


solve for 𝑚. solve for 𝑓.
−3𝑛 2𝑑−3𝑒
(A) 𝑚 = (A) 𝑓 =
3𝑛−15𝑝−8 11−2𝑒
−2𝑛 3𝑑−2𝑒+15
(B) 𝑚 = (B) 𝑓 =
15𝑝−3𝑛+8 2𝑒−11
3𝑛 15+3𝑑−2𝑒
(C) 𝑚 = (C) 𝑓 =
15𝑝+3𝑛−8 −2𝑒+11
(D) None of the above (D) None of the above

3. Given: 𝑉 = 𝑙 ∙ 𝑤 ∙ ℎ, solve for 𝑤. 8. Given: 3𝑝 − 3𝑞 = 9𝑞 + 5𝑝, solve


𝑉𝑙
for 𝑝.
(A) 𝑤 =

𝑉ℎ
(A) 𝑝 = −6𝑞
(B) 𝑤 = (B) 𝑝 = 6𝑞
𝑙
(C) 𝑤 =
𝑉 (C) 𝑝 = 9𝑞
𝑙ℎ (D) None of the above
(D) None of the above
1
1 9. Given: 𝐴 = ℎ(𝑏1 + 𝑏2 ), solve for 𝑏1 .
4. Given: 𝑉 = 𝜋𝑟 2 ℎ, solve for ℎ. 2
3
𝐴−𝑏2 ℎ
3𝑉 (A) 𝑏1 =
(A) ℎ = 𝑏2
𝜋𝑟 2 𝑏2ℎ−𝐴
𝑉 (B) 𝑏1 =
(B) ℎ = ℎ
3𝜋𝑟 2 𝐴−𝑏2ℎ
𝑉 (C) 𝑏1 =
(C) ℎ = ℎ
−3𝜋𝑟 2
(D) None of the above (D) None of the above

5 1
5. Given: 2𝑘 + 4 − ℎ = − ℎ − 10, 4
2 2 10. Given: 𝑉 = 𝜋𝑟1 𝑟2 𝑟3 , solve for 𝑟3 .
solve for ℎ. 3
3𝑉𝑟1
(A) ℎ = 7 − 𝑘 (A) 𝑟3 =
4𝜋𝑟1 𝑟2
(B) ℎ = 7 + 𝑘 3𝑉
(B) 𝑟3 =
(C) ℎ = 𝑘 − 7 4𝜋𝑟1 𝑟2
3𝑉𝑟2
(D) None of the above (C) 𝑟3 =
4𝜋𝑟1 𝑟2
(D) None of the above

4.4 Inequalities and Properties of Inequality


Definition:
Inequalities are mathematical statements like equations but instead of equality, inequalities state
inequality between algebraic expressions. The following symbols are used to express inequality
between expressions.

48
Symbol Meaning
< “is less than”
> “is greater than”
≤ “is less than or equal to”
≥ “is greater than or equal to”

Table 4.4.1 Symbols of inequality and their corresponding meaning


Example 1:
(a) 2𝑥 < 8
(b) 𝑎 – 𝑏 ≥ 0
(c) 3𝑠 + 2𝑡 – 9 > 5

Writing Sentences into Inequalities


We have learned from the previous lesson about the steps in writing sentences into equations.
When writing sentences into inequalities, the same process is also used.

Step 1: Identify your variables.


Identify and define the variables in your world problems. Ask yourself what the problem is
looking for.

Step 2: Translate and Combine.


The problem will be giving phrases that will aid us in the formulation of our inequality. We
translate the given phrases in the problem to mathematical expressions and combine them to
form the inequality.

Example 2:
The difference of two numbers is less than 10. The second number is twice the first number.
Solution:
Step 1: Let 𝑥 be the first number. The second number is twice the first so we have 2𝑥.
Step 2: 𝒙 − 𝟐𝒙 < 10

Example 3:
The absolute value of a number is greater than or equal to 1
Solution:
Step 1: Let 𝑦 be the number.

49
Step 2: |𝒚| ≥ 𝟏

Example 4:
A rectangular prism has length equal to the size of Marc’s feet. Half of it is the width of the prism
and twice of the size of Marc’s feet is the height. What is the volume of the rectangular prism?
Solution:
1
Step 1: Let 𝑓 be the size of Marc’s feet. The width is half the size of his feet so we have 𝑓 while
2
the height is twice the size of his feet so we have 2𝑓.
Step 2: The formula for the volume of a rectangular prism is 𝑉 = 𝑙 × 𝑤 × ℎ. We then replace 𝑙
1
with 𝑓, replace 𝑤 with 𝑓 and replace ℎ with 2𝑓. Thus, the volume of the rectangular prism is 𝑽 =
2
𝟏
(𝒇) ( 𝒇) (𝟐𝒇).
𝟐

Properties of Inequality

Properties of Inequality

For all real numbers 𝑥 and 𝑦, only one of the following holds
Property of Comparison true:
𝑥 < 𝑦, 𝑥 = 𝑦, or 𝑥>𝑦
For all real numbers 𝑥, 𝑦, and 𝑧:
Transitive Property of Order (a) If 𝑥 < 𝑦 and 𝑦 < 𝑧, then 𝑥 < 𝑧;
(b) If 𝑧 > 𝑦 and 𝑦 > 𝑥, then 𝑧 > 𝑥.
For all real numbers 𝑥, 𝑦, and 𝑧:
Addition Property of Order (a) If 𝑥 < 𝑦, then 𝑥 + 𝑧 < 𝑦 + 𝑧;
(b) If 𝑥 > 𝑦, then 𝑥 + 𝑧 > 𝑦 + 𝑧.
For all real numbers 𝑥, 𝑦, and 𝑧 such that
Multiplication Property of 𝑧 is positive: 𝑧 is negative:
Order
(a) If 𝑥 < 𝑦, 𝑡ℎ𝑒𝑛 𝑥𝑧 < 𝑦𝑧; (a) If 𝑥 < 𝑦, 𝑡ℎ𝑒𝑛 𝑥𝑧 > 𝑦𝑧;
(b) If 𝑥 > 𝑦, 𝑡ℎ𝑒𝑛 𝑥𝑧 > 𝑦𝑧. (b) If 𝑥 > 𝑦, 𝑡ℎ𝑒𝑛 𝑥𝑧 < 𝑦𝑧.

Lesson Exercise:
Look for the correct inequality of the following statements:
1. Half of 𝑚 decreased by 20 is less 2. Anna’s age (𝑥) is greater than twice
than 6. Chelsea’s age (𝑦).
(A) 2𝑚 − 20 < 6 (A) 𝑥 > 𝑦
1
(B) 𝑚 − 20 = 6 (B) 𝑥 > 2𝑦
2
1 (C) 𝑦 > 2𝑥
(C) 𝑚 − 20 < 6 (D) 𝑥 ≥ 2𝑦
2
1
(D) 𝑚 − 20 > 6
2
3. The average of 3 consecutive
integers is less than 50.

50
𝑥+𝑦+𝑧
(A) = 50 7. For all real numbers 𝑎 and 𝑏, one
2
𝑥𝑦𝑧 holds true, except what?
(B) < 50
2 (A) 𝑎 = 𝑎
𝑥+𝑥+1+𝑥+2
(C) < 50 (B) 𝑎 > 𝑏
2
𝑥+𝑥+1+𝑥+2 (C) 𝑎 < 0
(D) < 50
3 (D) 𝑎 < 𝑏
4. Five times the difference of 8 and a 8. Fill in the blanks: By the transitive
number is less than or equal to 2. property of order, if 𝑥 < 1 and 1 <
(A) 5𝑥 − 8 = 7 3, then, _____.
(B) 5 − 8𝑥 < 2 (A) 1 < 𝑥
(C) 5(8 − 𝑥) ≤ 2 (B) 𝑥 < 3
(D) 5(8 − 𝑥) = 2 (C) 𝑥 > 3
(D) 𝑥 ≤ 1
5. Six more than twice the number 𝑛 is
greater than or equal to 0. 9. If 𝑥 < 10, 𝑡ℎ𝑒𝑛 𝑥 + 2 < 10 + 2. What
(A) 6𝑛 + 2 > 0 property is used in the statement?
(B) 6 + 2𝑛 ≥ 0 (A) Property of Comparison
(C) 6(2𝑛) = 0 (B) Transitive Property of Order
(D) 6 + 2 + 𝑛 < 0 (C) Addition Property of Order
(D) Multiplication Property of Order
6. What property is illustrated in this
statement? Given that 𝑎 = 3, if 𝑥 < 10. Given that 𝑎 = −2, 𝑎𝑛𝑑 𝑏 > 4, 𝑡ℎ𝑒𝑛 −
𝑦, then 3𝑥 < 3𝑦. 2𝑏 < −8. What property is used in
(A) Property of Comparison the statement?
(B) Transitive Property of Order (A) Property of Comparison
(C) Addition Property of Order (B) Transitive Property of Order
(D) Multiplication Property of Order (C) Addition Property of Order
(D) Multiplication Property of Order

4.5 Solving General Linear Inequalities


The solution set of an inequality is the set of numbers or values which when substituted to the
variable makes the inequality true.

The properties of inequality are most useful in solving inequalities and the process of solving this
is almost the same as solving equations. The only difference is that when multiplying a negative
number or expression, the inequality symbol changes from “<” to “>” or vice versa or from “≤” to
“≥” or vice versa. This is also stated in the multiplication property of order.

Example 1:
Solve: 𝑥 + 10 < 2
Solution:

𝑆𝑢𝑏𝑡𝑟𝑎𝑐𝑡 10 𝑜𝑛 𝑏𝑜𝑡ℎ 𝑠𝑖𝑑𝑒𝑠.


𝑆𝑖𝑚𝑝𝑙𝑖𝑓𝑦.

51
The graph of this inequality is:

The open circle means that−8 is not included in the solution set.

Example 2:
Solve: 3𝑥 + 6 ≥ 15
Solution:

𝑆𝑢𝑏𝑡𝑟𝑎𝑐𝑡 6 𝑜𝑛 𝑏𝑜𝑡ℎ 𝑠𝑖𝑑𝑒𝑠.


𝑆𝑖𝑚𝑝𝑙𝑖𝑓𝑦.
𝐷𝑖𝑣𝑖𝑑𝑒 𝑏𝑜𝑡ℎ 𝑠𝑖𝑑𝑒𝑠 𝑏𝑦 3.
𝑆𝑖𝑚𝑝𝑙𝑖𝑓𝑦.

The graph of this inequality is:

The shaded circle means that 3 is included in the solution set.


Example 3:
Solve: −2𝑎 + 14 < 5𝑎
Solution:

𝑆𝑢𝑏𝑡𝑟𝑎𝑐𝑡 𝑏𝑜𝑡ℎ 𝑠𝑖𝑑𝑒𝑠 𝑏𝑦 5𝑎.


𝑆𝑖𝑚𝑝𝑙𝑖𝑓𝑦.
𝐷𝑖𝑣𝑖𝑑𝑒 𝑏𝑜𝑡ℎ 𝑠𝑖𝑑𝑒𝑠 𝑏𝑦 − 7.
𝑈𝑠𝑒 𝑡ℎ𝑒 𝑀𝑢𝑙𝑡𝑖𝑝𝑙𝑖𝑐𝑎𝑡𝑖𝑜𝑛 𝑃𝑟𝑜𝑝𝑒𝑟𝑡𝑦 𝑜𝑓 𝑂𝑟𝑑𝑒𝑟 𝑡ℎ𝑒𝑛 𝑠𝑖𝑚𝑝𝑙𝑖𝑓𝑦.

Note that when the inequality was divided by −7, the inequality changed from “<” to “>”.
The graph of this inequality is:

Example 4:

52
Solve: −𝑚𝑛 + 2 ≥ 3𝑚𝑛 + 10
Solution:

𝑃𝑢𝑡 𝑎𝑙𝑙 𝑡𝑒𝑟𝑚𝑠 𝑐𝑜𝑛𝑡𝑎𝑖𝑛𝑖𝑛𝑔 𝑚𝑛 𝑜𝑛 𝑡ℎ𝑒 𝑙𝑒𝑓𝑡 𝑠𝑖𝑑𝑒.


𝑆𝑖𝑚𝑝𝑙𝑖𝑓𝑦.

𝐷𝑖𝑣𝑖𝑑𝑒 𝑏𝑜𝑡ℎ 𝑠𝑖𝑑𝑒𝑠 𝑏𝑦 − 4 𝑎𝑛𝑑 𝑢𝑠𝑒 𝑡ℎ𝑒 𝑀𝑢𝑙𝑡𝑖𝑝𝑙𝑖𝑐𝑎𝑡𝑖𝑜𝑛 𝑃𝑟𝑜𝑝𝑒𝑟𝑡𝑦.


𝑆𝑖𝑚𝑝𝑙𝑖𝑓𝑦.

The graph of this inequality is:

Tips:
In solving general linear inequalities,
i. The rules in solving equations of the form 𝑎𝑥 + 𝑏 = 𝑐 hold true.
Example:
Solve: 5𝑥 − 4 > −9

𝐴𝑑𝑑 4 𝑜𝑛 𝑏𝑜𝑡ℎ 𝑠𝑖𝑑𝑒𝑠.


𝑆𝑖𝑚𝑝𝑙𝑖𝑓𝑦.
𝐷𝑖𝑣𝑖𝑑𝑒 𝑏𝑜𝑡ℎ 𝑠𝑖𝑑𝑒𝑠 𝑏𝑦 5.
𝑆𝑖𝑚𝑝𝑙𝑖𝑓𝑦.

ii. The rules in solving equations of the form 𝑎𝑥 + 𝑏 = 𝑐𝑥 + 𝑑 also hold true.
Example:
Solve: −10𝑎 + 6 ≤ −2𝑎 − 2

𝑃𝑢𝑡 𝑎𝑙𝑙 𝑡𝑒𝑟𝑚𝑠 𝑐𝑜𝑛𝑡𝑎𝑖𝑛𝑖𝑛𝑔 𝑎 𝑜𝑛 𝑡ℎ𝑒 𝑙𝑒𝑓𝑡 𝑠𝑖𝑑𝑒.


𝑆𝑖𝑚𝑝𝑙𝑖𝑓𝑦.

𝐷𝑖𝑣𝑖𝑑𝑒 𝑏𝑜𝑡ℎ 𝑠𝑖𝑑𝑒𝑠 𝑏𝑦 − 8 𝑎𝑛𝑑 𝑢𝑠𝑒 𝑡ℎ𝑒 𝑀𝑢𝑙𝑡𝑖𝑝𝑙𝑖𝑐𝑎𝑡𝑖𝑜𝑛 𝑃𝑟𝑜𝑝𝑒𝑟𝑡𝑦.


𝑆𝑖𝑚𝑝𝑙𝑖𝑓𝑦.
iii. The rules in solving equations containing grouping symbols also hold true.
Example:
Solve: 5(𝑟 + 2) > −3(2 − 𝑟)

𝑈𝑠𝑒 𝑡ℎ𝑒 𝐷𝑖𝑠𝑡𝑟𝑖𝑏𝑢𝑡𝑖𝑣𝑒 𝑃𝑟𝑜𝑝𝑒𝑟𝑡𝑦.


𝑃𝑢𝑡 𝑎𝑙𝑙 𝑡𝑒𝑟𝑚𝑠 𝑐𝑜𝑛𝑡𝑎𝑖𝑛𝑖𝑛𝑔 𝑟 𝑜𝑛 𝑡ℎ𝑒 𝑙𝑒𝑓𝑡 𝑠𝑖𝑑𝑒.
𝑆𝑖𝑚𝑝𝑙𝑖𝑓𝑦.
𝐷𝑖𝑣𝑖𝑑𝑒 𝑏𝑜𝑡ℎ 𝑠𝑖𝑑𝑒𝑠 𝑏𝑦 2.
𝑆𝑖𝑚𝑝𝑙𝑖𝑓𝑦.

53
iv. The rules in solving equations containing fractions and decimals also hold true.
Example:
1
Solve: (𝑥 + 1) ≥ 1.5(3 − 𝑥)
2

Lesson Exercise:
Solve for the following inequalities.
1. 10𝑘 − 5 < −20𝑘 + 25 (D) None of the above
(A) 𝑘 < 1
6. 10(𝑥𝑦 − 3) ≥ 5𝑥𝑦 + 15
(B) 𝑘 < 2
(C) 𝑘 < −1 (A) 𝑥𝑦 ≤ 9
(D) None of the above (B) 𝑥𝑦 ≥ 9
(C) 𝑥𝑦 ≥ 15
2. 5𝑏 − 3 ≥ 8𝑏 + 6 (D) None of the above
(A) 𝑏 ≤ 3 3 1
(B) 𝑏 ≥ 3 7. (𝑡 + 4) < (2𝑡 − 4)
4 3
(C) 𝑏 ≤ −3 (A) 𝑡 < −48
(D) None of the above
(B) 𝑡 < −50
(C) 𝑡 < −52
3. 2𝑎 − 4 ≤ 16
(D) None of the above
(A) 𝑎 ≤ 10
3 2
(B) 𝑎 ≤ 20 8. 𝑦 − ≥ 𝑦 − 3
2 5
(C) 𝑎 ≥ 10
15
(D) None of the above (A) 𝑦 ≥
6
5
4. −3 − 5𝑧 > 17 (B) 𝑦 ≤ −
2
2
(C) 𝑦 ≥
(A) 𝑧 > 4 5
(B) 𝑧 < −4 (D) None of the above
(C) 𝑧 > −4
(D) None of the above 9. 0.45𝑝 + 3.1 < 2.3𝑝 − 1.7
−3 − 5𝑧 > 17 96
(A) 𝑝 >
−5𝑧 > 17 + 3 37
24
−5𝑧 > 20 (B) 𝑝 >
7
20 31
𝑧 < (C) 𝑝 >
−5 47
𝑧 < −4 (D) None of the above

5. −2(3𝑠 − 2) ≤ −12𝑠 + 16 10. 1.1𝑞 − 3.2(𝑞 − 3) ≥ 1.5𝑞


14
(A) 𝑠 ≤ −2 (A) 𝑞 ≤
5
(B) 𝑠 ≤ 2 18
(C) 𝑠 ≥ 2 (B) 𝑞 ≤
3

54
8 (D) None of the above
(C) 𝑞 ≤
3

Chapter Exercise:
1. Find the equation in the statement: Four multiplied to a number and then added with nine
is equal to five.
(A) 9𝑥 + 4 = 5
(B) 4𝑥 + 9 = 5
(C) 5𝑥 + 9 = 4
(D) 4𝑥 + 9 = 4

2. Solve for 𝑥: − 2𝑥 + 5 = −31


(A) 𝑥 = −24
(B) 𝑥 = −20
(C) 𝑥 = 18
(D) 𝑥 = 21

3. Given 2𝑥 − 3𝑦 = 5𝑦 − 4, solve for 𝑦.


−𝑥−2
(A) 𝑦 =
4
2+𝑥
(B) 𝑦 =
4
2−𝑥
(C) 𝑦 =
4
2
(D) 𝑦 =
𝑥−4

4. Find the inequality in the statement: Thrice a number is less than five times the sum of the
number and two.
(A) 5(2 + 𝑥) < 3𝑥
(B) 5(2 + 𝑥) > 3𝑥
(C) 3𝑥 > 5(𝑥 − 2)
(D) 3𝑥 < 5(𝑥 − 2)

5. Solve for the inequality: −3𝑚 + 2 ≤ 2𝑚 + 7


(A) 𝑚 ≤ −1
(B) 𝑚 ≥ −1
(C) 𝑚 > 1
(D) 𝑚 < 1

6. A number multiplied by three and added to the product of the number and ten is twenty.
(A) 3𝑥 + 10𝑥 = 10
(B) 3𝑥 + 20𝑥 = 10

55
(C) 3𝑥 + 10𝑥 < 20
(D) 3𝑥 + 10𝑥 > 20

7. Solve for the equation: 12𝑦 − 3 = −39


(A) 𝑦 = −2
(B) 𝑦 = −4
(C) 𝑦 = −6
(D) 𝑦 = −3

8. Given −3𝑏 + 2𝑐 = 2𝑏𝑐 − 3𝑐, solve for 𝑐.


3𝑏
(A) 𝑐 =
5−2𝑏
−3𝑏
(B) 𝑐 =
2𝑏+5
3𝑏
(C) 𝑐 =
2𝑏−5
5
(D) 𝑐 =
2𝑏−3

9. Five subtracted from ten times a number is less than or equal to three.
(A) 10𝑥 − 5 < 3
(B) 10𝑥 − 5 ≥ 3
(C) 3 ≥ 10𝑥 − 5
(D) 3 > 10𝑥 − 5

10. −8𝑘 + 3 > −2𝑘 + 15


(A) 𝑘 < 2
(B) 𝑘 > −2
(C) 𝑘 < −2
(D) 𝑘 ≤ 2

11. The difference of the cube and the square of a number is 100.
(A) 𝑥 2 − 𝑥 3 = 100
(B) 𝑥 3 − 𝑥 2 ≤ 100
(C) 𝑥 3 − 𝑥 2 ≥ 100
(D) 𝑥 3 − 𝑥 2 = 100

12. 5𝑧 − 3 = 4𝑧 + 5
(A) 𝑧 = 5
(B) 𝑧 = 8
(C) 𝑧 = 11
(D) 𝑧 = 5

13. 2𝑘 + 3𝑗 = 5 − 5𝑘𝑗 for 𝑘


3𝑗−5
(A) 𝑘 =
5𝑗−2
5+3𝑗
(B) 𝑘 =
2+5𝑗
5−3𝑗
(C) 𝑘 =
2+5𝑗
3𝑗+5
(D) 𝑘 =
2𝑗−5

56
14. A The temperature in the Arctic can be written in this inequality: twenty subtracted from
twice a number is less than one
(A) 20 − 2𝑥 < 1
(B) 2𝑥 − 20 > 1
(C) 2𝑥 − 20 ≥ 1
(D) 2𝑥 − 20 < 1

15. 𝑎𝑏 − 3 ≥ −2𝑎𝑏
(A) 𝑎 ≥ 𝑏
(B) 𝑎𝑏 ≥ 3
(C) 𝑎𝑏 ≥ −1
(D) 𝑎𝑏 ≥ 1

16. The product of seven and a number subtracted from one is negative ten.
(A) 7𝑒 − 1 = −10
(B) 7𝑒 + 1 = 10
(C) 1 − 7𝑒 = −10
(D) 1 + 7𝑒 = −10

17. −3𝑥𝑦 − 2𝑥𝑦 + 5 = 12𝑥𝑦 − 4 + 𝑥𝑦


1
(A) 𝑥𝑦 =
3
(B) 𝑥𝑦 = 2
1
(C) 𝑥𝑦 =
2
(D) 𝑥𝑦 = −2

3 3ℎ 2
18. (𝑘 − 2) + = − ℎ for ℎ
4 5 5
3
(A) ℎ = (2 + 𝑘)
4
−3
(B) ℎ = (𝑘 + 2)
4
3
(C) ℎ = (−2 − 𝑘)
4
(D) None of the above

19. The top of the hill is five times higher than the plateau. Write an inequality for the
statement.
(A) 5𝑥 > 𝑥
(B) 5𝑥 > 𝑦
(C) 5𝑦 > 𝑥
(D) 𝑥 > 5𝑥

20. −2ℎ + 3(ℎ − 5) < −5(3 − ℎ)


(A) ℎ < 0
(B) ℎ > 0
(C) ℎ > −0

57
(D) None of the above

21. Half of a number plus one third of it is equal to fifteen.


𝑥 𝑥
(A) + = 15
2 3
1 1
(B) + = 15
2 3
𝑥 1
(C) + = 15
2 3
1 𝑥
(D) + = 15
2 3

22. −10[2 + 3(4 − 𝑎)] = 5(𝑎 − 3)


(A) 𝑎 = 5
(B) 𝑎 = −5
(C) 𝑎 = 8
(D) None of the above

23. 0.25𝑚𝑛 + 2𝑚 = 3.5𝑛 for 𝑛


−4𝑚
(A) 𝑛 =
𝑚+14
4𝑚
(B) 𝑛 =
14−𝑚
−4𝑚
(C) 𝑛 =
14−𝑚
(D) None of the above

24. Jed’s pocket money for a trip is five times greater than twice the pocket money of his
sister. Write an inequality for the statement.
(A) 2(5𝑥) > 2
(B) 2(5𝑥) > 2𝑥
(C) 5(2𝑥) > 5𝑥
(D) 5(2𝑥) > 𝑥

25. −10(𝑝 − 4) + 5(2𝑝 + 1) ≤ 20 + 𝑝


(A) 𝑝 ≥ 25
(B) 𝑝 ≤ 25
(C) −25 ≤ 𝑝
(D) None of the above

26. What property is used in this statement: If 5𝑥 = 2 then 2 = 5𝑥.


(A) Substitution Property of Equality
(B) Reflexive Property of Equality
(C) Symmetric Property of Equality
(D) Addition Property of Equality

27. 2(3𝑏 − 7) + 5(𝑏 − 4) = −8(3 − 𝑏)


2
(A) 𝑏 =
3
4
(B) 𝑏 =
5
10
(C) 𝑏 =
4
(D) None of the above

28. 2𝑝𝑞 − 3𝑝 = 5𝑞 − 10 for 𝑞

58
3𝑝+10
(A) 𝑞 =
2𝑝+5
3𝑝+10
(B) 𝑞 =
2𝑝−5
3𝑝−10
(C) 𝑞 =
2𝑝+5
(D) None of the above

29. What property is used in the statement: If 𝑚 < 3, then 𝑚 − 2 < 3 − 2.


(A) Property of Comparison
(B) Transitive Property of Order
(C) Addition Property of Order
(D) Multiplication Property of Order

30. −7𝑠 − 40 > −11𝑠


(A) 𝑠 > 10
(B) 𝑠 > 12
(C) 𝑠 < −12
(D) None of the above

31. What property is used in this statement: If 3𝑎 = −5, 𝑡ℎ𝑒𝑛 𝑎 = −5/3.


(A) Addition Property of Equality
(B) Multiplication Property of Equality
(C) Subtraction Property of Equality
(D) Division Property of Equality

5 Word Problems Involving One Variable


5.1 Translating Sentences into Equations
Applications of Algebra in solving real-life problems are usually presented in words or in text
form. We need to translate English phrases and sentences into mathematical phrases and
sentences.
Lesson Proper:

A mathematical sentence is a group of symbols that expresses a complete mathematical


thought. Whether numerical or algebraic, it expresses either an equality or inequality
between numbers. They are either equations or inequalities depending on the symbol, =
used for equations and the symbols < , > , ≤ , and ≥ for inequalities.

English Sentences Mathematical


Sentences
1. Four plus thirteen equals seventeen. 4 + 13 = 17
2. The square of three is not equal to one 32 ≠ (2) (5)
times eight.

59
3. The average of two, three, and four is 2+3+4
< 10
lesser than ten. 3
4. Eight increased by seven is greater than 8 + 7 > 12
twelve.
5. Nine minus eight equals one. 9−8= 1

Table 5.1.3 English Sentences to Mathematical Sentences


Notice that all mathematical statements above are true. Now, consider the English
translations of the following mathematical sentences:

Mathematical Sentences English Sentences


1. 2(4 + 1) = 10 Twice the sum of 4 and 1 is 10.
9+3 The sum of 9 and 3 divided by 4 is greater
2. >2
4
than 2.
40+20 The sum of 40 and 20 divided by the
3. =3
40−20
difference of 40 and 20 is equal to 3.
4. 122 − 44 = 100 The square of 12 decreased by 44 equals
100.
5. √36 < 8 The square root of 36 is less than 8.

Table 5.1.4 English translations of the following mathematical sentences.


We use variables or letters in the alphabet to represent definite but unknown numbers. For
example, “a certain number diminished by 5” may be written as “𝑥 − 5”, where 𝑥 represents
the variable.
Lesson Exercise:
Translate the following English phrases and sentences into mathematical phrases and
sentences:

1. The product of 18 and z


18
A.
𝑧
B. √18𝑧
C. 18 ∙ 𝑧
D. 18 < 𝑧
Correct answer: C
Explanation:

60
Symbol Meaning
∙ the product of

(E) The average of p, q, and r


𝑝+𝑞+𝑟
a.
3
b. 𝑝 + 𝑞 + 𝑟
𝑝
c.
𝑞+𝑟
d. 𝑟√𝑝𝑞
Correct answer: A
Solution or Explanation:
Average is the sum of the item values divided by the number of items.
(F) The area A of a square whose side is b cm long
a. 𝐴 = 𝑏 2
b. 𝐵 = 𝑎2
c. 2 = 𝑎𝑏
d. 𝐴 = 2𝑏
Correct answer: A
Solution or Explanation:
The area of the square is obtained by getting the product of its length and width.
Since the length and width of the square are equal, its area is given by: 𝑨𝒔𝒒𝒖𝒂𝒓𝒆 =
(𝒔)(𝒔) = 𝒔𝟐 , where 𝑠 is the length of the side of the square or 𝑠 = 𝑙 = 𝑤.
(G) The quotient of a certain number and 5 increased by 16 is equal to 34.
𝑥
a. +5 =𝑥
16
34
b. + 16 = 5
𝑥
16
c. +5 =𝑥
34
𝑥
d. + 16 = 34
5
Correct answer: D
Solution or Explanation:
Symbol Meaning
+ increased by
= is equal to
/ the quotient
of

(H) The difference between 24 and thrice a given number is 10.


a. 24 − 10 = 3𝑥
b. 24 − 3𝑥 = 10
c. 24 − 3𝑥 = 10
d. 10 − 3𝑥 = 24

61
Correct answer: B
Solution:
Symbol Meaning
− the difference
of
= is equal to, is

(I) Twice a certain number increase by 31 gives 55.


a. 3𝑥 + 55 = 31
b. 2𝑥 + 55 = 31
c. 2𝑥 + 31 = 55
d. 3𝑥 + 31 = 55
Correct answer: C
Solution:
Symbol Meaning
+ increased by
= gives

(J) Five less than four times a number m


a. 5𝑚 − 4
b. 5 − 4𝑚
c. (5 − 4)𝑚
d. 4 − 5𝑚
Correct answer: C
Solution:
Symbol Meaning
− less than
∙ times
(K) The perimeter P of a square of side s
a. 𝑃 = 4𝑠
b. 𝑃 = 𝑠4
c. 𝑆 = 4𝑝
d. 𝑆 = 4𝑝
Correct answer: A
Solution:
Perimeter of a square: 𝑷 = 𝟒𝒔, where 𝑠 is the length of the side.
(L) Three times the sum of 5 and a certain number equals 27.
a. 3𝑥 + 5 = 27
b. 3 + 5𝑥 = 27

62
c. 3 + 5 = 27𝑥
d. 3(5 + 𝑥) = 27
Correct answer: D
Solution:
Symbol Meaning
+ the sum of
∙ times
= equals

(M) One-half the sum of the numbers m and a


𝑚 1
a. +
𝑎 2
𝑚+𝑎
b.
2
2
c.
𝑚+𝑎
𝑚
d.
2+𝑎

Correct answer: B
Solution:
Symbol Meaning
+ the sum of

(N) Arvin’s age 5 years ago if he is 𝑥 years old now


a. 𝑥 − 5
b. 5 − 𝑥
c. 5𝑥
𝑥
d. √5
Correct answer: A
Explanation:
Subtract 5 from his present age, 𝑥, because of “5 years ago”
(O) Three less than the product of 𝑎 and 𝑏
a. 3 − 𝑎𝑏
b. 3𝑎𝑏
2
c. √3𝑎𝑏
d. 3𝑎𝑏
Correct answer: A
Explanation:
Symbol Meaning
− less than
∙ the product of

63
(P) Five less than four times a number 𝑚
4
a. 5 −
𝑚
b. 5 + 4𝑚
c. 5 − 4𝑚
d. 5 ∙ 4𝑚
Correct answer: C
Explanation:
Symbol Meaning
− less than
∙ times
(Q) The perimeter 𝑃 of a rectangle (𝑥 + 3)cm long and (𝑦 + 4)cm wide
a. 𝑃 = (𝑥 + 3)2 + (𝑦 + 4)2
b. 𝑃 = (𝑥 + 3)2 − (𝑦 + 4)2
c. 𝑃 = (𝑥 + 3)2 ∙ (𝑦 + 4)2
d. 𝑃 = 2(𝑥 + 3) + 2(𝑦 + 4)
Correct answer: D
Explanation:
The perimeter 𝑃 of a rectangle is equal to the sum of twice its length 𝑙 and twice
its width 𝑤.
(R) The difference of (2𝑎 + 6) and 7𝑐
a. 7𝑐(2𝑎 + 6)
b. (2𝑎 + 6) − 7𝑐
c. 7𝑐 √2𝑎 + 6
2𝑎+6
d.
7𝑐

Correct answer: B
Explanation:
Symbol Meaning
− the difference
of
Translate each of the following mathematical sentences to English sentences.
(S) 43 = 12
a. The cube of four is equal to twelve.
b. The square of four is equal to twelve.
c. The cube of four is not equal to twelve.
d. The cube of four is greater than twelve.
Correct answer: A

Explanation:

64
Symbol Meaning
= is equal to
𝑥3 cube

(T) 11 + 7 > 11 − 7
a. Eleven decreased by seven is greater than eleven increased by seven.
b. Eleven increased by seven is less than eleven decreased by seven.
c. Eleven increased by seven is greater than or equal to eleven decreased by
seven.
d. Eleven increased by seven is greater than eleven decreased by seven.
Correct answer: D

Explanation:
Symbol Meaning
> is greater
than
+ increased by
− decreased by

12
(U) =1
12
a. Twelve plus twelve is equal to one.
b. Twelve diminished by itself is equal to one.
c. Twelve divided by itself is equal to one.
d. The square root of twelve is equal to one.
Correct answer: C
Explanation:
Symbol Meaning
/ divided by
= is equal to
(V) 2 ∙ 𝑚
a. the product of 2 and 𝑚
b. the quotient of 2 and 𝑚
c. the sum of 2 and 𝑚
d. the difference of 2 and 𝑚
Correct answer: A
Explanation:
Symbol Meaning
∙ the product of

65
(W)2 + 4 ≠ 4 + 3
a. The sum of two and four is not equal to the sum of four and three.
b. The difference of two and four is equal to the difference of four and three.
c. The sum of two and four is equal to the sum of four and three.
d. The sum of two and four is less than or equal to the sum of four and three.
Correct answer: A
Explanation:
Symbol Meaning
+ the sum of
≠ is not equal
to

5.2 Integer Problems


Lesson Proper
Any word problems that ask for finding the integers are integer word problems.
If the following statements are present in the word problems:
a. What are the values of the integers?
b. Find the integers.
c. Find the consecutive integers.

Consecutive integers are integers that follow in sequence, each number being 1 more than the
previous number, represented by n, n +1, n + 2, n + 3, ..., where n is any integer.
For example: 23, 24, 25, …
There are two tuypes of consecutive integers.
(a) Consecutive Even Integers - starts with an even number and each number in the
sequence is 2 more than the previous number then we will get consecutive even
integers.
For example: 16,18, 20, …
(b) Consecutive odd integers - starts with an odd number and each number in the sequence
is 2 more than the previous number then we will get consecutive odd integers.
For example: 33, 35, 37, …
In solving integer word problems identify the variable and express the other integers in terms of
that variable. Specifically, when solving consecutive odd or even integers, the expressions
always differ by 2 and are in this form:
𝑛, 𝑛 + 2, 𝑛 + 4, … , 𝑤ℎ𝑒𝑟𝑒 𝑛 ∈ 𝑍

Lesson Exercise:
Solve the following word problems:
1. The sum of the least and greatest of 3 consecutive integers is 60. What are the values of the
3 integers?

66
Solution:
Step 1: Assign variables :
Let x = least integer
x + 1 = middle integer
x + 2 = greatest integer
Translate sentence into an equation.
Sentence: The sum of the least and greatest is 60.
Rewrite sentence:
x + (x + 2) = 60
Step 2: Solve the equation
Combine like terms
2x + 2 = 60
Isolate variable x
2x =58

Step 3: Check your answer


29 + 29 + 2 = 60
The question wants all the 3 consecutive numbers: 29, 30 and 31
Answer: The 3 consecutive numbers are 29, 30 and 31.

2. The lengths of the sides of a triangle are consecutive odd numbers. What is the length of the
longest side if the perimeter is 45?
Solution:
Step 1: Being consecutive odd numbers we need to add 2 to the previous number.
Assign variables :
Let x = length of shortest side
x + 2 = length of medium side
x + 4 = length of longest side
Sketch the figure

67
Step 2: Write out the formula for perimeter of triangle.
P = sum of the three sides
Step 3: Plug in the values from the question and from the sketch.
45 = x + x + 2 + x + 4
Combine like terms
45 = 3x + 6
Isolate variable x
3x = 45 – 6
3x = 39
x =13
Step 3: Check your answer
13 + 13 + 2 + 13 + 4 = 45
Be careful! The question requires the length of the longest side.
The length of longest = 13 + 4 =17
Answer: The length of longest side is 17

3. John has a board that is 5 feet long. He plans to use it to make 4 shelves whose lengths are
to be a series of consecutive even numbers. How long should each shelf be in inches?
Solution:
Step 1: Being consecutive even numbers we need to add 2 to the previous number.
Assign variables :
Let x = length of first shelf
x + 2 = length of second shelf
x + 4 = length of third shelf
x + 6 = length of fourth shelf
Step 2: Convert 5 feet to inches
5 × 12 = 60
Step 3: Sum of the 4 shelves is 60
x + x + 2 + x + 4 + x + 6 = 60
Combine like terms
4x + 12 = 60
Isolate variable x
4x = 60 – 12
4x = 48
x = 12

68
Step 3: Check your answer
12 + 12 + 2 + 12 + 4 + 12 + 6 = 60
The lengths of the shelves should be 12, 14, 16 and 18.
Answer: The lengths of the shelves in inches should be 12, 14, 16 and 18.

4: The sum of two consecutive integers is -29. Find the two integers.
Solution:
Let x = first integer
Let x + 1 = second integer.

Equation setup:
x + (x + 1) = -29
x = -15
Answer: -15 and -14

5. The sum of three consecutive integers is 15. Find the three integers.
Solution:
Let x = first integer
Let x + 1 = second integer.
Let x + 2 = third integer.

Equation setup:
x + (x + 1) + (x + 2) = 15
x=4
Answer: 4, 5 and 6
6. The sum of two consecutive integers is 55. Find the two integers.
Solution:
Let x = first integer
Let x + 1 = second integer.

Equation setup:
x + (x + 1) = 55
x = 27
Answer: 27 and 28

7. The sum of three consecutive integers is -21. Find the smallest integer.
Solution:
Let x = first integer
Let x + 1 = second integer.
Let x + 2 = third integer.

69
Equation setup:
x + (x + 1) + (x + 2) = -21
x = -8
The smallest integer is x = -8

8. The sum of two odd consecutive integers is -16. Find the two integers.
Solution:
Let x = first integer
Let x + 2 = second odd integer.

Equation setup:
x + (x + 2) = -16
x = -9,
x + 2 = -9 + 2 = -7, second odd integer.
Answer: -9 and -7

9. The sum of three even consecutive integers is -36. Find the three integers.
Solution:
Let x = first integer
Let x + 2 = second even integer.
Let x + 4 = third even integer.

Equation setup:
x + (x + 2) + (x + 4) = -36
x = -14
Answer: -14, -16 and -18

10. The sum of three odd consecutive integers is 99. Find the largest integer.
Solution:
Let x = first integer
Let x + 2 = second odd integer.
Let x + 4 = third odd integer.

Equation setup:
x + (x + 2) + (x + 4) = 99
x = 31
The largest integer is x + 4 = 35

70
CHAPTER TEST:
1. The sum of an integer and three more than three times the next consecutive integer is
30. Find the integers.
a) 5 and 6
b) 6 and 7
c) 7 and 8
d) 8 and 9
SOLUTION:
Let x = first integer
X + 1 = second integer
X + 3 + 3(x + 1) = 30
X + 3 + 3x + 3 = 30
4x + 6 = 30
4x = 24
X=6
X+1=7
2. The sum of two consecutive integers is 99. Find the value of the smaller integer.
a) 54, 55
b) 44, 45
c) 40, 59
d) 49, 50
SOLUTION:
Let x = first number, x + 1 = second number;
X + x + 1 = 99
2x + 1 = 99
2x + 1 – 1 = 99 – 1
2x = 98
X = 49
X + 1 = 50
Answer: 49, 50

71
3. The sum of two consecutive odd integers is 40. What are the integers?
a) 17, 19
b) 11, 29
c) 17, 23
d) 19, 21
SOLUTION:
Let x = first number
X + 2 = second number
X + (x + 2) = 40
2x + 2 = 40
2x = 38
X = 19
X + 2 = 21
4. The sum of three consecutive even integers is 30. Find the integers.
a) 12, 14, 16
b) 10, 12, 14
c) 6, 10, 14
d) 8, 10, 12
SOLUTION:
Let x = first number
X + 2 = second number
X + 4 = 3rd number
X + (x + 2) + (x + 4) = 30
3x + 6 = 30
3x = 24
X=8
X + 2 = 10
X + 4 = 12
Answer: 8, 10, 12

5. The square of an integer minus three times the integer is 18. Find all possible solutions.
a) 3, 6
b) 3, -6
c) -3, -6
d) -3, 6

72
SOLUTION:
Let x = an integer
Equation setup:
x2 - 3x = 18
x 2 - 3x - 18 = 0
Solve to get x = 6,-3
6. The product of two consecutive negative even integers is 24. Find the numbers.
a) -8, -3
b) -8, 3
c) -4, 6
d) -6, -4
Solution: Let x = first number, x + 2 = second number.
x(x+2) = 24
x2 + 2x = 24
x2 + 2x – 24 = 0, the given expression can be factored to
(x + 6) (x – 4) = 0
X + 6 = 0 or x – 4 = 0
X = -6 and x = 4
Since we were asked for negative integers, we take x = -6.
And x + 2 = -6 + 2 = -4
The answer is -6 and -4.

7. Twice the larger of two numbers is three more than five times the smaller, and the
sum of four times the larger and three times the smaller is 71. What are the numbers?
a) 6, 12
b) 7, 13
c) 8, 15
d) 5, 14
Solution:
The best first step is to start labelling:
the larger number: x
the smaller number: y
twice the larger: 2x
three more than five times the smaller: 5y + 3
relationship between ("is"): 2x = 5y + 3

73
four times the larger: 4x
three times the smaller: 3y
relationship between ("sum of"): 4x + 3y = 71
Now I have two equations in two variables:
2x = 5y + 3
4x + 3y = 71
I will solve, say, the first equation for x:
x = (5/2)y + (3/2)
Then I'll plug the right-hand side of this into the second equation in place of the "x":
4[ (5/2)y + (3/2) ] + 3y = 71
10y + 6 + 3y = 71
13y + 6 = 71
13y = 65
y = 65/13 = 5
Now that I have the value for y, I can solve for x:
x = (5/2)y + (3/2)
x = (5/2)(5) + (3/2)
x = (25/2) + (3/2)
x = 28/2 = 14
As always, I need to remember to answer the question that was actually asked. The solution
here is not "x = 14", but is the following sentence:
The larger number is 14, and the smaller number is 5.
8.) When 4 times a number is increased by 40, the answer is the same as when 100 is
decreased by the number. Find the number.
A) 10
b) 11
c) 12
d) 13
Solution:
Let x be the number.
“4 times a number is increased by 40” is equivalent to 4x + 40
“100 is decreased by the number” to algebraic notation, 100 – x
“the same” means 4x + 40 = 100 – x
4x + x = 100 – 40
5x = 60
X = 12
Therefore, 12 is the number.

74
9). One number is twice another number. If the 12 is subtracted from the larger number the
result is 7 more than the smaller number. Find the numbers.
a) 17, 34
b) 18, 36
c) 19, 38
d) 20, 40
Solution: Let x be the first number and 2x be the second number.
2x – 12 = x + 7
2x – x = 7 + 12
X = 19, 2x = 38

10) The smaller of two numbers is 18 less than the larger number. If 5 times the larger
number exceeds 4 times the smaller number by 100. Find the numbers.
a) 48, 30
b) 38, 20
c) 28, 10
d) 36, 18
Solution:
Let x = larger number
Since smaller number is 18 less than the larger number, then
x – 18 is the smaller number
5x – 4(x – 18) = 100
5x – 4x + 72 = 100
X = 100 – 72
X = 28
X – 18 = 28 – 18 = 10
The numbers are 28 and 10.

5.3 Age Problems


Lesson Proper
Any word problems that ask or discuss about age of people is age problems.
There are three different ways in solving age problems.
(a) If the age problem involves a single person, then it is similar to solving an Integer Problem.
(b) When the problem involves ages in the past, present, and future, it is recommended to
make a table to organize the given data in the problem.
(c) If the age problem involves the ages of two or more people, then using a table is
recommended. A table will help you in organizing the information and in writing down the
equation.

75
Example:
1) Eight years from now Peter will be 26 years old. The current sum of the ages of
Peter and Paul is 53. How old is Paul right now.
2) Five years ago, John’s age was half of the age he will be in 8 years. How old is he
now?
3) John is twice as old as his friend Peter. Peter is 5 years older than Alice. In 5 years,
John will be three times as old as Alice. How old is Peter now?

Lesson Exercise:
Solve the following word problems:
1. Five years ago, John’s age was half of the age he will be in 8 years. How old is he now?
Solution:
Step 1: Let x be John’s age now. Look at the question and put the relevant expressions above
it.

Step 2: Write out the equation.

Isolate variable x

Answer: John is now 18 years old.

2. Mary is 44 years old. The sum of the ages of Mary and John is 52. How old is John?
SOLUTION: Let x be John’s age:
“sum of the ages of Mary and John” is equivalent to 44 + x = 52
X = 52 – 44 = 8
John’s age is 8.

76
3. 8 years from now Peter will be 26 years old. The current sum of the ages of Peter
and Paul is 53. How old is Paul right now?
SOLUTION: Let x be Paul’s age.
“8 years from now Peter will be 26 years old” means Peter’s current age is 26 – 8 = 18
Now 8 years from now
Peter’s age 18 26
Paul’s age x

X + 18 = 53
X = 35
Therefore, Paul’s age is 35.

4. 6 years from now John will be 15 years old. In 9 years, the sum of the ages of John and
Pearl will be 38. How old is Pearl right now?
SOLUTION: Let x be Pearl’s age.
Now 6 years from now In 9 years
Pearl’s age x x+6 x+9
Paul’s age 9 15 18

x + 9) + 18 = 38
X + 27 = 38
X = 38 – 27 = 11
Pearl’s age is 11.

5. 15 years from now Peter will be twice as old as Miguel. The current sum of the ages of Peter
and Miguel is 18. How old is Miguel right now?
Solution

Let x = Miguel’s age


y = Peter’s age
Now In 15 years
Miguel’s age X X + 15
Peter’s age Y Y + 15

‘15 years from now Peter will be twice as old as Miguel’ to algebraice expression (y + 15) = 2(x+
15)

77
‘the current sum of the ages of Peter and Miguel is 18’ to algebraic expression y + x = 18.
We have 2 equations:
y + x = 18
y + 15 = 2(x+15) => y + 15 = 2x + 30
we solve this by substitution, y = 18 – x
y + 15 = 2x + 30
(18 – x ) + 15 = 2x + 30
33 – x = 2x + 30
-x – 2x = 30 – 33
-3x = -3 divide by -3 both sides
X=1
Miguel’s age = 1

6. 9 years from now Sharon will be 18 years old. In 6 years, the sum of the ages of Sharon and
Paul will be 71. How old is Paul right now?
SOLUTION:
Let x be Paul’s current age
Now In 6 years In 9 years
Paul’s age x X+6
Sharon’s age 18 – 9 = 9 9+6 = 15 18

“In 6 years, the sum of the ages of Sharon and Paul will be 71” means (x + 6) + 15 = 71
X + 21 = 71
X = 50
Paul’s age is 50.

7. In three more years, Miguel's grandfather will be six times as old as Miguel was last year.
When Miguel's present age is added to his grandfather's present age, the total is 68. How old is
each one now?
Let x = grandfather’s present age
y = miguel’s present age
Now In 3 more years
Grandfather’s age X X+3
Miguel’s age Y Y+3

78
“In three more years, Miguel's grandfather will be six times as old as Miguel was last year”
means
(x + 3) = 6(y - 1)
x + 3 = 6y – 6
x – 6y = -6 – 3
x – 6y = -9
“Miguel's present age is added to his grandfather's present age, the total is 68” means
x + y = 68
we have 2 equations:
x – 6y = -9
x + y = 68
We solve this by eliminating x:
(x – 6y) – (x + y) = -9 – 68
X – 6y –x – y = - 77
-7y = -77
Y = 11
X + y = 68
X = 68 – y = 68 – 11 = 57
Hence, Miguel’s age is 11 and Grandfather’s age is 57.

8. One-half of Heather's age two years from now plus one-third of her age three years ago is
twenty years. How old is she now?
This problem refers to Heather's age two years in the future and three years in the past.
So I'll pick a variable and label everything clearly:
age now: H
age two years from now: H + 2
age three years ago: H – 3
Now I need certain fractions of these ages:
one-half of age two years from now: ( 1/2 )(H + 2) = H/2 + 1
one-third of age three years ago: ( 1/3 )(H – 3) = H/3 – 1
The sum of these two numbers is twenty, so I'll add them and set this equal to 20:
H
/2 + 1 + H/3 – 1 = 20
H
/2 + H/3 = 20
3H + 2H = 120
5H = 120
H = 24
Heather is 24 years old.

79
9. Bob is currently twice as old as Steve. Twenty years ago Bob was 6 times as old as Steve.
What are their current ages?
Let x be Steve’s age
2x be Bob’s age
(2x – 20) = 6(x – 20)
2x – 20 = 6x – 120
2x -6x = -120 + 20
-4x = -100
X = 25
2x = 2(25) = 50
Hence, Steve’s age is 25 and Bob’s age is 50.

10. Pol is 10 years younger than Greg. In 7 years, he will be 10 years more than one half as old
as Greg. Find their age at present.
Let x = Pol’s age
X + 10 = Greg’s age
In 7 years,
(X + 7) – 10 = ½ (x + 10 + 7)
X - 3= ½ x + 17/2, to avoid fractions we multiply both sides by 2
2(x – 3) = x + 17
2x – 6 = x + 17
2x – x = 17 + 6
X = 23
X + 10 = 23 + 10 = 33
Hence, Pol’s age is 23 and Greg’s age 33.

CHAPTER TEST:
1. Mike is 18 years old. The sum of the ages of Mike and Peter is 39.
How old is Peter?
e) 18
f) 19
g) 20
h) 21
Solution:
Let x = Peter’s age
X + 18 = 39

80
X = 39 - 18
X = 21
Therefore, Peter’s age is 21

2. 9 years from now Maria will be 17 years old. The current sum of the ages of Maria and
Jose is 41.
How old is Jose right now?
e) 23
f) 32
g) 33
h) 34

SOLUTION:
Let x = Jose’s age;
17 – 9 = 8 Maria’s age
X + 8 = 41
X = 41 -8 = 33
Answer: Jose’s age is 33
3. 5 years from now Kate will be 48 years old. In 10 years, the sum of the ages of Kate and
Sharon will be 90.
How old is Sharon right now?
a. 25
b. 26
c. 27
d. 28
SOLUTION:
Let x = Sharon’s age
Current’s age 5years from now 10 years
Sharon x X+5 X + 10
Kate 48 – 5 = 43 48 43 + 10 = 53

(x + 10) + 53 = 90, In 10 years, the sum of the ages of Kate and Sharon will be 90.
X + 63 = 90
X = 90 – 63 = 27
Answer: Sharon’s age is 27 years old.

81
4. 6 years from now Peter will be 43 years old.
In 13 years, the sum of the ages of Peter and Steven will be 87.
How old is Steven right now?
a) 22
b) 23
c) 24
d)25

Solution: Let x be Steven’s age


Current’s age 6 years from now 13 years from now
Peter 43-6 = 37 43 37 + 13 = 50
Steven X X+6 X + 13

(x + 13) + 50 = 87
X + 63 = 87
X = 87 – 63 = 24
Answer: Steven’s age is 24.

5.) 12 years from now Tiffany will be twice as old as Maria.


The current sum of the ages of Tiffany and Maria is 57.
How old is Maria right now?
a) 12
b) 13
c) 14
d) 15
Solution: Let x be Maria’s age.
Current’s age In 12 years
Tiffany 2x + 24 – 12 = 2x + 12 2(x+12) = 2x + 24
Maria X X + 12

2x + 12 + x = 57
3x = 57 – 12
3x = 45
X = 15

82
6.) John is twice as old as his friend Peter. Peter is 5 years older than Alice. In 5 years, John will
be three times as old as Alice. How old is Peter now?
a) 5
b) 10
c) 15
d) 20
Solution:

Step 1: Set up a table.


age now age in 5 yrs
John
Peter
Alice
Step 2: Fill in the table with information given in the question.
John is twice as old as his friend Peter. Peter is 5 years older than Alice. In 5 years, John will be
three times as old as Alice. How old is Peter now?
Let x be Peter’s age now. Add 5 to get the ages in 5 yrs.
age now age in 5 yrs
John 2x 2x + 5
Peter x x+5
Alice x–5 x–5+5
Write the new relationship in an equation using the ages in 5 yrs.
In 5 years, John will be three times as old as Alice.
2x + 5 = 3(x – 5 + 5)
2x + 5 = 3x
Isolate variable x
x=5
Answer: Peter is now 5 years old.

5.4 Money or Cost Problems


Word problems that deal with the value, number of pieces of bills or coins, and denomination of
money are called money problems.
Frequently asked questions on money problems are:

83
(a) How many bills of each type does he have?
(b) How many P5 coins did she get?
(c) How much money did she pay?
In order to properly solve money problems, it is recommended that a table is constructed which
includes the kind of denomination, the number of pieces of coins or bills, and its value. But in
other cases, these will be changed depending on the specific pro9blem given. To better
understand this, a few examples are provided below.
Examples:
(a) A man’s wallet contains 9 bills in P500 and P100. His total money is P2900. How many
P500 and P100 does he have?
Solution:
Let x: x – number of P500 bills
(9 – x) – number of P100 bills
Remember that the (kind of denomination x number of pieces of money) = value
Denomination Number of Value
Pieces
P100 9–x 100(9 – x)
P500 x 500x
Total P2900
Looking at the table, adding the two values and equating it to the total gives us:
100(9 – x) + 500x = 2900
900 – 100x + 500x = 2900
400x = 2900 – 900
x = 2000
4000
x = 5
9-x = 4
Therefore, there are 5 P500 bills and 4 P100 bills.
(b) A debtor borrowed P1000 and P200 bills in a loan company. The number of P200 bills is
two less than thrice the number of P1000 bills. If he borrowed a total of P6000, how
many P1000 and P200 bills did he borrow?
Solution:
Let: x – number of P1000 bills
(3x – 2) – number of P200 bills
Denomination Number of Pieces Value
P200 3x - 2 200(3x – 2)
P1000 x 1000x
Total P6000

Looking at the table, adding the two values and equating it to the total give us:
200(3x – 2) + 1000x = 6000
600x – 400 + 1000x = 6000
1600x = 6400
x = 6400

84
1600
x = 4
x = 10
Therefore, he borrowed 4 P1000 bills and 10 P200 bills.

(c) In order to play in the park’s playground, children who are 15 years old and below need
to pay P5 and P10 for those above 15. In a day, a total of 50 get in the playground. If
they can collect a total of P340 in a day, how many children below 15 years old enter the
playground?
Solution:
Let: x – number of people above 15 years old
(50 – x) – Number of children 15 years old and below

Age Brackets Entrance Fee Number of People Sales


15 years and below P5 50 – x 50(50 – x)
Above 15 P10 x 10x
Total P340
Looking at the table, adding the sales and equating it to the total gives us:
50(50 – x) +10x = 340
250 – 5x + 10x = 340
5x = 340 – 250
X = 90
5
X = 18
50 – x = 32

Therefore, there are 32 children aged 15 years old and below who enter the playground in a day.
Exercises:
1. A cashier remitted her sales which sums up to P15,800. She forgot how many P1000, P500
and P100 bills she gave but she can still remember that there are equal numbers of P1000 and
P500 bills and the number of P100 bills is two less than the number of P500 bills. How many
P100 bills did she remit?
a. 5
b. 6
c. 7
d. 8
Solution:
Denomination Number of Pieces Value
P100 x-2 100(x – 2)
P500 x 500x
P1000 x 1000x

85
Total P15, 800
100(x – 2) + 500x + 1000x = 15800
100x – 200 + 1500x = 15800
1600x = 16000
x = 16000
1600
x = 10
x–2 = 8
Therefore, there are 8 P100 bills.
2. You have P20 and P50 bills. There are 15 more P20 bills than twice the P50 bills. The
total money you have is P1, 200. How many P20 bills do you have?
a. 25
b. 35
c. 45
d. 60
Solution:
Denomination Number of Value
Pieces
P5 15 + 2x 20(15 + 2x)
P10 x 50 x
Total P1,200
20(15 + 2x) + 50x = 1200
300 + 40x + 50x = 1200
90x = 1200 – 300
x = 900
90
x = 10
15 + 2x = 15 + 2(10)
15 + 2x = 35
Therefore, you have 35 P20 bills.
3. Mimi has twice as many P5 coins than P10 coins. The total value of her money is P100.
How many P5 coins did she have?
a. 5
b. 9
c. 10
d. 12
Solution:
Denomination Number of Pieces Value

86
P5 2x 10x
P10 x 10x
Total P100
10x + 10x = 100
20x = 100
x = 100
20
x = 5
2x = 10
Therefore, Mimi has 10 P5 coins.
4.Charles dropped P1 and P5 coins in a vending machine in order to buy a certain snack. If
he dropped 3 more P1 coins than twice the P5 coins, and the snack costs P17, how many
P1 coins did he drop?
a. 2
b. 5
c. 7
d. 8
Solution:
Denomination Number of Pieces Value
P1 3 + 2x 3 + 2x
P5 x 5x
Total P17
3 + 2x + 5x = 17
7x = 17 – 3
7x = 14

x = 14
7
x = 2
3 + 2x = 7
Therefore, Charles dropped 7 P1 coins to the vending machine.
5. A bank teller received P100, 550 to be deposited to the client’s account. She received
P1000, P100, and P50 bills. If the P100 bills are seven more than 5 times the P50 bills and
12 more than twice the P100 bills are the number of pieces of P1000 bills, then how many
P1000 bills are there?
a. 84
b. 96
c. 98

87
d. 102
Solution:
Denomination Number of Value
Pieces
P1000 2(5x + 7) + 12 1000[2(5x + 7) +
12]
P100 5x + 7 100(5x + 7)
P50 x 50x
Total P100,550
1000[2(5x + 7) + 12] + 100(5x + 7) + 50x = 100 550
1000[2(5x + 7) + 12] + 500x + 100 + 50x = 100 550
10 000x + 26 000 + 550x + 700 = 100 550
10 550x + 26 700 = 100 550
10 550x = 100 550 – 26 700
x = 73 850
10 550
x = 7
2(5x + 7) + 12 = 2[5(7) + 7] + 12
2(5x + 7) + 12 = 2[42] + 12
2(5x + 7) + 12 = 96
Therefore, the teller received 96 pieces of P1000 bills.
6. Marie wants to buy a pair of shoes that costs P560. She has a P100, P50 and P20 bills.
If there are equal number of P100 and P20 bills and the number of P50 bills is two les
than twice the number of P100 bills, how many P50 bills does Marie have?
a. 4
b. 3
c. 2
d. 1

Denomination Number of Value


Pieces
P20 x 20x
P50 2x - 2 50(2x – 2)
P100 x 100x
Total P560
20x + 50(2x – 2) + 100 = 560
120x + 100x – 100 = 560

88
220x = 560 + 100
220x = 660
x = 660
220
x = 3
2x – 2 = 4
Therefore Marie has 4 P50 bills.
7. Ten thousand tickets were sold out for Katy Perry’s concert in Manila. VIP seat were
sold at P5000 each and the regular seats were sold at P2500 each. If they collected a
total P32,500,000, how many VIP’s watched the concert?
a. 2500
b. 3000
c. 7000
d. 7500
Number of Rate per Sales
tickets sold Ticket
VIP X 5 000 5 000x
Regular 10, 000 - x 2 500 2 500(10 000 –
x)
Total P32, 500, 000

5 000x + 2 500(10 000 – x) = 32 500 000


1 1
(5 000x + 25 000 000 – 2500x) = (32 500 000)
100 100

50x + 250 000 – 25x = 325 000


25x = 325 000 – 250 000
75 000
x =
25

x = 3000
Therefore, there are a total of 3 000 VIP’s who watched Katy Perry’s concert.
8. On the first day of showing of a film, 856 tickets were sold. Tickets were priced
differently for students and non-students. Regular rate is P100 and a 20% discount is
given to students. If the total sales for the day are P73, 480, how many of each type of
ticket were sold?
a. regular ticket – 250; student’s ticket – 606
b. regular ticket – 300; student’s ticket – 556
c. regular ticket – 350; student’s ticket – 506
d. regular ticket – 400; student’s ticket – 456

Number of Rate per Ticket Sales


tickets sold

89
Regular x 100 100x
Students 856 - x 100 – (20% of 100) 80(856 – x)
= 100 – (.20)
= (100 – 20)
= 80
Total P73 480
100x + 80(856 – x) = 73 480
100x + 68480 – 80x = 73 480
20x = 73 480 – 68 480
20x = 5 000
5 000
x =
20

x = 250
856 – x = 606
Therefore, there are 250 regular tickets sold and 606 student tickets were sold.
9. Karl’s piggy band contains P486 with P1, P5 and P10 coins. If the number of P1 coins is
10 more than the P5 coins and the number of P5 coins is 10 more than the P10 coins,
how many coins does Karl have?
a. 16
b. 26
c. 36
d. 46
Denomination Number of Value
Pieces
P10 x 10x
P5 10 + x 5(10 + x)
P1 10 + 10 + x = 20 20 + x
+x
Total P486
10x + 5(10 + x) + 20 + x = 486
11x + 20 + 50 + 5x = 486
16x = 486 – 70
16x = 416
416
x =
16

x = 26
Therefore, Karl has 26 P10 coins.

90
10. Don Juan’s farm which is worth P10 million is to be apportioned to his two sons John
1
and Jude. Each of them shall first receive of the share and the rest shall be divided
4
such that John gets half a million more than twice the share of Jude. How much total
share will John receive?
a. P3, 500, 000
b. P1, 500, 000
c. P4, 000, 000
d. P6, 000, 000
Solution:
Since the two brothers will each have one-fourth of the 10 million, then they each get:
1
(10 000 000) = P2 500 000
4

Hence, the remaining money to be divided is P5 000 000.


Corresponding Share
John 500 000 + 2x
Jude x
Total P5 000 000
500 000 + 2x + x = 5 000 000
3x = 4 500 000
4 500 000
x =
3

x = 1 500 000
x = 3 500 000

Initially, John already received his share worth P2 500 000. Adding his other share, then he gets
a total P6 million worth of shares.
5.5 Geometry Problems
It requires a working knowledge of both Geometry and Algebra to solve Geometry-
related problems. We shall recall the formulas for finding the perimeter or area of simple
geometric figures.
Lesson Proper:
Recall the following concepts:
1. A line segment is the union of two points and all points between them. It has definite
length.
2. Two angles are complementary if the sum of their measures is 90°. Two angles are
supplementary if the sum of their measures is 180°. The sum of the measures of the
angles of a triangle is 180°.
3. The perimeter of a triangle is equal to the sum of the lengths of all its three sides. The
area of a triangle is equal to one-half the product of the base and the altitude.

91
4. The perimeter 𝑃 of a rectangle is equal to the sum of twice its length 𝑙 and twice its width 𝑤,
that is, 𝑷 = 𝟐𝒍 + 𝟐𝒘. The area 𝐴 of a rectangle is equal to the product of its length and
width, that is, 𝐴 = 𝑙𝑤.
5. A square is a rectangle with all sides congruent. The area 𝐴 of a square is equal to the
square of the length of its sides 𝑠, that is, 𝑨 = 𝒔𝟐 . The perimeter 𝑃 of a square is equal to
four times the length of its side 𝑠, that is, 𝑷 = 𝟒𝒔.
6. A rhombus is a parallelogram with all sides congruent. The area of a rhombus is equal to
half the product of its diagonals. The diagonal of a rhombus is the segment joining two
opposite vertices.
7. The area of a trapezoid is equal to one-half the product of its altitude and the sum of its
bases. In symbol, we write
𝒉(𝑩𝟏 + 𝒃𝟐 )
𝑨= , where 𝐵 = upper base
𝟐

𝑏 = lower base
ℎ = altitude
8. A circle is a set of points on the plane equidistant from a given fixed point. The area 𝐴 and
circumference 𝐶 of a circle having a radius 𝑟 are given by the following formulas:
22
𝑨 = 𝝅𝒓𝟐 and 𝐶 = 2𝜋𝑟, where 𝝅 = 𝟑. 𝟏𝟒𝟏𝟔 or
7

Example:
1. A piece of wooden rod, 48 cm long, is divided into three parts. The second part is
longer than the first part by 1 cm while the third part is 2 cm longer than the first. How
long is each part?
Solution: Let 𝑥 = first part
𝑥 + 1 = second part
𝑥 + 2 = third part
Equation: 𝑥 + (𝑥 + 1) + (𝑥 + 2) = 48
Solving the equation, we have
𝑥 + (𝑥 + 1) + (𝑥 + 2) = 48
3𝑥 + 3 = 48
3𝑥 = 45
𝑥 = 15 (first part)
𝑥 + 1 = 16 (second part)
𝑥 + 1 = 17 (third part)
Therefore, the lengths of the parts are 15cm, 16cm, and 17cm.
Checking, we obtain 15 + 16 + 17 = 48
48 = 48

2. The measure of a given angle is twice the measure of another. What are their
measures if they are complementary to each other?
Solution: Let 𝑥 = measure of the smaller angle
2𝑥 = measure of its complement
Equation: 𝑥 + 2𝑥 = 90
Solving the equation, we obtain
𝑥 + 2𝑥 = 90
3𝑥 = 90

92
𝑥 = 30
2𝑥 = 60
Therefore, the measures of the angles are 30° and 60°.

3. If a square floor has a side of 2m, how many 2cm x 2cm tiles are needed to
completely cover this floor?
Solution: The square floor is 2m or 200cm on one side. Thus, its area is
𝐴 = (200 𝑐𝑚)2
= 40 000 𝑠𝑞 𝑐𝑚
Each tile has an area of (2 cm)2 or 4 sq cm. Therefore, the number of tiles
40 000
needed is or
4

10 000.
Lesson Exercise:
Solve the following Geometry word problems.
1. Each leg of an isosceles triangle is 11cm longer than the base. The perimeter of the
triangle is 64cm. Find the length of the base.
a. 20 cm
b. 18 cm
c. 14 cm
d. 10 cm
Correct answer: C
Solution:
An isosceles triangle has two congruent sides. Each of these sides is called a
leg. The third side of an isosceles triangle is called the base.
Let 𝑥 = length of the base
𝑥 + 11 = length of one leg
The perimeter is 64 cm. Hence,
𝑥 + (𝑥 + 11) + (𝑥 + 11) = 64
Solving the equation, we have
𝑥 + (𝑥 + 11) + (𝑥 + 11) = 64
3𝑥 + 22 = 64
3𝑥 = 42
𝑥 = 14
𝑥 + 11 = 25
Therefore, the length of the base is 14 cm.

93
2. The perimeter of a rectangular vegetable garden is 54 m. If the length of the garden is
3m longer than its width, find the dimensions of the garden.
A. 15 m by 12 m
B. 10 m by 30 m
C. 8 m by 12 m
D. 15 m by 16 m
Correct answer: A
Solution:
Let 𝑥 = the width of the rectangular garden
𝑥 + 3 = the width of the rectangular garden
Equation: 54 = 2(𝑥 + 3) + 2𝑥 from the formula 𝑃 = 2𝑙 + 2𝑤
Solving the equation, we get 𝑥 = 12 and 𝑥 + 3 = 15.
Therefore, the width is 12 m and the length is 15 m. The rectangular garden has
dimensions by 15 m by 12 m.

3. A rhombus has an area of 100 sq cm. If its shorter diagonal is 10 cm, find the length of
the longer diagonal.
A. 14 cm
B. 16 cm
C. 12 cm
D. 20 cm
Correct answer: D
Solution:
Let 𝑥 = the length of the longer diagonal
Recall that the area of a rhombus is equal to one-half the product of its
diagonals. So,
1
( ) (10)(𝑥) = 100
2

Solving the equation, we obtain


1
( ) (10)(𝑥) = 100
2

5𝑥 = 100
𝑥 = 20
Therefore, the length of the longer diagonal is 20 cm.

4. If the circumference and area of a circle are numerically equal, find the circumference
and area of the circle.
A. 𝐴 = 3𝜋 sq units and 𝐶 = 3𝜋 units
B. 𝐴 = 4𝜋 sq units and 𝐶 = 4𝜋 units
C. 𝐴 = 5𝜋 sq units and 𝐶 = 5𝜋 units

94
D. 𝐴 = 6𝜋 sq units and 𝐶 = 6𝜋 units
Correct answer: B
Solution:
Since 𝐶 = 𝐴,
2𝜋𝑟 = 𝜋𝑟 2
Solving for 𝑟, we obtain 𝑟 = 2.
Therefore, 𝐴 = 𝜋𝑟 2 = (𝜋)(2)2 = 4𝜋 sq units and
𝐶 = 2𝜋𝑟 = (2𝜋)(2) = 4𝜋 units.

5. The measures of the three angles of a triangle are in the ratio 3:4:5. What are the
measures of the angles?
A. 50°, 50°, 80°
B. 45°, 60°, 75°
C. 100°, 35°, 45°
D. 30°, 75°, 75°
Correct answer: B
Solution:
Let 3𝑥 = first angle
4𝑥 = second angle
5𝑥 = third angle
Since the sum of the angles in a triangle is 180°, we obtain
3𝑥 + 4𝑥 + 5𝑥 = 180
12𝑥 = 180
𝑥 = 15
3𝑥 = 45
4𝑥 = 60
5𝑥 = 75
Therefore, the measures of the angles are 45°, 60°, and 75° respectively.

6. The base of a triangle is 6 cm. If the area of a triangle is 30 sq cm, find its altitude.
A. 10 cm
B. 7 cm
C. 14 cm
D. 5 cm
Correct answer: A
Solution:
Let ℎ = the altitude of the triangle

95
6ℎ 𝑏ℎ
Equation: 30 = from the formula 𝐴 =
2 2

Solving the equation, we get ℎ = 10.


Therefore, the altitude of the triangle is 10 cm.
7. The second angle of the triangle is three times the first angle, and the third angle is 5
times the first. Find the measure of each angle.
a. 20°, 80°, 80°
b. 30°, 60°, 90°
c. 20°, 60°, 100°
d. 110°, 35°, 35°
Correct answer: C
Solution:
Let 𝑥 = first angle
3𝑥 = second angle
5𝑥 = third angle
Since the sum of the angles in a triangle is 180°, we obtain
𝑥 + 3𝑥 + 5𝑥 = 180
9𝑥 = 180
𝑥 = 20
3𝑥 = 60
5𝑥 = 100
Therefore, the measure of each angle are 20°, 60°, and 100°.

8. The area of a triangle is 54 sq cm. Find its altitude if the measure of its base is 18 cm.
A. 4 cm
B. 7 cm
C. 8 cm
D. 6 cm
Correct answer: D
Solution:
Let ℎ = the altitude of the triangle
18ℎ 𝑏ℎ
Equation: 54 = from the formula 𝐴 =
2 2

Solving the equation, we get ℎ = 6.


Therefore, the altitude of the triangle is 6 cm.

9. A triangle has a perimeter of 21 cm. What are its dimensions if the length of the second
side is 1 cm more than the length of the first and the third side is twice as long as the
first?
a. 4 cm, 5 cm, 9 cm

96
b. 5 cm, 6 cm, 10 cm
c. 8 cm, 9 cm, 13 cm
d. 6 cm, 7 cm, 11 cm
Correct answer: B
Solution:
Let 𝑥 = length of the first side
𝑥 + 1 = length of second side
2𝑥 = length of third side
The perimeter is 21 cm. Hence,
𝑥 + (𝑥 + 1) + (2𝑥) = 21
Solving the equation, we have
𝑥 + (𝑥 + 1) + (2𝑥) = 21
4𝑥 + 1 = 21
4𝑥 = 20
𝑥=5
𝑥+1 =6
2𝑥 = 10
Therefore, the dimensions of the triangle are 5 cm, 6 cm, and 10 cm.

10. The length of a rectangle is 3 cm more than twice the width. Find the area when its
perimeter is 30 cm.
A. 110 sq cm
B. 25 sq cm
C. 53 sq cm
D. 44 sq cm
Correct answer: D
Solution:
Let 𝑤 = width of the rectangle
2𝑤 + 3 = length of the rectangle
𝐴 = area of the rectangle
Equation 1: 30 = 2(2𝑤 + 3) + 2(𝑤) from the formula 𝑃 = 2𝑙 + 2𝑤
Solving the equation, we have
2(2𝑤 + 3) + 2(𝑤) = 30
6𝑤 + 6 = 30
6𝑤 = 24
𝑤=4
2𝑤 + 3 = 11

97
Equation 2: 𝐴 = 11(4) from the formula 𝐴 = 𝑙𝑤
Therefore, the area of a rectangle is 44 sq cm.

11. What is the area and the perimeter of a square farm lot whose side measures 21 m?
A. 𝐴 = 336 sq m & 𝑃 = 45 m
B. 𝐴 = 225 sq m & 𝑃 = 36 m
C. 𝐴 = 441 sq m & 𝑃 = 84 m
D. 𝐴 = 121 sq m & 𝑃 = 50 m
Correct answer: C
Solution:
Let 𝐴 = area of the farm lot
𝑃 = perimeter of the farm lot
Equation 1: 𝐴 = 212 from the formula 𝐴 = 𝑠2
Equation 2: 𝑃 = 4(21) from the formula 𝑃 = 4𝑠
Solving the equations, we have
𝐴 = 212
𝐴 = 441 sq m
𝑃 = 4(21)
𝑃 = 84 m
Therefore, the area of the square farm lot is 441 sq m & its perimeter is 84 m.

12. The area of a trapezoid is 72 sq cm. One base is 10 cm long and the other is 14 cm.
Find its altitude.
A. 6 cm
B. 7 cm
C. 5 cm
D. 10 cm
Correct answer: A
Solution:
Let ℎ = altitude
ℎ(10+ 14) ℎ(𝐵1 + 𝑏2 )
Equation: 72 = from the formula 𝐴 =
2 2

Solving the equation, we have


ℎ(10+ 14)
72 =
2
ℎ(24)
72 =
2

98
24ℎ
72 =
2

144 = 24ℎ
ℎ= 6
Therefore, the altitude of the trapezoid is 6 cm.

13. A trapezoid has an area of 240 sq cm and its altitude is 16 cm long. Find the length of its
two bases if one base is 3 cm longer than the other.
A. 13.5 cm & 16.5 cm
B. 5.5 cm & 20 cm
C. 7.5 cm & 17.5 cm
D. 11.5 cm & 15.5 cm
Correct answer: A
Solution:
Let 𝑥 = first base
𝑥 + 3 = second base
16[ (𝑥) +(𝑥+3) ] ℎ(𝐵1 + 𝑏2 )
Equation 1: 240 = from the formula 𝐴 =
2 2

Solving the equation, we have


16[ (𝑥) +(𝑥+3) ]
240 =
2
16(2𝑥+3)
240 =
2
32𝑥+48
240 =
2

480 = 32𝑥 + 48
32𝑥 = 432
𝑥 = 13.5
𝑥 + 3 = 16.5
Therefore, the lengths of the bases of the trapezoid are 13.5 cm & 16.5 cm.

14. The sum of the measures of two angles is 180°. Three times the measure of one angle
is 24 less than the measure of the other angle. What is the measure of each angle?
A. 80° & 100°
B. 39° & 141°
C. 90° & 90°
D. 49° & 131°
Correct answer: B
Solution:
Let 𝑥 = the measure of the small angle
𝑦 = the measure of the large angle

99
3𝑥 = 𝑦 − 24, the relationship between the two angles
The system of equations consists of:
𝑥 + 𝑦 = 180 (1)
{
3𝑥 = 𝑦 − 24 (2)
Solve for 𝑦 in terms of 𝑥 in equation (1).
𝑦 = 180 − 𝑥
Substitute 180 − 𝑥 for 𝑦 in equation (2).
3𝑥 = (180 − 𝑥) − 24
3𝑥 = 156 − 𝑥
4𝑥 = 156
𝑥 = 39
𝑦 = 180 − 39
𝑦 = 141
The solution of the system is (39, 141).
Therefore, the measure of the small angle is 39° and the measure of the large angle is 141°.

15. The circumference of a circle and the perimeter of a square are both 40 cm. Which has a
greater area, the circle or the square?
A. the same
B. square
C. circle
D. none of the above
Correct answer: C
Solution:
Recall that the circumference of a circle is,
𝐶 = 2𝜋𝑟
And the perimeter of a square is,
𝑃 = 4𝑠
Equation 1: 40 = 2𝜋𝑟 from the formula 𝐶 = 2𝜋𝑟
Solving the equation, we have
40 = 2𝜋𝑟
𝑟 = 20𝜋 cm
Equation 2: 40 = 4𝑠 from the formula 𝑃 = 4𝑠
Solving the equation, we have
40 = 4𝑠
𝑠 = 10 cm

100
Substitute 𝑟 in the formula for the area of a circle.
𝐴 = 𝜋𝑟 2
𝐴 = 𝜋(20𝜋)2
𝐴 = 40𝜋 3 sq cm
Substitute 𝑠 in the formula for the area of a square.
𝐴 = 𝑠2
𝐴 = 102
𝐴 = 100 sq cm
Therefore, the circle has a greater area.

16. A triangle has a perimeter of 50. If 2 of its sides are equal and the third side is 5 more
than the equal sides, what is the length of the third side?
a. 20
b. 30
c. 40
d. 60
Correct answer: A
Solution:
Let 𝑥 = length of the equal sides
𝑥 + 5 = length of the third side
Recall that perimeter of a triangle is,
𝑃 = 𝑠𝑢𝑚 𝑜𝑓 𝑡ℎ𝑒 𝑡ℎ𝑟𝑒𝑒 𝑠𝑖𝑑𝑒𝑠
So, 50 = 𝑥 + 𝑥 + 𝑥 + 5
Solving the equation, we have
50 = 3𝑥 + 5
Isolate variable 𝑥,
3𝑥 = 50 − 5
3𝑥 = 45
𝑥 = 15
𝑥 + 5 = 20
Therefore, the length of the third side is 20.

17. A rectangle is 4 times as long as it is wide. If the length is increased by 4 inches and the
width is decreased by 1 inch, the area will be 60 square inches. What were the
dimensions of the original rectangle?
a. 3 in by 12 in
b. 4 in by 16 in

101
c. 6 in by 24 in
d. 2 in by 8 in
Correct answer: B
Solution:
Let 𝑥 = original width of the rectangle
4𝑥 + 4 = length of the rectangle
𝑥 − 1 = width of the rectangle
Recall that the formula for area of rectangle is,
𝐴 = 𝑙𝑤
Plug in the values from the question and from the sketch.
60 = (4𝑥 + 4)(𝑥 − 1)
Simplifying we have,
60 = 4𝑥 2 − 4𝑥 + 4𝑥 − 4

Put in quadratic form


4𝑥 2 − 4 − 60 = 0
4𝑥 2 − 64 = 0
This quadratic can be rewritten as a difference of two squares
(2𝑥)2 − (8)2 = 0
Factor difference of two squares
(2𝑥)2 − (8)2 = 0
(2𝑥 − 8)(2𝑥 + 8) = 0
We get two values for 𝑥.
2𝑥 − 8 = 0 → 2𝑥 = 8 → 𝑥 = 4
2𝑥 + 8 = 0 → 2𝑥 = −8 → 𝑥 = −4
Since 𝑥 is a dimension, it would be positive. So, we 𝑥 = 4.
The question requires the dimensions of the original rectangle.
The width of the original rectangle is 4.
The length is 4 times the width → 4 × 4 = 16
Therefore, the dimensions of the original rectangle are 4 and 16.

18. In a quadrilateral two angles are equal. The third angle is equal to the sum of the two
equal angles. The fourth angle is 60° less than twice the sum of the other three
angles. Find the measures of the angles in the quadrilateral.
a. 85°, 85°, 70°, & 120°
b. 65°, 65°, 10°, & 220°
c. 45°, 45°, 70°, & 200°

102
d. 35°, 35°, 70°, & 220°
Correct answer: D
Solution:
Let 𝑥 = size of one of the two equal angles
𝑥 + 𝑥 = third angle
20(𝑥 + 𝑥 + 𝑥 + 𝑥) − 60 = fourth angle
The sum of angles in a quadrilateral is 360°
Plug in the values from the question
360 = x + x + (x + x) + 2(x + x + x + x)– 60
Combine like terms
360 = 4x + 2(4x) – 60
360 = 4x + 8x – 60
360 = 12x − 60
Isolate variable 𝑥
12𝑥 = 420
𝑥 = 35
The question requires the values of all the angles.
Substituting x for 35, you will get: 35, 35, 70, 220.
Therefore, the values of the angles are 35°, 35°, 70°, & 220°.

19. A square has a perimeter of 24 inches. What is the area of the square?
a. 34 sq in
b. 32 sq in
c. 36 sq in
d. 38 sq in
Correct answer: C
Solution:
Let 𝑥 = area of the square
Recall that the perimeter of a square is,
𝑃 = 4𝑠
We obtain 𝑠 = 6 from the formula 24 = 4𝑠.
Substitute 𝑠 in the equation 𝐴 = 𝑠2 ,
𝐴 = 𝑠2
𝐴 = 62
𝐴 = 36 in

103
Therefore, the area of the square is 36 in.

20. A square kitchen has an area of 100 square feet. What is the kitchen’s perimeter?
a. 42 ft
b. 40 ft
c. 36 ft
d. 38 ft
Correct answer: B
Solution:
Let 𝑥 = kitchen’s perimeter
Recall that the area of a square is,
𝐴 = 𝑠2
We obtain 𝑠 = 10 from the formula 100 = 𝑠2 .
Substitute 𝑠 in the equation 𝑃 = 4𝑠,
𝑃 = 4𝑠
𝑃 = 4(10)
𝑃 = 40
Therefore, the perimeter of the square kitchen is 40 ft.

5.6 Percent Problems


When you learned how to translate simple English statements into mathematical expressions,
you learned that "of" can indicate "times". This frequently comes up when using percentages.
If you need to find 16% of 1400, you first convert the percentage "16%" to its decimal form;
namely, the number "0.16". (When you are doing actual math, you need to use actual numbers.
Always convert the percentages to decimals!) Then, since "sixteen percent OF fourteen
hundred" tells you to multiply the 0.16 and the 1400, you get: (0.16)(1400) = 224. This says
that 224 is sixteen percent of 1400.
Percentage problems usually work off of some version of the sentence "(this) is (some
percentage) of (that)", which translates to "(this) = (some decimal) × (that)". You will be given
two of the values, or at least enough information that you can figure two of them out. Then you'll
need to pick a variable for the value you don't have, write an equation, and solve for that
variable.
 What percent of 20 is 30?
We have the original number (20) and the comparative number (30). The unknown in
this problem is the rate or percentage. Since the statement is "(thirty) is (some
percentage) of (twenty)", then the variable stands for the percentage, and the equation
is:
30 = (x)(20)
30 ÷ 20 = x = 1.5

104
Since x stands for a percentage, I need to remember to convert this decimal back into a
percentage:

1.5 = 150%
Thirty is 150% of 20.

 What is 35% of 80?


Here we have the rate (35%) and the original number (80); the unknown is the
comparative number which constitutes 35% of 80. Since the exercise statement is
"(some number) is (thirty-five percent) of (eighty)", then the variable stands for a number
and the equation is:
x = (0.35)(80)
x = 28
Twenty-eight is 35% of 80.
 45% of what is 9?
Here we have the rate (45%) and the comparative number (9); the unknown is the
original number that 9 is 45% of. The statement is "(nine) is (forty-five percent) of (some
number)", so the variable stands for a number, and the equation is:
9 = (0.45)(x)
9 ÷ 0.45 = x = 20
Nine is 45% of 20.
The format displayed above, "(this number) is (some percent) of (that number)", always holds
true for percent. In any given problem, you plug your known values into this equation, and then
you solve for whatever is left.
 Suppose you bought something that was priced at $6.95, and the total bill
was $7.61. What is the sales tax rate in this city? (Round answer to one decimal
place.)
The sales tax is a certain percentage of the price, so I first have to figure what the actual
tax was. The tax was:
7.61 – 6.95 = 0.66
Then (the sales tax) is (some percentage) of (the price), or, in mathematical terms:
0.66 = (x)(6.95)
Solving for x, I get:
0.66 ÷ 6.95 = x = 0.094964028... = 9.4964028...%
The sales tax rate is 9.5%.
In the above example, I first had to figure out what the actual tax was. Many percentage
problems are really "two-part-ers" like this: they involve some kind of increase or decrease
relative to some original value. Warning: Always figure the percentage of change relative to
the original value.

105
 Suppose a certain item used to sell for seventy-five cents a pound, you see that
it's been marked up to eighty-one cents a pound. What is the percent increase?
First, I have to find the absolute increase: Copyright © Elizabeth Stapel 1999-2011 All
Rights Reserved
81 – 75 = 6
The price has gone up six cents. Now I can find the percentage increase over the
original price.
Note this language, "increase/decrease over the original", and use it to your advantage: it will
remind you to put the increase or decrease over the original value, and then divide.
This percentage increase is the relative change:
6
/75 = 0.08
...or an 8% increase in price per pound.
 Cost – the price that the business pays for the product.
 Selling price – price for which business sells a product to a customer
 Markup – the amount added to the cost to obtain the selling price
 Markup rate – the markup divided by the cost
 Discount – the amount subtracted by the retailer to the regular price of a product during
a promotional sale
 Discount rate – the discount divided by the regular price

Therefore, we have the following formula:


(1) 𝑆𝑒𝑙𝑙𝑖𝑛𝑔 𝑝𝑟𝑖𝑐𝑒 (𝑆) − 𝑀𝑎𝑟𝑘𝑢𝑝 (𝑀) + 𝐶𝑜𝑠𝑡(𝐶)
𝑀𝑎𝑟𝑘𝑢𝑝 (𝑀)
(2) 𝑚𝑎𝑟𝑘𝑢𝑝 𝑟𝑎𝑡𝑒 (𝑚) =
𝐶𝑜𝑠𝑡(𝐶)

𝑚∙𝐶 =𝑀
Since 𝑀 = 𝑚𝐶, we substituted this value of M to (1) hence, we have:
𝑆=𝑀+𝐶
(3) 𝑆 = 𝑚𝐶 + 𝐶
𝑆 = 𝐶(𝑚 + 1)
(4) 𝑆𝑎𝑙𝑒 𝑃𝑟𝑖𝑐𝑒 (𝑆𝑃) = 𝑅𝑒𝑔𝑢𝑙𝑎𝑟 𝑃𝑟𝑖𝑐𝑒 (𝑅) − 𝐷𝑖𝑠𝑐𝑜𝑢𝑛𝑡 (𝐷)
𝐷𝑖𝑠𝑐𝑜𝑢𝑛𝑡 (𝐷)
(5) 𝑑𝑖𝑠𝑐𝑜𝑢𝑛𝑡 𝑟𝑎𝑡𝑒 (𝑑) =
𝑅𝑒𝑔𝑢𝑙𝑎𝑟 𝑃𝑟𝑖𝑐𝑒 (𝑅)
𝑑∙𝑅 = 𝐷
Since 𝐷 = 𝑑 ∙ 𝑅, we substitute this value of D to (4) hence, we have:
𝑆𝑃 = 𝑅 − 𝐷
(6) 𝑆𝑃 = 𝑅 − 𝑑 ∙ 𝑅
𝑆𝑃 = 𝑅(1 − 𝑑)
Formulae (1) –(3) are used to markup problems and formulae (4) – (6) are used to discount
problems. Below, some examples of markup and discount problems are shown.

106
Examples:
 The markup of certain pair of shoes is P100. If the selling price is P699, how much is the
markup rate?
Solution:
Given: Selling Price – P699
Markup – P100
m-?
Using formula (1):
𝑆 =𝑀+𝐶
699 = 100 + 𝐶
599 = 𝐶

After getting the cost, use formula (3):


𝑆 = 𝐶(𝑚 + 1)
699 = 599(𝑚 + 1)
699 = 599𝑚 + 599
699 − 599 = 599𝑚
100
=𝑚
599
𝑚 ≈ 0.1669 𝑜𝑟 16.69%
Therefore, the markup rate is 16.69%.
 A jewelry vendor buys a ten-set jewelry which cost a total of P1 300 and sells each of
them for P190 each. What is the markup rate?
Solution:
Given: Selling Price – P190
Cost – P130
A ten-set jewelry was bought for a total of P1 300, for every jewelry item, the cost is P130.
To get the markup rate given the selling price and the cost, we use formula (3).
𝑆 = 𝐶(𝑚 + 1)
190 = 130(𝑚 + 1)
190 = 130𝑚 + 130
−130𝑚 = 130 − 190
−130𝑚 = −60
−60
𝑚=
−130
𝑚 ≈ 0.4615 𝑜𝑟 46.15%

107
Therefore, the markup rate is 46.15%
 In a back-to-school sale, notebooks were on sale and now cost only P8 each. If the
discount rate is 20%, what is the regular price?
Solution:
Given: Sale Price – P8
Discount rate – 20% or 0.20
Using formula (6):
𝑆𝑃 = 𝑅(1 − 𝑑)
8 = 𝑅(1 − 0.20)
8 = 𝑅(0.80)
8
=𝑅
0.80
𝑅 = 10
Therefore, the regular price of a notebook is P10.
 Designer bags sold by a mall are discounted by 50%. If Myra bought a designer bag for
only P30, 650, what was the original price?
Solution:
Given: Sale Price – P30, 650
Discount rate – 50% or 0.50
Using formula (6):
𝑆𝑃 = 𝑅(1 − 𝑑)
30 650 = 𝑅(1 − 0.50)
30 650 = 𝑅(0.50)
30 650
=𝑅
0.50
𝑅 = 61 300
Therefore the original price of the designer bag is P61 300.

Lesson Exercise:
1. What is 17% of 198?
a. 1164.71
b. 3366
c. 33.66
d. 197.83
Solution:
𝑥 = 0.17(198)

108
𝑥 = 33.66

2. Five hundred seventy is what percent of 200?


a. 2.85%
b. 258%
c. 0.285%
d. 28.5%
Solution:
570 = 𝑥(200)
570
=𝑥
200
𝑥 = 2.85 𝑜𝑟 285%

3. Two hundred sixty-four is 255 of what?


a. 66
b. 256
c. 1044
d. 1056
Solution
264 = 25%(𝑥)
264 = 0.25𝑥
264
=𝑥
0.25
𝑥 = 1 056

4. A coin collector bought a century old coin for P5 800 and sold it for P10 400. Find the markup
rate.
a. 68.31%
b. 79.31%
c. 84.29%
d. 86.79%
Solution:
Given: Cost – 5800
Selling Price – 10 400
𝑆 = 𝐶(𝑚 + 1)

109
100 400 = 5800(𝑚 + 1)
10 400 = 5800𝑚 + 5800
10 400 − 5800 = 5800𝑚
4600 = 4500𝑚
4600
=𝑚
5800
𝑚 ≈ 0.7931 𝑜𝑟 79.31%
Therefore, the coin collector used a markup rate of 79.31%.

5. Mr. Cruz bought two dozens of basketball for P4 200 and sold each of them. If the markup
rate is 10%, what is the selling price of each ball?
a. P192.50
b. P185.50
c. P200.50
d. P266.50
Solution:
4200
Given: 𝐶𝑜𝑠𝑡 − = 175
24

markup rate – 10% or 0.10


𝑆 = 𝐶(𝑚 + 1)
𝑆 = 175(0.10 + 1)
𝑆 = 192.50
Therefore, selling price for each basketball is P192.50.

6. A supermarket uses a 35% markup on cost. What is the cost of a canned good if it sells
P35.50?
a. P23.30
b. P22.50
c. P26.30
d. P27.50
Solution:
Given: markup rate – 35% or 0.35
Selling price – 35.50
𝑆 = 𝐶(𝑚 + 1)
35.50 = 𝐶(0.35 + 1)
35.50 = 1.35𝐶

110
35.50
=𝐶
1.35
𝐶 = 26. 296 ≈ 26.30
Therefore, the cost for a canned good is P26.30.

7. A cellular phone retailer uses a markup rate of 44%. What is the selling price of a cellular
phone that cost the retailer P3, 560?
a. P5126.60
b. P5126.40
c. P1566.40
d. P2472.22
Solution:
Given: markup rate – 0.44
Cost – 3560
𝑆 = 3560(0.44 + 1)
𝑆 = 3560(1.44)
𝑆 = 2 126.40
Therefore, the selling price of a cellular phone is P5 126.40.

8. Teddy got a 15% discount when he bought a coat and tie. If the coat and tie originally cost
P1680.50, how much discount did Teddy get?
a. P1120.33
b. P252.06
c. P112.03
c. P2520.75
Solution:
Given: discount rate – 15% or 0.15
Regular Price – 1 680.50
𝐷 =𝑑∙𝑅
𝐷 = 0.15(1680.5)
𝐷 = 252.08
Therefore, Teddy got a discount of P252.08.

9. Find the discount rate of a pair of shoes if its sale price is twenty pesos less than the regular
price which is equivalent to P 560.90.
a. 0.03%

111
b. 3.03%
c. 3.57%
d. 35.7%
Solution:
Given: Regular Price – 560.9
Sale Price – 560.9-20=540.9
𝑆 = 𝑅(1 − 𝑑)
540.9 = 560.9(1 − 𝑑)
540.9 = 560.9 − 560.9𝑑
560.9𝑑 = 560.9 − 540.9
20
𝑑=
560.9

𝑑 ≈ 0.0357 𝑜𝑟 3.57%
Therefore, the discount rate for a pair of shoes is 3.5%.

10. The sale price of a dining set is P15, 000 which is 18% off the original price. How much is
the discount?
a. P3 292.68
b. P18 292.68
c. P8 297.68
c. P13 292.68
Solution:
Given: Sale Price – 15 000
Discount rate – 18% or 0.18
𝑆𝑃 = 𝑅(1 − 𝑑)
15000 = 𝑅(1 − 0.18)
15000 = 𝑅(0.82)
15000
𝑅=
0.82
𝑅 = 18 292.68
After getting the regular price, now, we solve for the discount.
𝑆𝑃 = 𝑅 − 𝐷
15000 = 18,292.68 − 𝐷
𝐷 = 18,292.68 − 15,000
𝐷 = 3,292.68
Therefore, the discount is P3,292.68.

112
5.7 Investment Problems
Investment word problems usually deal with the following terms:
 Interest – the amount of money received by an investor from an investment ( in
investment problems, annual simple interest is usually used)
 Principal – the amount of money invested excluding the interest earned.
 Rate – the percentage by which an investor expects his investment to increase
To solve for investment word problems, the formula below is used.
Interest (I) = Principal (P) x rate (r)
In this lesson, multiple investments are discussed. Hence, a table containing the rate, principal,
and interest must be created in order to identify the given, and find out what is asked in the
problem and eventually formulate an equation which will be used in looking for the missing
information.
Examples:
(a) Ria deposited a certain amount of money in the bank with a 9% annual simple interest
and deposited in another bank P5,000 more than the first with a 7% annual simple interest.
If the total interest earned on both investments for a year is P8,560, how much was
deposited in each bank?
Solution:
Let: x – amount invested in the first bank
(x + 500) – amount invested in the second bank
P r I
Bank 1 x 0.09 0.09x
Bank 2 x + 5000 0.07 0.07(x + 5000)
Total x + x + 5000 ----- 8,560
amount of interest earned from Bank 1 + amount of interest earned from Bank2 = P8,560
0.09x + 0.07(x + 500) = 8560
0.09x + 0.07x + 350 = 8560
0.16x = 8560 – 350
8210
x =
0.16.

x = 51,312.50
x + 5000 = 56,312.50
Therefore, P51, 312.50 is invested in the first bank and P56,312.50 is invested in the second
bank.
(b) Mrs. De la Fuentes received here Christmas bonus of P15,000 and invested it in stocks
paying 9% annual simple interest and in bonds paying 12% annual simple interest. If she
receives P1,650 after a year, how much was invested at each rate?
Solution:

113
Let: x – amount invested in stocks
(15,000 – x) – amount invested in bonds
P r I
Stocks x 0.09 0.09x
Bonds 15000 - x 0.12 0.12(1500 – x)
Total 15,000 ----- 1,650
amount of interest in stocks + amount of interest in bonds = P1,650
0.09x + 0.12(1500 – x) = 1650
0.09x + 1800 – 0.12x = 1650
-0.03x = 1650 – 1800
−150
x =
−0.03

x = 5000
1500 – x = 10000
Therefore, at 9% interest rate, Mrs. De la Fuentes receives an interest f P5,000 and she receives
P10,000 from the 12% interest rate.

(c) A manager invested his money in a trust bank paying 10% annual simple interest and
invested P3,000 less than twice his first investment in stocks paying 6% annual simple
interest. if the interest

0.10x = 0.06(2x – 3000)


0.10x = 0.12x – 180
0.10x – 0.12x = -180
-0.02x = -180
−180
x =
−0.02

x = 9000
2x – 3000 = 2(9000) – 3000
2x – 3000 = 15,000

P r I
Stocks 2x – 3000 0.06 0.06(2x – 3000)
Bonds x 0.10 0.10x
Total 2x – 3000 + x ----- 0.06(2x – 3000) = 0.10x
amount of interest in the trust bank = amount of interest in stocks

114
0.10x = 0.06(2x – 3000)
0.10x = 0.12x – 180
0.10x – 0.12x = -180
-0.02x = -180
−180
x =
−0.02

x = 9000
2x – 3000 = 2(9000) – 3000
2x – 3000 = 15,000
Therefore, the manager invested P15,000 in stocks.

Lesson Exercise:
1. A financial consultant deposited her commissions in two banks. The firs bank pays 8% annual
simple interest and the other bank pays 6.5% annual simple interest. If the financial consultant
invested P15, 000 more on the second bank and collected a total of P8,341 interest after a year,
how much was invested in the bank paying 6.5% interest?
a. P50, 800
b. P65, 800
c. P66, 400
d. P67, 800
Solution:
Let: x – Amount invested in the first bank
(x + 15, 000) – Amount invested in the second bank

P r I
Bank1 x 0.08 0.08x
Bank2 x + 15000 0.065 (x + 15000) 0.065(x + 15000)
Total x + x + 15000 ---- 8, 341

Amount of interest earned from Bank1 + amount of interest earned from Brank 2 = P8, 341
0.08x + 0.065(x + 15000) = 8341
0.08x + 0.065x + 975 = 8341
0.145x = 8341 – 975
7366
x =
0.145

x = 50, 800
x = 65, 800

115
Therefore, P65, 800 is invested in the second bank.

2.You have P50, 000 to invest in two investment banks. The first investment bank yields 7%
interest. You aim to receive P5, 000 interest income after a year for your tuition payment. How
much money should be invested in the second bank?
a. P25, 000
b. P28, 000
c. P30, 000
d. P32, 000
Solution:
Let: x – the amount invested in the first investment bank
(50, 000 – x) – the amount invested in the second investment bank

P r I
Investment Bank 1 x 0.13 0.13x
Investment Bank2 50,000 - x 0.07 0.07(50,000 – x)
Total 50,000 ---- 5,000
Amount of interest earned from Investment Bank 1 + amount of interest earned from Investment
Bank 2 = P5, 000
0.13x + 0.07(50,000 – x) = 5000
0.13x + 3500 – 0.7x = 5000
0.06x = 5000 – 3500
1500
x =
0.06

x = 25, 000
50,000 – x = 25, 000
Therefore, the amount invested in the two banks is equal and thus, P25, 000 is invested in the
second bank.

3.A business man invests a certain amount of money in bonds paying 11.5% interest and invests
thrice his first investment in bank accounts paying 8.5% interest. If he receives an annual income
of P7, 400, how much did he invest in bank accounts?
a. P20, 000
b. P25, 000
c. P35, 000
d. P60, 000
Solution:

116
Let: x – the amount invested in bonds.
3x – the amount invested in bank accounts
P r I
Bonds x 0.115 0.115x
Bank Accounts 3x 0.085 0.085(3x)
Total x + 3x ---- 7, 400
Amount of interest earned from bonds + amount of interest earned from bank accounts = P7, 400
0.115x + 0.085(3x) = 7, 400
0.115x + 0.255x = 7, 400
0.37x = 7, 400
7400
x =
0.37

x = 20, 000
3x = 60, 000
Therefore, the business man invested P60, 000 in bank accounts.
4. Bryan invested in stocks paying a 6.8% dividend while Mindy invested P4000 more
than Byran in tax-free bonds paying 7.2% annual simple interest. If Mindy’s income
from investment is P1, 200 more than Bryan’s, how much money did Mindy invest?
a. P228, 000
b. P235, 000
c. P232, 000
d. P218, 000
Solution:
Let x: x – the amount invested in stocks
(4000 + x) – the amount invested in bonds
P r I
Bryan (Stocks) x 0.068 0.068x
Mindy (Bonds) X + 4000 0.072 0.072(4000 + x)
Total x + x + 4000 ---- 0.072(4000 + x) = 0.068x + 1200
Solution:
Let:
x – the amount of interest earned by Bryan + 1, 200 = amount of interest earned by Mindy
0.072(4000 + x) = 0.068x + 1200
288 + 0.072x = 0.068x + 1200
0.072x – 0.068x = 1200 – 288
0.004x = 912
912
x =
0.004

117
x = 228, 000
4000 + x = 232, 000
Therefore, Mindy invested P232, 000 in bonds.
5. Carissa invested P3, 000 more in stocks than in bonds. The bonds paid 10.5% interest
and stocks paid 8%. If the income from each investment was the same, how much
interest did she receive from bonds?
a. P9, 600
b. P1, 008
c. P7, 250
d. P5, 340
Solution:
Let: x – the amount invested in bonds
(3000 – x) – the amount invested in stocks
P r I
Stocks 300 + x 0.08 0.08(3000 + x)
Bonds x 0.105 0.105x
Total x + x + 3000 ---- 0.08(3000 + x) = 0.105x
amount of interest earned from stocks = amount of interest earned from bonds
0.08(3000 + x) = 0.105x
240 + 0.08x = 0.105x
240 = 0.105x – 0.08x
240 = 0.025x
240
x =
0.025

x = 9, 600
0.105x = 1, 008
Therefore, Carissa received P1, 008 from bonds.
6. Half of Trisha’s money is invested at 10% interest, one-fourth at 9%, and the remaining
money at 8%. If she earns an annual income of P1, 850 from her investments, how
much money did Trisha invest?
a. P15, 000
b. P18, 000
c. P20, 000
d. P25, 000
Solution:
Let:
x – the amount of money invested by Trisha to get the remaining money Trisha invested in 8%
interest rate, we have:
1 1 4𝑥−2𝑥−𝑥 𝑥
x- x- x= =
2 4 4 4

118
P r I
1 0.10 1
x 0.10( x)
2 2
1 0.09 1
x 0.09( x)
4 4
1 0.08 1
x 0.08( x)
4 4

Total x ---- 1, 850


sum of the interests of the three different investments = 1, 850
1 1 1
0.10 ( x) + 0.09( x) + 0.08( x) = 1, 850
2 4 4

0.05x + 0.225x + 0.02x = 1, 850


0.0925x = 1, 850
x = 20, 000
Therefore, Trisha invested P20, 000.
7. Sheila invested some of her money at 8.75% interest per year and half as much of her
money at 6.2%. How much money did she invest at the lesser interest rate if the total
annual interest was P4, 147.50?
a. P35, 000
b. P17, 500
c. P25, 500
d. P12, 250
Solution:
Let: x – the amount invested at a rate of 8.75%
1
x – the amount of money invested at a rate of 6.2%
2

P r I
x 0.0875 0.0875x
1 0.062 1
x 0.062( x)
2 2

Total 1 ----
X+ x
2

sum of the interests of the two different investments = 4, 147.50


1
0.0875x + 0.062( x) = 4, 147.5
2

0.087x + 0.031x = 4, 147.5


0.1185x = 4, 147.5
4,147.5
x =
0.1185

x = 35, 000
1
x = 17, 500
2

Therefore, Sheila invested P17, 500 at the lesser interest rate.

119
8. Carla invested P25, 000 in two different banks. Some of her was invested at 5%
interest rate and the rest at 11% interest rate. If in a year, the two investments yield
the same interest, how much interest did she receive from each bank?
a. P859.38
b. P1, 718.75
c. P3, 437.50
d. P17, 187.50
Solution:
Let: x – the amount of money invested at a rate of 15%
(25, 000 – x) – the amount of money invested at a rate of 12%

P r I
Bank 1 x 0.05 0.05x
Bank 2 2 5000 - x 0.11 0.11(25, 000 – x)
Total 25, 000 ---- 0.05x = 0.11(25, 000 – x)
amount of interest from Bank 1 = amount of interest from Bank 2
0.05x = 0.11(25, 000 – x)
0.05x = 2, 750 – 0.11x
0.05x + 0.11x = 2, 750
0.16x = 2, 750
2,750
x =
0.16

x = 17, 187.5
0.05x = 859.375 ≈ 859.38
Therefore, Carla received P859.38 out of her investments from each bank.
9. An investment of P8, 000 is made at an annual simple interest rate of 7%. How much
additional money must be invested at an annual simple interest rate of 10% so that
the total annual interest earned is 8% of the total investment?
a. P3, 550
b. P4, 000
c. P4, 500
d. P5, 000
Solution:
Let x be the additional investment made at 10% interest rate.
P r I
Bank 1 x 0.05 0.05x
Bank 2 2 5000 - x 0.11 0.11(25, 000 – x)
Total 25, 000 ---- 0.05x = 0.11(25, 000 – x)

120
amount of interest from 8, 000 investment + amount of interest from additional investment = 8%
of total investment
560 + 0.10x = 0.08(8000 + x)
560 + 0.10x = 640 – 0.08x
0.10x – 0.08x = 640 - 560
0.02x = 80
80
x =
0.02

x = 4, 000
Therefore, the additional investment made is P4, 000.
10. An investment of P15, 000 is made at an annual simple interest rate of 8.5%. How
much additional money must be invested at an annual simple interest rate of 13% so
that the total annual interest earned is one-tenth of the total investment?
a. P7, 000
b. P7, 500
c. P8, 500
d. P8, 850
Solution:
Let x be the additional investment made at 10% interest rate
P r I
15, 000 0.085 0.085(15,000) = 1, 275
x 0.13 0.13x
Total 15, 000 + x ---- 1
(15, 000 + x)
10
1
amount of interest from P15, 000 investment + amount of interest from additional investment =
10
of total investment
1
1, 275 + 0.13x = (15, 000 + x)
10

1, 275 + 0.13x = 1, 500 + 0.10x


0.13x – 0.10x = 1, 500 – 1, 275
0.03x = 225
225
x =
7,500

x = 7, 500
Therefore, the additional investment made is P7, 500.
5.8 Mixture Problems
Problems that deal with creating mixtures of two or more things and then identifying specific
quantities (such as percentage, weight, price, etc.) of the resulting mixture called mixture word
problems.
There are two kinds of mixture problems:

121
(a) Value mixture problems
- Deals with mixture problems that involve mixing ingredients with different prices into
one mixture
- uses the formula V = A x C where V is the value of the ingredient, A is the amount of
the mixture and C is the cost per unit of the mixture
(b) Percent mixture problems
-Deals with mixture problems that involve percentages of solutions such as chemicals,
alcohols, etc.
-Uses the formula Q = P x A where Q is the quantity of the substance in the solution, P is
the percent of concentration of the substance, and A is the amount of the solution.
Examples:
(a) A food store sells a mixture of cashew nuts and roasted nuts. Cashew nuts sell for
P200/kg and roasted nuts sell for P120/kg. How many kilograms of each should be
mixed to make 14kg of this snack worth P130/kg?
Solution:
The problem asks for the number of kilograms of cashew nuts and the number of kilograms
of roasted nuts.
Let x be the number of kilograms of raisins.
Then 14 – x is the number of kilograms of nuts.
Number of Kilograms Cost Value
Cashew nuts x P 200 200x
Roasted nuts 14 –x P 120 120(14 – x)
Total 14 P130 14(130) = 1, 820
value of cashews + value of nuts = total value of mixture
200x + 120(14 – x) = 1, 820
200x + 1, 680 – 120x = 1, 820
80x = 1, 820 – 1, 680
80x = 140
140
80x =
80

x = 1.75
14 – x = 12.25
Therefore, 1.75kg of cashew and 12.25kg of roasted nuts are needed.
(b) A health food store sells a mixture of raisins and roasted nuts. Raisins sell for $3.50/kg
and nuts sell for $4.75/kg. How many kilograms of each should be mixed to make 20kg of
this snack worth $4.00/kg
The problem asks for the number of kilograms of raisins and the number of kilograms of nuts.
Let 𝑥 be the number of kilograms of raisins.
Then 20 − 𝑥 is the number of kilograms of nuts.

122
Number of kg × Price per kg = Cost
Raisins 𝑥 3.50 3.5𝑥
Nuts 20 − 𝑥 4.75 4.75(20 − 𝑥)
Total 20 4.00 80
Cost of raisins + cost of nuts = total cost of mixture
3.5𝑥 + 4.75(20 − 𝑥) = 80
350𝑥 + 475(20 − 𝑥) = 8000
350𝑥 + 9500 − 475𝑥 = 8000
−125𝑥 = −1500
𝑥 = 12
20 − 𝑥 = 8
∴ 12 kg of raisins and 8 kg of nuts are needed.
(c) A laboratory assistant was tasked to mix 30mL of a solution that is 60% acid with 45m
solution that is 40% acid. What is the percent of the mixture?
Solution:
The problem asks for the percent of acid of the mixture.
Let x be the percent of acid of the mixture.
Number of milliliters Acid Percentage Amount of Acid
60% Solution 30 60% = 0.6 30(0.6) = 18
40% Solution 45 45% = 0.4 45(0.4) = 18
Total 75 x 75x
Amount of acid in the 60% solution + amount of acid in the 40% solution = total amount
of acid
18 + 18 = 75x
36 = 75x
36
x =
75
x = 0.48 or 48%
Therefore, the percent of acid of the mixture is 48%.
(d) If you were to mix a 600mL juice drink that is 10% orange juice, how much orange juice
should be added to make a drink that is 25% orange juice?
Solution:
The problem asks for the amount of orange juice added to the 10% orange juice drink.
Let x be the amount of orange juice added.
Amount of Juice Orange Juice Amount of Orange
Drink (in mL) Percentage Concentrate (in mL)
Original Juice 600 10% = 0.10 600(0.10) = 60
Drink
Orange Juice x 100% = 1 x(1) = x
Added

123
New Juice Drink 600 + x 25% = 0.25 60 + x
amount of new juice drink orange juice % = amount of orange concentrate
(600 + x) (0.25) = 60 + x
150 + 0.25x = 60 + x
150 – 60 = x – 0.25x
90 = 0.75x
90
x =
0.75
x = 120
Therefore, 120 mL of orange juice should be added to make a drink that is 25% orange
juice.
Lesson Exercises:
1. An auto mechanic has 250mL of battery acid solution that contains 40% acid. The
mechanic needs to add water into the solution to dilute it so that it is only 25% acid. How
much water should be added?
a. 100mL
b. 150mL
c. 175mL
d. 200mL
Solution:
The problem asks for the amount of water to be added in the solution.
Let x be the amount of water to be added in the solution.
Amount (in mL) Acid Percentage Amount of Acid
Battery Acid Solution 250 40% = 0.40 250(0.40) = 100
Water x 0% x(0)
Total 250 + x 25% = 0.25 100 + 0 = 0
Note that the amount of acid in water is 0%.
Total amount of the two liquids acid percentage of the mixture = total amount of acid
0.25 (250 + x) = 100
62.50 + 0.25x = 100
0.25x = 100 – 62.50
37.50
x =
0.25

x = 150
Therefore, 150mL of water must be added to the batter acid solution.

2. A candy shop mixes different sweets in order to create unique snacks for the customers.
If the owner mixes mallows that sell for P100/kg and chocolate sprinkles that sells for
P80/kg, how many kilograms of mallows should be mixed to get a 5kg snack worth
P90/kg?
a. 1.75kg
b. 1.80kg

124
c. 2.20kg
d. 2.50kg
Solution:
The problem asks for the amount of mallows to be mixed.
Let x be the amount of mallows in the snack.
Number of Kilograms Cost Value
Mallows x P100 100x
Chocolate Sprinkles 5 -x P80 80(5-x)
Total 5 P90 5(90) = 450
value of mallows + value of chocolate sprinkles = total value of mixture
100x + 80(5 – x) = 450
100x + 400 – 80x = 450
20x = 450 – 400
50
x =
20

x = 2.5
Therefore, 2.5kg of mallows is mixed in the snack.

3. A mother makes a liter of juice drink that is 60% mango concentrate and mixes it with 2
liters of juice drink that 30% mango concentrate. What is the percentage of mango
concentrate in the new juice drink?
a. 40%
b. 45%
c. 50%
d. 90%
Solution:
The problem asks for the percentage of mango concentrate in the new juice drink.
Amount of Juice Mango Concentrate Amount of Mango
Drink (in Liters) Percentage Concentrate (in Liters
60% concentrate 1 60% = 0.60 1(0.60) = 0.60
30% concentrate 2 30% = 0.30 2(0.30) = 0.60
Total 3 x 0.60 + 0.60 = 1.20
total amount of juice drink mango juice% = amount of mango concentrate
3x = 1.20
1.20
x =
3

x = 0.40 or 40%
Therefore, the juice drink mixture contains 40% mango concentrate.

125
4. A chef creates a solution that contains 25% chicken broth and mixes it with a solution that
contains 70% chicken broth. If he made a mixture that is a total of 120 liters containing
38% chicken broth, how much of the 25% chicken broth solution should be mixed?
a. 80.5mL
2
b. 82
3
2
c. 83 mL
3
1
d. 85 mL
3

Solution:
The problem asks for the amount of 25% chicken broth solution.
Amount of Chicken Broth Amount of Chicken Broth in
Solution (in Liters) Percentage the Solution(in Liters)
25% chicken broth x 25% = 0.25 x(0.25) = 0.25x
70% chicken broth 120 – x 70% = 0.70 (120 – x)0.70
Total 120 38% = 0.38 0.25x + (120 – x) 0.70
Total amount of solution chicken broth % = amount of chicken broth in the solution
120(0.38) = 0.25x + (120 – x) 0.70
45.6 = 0.25x + 84 – 0.70x
0.70x – 0.25x = 84 – 45.6
0.45x = 38.4
38.4
x =
0.45
1
x = 85
3
1
Therefore, 85 of the 25% chicken broth solution should be mixed.
3

5. Ten gallons of regular gasoline is added to 15 gallons of premium gasoline and now costs
P60.50 per gallon. If premium gasoline costs P2.50 more than the regular gasoline, find
the price per gallon for the premium gasoline.
a. P59.00
b. P59.50
c. P61.50
d. P62.00
Solution:
The problem asks for the price per gallon of the premium gasoline.
Let x be the price of the regular gasoline and x + 2.50 be the price of the premium gasoline.
Amount of
Price per Liter Value
Gasoline (in Liters)
Regular Gasoline 10 x 10x
Premium Gasoline 15 x + 2.50 15(x + 2.50)
Total 25 60.50 10x + 15(x + 2.50)

126
Total amount of gasoline price per liter = value
25(60.50) = 10x + 15(x + 2.50)
1,512.50 = 10x + 15x + 37.5
1,512.50 – 37.50 = 25x
1,475 = 25x
1,475
x =
25

x = 59
x + 2.50 = 61.50
Therefore, premium gasoline is sold at P61.50/liter.
6. Pure silver that is 40% alloy is to be mixed with pure silver that is 60% alloy to obtain 20lbs
of silver that contains 44% alloy. How many pounds of pure silver which is 40% allow must
be mixed?
a. 14lbs
b. 16lbs
c. 17lbs
d. 18lbs
Solution:
The problem asks for the number of pounds of pure silver which is 40% alloy.
Let x be the price of the regular gasoline and x + 2.50 be the price of the premium gasoline.
Amount of Silver (in lbs) Alloy Percentage Amount of alloy
40% alloy x 40% = 0.40 0.40x
60% alloy 20 - x 60% = 0.60 0.60(20 – x)
Total 20 44% = 0.44 0.40x + 0.60(20 – x)
Total amount of pure silver alloy percentage = total amount of alloy
20(0.44) = 0.40x + 0.60(20 – x)
8.8 = 0.40x – 12 – 0.60x
8.8 - 12 = -0.20x
-3.20 = -0.20x
−3.20
x =
−0.20

x = 16
Therefore, 16lbs of pure silver containing 40% alloy should be mixed.

7. In a debut the waiter was tasked to mix 5 gallons of Brand X fruit punch and 3 gallons of
Brand Y fruit punch. If brand X has 35% fruit juice while Brand Y has 65% fruit juice, what
percent of the mixture is fruit juice?
a. 46.25%
b. 48.75%

127
c. 56.50%
d. 100%
Solution:
The problem asks for the percent of mixture of the fruit juice.
Let x be the percentage of fruit juice of the mixture.
Amount of Fruit Fruit Juice Amount of Fruit juice in
punch (in galloons) Percentage the Punch (in gallons)
Brand X Fruit Punch 5 35% = 0.35 5(0.35) = 1.75
Brand Y Fruit Punch 3 65% = 0.65 3(0.65) = 1.95
Total 8 x 1.75 + 1.95
total amount of fruit punch fruit juice percentage = total amount of fruit juice in the mixture
8x = 1.75 + 1.95
8x = 3.7
3.7
x =
8

x = 0.4625
x = 46.25%
Therefore, 46.25% of the mixture is fruit juice.

8. A 30mg alloy containing 40% nickel is melted and mixed with another alloy weighing 12mg
containing 35% nickel. How many percent of nickel is the resulting alloy?
a. 75%
b. 49.36%
c. 33.79%
d. 38.57%
Solution:
The problem asks for the percent of nickel in the resulting alloy.
Let x be the nickel percentage of the resulting alloy.
Amount of Alloy (in mg) Nickel Percentage Amount of Nickel
Alloy 1 30 40% = 0.40 0.40(30) = 12
Alloy 2 12 35% = 0.35 0.35(12) = 4.2
Total 42 x 12 + 4.2
total amount of alloy nickel percentage = total amount of nickel
42x = 12 + 4.2
42x = 16.2
16.2
x =
42

x ≈ 0.3857

128
x ≈ 38.57%
Therefore, the resulting alloy contains 38.57% nickel.

9. A hundred milliliters of a 10% sugar solution is mixed with 150mL of a 58% sugar solution.
Find the concentration of the new mixture.
a. 29.2%
b. 38.8%
c. 43.5%
d. 68%
Solution:
The problem asks for concentration of the new mixture.
Let x be the percentage of sugar in the new mixture.
Amount of Sugar
Sugar Percentage Amount of Nickel
Solution (in mL)
10% sugar solution 100 10% = 0.10 0.10(100) = 10
58% sugar solution 150 58% = 0.58 0.58(150) = 87
Total 250 x 10 + 87
total amount of alloy nickel percentage = total amount of nickel
250x = 10 + 87
250x = 97
97
x =
250

x = 0.388
x = 38.8%
Therefore, the concentration of the new mixture is 38.8% sugar.

10. If 10kg of white sugar costing P60/kg is mixed with 15kg of brown sugar costing P55/kg,
how much is the cost of 8kg of the white and brown sugar mixture?
a. P57
b. P205
c. P315
d. P456
Solution:
The problem asks for the cost of 8kg of white and brown sugar mixture.
Let x be the cost of 8kg of white and brown sugar mixture.
Amount of Sugar (in kg) Price of Sugar Value
White sugar 10 60 10(60) = 60
Brown sugar 15 55 15(55) = 825

129
Total 25 x 600 + 825
total amount of sugar price of sugar mixture = value of the mixture\
25x = 600 + 825
25x = 1, 425
1,425
x =
25

x = 57
8x = 456
Therefore, 8kg of the brown and white sugar mixture costs P456.00.

5.9 Work Problems


Work requires the expenditure of time and effort. Some jobs take a shorter time to finish
than others.
Learning Objectives:
1. At the end of this lesson, students must be able to know the solution of some Work-
related word problems.
2. The student should be able to learn Work-related problems.
3. The student must be able to solve word problems involving work.
Lesson Proper:
The most important factor in work is the efficiency of each working unit. This refers to how
fast each worker can finish a given job. In more concrete terms, efficiency refers to the rate of
work done. This can be expressed as follows:
1
𝑊𝑜𝑟𝑘 𝑟𝑎𝑡𝑒 𝑜𝑓 𝑎𝑛 𝐼𝑛𝑑𝑖𝑣𝑖𝑑𝑢𝑎𝑙 =
𝑇𝑜𝑡𝑎𝑙 𝑇𝑖𝑚𝑒 𝑆𝑝𝑒𝑛𝑡 𝑡𝑜 𝐹𝑖𝑛𝑖𝑠ℎ 𝑡ℎ𝑒 𝐽𝑜𝑏
Moreover, we can determine the total time at which two or three individuals working together
at the same time can finish a given job. Under this arrangement, it is assumed that the rate of
work of each individual is known. Thus, if A can do a certain job alone in 3 hours and B can do
the same job alone in 4 hours, then the equation relating their work rates with the total time to
finish the job may be written as follows:
𝑊𝑜𝑟𝑘 𝑅𝑎𝑡𝑒 𝑜𝑓 𝐴 + 𝑊𝑜𝑟𝑘 𝑅𝑎𝑡𝑒 𝑜𝑓 𝐵 = 𝑊𝑜𝑟𝑘 𝑅𝑎𝑡𝑒 𝑜𝑓 𝐴 𝑎𝑛𝑑 𝐵 𝑇𝑜𝑔𝑒𝑡ℎ𝑒𝑟
1 1 1
+ =
𝑇𝑜𝑡𝑎𝑙 𝑡𝑖𝑚𝑒 𝐴 𝑐𝑎𝑛 𝑇𝑜𝑡𝑎𝑙 𝑡𝑖𝑚𝑒 𝐵 𝑐𝑎𝑛 𝑇𝑜𝑡𝑎𝑙 𝑡𝑖𝑚𝑒 𝑡ℎ𝑒 𝑗𝑜𝑏
𝑓𝑖𝑛𝑖𝑠ℎ 𝑡ℎ𝑒 𝑗𝑜𝑏 𝑎𝑙𝑜𝑛𝑒 𝑓𝑖𝑛𝑖𝑠ℎ 𝑡ℎ𝑒 𝑗𝑜𝑏 𝑎𝑙𝑜𝑛𝑒 𝑐𝑎𝑛 𝑏𝑒 𝑓𝑖𝑛𝑖𝑠ℎ𝑒𝑑 𝑏𝑦 𝐴 𝑎𝑛𝑑 𝐵 𝑡𝑜𝑔𝑒𝑡ℎ𝑒𝑟

Example:
1. Problem:
Mae can finish a project in 12 hours and Abi in 6 hours alone. How long will it take them
to finish the project together?
Solution:

130
Let 𝑥 = number of hours that will take Mae and Abi to finish the project together. Since
1
Mae can finish the project in 12 hours, he can do of the project in 1 hour. Similarly, Abi
12
1
can do of the project 1 hour. Then, if 𝑥 is the number of hours it will take both of them to
6
1
finish the project, they can do of the project in one hour. Thus, we have the equation
𝑥
1 1 1
+ =
12 6 𝑥

Multiplying both members by 36𝑥, the LCD, we have


1 1 1
36𝑥 ( + = ) which when simplified gives
12 6 𝑥

3𝑥 + 6𝑥 = 36
Combining similar terms and performing the indicated operations, we find
9𝑥 = 36
36
𝑥=
9

𝑥=4
Thus, Mae and Abi can finish the project in 4 hours.

2. Problem:
Justin can paint a house in two hours, and Aaron can paint the same house in 8 hours.
How long does it take for them to do it together?
Solution:
Let 𝑥 = number of hours that will take Justin and Aaron to finish painting together. Since
1
Justin can paint a house in 2 hours, he can do of the project in 1 hour. Likewise, Aaron can
2
1
do of the project 1 hour. Then, if 𝑥 is the number of hours it will take both of them to finish
8
1
the project, they can do of the project in one hour. Thus, we have the equation
𝑥
1 1 1
+ =
2 8 𝑥

Multiplying both members by 8𝑥, the LCD, we have


1 1 1
8𝑥 ( + = ) which when simplified gives
2 8 𝑥

4𝑥 + 𝑥 = 8
Combining similar terms and performing the indicated operations, we find
5𝑥 = 8
8
𝑥=
5
8
Therefore, Justin and Aaron can paint a house together in hours.
5

131
Lesson Exercise:
Solve the following work problems.

1. Arvin can finish building a fence for a vegetable garden in 6 hours and Mark can do the
same work in 8 hours. In how many hours can they finish building the fence if they will
work together?
3
E. 3 hours
9
5
F. 3 hours
7
3
G. 3 hours
7
3
H. 4 hours
7

Correct answer: C
Solution:
Let 𝑥 = number of hours they can finish building the fence together
1
Since Arvin can finish building the fence in 6 hours, he can do of the project in 1
6
1
hour. Also, Mark can do of the project in 1 hour. Then, if 𝑥 is the number of hours it will
8
1
take both of them to finish building the fence, they can do of the project in one hour.
𝑥
Thus, we have the equation
1 1 1
+ =
6 8 𝑥

Multiplying both members by 24𝑥, the LCD, we have


1 1 1
24𝑥 ( + = ) which when simplified gives
6 8 𝑥

4𝑥 + 3𝑥 = 24
7𝑥 = 24
3
𝑥=3
7
3
Therefore, Arvin and Mark can finish building the fence in 3 hours.
7

2. Jacob can finish typing the document in 8 hours and Michael can do the same work in 3
hours. If they will work together, how long will it take them to finish typing the document?
2
E. 2 hours
11
5
F. 2 hours
7
5
G. 4 hours
11
5
H. 7 hours
7

Correct answer: A
Solution:
Let 𝑥 = number of hours it will take them to finish typing together

132
1
Since Jacob can finish typing in 8 hours, he can do of the project in 1 hour. Also,
8
1
Michael can do of the project in 1 hour. Then, if 𝑥 is the number of hours it will take
3
1
both of them to finish typing, they can do of the project in one hour. Thus, we have the
𝑥
equation
1 1 1
+ =
8 3 𝑥

Multiplying both members by 24𝑥, the LCD, we have


1 1 1
24𝑥 ( + = ) which when simplified gives
8 3 𝑥

3𝑥 + 8𝑥 = 24
11𝑥 = 24
2
𝑥=2
11
2
Therefore, Jacob and Michael can finish typing the document in 2 hours.
11

3. Joshua, Matthew, and Daniel have decided to spray their mango farm with insecticide to
remove the pests from the farm. Working alone, Joshua needs 7 hours to spray the
farm, Matthew needs 6 hours, and Daniel needs 3 hours. How long will it take them to
spray their mango farm if they will work together?
12
E. 1 hours
14
11
F. 1 hours
13
11
G. 2 hours
13
18
H. 2 hours
20

Correct answer: B
Solution:
Let 𝑥 = number of hours it will them to spray the cornfield together
1
Since Joshua can finish spraying the farm in 7 hours, he can do of the project in 1
7
1 1
hour. Also, Matthew can do and Daniel can do of the project in 1 hour. Then, if 𝑥 is
6 3
1
the number of hours it will take them to finish spraying the farm together, they can do
𝑥
of the project in one hour. Thus, we have the equation
1 1 1 1
+ + =
7 6 3 𝑥

Multiplying both members by 24𝑥, the LCD, we have


1 1 1 1
42𝑥 ( + + = ) which when simplified gives
7 6 3 𝑥

6𝑥 + 7𝑥 + 14𝑥 = 42
27𝑥 = 42
15
𝑥=1
27

133
Therefore, Joshua, Matthew, and Daniel can finish spraying their mango farm
15
together in 1 hours.
27

4. Chris can trim the grass in a garden in 6 hours. Ethan can do it in 5 hours. How long will
it take them to trim the grass if they work together?
7
E. 3 hours
12
11
F. 3 hours
12
12
G. 2 hours
15
11
H. 2 hours
12

Correct answer: D
Solution:
Let 𝑥 = number of hours they can finish trimming the grass together
1
Since Chris can finish weeding a garden in 6 hours, he can do of the project in 1
6
1
hour. Also, Ethan can do trimming in of an hour. Then, if 𝑥 is the number of hours it will
5
1
take both of them to finish trimming the grass, they can trim the grass in of an hour.
𝑥
Thus, we have the equation
1 1 1
+ =
6 5 𝑥

Multiplying both members by 30𝑥, the LCD, we have


1 1 1
30𝑥 ( + = ) which when simplified gives
6 5 𝑥

5𝑥 + 6𝑥 = 30
11𝑥 = 30
8
𝑥=2
11
8
Therefore, Chris and Ethan can together trim the grass in just 2 hours.
11

5. Joseph can paint the whole house in ten hours, and William in nine hours. How long
would it take the two of them to paint the entire house?
E. 4 hours
24
F. hours
5
24
G. hours
7
15
H. hours
5

Correct answer: B
Solution:
Let 𝑥 = number of hours it takes to them to paint the house together

134
1
Since Joseph can paint the entire house in 10 hours, he can do of the job per
10
1
hour. And William can do per hour. Then, if 𝑥 is the number of hours it will take both of
9
1
them to paint the whole house, they can paint the house in of an hour. Thus, we have
𝑥
the equation
1 1 1
+ =
10 9 𝑥

Simplifying the equation, we have


21 1
= which when simplified gives
90 𝑥
90
𝑥=
21
90
Therefore, they can complete their work together in just hours.
21

6. The first tube can fill a pool 2.25 times faster than the second tube. When both tubes are
opened, they fill the pool in five hours. How long would it take to fill the pool if only the
first tube is used?
e. 16.25 hrs
f. 11.5 hrs
g. 9 hrs
h. 11.25 hrs
Correct answer: A
Solution:
Let 𝑥 = hours to complete the job if the second tube is used
2.25𝑥 = hours to complete the job if the first tube is used
Completed per hour:
1
second tube:
𝑥
1
first tube:
2.25𝑥
1
together:
5

Adding their labor:


1 1 1
. + =
2.25𝑥 𝑥 5

Simplifying the equation:


3.25 1
=
2.25𝑥 5

2.25𝑥 = 16.25
𝑥 ≈ 7.22
Then, 2.25𝑥 = 16.25, therefore the first tube takes 16.25 hours.
7. If five girls can do a job in fifteen days, how many would it take to do the job in twenty
days?

135
E. 6
F. 5
G. 10
H. 3
Correct answer: D
Solution:
Convert this to man-days. If it takes five girls to do a job in twenty-five days, then:
(5 𝑔𝑖𝑟𝑙𝑠) × (15 𝑑𝑎𝑦𝑠) = 75 𝑚𝑎𝑛 − 𝑑𝑎𝑦𝑠
That is, the entire job requires 75 man-days.
(𝑥 𝑔𝑖𝑟𝑙𝑠) × (25 𝑑𝑎𝑦𝑠) = 75 𝑚𝑎𝑛 − 𝑑𝑎𝑦𝑠
Simplifying the equation,
25𝑥 = 75
𝑥=3
Therefore, 3 girls are needed to do the job in twenty-five days.
8. Anthony can finish a job in 5 hours. When working at the same time as David, they can
finish the job in 4 hours. How long does it take for David to finish the job if he works alone?
E. 2 hours
F. 10 hours
G. 20 hours
H. 5 hours
Correct answer: C
Solution:
Let 𝑥 = number of hours Anthony needs to finish the job
𝑦 = number of hours David needs to finish the job
We already know that 𝑥is equal to 5. When working at the same time, they
need 4 hours. Using the formula,
1
1 1
𝑡𝑖𝑚𝑒=𝑥+𝑦

1
1 1
4=5+𝑦

Rearranging the terms, we get:


1 1 1
+ =
5 𝑦 4
1 1 1
= −
𝑦 4 5
1 1
=
𝑦 20

𝑦 = 20
Therefore, David needs 20 hours to complete the job alone.

136
9. A barrel can be filled by tube A in 2 hours and by tube B in 4 hours. When the barrel is
full, it can be drained by tube C in 3 hours. If the barrel is initially empty and all three tubes
are open, how many hours will it take to fill up the barrel?
2
E. 2 hours
5
7
F. 2 hours
16
10
G. 8 hours
20
3
H. 7 hours
12

Correct answer: A
Solution:
Let 𝑥 = time taken to fill up the barrel
Since tube C drains the water, it is subtracted.
1 1 1 1
+ − =
2 4 3 𝑥

Multiplying both sides with 12


1 1 1 1
+ − =
2 4 3 𝑥
12
6+3−4=
𝑥
12
5=
𝑥

5𝑥 = 12
12 2
𝑥= =2
5 5
2
Therefore, the time taken to fill the barrel is 2 hours.
5

10. Alex can cultivate plants alone in 1 hour. It takes Nick 3 hours to cultivate the same
plants. If they work together, how long will it take them to cultivate the plants together?
3
E. hour
10
3
F. hour
4
1
G. hour
2
2
H. hour
7

Correct answer: B
Solution:
Let 𝑥 = time it takes to cultivate the plants together
Solving the problem,
1 1 1
+ =
1 3 𝑥

Finding a common denominator


3 1 1
+ =
3 3 𝑥

137
4 1
=
3 𝑥

Cross multiplying
4𝑥 = 3
3
𝑥=
4
3
Therefore, it will take them of an hour working together.
4

11. It takes Sophy 7 hours to clean her room if she works alone and it takes Issa 5 hours to
clean the same room if she works alone. How long will it take them to clean the room if they
work together?
2
E. 2 hours
5
2
F. 2 hours
7
11
G. 2 hours
12
3
H. 3 hours
5

Correct answer: C
Solution:
Let 𝑥 = the amount of time it takes to clean the room (in hours) if the two work
together.
Sophy would need 7 hours if she did the entire job by herself, so her
1 1
working rate is of the job in an hour. Likewise, Issa’s rate is of the job in an hour.
7 5
1 1 1
+ =
7 5 𝑥

Solving the equation, we have


5𝑥 + 7𝑥 = 35
12𝑥 = 35
35 11
𝑥= =2
12 12
11
Therefore, the two females can clean the room in only 2 hours together.
12

12. Diego can dig a 20 ft by 20 ft den in six hours working alone. Max can dig the same den
in seven hours. How long would it take them if they worked together?
a. 5.90 hours
b. 4.12 hours
c. 2.45 hours
d. 3.23 hours
Correct answer: D
Solution:
Let 𝑥 = the amount of time it takes them if they worked together.

138
1
Diego would need 6 hrs to dig a den, so his working rate is of the job in an hour.
6
1
Likewise, Max can dig a den in 7 hrs, his rate is of the job in an hour.
7
1 1 1
+ =
6 7 𝑥

Solving the equation, we have


7𝑥 + 6𝑥 = 42
13𝑥 = 42
42
𝑥= ≈ 3.23
13

Therefore, it takes 3.23 hours for them to dig the den together.
13. It takes Luis nine hours to clean the loft. Ian can clean the same loft in six hours. Find
how long it would take them if they worked together.
a. 3.6 hours
b. 2.40 hours
c. 4.3 hours
d. 3.12 hours
Correct answer: A
Solution:
Let 𝑥 = the amount of time it would take them if they worked together.
1
Luis would need 9 hrs to clean an attic, so his working rate is of the job in an
9
1
hour. Likewise, Ian can also clean the attic in 6 hrs, his rate is of the job in an hour.
6
1 1 1
+ =
9 6 𝑥

Solving the equation, we have


6𝑥 + 9𝑥 = 54
15𝑥 = 54
54
𝑥= = 3.6
15

Therefore, it takes 3.6 hours for them to clean the loft together.
14. Anna can pour a large concrete driveway in two hours. Owen can pour the same
driveway in four hours. Find how long it would take them if they worked together.
a. 1.12 hours
b. 1.33 hours
c. 8.73 hours
d. 1.21 hours
Correct answer: B
Solution:
Let 𝑥 = the amount of time it takes them if they worked together.

139
Anna would need 2 hrs to pour a large concrete driveway, so her
1
working rate is of the job in an hour. Likewise, Owen can do the job in 4 hrs,
2
1
his rate is of the job in an hour.
4
1 1 1
+ =
2 4 𝑥

Solving the equation, we have


2𝑥 + 𝑥 = 4
3𝑥 = 4
4
𝑥 = ≈ 1.33
3

Therefore, it takes 1.33 hours for them to pour a large concrete driveway
together.
15. Working alone, Thomas can change car’s oil in five hours. Kevin can change oil of the
same car in nine hours. If they worked together how long would it take them?
a. 3.21 hours
b. 9.8 hours
c. 8.54 hours
d. 11.21 hours
Correct answer: A
Solution:
Let 𝑥 = the amount of time it takes them if they worked together.
1
Thomas would need 5 hrs to change car’s oil, so his working rate is of the job in
5
1
an hour. Likewise, Kevin can oil the same car in 9 hrs, his rate is of the job in an hour.
9
1 1 1
+ =
5 9 𝑥

Solving the equation, we have


9𝑥 + 5𝑥 = 45
14𝑥 = 45
45
𝑥= = 3.21
14

Therefore, it takes 3.21 hours for them change car’s oil together.
16. Working alone, Eli can sweep a terrace in 30 minutes. Joe can sweep the same terrace
in 20 minutes. If they worked together how long would it take them?
a. 36.35 mins
b. 1.8 mins
c. 12.05 mins
d. 8.35 mins
Correct answer: C
Solution:
Let 𝑥 = the amount of time it takes them if they worked together.

140
1
Eli would need 30 mins to sweep a terrace, so his working rate is of the job in
30
1
every minute. Likewise, Joe can sweep the same terrace in 20 mins, his rate is of the
20
job in every minute.
1 1 1
+ =
30 20 𝑥

Solving the equation, we have


1
= 0.083
𝑥

0.083𝑥 = 1
𝑥 ≈ 12.05
Therefore, it takes 12.05 minutes for them to sweep a terrace together.
17. Caleb can tar a wall in 8 hours. One day his friend Hanna helped him and it only took 4
hours. How long would it take Hanna to do it alone?
a. 10.01 hours
b. 9 hrs
c. 6.5 hrs
d. 9.31 hours
Correct answer: A
Solution:
Let 𝑥 = the amount of time it takes Hanna to do it alone
1
Caleb can tar a wall in 8 hrs, so his working rate is of the job in an hour. With
8
1
Hanna help, it only took 4 hrs. So their working rate is of the job in an hour.
4
1 1 1
+ =
8 𝑥 4

Solving the equation, we have


4𝑥 + 32 = 8𝑥
4𝑥 = 32
𝑥=8
Therefore, it takes 8 hrs too for Hanna to tar a wall alone.
18. Working alone, it takes Belle 11 hours to harvest the fruits. Tara can harvest the same
fruits in 16 hours. Find how long it would take them if they worked together.
a. 18 hours
b. 9.65 hrs
c. 4.35 hrs
d. 6.52 hours
Correct answer: D
Solution:
Let 𝑥 = the amount of time it takes them to finish the work together

141
1
Belle would need11 hrs to harvest the fruits, so her working rate is of the job in an
11
1
hour. Likewise, Tara can harvest the same fruits in 16 hrs, her rate is of the job in an
16
hour.
1 1 1
+ =
11 16 𝑥

Solving the equation, we have


16𝑥 + 11𝑥 = 176
27𝑥 = 176
176
𝑥= = 6.52
27

Therefore, it takes 6.52 hours for them to harvest the fruits together.
19. Chasey can input data into the computer system in 40 minutes, and Bran can input the
same data in 60 minutes. How long will it take both of them, to input the data together?
a. 24mins
b. 12mins
c. 45mins
d. 32mins
Correct answer: A
Solution:
Let 𝑡 = the amount of time it takes Hanna to do it alone
Chasey’s rate for doing the job is 1/40 of the job per minute. Bran’s rate is 1/60 of
the job per minute. Then the sum of the work that Chasey does and the work that Bran
does must equal one job:
1 1
( )𝑡 + ( )𝑡 = 1
40 60

Simplifying, we get:
60𝑡 + 40𝑡 = 2400
𝑡 = 24
Therefore, it takes 24mins for them to input the data.
20. Faith and Lucy can type the article in 8 hours. Faith can type the same article alone in 20
hours. How long would it take Lucy to type the article?
a. 5hrs & 20mins
b. 6hrs & 30mins
c. 11hrs & 25mins
d. 13hrs & 20mins
Correct answer: D
Solution:
Let 𝑟 = the rate that Lucy works
1
The rate that Faith works is of the job per hour. To do one job in 8 hours we
20
have

142
1
1 = (8) + 𝑟(8)
20
To solve for 𝑟, multiply by 20:
20 = 8 + 20𝑟(8)
12 = 8(20)𝑟
So,
12 3
𝑟= = of the job per hour.
8(20) 40
To type the entire article alone, Lucy takes
𝑤 1 40 1
𝑡= = = = 13
𝑟 3/40 3 3
1
Therefore, it takes Lucy 13 hr to type the article.
3

5.10 Uniform Motion Problems


Uniform Motion problems are problems that deal with objects that move at a fixed rate. To solve
for a uniform motion problem, we use the equation:

𝑑𝑖𝑠𝑡𝑎𝑛𝑐𝑒 (𝑑) = 𝑟𝑎𝑡𝑒 (𝑟) 𝑥 𝑡𝑖𝑚𝑒(𝑡)


Examples:
(a) Two swimmers leave at 6:00 am from two islands which are 1500 miles apart. If they
swam at speeds of 120mph and 130mph, at what time will the two swimmers meet?
Solution:
Let t be the time the two swimmers meet.
Rate Time Distance
Swimmer 1 120 mph t 120t
Swimmer 2 130mph t 130t
Total ------ ----- 1500
distance covered by Swimmer 1 + distance covered by Swimmer 2 = 1500
120t + 130t = 1500
250t = 1500
1500
t =
250

t = 6 hours
Therefore, after six hours, the two swimmers meet each other.

(b) Michelle walked from home towards the park at a rate of 10kph. Going back home, she
jogged at a rate of 15kph following the same route. If the trip from home to park then
back home took 5 hours, how long was Michelle’s house from the park?
Solution:

143
Let t be the time Michelle spent in walking from her home to the park.

Rate Time Distance


Walking 10 mph t 10t
Jogging 15mph 5-t 15(5 – t)
Total ------ ----- 10t = 15(5 – t)
distance Michelle walked = distance Michelle jogged
10t = 15(5 – t)
10t = 75 – 15t
10t + 15t = 75
25t = 75
75
t =
25

t = 3
10t = 30km
Therefore, Michele’s house is 30km away from the park.

(c) Jean traveled from her house toward the downtown area at an average speed of 60kph.
Her friend Sheena wanted to meet her there and left an hour after Jean left. Sheena
traveled a similar direction at a rate of 75kph. How long will it take Sheena to meet
Jean?
Solution:
Let t be the time it will take Sheen to meet Jean.
Let t + 1 be the time for Jean to reach the downtown area.
Rate Time Distance
Sheena 75kph t 75t
Jean 60kph t+1 60(t + 1)
Total ------ ----- 75t = 60(t + 1)
distance Jean traveled = distance Sheena traveled
75t = 60(t + 1)
75t = 60t + 60
75 – 60t = 60
15t = 60
60
t =
15

t = 4 hours

144
Therefore, it will take Sheena four hours to meet Jean.

(d) Two jets depart from an airport and travel in opposite directions. If the first jet flies at a
rate of 120 miles faster than the second jet, and after four hours they are 3040 miles
apart, what is the rate of the first jet?
Solution:
Let x be the rate of the second jet.
Let x + 120 be the rate of the first jet.
Rate Time Distance
First Jet x + 120 4 4(x + 120)
Second Jet x 4 4x
Total ------ ----- 3,040
distance of first jet + distance of second jet = total distance
4(x + 120) + 4x = 3,040
4x + 480 + 4x = 3,040
8x = 3,040 – 480
8x = 2,560
2,560
x =
8

x = 320 miles
x + 120 = 440 miles
Therefore, the first jet has a rate of 440mph
Lesson Exercises:
1. A shipment carrier from the Philippines sent shipments to Singapore. The trip towards
Singapore took 4 hours and going back, it only took 3 hours. If the trip from Philippines to
Singapore averages 150mph, what is the rate traveled by the carrier from Singapore to
Philippines if it traveled the same route?
a. 150mph
b. 200mph
c.250mph
d. 300mph

Solution:
Let x be the rate of the carrier from Singapore to Philippines.
Rate Time Distance
Philippines to Singapore 150 4 150(4) = 600

145
Singapore to Philippines x 3 3x
Total ------ ----- 600 = 3x
distance traveled Phil. to Singapore = distance traveled from Singapore to Phil.
600 = 3x
600
= x
3

200mph = x
Therefore, the carrier flew at a rate of 200mph from Singapore to Philippines.

2. Pete left the park and drove towards the mall. Patrick left two hours later at a rate of 60kph in
an effort to catch up to Pete. In three hours, Patrick was able to meet Pete in the mall. What is
Pete’s average rate from the park to the mall?
a. 15kph
b. 18kph
c. 20kph
d. 23kph

Solution:
Let x be Pete’s rate from the park to the mall.
Rate Time Distance
Pete x 3 3x
Patrick 60kph 1 60
Total ------ ----- 3x = 60
Distance Pete travelled = distance Patrick travelled
3x = 60
60
x =
3

x = 20kph
Therefore, Pete traveled at a rate of 20kph.

3. A jeepney bound for Mandaue City leaves the Ayala Terminal and averages at a speed
of 45kph. Thirty minutes later, another jeepney averaging at 65 kph leaves the same
terminal. In how many hours will the second jeepney overtake the first?
a. 1.125 hours
c. 1.5 hours
c.1.25hours
d. 2.0 hours

146
Solution:
1
Let ( + t) be the time of the first jeepney.
2

Let t be the time of the second jeepney.


Rate Time Distance
First Jeepney 45 1 1
45( + 𝑡)
+𝑡 2
2
Second Jeepney 65 t 65t
Total ------ ----- 1
45( + t) = 65t
2

distance of first jeepney = distance of second jeepney


1
45( + t) = 65t
2
45
+ 45t = 65t
2
45
= 65t – 45t
2
45
= 20t
2

45 = 40t
45
t =
40

t = 1.125 hours
Therefore, it takes 1.125 hours for the second jeepney to overtake the first.

4. Two Ceres buses leave the South Bus Terminal at 4:00am and travel in opposite
directions. The first bus travels at a rate of 62kph and the second at 48kph. At what time
will the two buses be 1210km apart?
a. 12:00 noon
b. 1:00 pm
c. 2.00 pm
d. 3:00 pm
Solution:
Let t be the time traveled by the two different buses.
Rate Time Distance
First Bus 62kph t 62t
Second Bus 48kph t 48t
Total ------ ----- 1210 km
distance traveled by first bus + distance traveled by second bus = total distance of 2 buses
62t + 48t = 1210

147
110t = 1210
1210
t =
110

t = 11
Therefore, after 11 hours or at exactly 3:00 pm, the two buses will be 1210 km apart.

5. A jet from Cebu flew to Davao for 3 hours in two different speeds. The first part of the trip
was flown at 85mph and then it flew at a speed of 95mph. If the jet flew a distance of
275 miles, how many hours did the first part of the trip last?
a. 30 minutes
b. 1 hour
c. 1 hour and 30 minutes
d. 2 hours
Solution:
Let t be the time spent for first part of the trip from Cebu to Davao.
Let (3 –t) be the time spent for the remaining time of the trip.
Rate Time Distance
First Part 85 mph t 85t
Second Bus 95 mph 3-t 95(3 – t)
Total ------ ----- 275 miles
distance traveled for the first part + distance traveled for the second part = total distance from
Cebu to Davao
85t + 95(3 – t) = 275
85t + 285 – 95t = 275
-10t = 275 – 285
−10
t =
−10

t = 1
Therefore, for 1 hour, the jet traveled at a rate of 85mph.

6. Therese and Shiela are having a road trip towards Moalboal. They rode their respective
cars and it turned out that Therese came 2 hours earlier than Shiela. If Shiela arrived in
Moalboal after 8 hours, and her rate is 2mph less than Therese’s rate, find Shiela’s rate.
a. 4 mph
b. 5 mph
c. 6 mph
d. 8mph
Solution:
Let x be the rate of Therese

148
(x – 2) be the rate of Shiela
Rate Time Distance
Therese X mph 6 6x
Shiela (x -2) mph 8 8(x -2)
Total ------ ----- 6x = 8(x – 2)
distance traveled by Therese = distance traveled by Shiela
6x = 8(x – 2)
6x = 8x – 16
6x – 8x = -16
−16
-2x =
−2

x = 8
x–2 = 6
Therefore, Shiela traveled at a rate of 6 miles per hour.

7. Tom and Jerry wants to race towards the kitchen. Each of them is wearing their skate
shoes and they started at a distance of 200 meters apart. If Tom can skate at a speed of
2 meters per second while Jerry skates at 1.2 meters per second, how many seconds
will pass after they bump at each other?
a. 62.5 seconds
b. 64.5 seconds
c. 68.2 seconds
d. 70 seconds
Solution:
Let s be the number of seconds passed after they bump at each other.
Rate Time Distance
Tom 2 mps s 2s
Jerry 1.2 mps s 1.2s
Total ------ ----- 200
distance traveled by Tom + distance traveled by Jerry = total distance traveled by the two
2s + 1.2s = 200
3.2s = 200
200
s =
3.2

s = 62.5 seconds
Therefore, 62.5 seconds will pass after Tom and Jerry bump at each other.

149
8. Georgia left home and walked towards the playground at a speed of 0.54kph. An hour
later, her mom looks for her and upon knowing that Georgia is going to the playground,
her Mom ran in an effort to catch her up at a rate of 1.08kph. If her mom left at 5:00 in
the afternoon, what time did she find Georgia?
a. 5:45 pm
b. 6:00 pm
c. 6:28 pm
d. 6:35 pm

Solution:
Let t be the number of hours after the Mom found Georgia
Let (t + 1) be the time Georgia spent in walking through the playground.
Rate Time Distance
Mom 1.08 kph t 1.08t
Georgia 0.54 kph t +1 0.54(t + 1)
Total ------ ----- 1.08t = 0.54(t + 1)
distance ran by the Mom = distance Georgia walked
1.08t = 0.54(t + 1)
100(1.08t) = 100(0.54t + 0.54)
108t – 54t = 54
54t = 54
54
t =
54

t = 1 hour
Therefore, after an hour, at exactly 6:00 pm, Georgia’s mom found Georgia.

9. Janine and Carina are in opposite ends of Cebu. They have missed each other for 3
months and they plane to meet up. Janine drove her car at an average speed of 35
miles per hour while Carina drove at 40mph. If both of them left their respective homes
at 9:00 am, around what time will they be seeing each other if they are 225 miles apart?
a. 12:00 noon
b. 11:45 am
c. 1:30 pm
d. 2:00 pm
Solution:
Let t be the number of hours they have to travel before they will meet.
Rate Time Distance

150
Janine 35 kph t 35t
Carina 40 kph t 40t
Total ------ ----- 225
distance drove by Janine + distance drove by Carina = total distance they are apart
35t + 40t = 225
75t = 225
225
t =
75

t = 3 hours
Therefore, Janine and Carina will be seeing each other after three hours or at exactly 12:00
noon.

10. Dandy needs to buy a sack of rice for this family’s consumption for the month. He left his
house, rode a jeepney, and travelled north. Meanwhile, his wife Sally rode a jeepney
which travelled down south to buy some garments and clothes for their kids. If the
jeepney rode by Dandy had an average speed of 66kph and the jeepney rode by Sally
had 74kph, at what time will they be 637 km apart?
a. 4.65 hours
b. 3.75 hours
c. 4.55 hours
d. 5.15 hours
Solution:
Let t be the number of hours they travelled where they are 637 km apart.
Rate Time Distance
Jeepney (Dandy 66 kph t 66t
Jeepney (Sally) 74 kph t 74t
Total ------ ----- 637
distance travelled by Dandy + distance travelled by Sally = total distance they are apart
66t + 74t = 637
140t = 637
637
t =
140
t = 4.55 hours
Therefore, after 4.55 hours, Dandy and Sally are already 637 km apart.

5.11 Clock Problems


Clock angle problems are a type of mathematical problem which involve finding the
angles between the hands of an analog clock.
Lesson Proper:
Since there are 12 hours on a standard clock face we divided the total 360 degree
measurement by 12 to get 30. 360/12=30. This tells us that the space between any 2 hours is
30 degrees.

151
A clock's full measure is 360 degrees. Therefore, between each pair of numbers (i.e. 10
and 11) it is 30 degrees. We figured this out because there are 12 pairs, and 360 divided by 12
= equals 30. At 4:08, the clock looks something like this:

Clock angle problems relate two different measurements: angles and time.
A method to solve such problems is to consider the rate of change of the angle in degrees per
minute. The hour hand of a normal 12-hour analogue clock turns 360° in 12 hours (720 minutes)
or 0.5° per minute. The minute hand rotates through 360° in 60 minutes or 6° per minute.
Equation for the angle of the hour hand:
1 1
𝜃ℎ𝑟 = 𝑀𝐸 = (60𝐻 + 𝑀)
2 2
where
𝜃 is the angle in degrees of the hand measured clockwise from the 12
o’clock position.
𝐻 is the hours past 12 o’clock
𝑀 is the minutes past the hour
𝑀𝐸 is the minutes past 12 o’clock
Equation for the degrees on the minute hand:
𝜃𝑚𝑖𝑛 = 6𝑀
where
𝜃 is the angle in degrees of the hand measured clockwise from the 12
o’clock position.
𝑀 is the minute
Example:
The time is 5:24. The angle in degrees of the hour hand is:

152
1
𝜃ℎ𝑟 = (60 𝑥 50 + 24)=162
2

The angle in degrees of the minute hand is:


𝜃𝑚𝑖𝑛 = 6 𝑥 24 = 144
Equation for the angle between the hands
The angle between the hands can be found using the formula:
∆𝜃 = |𝜃ℎ𝑟 − 𝜃𝑚𝑖𝑛. |
1
= | (60𝐻 + 𝑀) − 6𝑀|
2
1
= | (60𝐻 − 11𝑀)|
2

Lesson Exercise:

1. An accurate clock shows 8 o'clock in the morning. Through how many degrees will the
hour hand rotate when the clock shows 2 o'clock in the afternoon?
A. 144º
B. 150º
C. 168º
D. 180º
Correct answer: D
Explanation:
360
Angle traced by the hour hand in 6 hours = ( 𝑥6) ° = 180°
12

2. The reflex angle between the hands of a clock at 10:25 is:


A. 180°
B. 192.5°
C. 195°
D. 197.5°
Correct answer: D
Solution or Explanation:

125 360 125 º 1º


Angle traced by hour hand in hrs = x = 312 .
12 12 12 2
360 º
Angle traced by minute hand in 25 min = x 25 = 150º.
60
1 º 1º 1
Reflex angle = 360º - 312 - 150 = 360º - 162 = 197 .
2 2 2
3. A clock is started at noon. By 10 minutes past 5, the hour hand has turned through:
A. 145°

153
B. 150°
C. 155°
D. 160°
Correct answer: C
Solution or Explanation:
Angle traced by hour hand in 12 hrs = 360º.
31 360 31 º
Angle traced by hour hand in 5 hrs 10 min. i.e., hrs = x = 155º.
6 12 6

4. A watch which gains 5 seconds in 3 minutes was set right at 7 a.m. In the afternoon of
the same day, when the watch indicated quarter past 4 o'clock, the true time is:24 − 10 =
3𝑥
7
A. 59 𝑚𝑖𝑛. 𝑝𝑎𝑠𝑡 3
12
B. 4𝑝𝑚
7
C. 58 𝑚𝑖𝑛. 𝑝𝑎𝑠𝑡 3
11
7
D. 2 𝑚𝑖𝑛. 𝑝𝑎𝑠𝑡 4
12

Correct answer: B
Solution:
37
Time from 7 a.m. to 4.15 p.m. = 9 hrs 15 min. = hrs.
4
3 min. 5 sec. of this clock = 3 min. of the correct clock.
37 1
hrs of this clock = hrs of the correct clock.
720 20
37 1 720 37
hrs of this clock = x x hrs of the correct clock.
4 20 37 4
= 9 hrs of the correct clock.
The correct time is 9 hrs after 7 a.m. i.e., 4 p.m.

5. How much does a watch lose per day, if its hands coincide every 64 minutes?
8
A. 32 𝑚𝑖𝑛.
11
B. 90𝑚𝑖𝑛
C. 36.5𝑚𝑖𝑛
D. 95𝑚𝑖𝑛
Correct answer: A
Solution:
55 min. spaces are covered in 60 min.
60 min. spaces are covered in 60 x 60 = 65 5 min.

154
55 11
min.
5 16
Loss in 64 min. = 65 - 64 = min.
11 11
16 1 8
Loss in 24 hrs = x x 24 x 60 = 32 min.
11 64 min. 11

6. At what time between 7 and 8 o'clock will the hands of a clock be in the same straight line
but, not together?

2
A. 5 min. past 7 B. 5 min. past 7
11

3 5
C. 5 min. past 7 D. 5 min. past 7
11 11

Correct Answer: D
Explanation:
When the hands of the clock are in the same straight line but not together, they are 30 minute
spaces apart.
At 7 o'clock, they are 25 min. spaces apart.
Minute hand will have to gain only 5 min. spaces.
55 min. spaces are gained in 60 min.
60 5
5 min. spaces are gained in x5 =5 min.
55 min 11
5
Required time = 5 min. past 7.
11

7. At what time between 5.30 and 6 will the hands of a clock be at right angles?

5 7
A. 43 min. past 5 B. 43 min. past 5
11 11

C. 40 min. past 5 D. 45 min. past 5

Correct Answer: B
Explanation:

155
At 5 o'clock, the hands are 25 min. spaces apart.
To be at right angles and that too between 5.30 and 6, the minute hand has to gain (25 + 15) =
40 min. spaces.
55 min. spaces are gained in 60 min.
60 7
40 min. spaces are gained in x 40 = 43 min.
55 min 11
7 min. past 5.
Required time = 43 11

8. The angle between the minute hand and the hour hand of a clock when
the time is 4.20, is:

A. 0º B. 10º

C. 5º D. 20º

Correct Answer: B
Explanation:
13 360 13 º
Angle traced by hour hand in hrs = x = 130º.
3 12 3
360 º
Angle traced by min. hand in 20 min. = x 20 = 120º.
60
Required angle = (130 - 120)º = 10º.

9. At what angle the hands of a clock are inclined at 15 minutes past 5?


A. 58 B. 64º
2

1º 1º
C. 67 D. 72
2 2

Correct Answer: C
Explanation:
21 360 21 º 1º
Angle traced by hour hand in hrs = x = 157
4 12 4 2
360 º
Angle traced by min. hand in 15 min. = x 15 = 90º.
60

156
1 º - 1º
90º
Required angle = 1572
= 2
67
10. At 3.40, the hour hand and the minute hand of a clock form an angle of:

A. 120º B. 125º

C. 130º D. 135º

Correct Answer: C
Explanation:
Angle traced by hour hand in 12 hrs. = 360º.
11 360 11 º
Angle traced by it in hrs = x = 110º.
3 12 3
Angle traced by minute hand in 60 min. = 360º.
360 º
Angle traced by it in 40 min. = x 40 = 240º.
60
Required angle (240 - 110)º = 130º

11. How many times are the hands of a clock at right angle in a day?

A. 22 B. 24

C. 44 D. 48

Correct Answer: C
Explanation:
In 12 hours, they are at right angles 22 times.
In 24 hours, they are at right angles 44 times.

12. The angle between the minute hand and the hour hand of a clock when the time is
8.30, is:

A. 80º B. 75º

C. 60º D. 105º

Correct Answer: B
Explanation:
Angle traced by hour hand in 17 hrs = 360 x 17 º = 255.

157
2 12 2

360 º
Angle traced by min. hand in 30 min. = x 30 = 180.
60
Required angle = (255 - 180)º = 75º.

13. How many times in a day, are the hands of a clock in straight line but opposite in
direction?

A. 20 B. 22

C. 24 D. 48

Correct Answer: B
Explanation:
The hands of a clock point in opposite directions (in the same straight line) 11 times in
every 12 hours. (Because between 5 and 7 they point in opposite directions at 6
o'clcok only).
So, in a day, the hands point in the opposite directions 22 times.

14. At what time between 4 and 5 o'clock will the hands of a watch point in
opposite directions?

A. 45 min. past 4 B. 40 min. past 4

4 6
C. 50 min. past 4 D. 54 min. past 4
11 11

Correct Answer: D
Explanation:
At 4 o'clock, the hands of the watch are 20 min. spaces apart.
To be in opposite directions, they must be 30 min. spaces
apart.

Minute hand will have to gain 50 min. spaces.


55 min. spaces are gained in 60 min.
60 6
50 min. spaces are gained in x 50 min.
55 min. or 5411

158
6
Required time = 54 min. past 4.
11

15.
At what time between 9 and 10 o'clock will the hands of a
watch be together?

A. 45 min. past 9 B. 50 min. past 9

1 2
C. 49 min. past 9 D. 48 min. past 9
11 11

Correct Answer: C
Explanation:
To be together between 9 and 10 o'clock, the minute hand has
to gain 45 min. spaces.
55 min. spaces gained in 60 min.
60 1
45 min. spaces are gained in x 45 min.
55 min or 4911
1
The hands are together at 49 min. past 9.
11

16. At what time, in minutes, between 3 o'clock and 4 o'clock,


both the needles will coincide each other?

1" 4"
A. 5 B. 12
11 11

4" 4"
C. 13 D. 16
11 11

Correct Answer: D
Explanation:
At 3 o'clock, the minute hand is 15 min. spaces apart from the hour hand.
To be coincident, it must gain 15 min. spaces.
55 min. are gained in 60 min.
15 min. are gained in 60 x 15 = 16 4 min.

159
55 11
min
4
The hands are coincident at 16 min. past 3.
11

17. How many times do the hands of a clock coincide in a day?

A. 20 B. 21

C. 22 D. 24

Correct Answer: C
Explanation:
The hands of a clock coincide 11 times in every 12 hours (Since between 11 and 1,
they coincide only once, i.e., at 12 o'clock).
AM
12:00
1:05
2:11
3:16
4:22
5:27
6:33
7:38
8:44
9:49
10:55
PM
12:00
1:05
2:11
3:16
4:22
5:27
6:33
7:38
8:44
9:49
10:55
The hands overlap about every 65 minutes, not every 60 minutes.
The hands coincide 22 times in a day.

160
18. How many times in a day, the hands of a clock are straight?

A. 22 B. 24

C. 44 D. 48

Correct Answer: C
Explanation:
In 12 hours, the hands coincide or are in opposite direction 22 times.
In 24 hours, the hands coincide or are in opposite direction 44 times a day.

19. A watch which gains uniformly is 2 minutes low at noon on Monday and is 4 min.
48 sec fast at 2 p.m. on the following Monday. When was it correct?

A. 2 p.m. on Tuesday B. 2 p.m. on Wednesday

C. 3 p.m. on Thursday D. 1 p.m. on Friday

Correct Answer: B
Explanation:
Time from 12 p.m. on Monday to 2 p.m. on the following Monday = 7 days 2 hours =
170 hours.
4 34
The watch gains 2+4 or min. in 170 hrs.
5 min. 5
34
Now, min. are gained in 170 hrs.
5
5
2 min. are gained in 170 x x2 = 50 hrs.
34 hrs
Watch is correct 2 days 2 hrs. after 12 p.m. on Monday i.e., it will be correct at 2
p.m. on Wednesday.

20. What time between 2 and 3 o’clock will the angle between the hands of the clock
be bisected by the line connecting the center of the clock and the 3 o’clock mark?
A.2:18:27.6
B. 2:16:00.0
C. 2:17:56.3
D. 2:19:03.1
Correct answer: A
solution :

161
5.12 Inequalities
In this course, you will learn how to solve inequalities. Solving an inequality means finding all of
its solutions. A solution of an inequality is a number which when substituted for the variable
makes the inequality a true statement.
Lesson Proper:

Symbol Meaning Implication


= equal equality between two
quantities
< less than strict inequality between
two quantities
> greater than
≤ less than or equal to combination of equality
and inequality
≥ greater than or equal to

In general, if variables x and y are any real numbers, 𝑥 < 𝑦 means that 𝑥 is to the left of 𝑦 on
the number line 𝑥 − 𝑦 < 0, and 𝑥 > 𝑦 means that x is to the right of y on the number line or𝑥 −
𝑦 > 0.
Here are some examples of inequalities expressed in algebraic symbols and in sentences.
Symbols Sentences

162
𝑥 + 8 > 10 𝑥 plus 8 is greater than 10.
2𝑥 − 6 < 20 2𝑥 minus 6 is less than 20.
4𝑥 ≤ 16 4𝑥 is less than or equal to 16.
5𝑥 − 1 ≥ 24 5𝑥 minus 1 is greater than or
equal to 24.
12 + 𝑥 ≠ 10
12 plus 𝑥 is not equal to 10.

To solve an inequality is to find values of the variable that will make the sentence true. In
solving an inequality, isolate the variable on one side of the inequality symbol. In doing so,
the following properties must be strictly observed:
Let a, b, and c be real numbers.
1. If a > b, then a+c > b+c.
2. If a>b, then a-c > b-c.
3. If a.b, then ac > bc when c > 0; ac < bc when c < 0.
4. If a > b, then a/c > b/c when c > 0; a/c < b/c when c < 0.
Inequalities are dealt within the same manner as equations. Consider the inequality x+8 > 5.
Adding -8 to both sides of the inequality, we have
x + 8 + (-8) > 5 + (-8)
x+0 > 5 - 8
x > -3
The addition property of inequality allows us to add the same number to both sides of the
inequality.

Tip:
1. If A, B, and C are real numbers and A ≥ B, then A + C ≥ B + C

Lesson Exercise:
1. If 𝑥 + 5 < 2, then
A. 𝑥 < 3
B. 𝑥 < −3
C. 𝑥 > 3
D. 𝑥 > −3
Correct answer: B
Solution:
Transpose 5 to the right side making it -3.
2. If −3𝑥 < 6, then
A. 𝑥 < −2
B. 𝑥 > −2
C. 𝑥<2

163
D. 𝑥 > 2
Correct answer: A
Solution:
Divide both sides by -3.
3. If 3𝑥 + 4 < 7, then
A. 𝑥 < 1
B. 𝑥 < −1
C. 𝑥 > 1
D. 𝑥 > −1
Correct answer: A
Solution:
Transpose 4 to the right side and divide both sides by 3.
4. 𝑥 ≥ 𝑎 means that
A. 𝑥 > 𝑎
B. 𝑥 < 𝑎 or 𝑥 = 𝑎
C. 𝑥 = 𝑎
D. 𝑥 > 𝑎 or 𝑥 = 𝑎
Correct answer: D
Solution:
≥ means greater than or equal to

5. Which of the following is/are true?


A. 5 > 3
B. -10 > 7
C. 0 ≤ 0
D. −8 ≤ −3
Correct answer: A
Solution:
Five is greater than 3.
6. The inequality -5< 𝑥 < 3 is equivalent to
A. (-5, 3)
B. [-5, 3)
C. {-5, 3]
D. (-5, 3]
Correct answer: A
Solution: ( , ) → lesser than or greater than
[ , ] → lesser than or equal to or greater than or equal to
7. [-4, 3) is equivalent to
A. -4 < x < 3

164
B. -4 ≤ 𝑥 < 3
C. -4 ≤ x ≤ 3
D. -4 < x ≤ 3
Correct answer: B
Solution:
( , ) → lesser than or greater than
[ , ] → lesser than or equal to or greater than or equal to
8. Which of the following is true?
A. −4 > 5
B. −8 > −22
1 1
C. >
3 2
D. 0.987 ≤ 0.876
Correct answer: B
Solution:
> → greater than
9. Which value of x is in the solution set of the inequality: -2x + 5 > 17?
A. -8
B. -6
C. -4
D. 12
Correct answer: B
Solution:
Transpose 5 to the right side and divide both sides by -2
10. Solve: 2x - 5 > x - 2
A. 𝑥 <3
B. 𝑥 >3
C. 𝑥 < −5
D. 𝑥 >2
Correct answer: B
Solution:
Transpose -5 to the other side, and x to the left side.
11. Describe the solution of the inequality 𝑥 + 3 ≤ 7.
A. All real numbers less than 4
B. All real numbers less than or equal to 10
C. All real numbers less than or equal to 4
D. All real numbers less than or equal to -4
Correct answer: C
Solution:
Transpose 3 to the other side
12. Which inequality is equivalent to 2 - 3x ≥ -4?

165
A. x≥2
B. x≤2
2
C. x ≤ --
3
D. none of the above
Correct answer: A
Solution:
Transpose 2 to the other side and divide both sides by -3
13. Describe the solution of the inequality -3x + 2 > 11.
A. All real numbers less than -3
B. All real numbers less than -3
C. All real numbers greater than -3
D. All real numbers less than 3
Correct answer: C
Solution:
Transpose 2 to the right side and divide both sides by -3
14. For which value of x is the inequality 5(3𝑥 + 4) ≤ 5𝑥 − 10 true?
A. x ≥ -3
B. x ≤ 1
C. x ≤ -1
D. x ≤ -3
Correct answer: D
Solution:
Simplify the equation by distributive property and transpose.
15. The solution of -3y - 1< -19 is:
A. y > - 6
B. y < 6
C. y < - 6
D. y > 6
Correct answer: B
Solution:
Transpose -1 to the other side and divide both sides by -3
16. The solution of -5x + 11 > 1 is:
A. x > -2
B. x > 2
C. x < 2
D. x < -2
Correct answer: B
Solution:
Transpose 11 to the other side and divide both sides by -5
𝑚
17. The solution of < -12 is:
3
A. m > 4

166
B. m > 36
C. m< -36
D. m < 4
Correct answer: C
Solution:
Multiply both sides by 3.
18. The solution of -2y>8 is:
A. y > -4
B. y > 4
C. y< -4
D. y < 4
Correct answer: A
Solution:
Divide both sides by -2
19. The solution of x + 17 < 8 is:
A. x > -9
B. x > 9
C. x< -9
D. x < -9
Correct answer: D

Solution:
Transpose 17 to the right side
20. Solve the inequality 3m + 3 > 9
A. m > 5
B. m > 2
C. m < 4
D. m < -9
Correct answer: B
Solution:
Transpose 3 to the right side and divide both sides by 3
6 Equations and Inequalities in Two Variables
6.1 Linear Equation in two variables
A Linear Equation in two variables is an equation with two variables usually multiplied with a
constant, no variables in the denominator, no exponents (Linear), and comes in the form of:
𝑎𝑥 + 𝑏𝑦 = 𝑝
𝑐𝑥 + 𝑑𝑦 = 𝑞

 Determining if an Ordered Pair is a solution to a Linear Equation in Two Variables

167
An ordered pair contains an x – value and a y – value. It is said to be a solution or a part of the
solution set of an equation if and only if the values in the ordered pair satisfies the given
equation (makes it true).
(3,4) is an example of an ordered pair, 3 as the x – value and 4 as the y – value.
Let’s say for example we have an equation:
𝑥 + 3𝑦 = 15
The ordered pair (3,4) will be considered a solution of the equation if it satisfies the equation.
The x – value will be substituted into the x variable, and the y – value is substituted into the y
variable.
Is (3) + 3(4) = 15 a true statement? If yes, then (3,4) is a solution of the equation.
3 + 12 = 15
15 = 15
Therefore (3,4) is a solution of the equation x + 3y = 15.
 Solving a System of Linear Equations Using Substitution
The method of substitution is done by solving one of the given for one of the variables, and then
plotting it into the other equation (substituting) to solve for the other variable.
Let us take the following as an example:
2𝑥 − 3𝑦 = −2
𝑦 = −4𝑥 + 24
Luckily, we already have a value for one of our variables y which is (-4x + 24).
Now all we need to do is plug the value of y into the y variable of the first equation.
2𝑥 − 3(−4𝑥 + 24) = −2
2𝑥 − (−12𝑥 + 72) = −2
2𝑥 + 12𝑥 − 72 = −2
14𝑥 = 70
𝑥=5
Now, all we need to do is plug the value of x (which is 5) into the second equation to get the
numerical value of y.
𝑦 = −4(5) + 24 = −20 + 24 = 4
So the solution is (5,4)
 Solving as System of Linear Equations by Graphing
The solution set of the equation is simply the ordered pair at the point of intersection or merely
the coordinates of the point of intersection of the two lines.

The graph of equations within a system can tell us how many solutions exist by looking at it.
 If the graphs of the equations intersect, there is no solution.

168
 If the graphs of the equations do not intersect (parallel), then there is no solution for the
equations.
 If the graphs of the equations are the same, then there are an infinite number of possible
solutions.
 Solving A System of Linear Equations by the Elimination Method.
Another way of Solving linear system is to use the elimination method.
1. Decide whether which of the two variables is easier to eliminate (variables
with same coefficients/ variables with the same coefficients but with
opposite signs).
2. Eliminate the selected variable via addition/ subtraction.
3. After the elimination, determine the numerical value of the other variable.
4. Plug and solve for the numerical value of the eliminated variable.
Example:
3𝑦 + 2𝑥 = 6
5𝑦 − 2𝑥 = 10
In this case, x is the variable that is to be eliminated (numerical coefficients are equal but of
different signs).
The two equations are to be added for the x variable to be eliminated.
After eliminating x, solve for y:
8𝑦 = 16
𝑦=2
Plug in the value of y into either of the given equations, and look for the numerical value of x.
3𝑦 + 2𝑥 = 6
3(2) + 2𝑥 = 6
6 + 2𝑥 = 6
𝑥=0
In a case where in the numerical coefficients of a certain variable is different, we need to
manipulate one equation for the coefficients to be equal (multiplying one equation with a certain
number is the most common process to make the coefficients equal).

Lesson Exercise:
1. Which of the following ordered pairs is a solution to the equation y = 3x + 5?
a.) (-2, -1)
b.) (3, 2)
c.) (0, 2)
Solution:
(-1) = 3(-2) + 5
-1 = -6 + 5

169
-1 = -1

2. Is (2,1) a solution of the equation y = 6x + 5?


a) True
b) False
Solution:
(1) = 6(2) + 5
1 = 12 + 5
1 = 17

3. Which of the following ordered pairs is a solution to the given system?


𝑥+𝑦 = 8
3𝑥 + 2𝑦 = 21
a) (4,4)
b) (5,3)
c) (6,2)
d) (7,1)

Solution:
Using the ordered pair (5,3),
Substitute in the first equation:
5+3 =8
8=8
Substitute in the second equation:
3(5) + 2(3) = 21
15 +6 = 21
21 = 21

For numbers 5-6, solve via substitution:


5. Determine the solution set for the following system.
2𝑥 − 5𝑦 = 1
3𝑥 + 𝑦 = −7
a. (3,2)
b. (-2, 0)
c. (-2, -1)
d. (1,2)

170
Solution:
Based on the given, a value for y can be easily taken which is 𝑦 = −3𝑥 − 7
Plugging it into the first equation, we get:
2𝑥 − 5(−3𝑥 − 7) = 1
2𝑥 + 15𝑥 + 35 = 1
17𝑥 = −34
𝑥 = −2
Now that we have the value of -2, all we need to do is plug it into any of the 2 equations in the
system (we will use eq. 1).
2(−2) − 5𝑦 = 1
−4 − 5𝑦 = 1
−5𝑦 = 5
𝑦 = −1

6. Determine the solution set for the following system.


2𝑥 + 3𝑦 = 7
𝑥=2
a. (2,1)
b. (2,2)
c. (2,-3)
d. (2, -2)
Solution:
A value for x is already provided and so, all we need to do is look for the value of y.
By plugging x into the first equation, we get:
2(2) + 3𝑦 = 7
4 + 3𝑦
3𝑦 = 3
𝑦=1

For number 7-10, solve using the elimination method.


7. Determine the solution set for the following system:
𝑥+𝑦 = 4
𝑥−𝑦 = 6
a. (3,-2)
b. (1,2)

171
c. (5, -1)
d. (2, -3)
Solution:
In this case we can eliminate either x or y. (We will eliminate y using addition.)
After addition we have:
2𝑥 = 10
𝑥=5
Now we can plug the value of y into any equation to get the numerical value of x.
5+𝑦 =4
𝑦 = −1

8. Determine the solution set for the following system.


2𝑥 + 𝑦 = 3
𝑥−𝑦 = 3
a. (5,2)
b. (2, -1)
c. (3, -1)
d. (1, 4)
Solution:
We can easily eliminate y in this case because they both have the same coefficients. After
eliminating y, we get:
3𝑥 = 6
𝑥= 2
And by plugging the value of x into any equation, we get:
2(2) + 𝑦 = 3
4+𝑦 =3
𝑦 = −1

9. Determine the solution set for the following system:


4𝑥 + 3𝑦 = 15
2𝑥 − 5𝑦 = 1
a. (2,0)
b. (1,-1)
c. (3,2)
d. (3,1)

172
Solution:
This is the case wherein variable don’t have the same coefficient. In this case, we need to
multiply the second equation by 2 so that the x-coefficients of the two equations will be equal.
After multiplying by 2, we get:
4𝑥 − 10𝑦 = 2
Now we can already eliminate x, which gives us equation 3:
13𝑦 = 13
𝑦=1
Now we can plug the y-value into either equation 1 or 2:
2𝑥 − 5(1) = 1
2𝑥 − 5 = 1
2𝑥 = 6
𝑥=3

10. Determine the solution set for the following system.


3𝑥 + 5𝑦 = 4
−2𝑥 + 3𝑦 = 10
a. (-2,2)
b. (-1,3)
c. (-3,1)
d. (1,1)
Solution:
In this case, we need to multiply a number to each of the equations for them to have the same
coefficients. This can be done by multiplying equation 1 by (2) and multiplying equation 2 by (3).
We now have:
6𝑥 + 10𝑦 = 8
−6𝑥 + 9𝑦 = 30
Now, we can easily eliminate x:
19𝑦 = 38
𝑦=2
Plug and solve:
3𝑥 + 5(2) = 4
3𝑥 + 10 = 4
3𝑥 = −6
𝑥 = −2

173
6.2 Graph of Linear Equations
Using a Table of Values to Graph Linear Equations
You can graph any equation using a table of values. A table of values is a graphic organizer or
chart that helps you determine two or more points that can be used to create your graph.

Why Use a Table of Values?

In order to graph a line, you must have two points. For any given linear equation, there are an
infinite number of solutions or points on that line.
If you just find two of the solutions, then you can plot your two points and draw a line through.
This will be the line that represents the equation. Every point on that line is a solution to the
equation.
In my table, I have 4 columns as described below:
 The first column is for the x coordinate. For this column, I can choose any number I
wish. Try to choose numbers that can be graphed on your graph. For example, if your x
axis only extends to 10, don't choose 12 as an x coordinate.

 The second column is for substituting x into the equation in order to solve for y. So,
whatever value I chose for x, I will substitute back into the equation and solve to find the
y value.

 The third column is for the y value. After substituting your x value into the equation, your
answer is the y coordinate.

 The last column is for your ordered pair. Your ordered pair is the x value and the y
value. This is the point on your graph.
Let's look at a few examples, and it will all make more sense.

174
In this first example, I chose -2, 0, and 2 as my x coordinates.
After substituting those values into the equation: y = 2x +1, I found my y values to be: -3, 1, and
5.
Therefore, the ordered pairs that I found on my graph were: (-2,-3), (0,1), and (2,5).
I plotted those points on my graph.
I then used my ruler and drew a straight line through those points. This is the line for the
equation, y = 2x +1.
If you had done this problem on your own, you may have found three different points using the
table of values. That's ok, because even if your three points are different, your line will still look
exactly the same!

175
We can also find other solutions for the equation just by reading the graph. I see that (3,7) is a
point on the graph. If I substitute 3 for x into the equation, I will get 7 as my y coordinate.
This line goes on forever, so there are infinite solutions to the equation.

In this first example, I chose -2, 0, and 2 as my x coordinates.


After substituting those values into the equation: y = 2x +1, I found my y values to be: -3, 1, and
5.
Therefore, the ordered pairs that I found on my graph were: (-2,-3), (0,1), and (2,5).
I plotted those points on my graph.

176
I then used my ruler and drew a straight line through those points. This is the line for the
equation, y = 2x +1.
If you had done this problem on your own, you may have found three different points using the
table of values. That's ok, because even if your three points are different, your line will still look
exactly the same!
We can also find other solutions for the equation just by reading the graph. I see that (3,7) is a
point on the graph. If I substitute 3 for x into the equation, I will get 7 as my y coordinate.
This line goes on forever, so there are infinite solutions to the equation.
Graphing a Linear Equation
Using Slope Intercept Form
Using slope intercept form is one of the quickest and easiest ways to graph a linear equation.
Before we begin, I need to introduce a little vocabulary. We are going to talk about x and y
intercepts.
An x intercept is the point where your line crosses the x-axis. The y intercept is the point
where your line crosses the y-axis.
We are only going to focus on the y intercept in this lesson, but you'll need to know x intercept
for later.
Let's take a look at intercepts

177
Graphing Linear Equations in Standard Form
Let's get started! If you have a standard form equation, you can rewrite it in slope intercept form.
Let's look at a couple of examples.

178
179
Notice that the slope in this equation is negative. This means that our line must be "falling" from
left to right.
Always double check your line and your slope. If your slope is positive, then your line
should "rise" from left to right. If your slope is negative, then your line should "fall" from left to
right.

Here's a quick summary of this lesson:


Rules for Graphing Using Slope Intercept Form
 Your y intercept is always the first point that you plot on the line. Your point will always
be (0, b).

 Then use your slope to plot your next point.


 If you have two points, you can draw a straight line and this is the line that represents
your equation. Any point on that line is a solution to the equation.
Graphing Linear Equations in Standard Form
You have learned many techniques for graphing linear equations that are written in slope
intercept form! Yes... if the equation is already written in slope intercept form, graphing is pretty
easy! However, sometimes you will see equations that are written in standard form.

If the equation is presented in standard form, then you are not able to identify the slope and y-
intercept that are needed for graphing.
So... what should you do?
There are actually two different techniques that you can use for graphing linear equations that
are written in standard form. You can use either method, so I'm going to demonstrate both
methods and you may come to favor one over the other.
In this particular lesson, we are going to study how to convert the standard form equation into
an equation written in slope intercept form.

180
Example 2

181
This shouldn't be too hard, since you've already mastered the skills for solving equations and
the skills for graphing in slope intercept form.
Using the X and Y Intercept to Graph Linear Equations
You've learned one way to graph a standard form equation - by converting it to slope intercept
form. There is another way to graph standard form equations, and that is to find the x and y
intercepts.

182
183
To find the X Intercept: Let y = 0
To find the Y Intercept: Let x = 0
Example 1

Example 2

184
185
Source: http://www.algebra-class.com/

6.3 Slopes
Slope is a very important property when we talk about equations especially linear
equations. We can already graph a linear equation given only a point and the slope. In this
module, we get a little bit deeper on the topic – slope.

Slope – denoted as m

186
Given (x1 , y1) and (x2, y2) as points on a line, the slope of the line is defined as follows:
m = change in the y direction ÷ change in the x direction
m = rise ÷ run
m = ∆y ÷ ∆x
m = ( y2 – y1 ) ÷ ( x2 – x1 )
NOTE:
If line 2 ,with slope m 2, is perpendicular to line 1, with slope m 1, then m1 = -1/m2.
If line 2 ,with slope m 2, is parallel to line 1, with slope m 1, then m1 = m2

Examples
Find the slope of the following:
1. ( 2 , 1) and ( - 3, 0) 2. (0,0) and (1, -3) 3. (4, -2) and (1, 3)

Solutions:
1. Step 1: Label the points.
(NOTE: All x’s can only be among the first value in the ordered pair. All y’s
can only be among the second value in the ordere pair. And also if the first
value in the first ordered pair is x1 , then the second value in the second
ordered pair is automatically y1. )
x1 = 2 , y1= 1, x2 = -3 & y2 = 0

Step 2: Solve for m.


m=(0–1)
( -3 – 2)
m = 1/5
2. Step 1: Label the points.

187
x1 = 0 , y1= 0, x2 = 1 & y2 = -3

Step 2: Solve for m.


m = ( -3 – 0 )
( 1 – 0)
m = -3

3. Step 1: Label the points.


x1 = 4 , y1= -2, x2 = 1 & y2 = 3

Step 2: Solve for m.


m = ( 3 – (-2) )
( 1 – 4)
m = -5/3

Using the slope, we can already determine the geometric interpretation or graph of the equation.

GEOMETRIC INTERPRETATION USING THE SLOPE:

EXAMPLE:
Using the slopes we solved above, what is the geometric interpretation?
1. Since m = 1/5 and 1/5 is positive, therefore the geometric
interpretation would be like this:

2. Since m = -3 and -3 is negative, therefore the geometric interpretation


would be like this:

188
3. Since m = -5/3 and -5/3 is negative, therefore the geometric
interpretation would be like this:

EXERCISE
1. Which would best describe “slope”?
a. Steepness of the line
b. Distance between two points
c. Ratio of rise to the distance between two points
d. A vertical line passing through the midpoint of the line.
Answer:
Slope is defined as how steep the graph of an equation is or as the ratio of rise to
run. It is not related to distance between two points.

2. What is the symbol for slope?


a. n
b. m
c. x
d. y
Answer:
The symbol m is used to denote slope.

3. The slope of a horizontal line is ____.


a. 0
b. 1
c. -1
d. Undefined
Answer:
The slope of a horizontal line is zero since y2=y1.
m=[(y2-y1)/(x2-x1)]
m=[(y1-y1)/(x2-x1)]
m=0/( x2-x1)
m=0

189
4. The slope of a vertical line is ____.
a. 0
b. 1
c. -1
d. Undefined
Answer:
The slope of a vertical line is undefined since x2=x1.
m=[(y2-y1)/(x2-x1)]
m=[(y2-y1)/(x1-x1)]
m=( y2-y1)/0
m= undefined

5. The slope of line 2x – y = 1 is 2. Any line parallel to this has slope ____.
a. -1/2
b. -2
c. -1
d. 2
Answer:
Parallel lines have the same slope. Therefore if the slope of line 2x-y=1 is 2, any
line parallel to this line has also the slope 2.

6. A line has slope 2/3. Any line perpendicular to this has slope ____.
a. -2/3
b. 2/3
c. -3/2
d. 3/2
Answer:
If line 2 ,with slope m 2, is perpendicular to line 1, with slope m 1, then m1 = -1/m2.
Therefore, any line perpendicular to that line would have the slope -3/2.

7. The slope of a nonvertical line passing through two points P(x 1,y1) and Q(x2,y2), is ____.
a. (x2-x1)/(y2-y1)
b. (y2-y1)/(x2-x1)
c. 1/y2
d. [1/(y2-x2)] + [1/(y1-x1)]
Answer:
The definition of slope is the ratio of rise to run. Rise refers to the change in the
y-coordinates while run refers to the change in the x-coordinates. Therefore, the
slope would be m=(y2-y1)/(x2-x1).

8. The change in the x-coordinate corresponds to the ____.


a. Rise
b. Run
c. Slope
d. Distance
Answer:
The change in the x-coordinate refers to the run since it involves the movement
from left to right.

9. The change in the y-coordinate corresponds to the ____.

190
a. Rise
b. Run
c. Slope
d. Distance
Answer:
The change in the y-coordinate refers to the rise since it involves the movement
upward or downward.

10. Which of the following slopes corresponds to the slope of the steepest line?
a. 5
b. -6
c. 1
d. 0
Answer:
The steepest lines are those for which the absolute value of the slope is largest.
From the choices, the absolute value of -6 is the largest; therefore, -6 corresponds to
the slope of the steepest line.

11-18. Find the slope of the line through P and Q.


11. P(-2,4), Q(0,0)
a. -2
b. 2
c. 1
d. 6
Answer:
(1) Label the points.
x1= -2 y1=4
x2= 0 y2=0
(2) Solve for m.
m = (0-4)/[0-(-2)]
m = -4/2
m = -2

12. P(-1,-1), Q(-15,1)


a. -7
b. 7
c. -1/7
d. 1/7
Answer:
(1) Label the points.
x1= -1 y1=-1
x2= -15 y2=1
(2) Solve for m.
m = [1-(-1)]/[-15-(-1)]
m = 2/-14
m = -1/7

13. P(-1,-1), Q(-15,1)


a. -1/9
b. -8/3

191
c. 8/3
d. -8/9
Answer:
(1) Label the points.
x1= -1 y1=-1
x2= -15 y2=1
(2) Solve for m.
m = [2-(-6)]/(-6-3)
m = 8/-9
m = -8/9

14. P(0,0), Q(-4,6)


a. -1/2
b. -2/3
c. -3/2
d. 2/3
Answer:
(1) Label the points.
x1= 0 y1=0
x2= -4 y2=6
(2) Solve for m.
m = (6-0)/(-4-0)
m = 6/-4
m = -3/2

15. P(-10,-12), Q(2,3)


a. 4/5
b. -4/5
c. 5/4
d. -5/4
Answer:
(1) Label the points.
x1= -10 y1=-12
x2= 2 y2=3
(2) Solve for m.
m = [3-(-12)]/[2-(-10)]
m = 15/12
m = 5/4

16. P(2,-5), Q(-6,-5)


a. -8
b. Undefined
c. 0
d. 1
Answer:
(1) Label the points.
x1= 2 y1=-5
x2= -6 y2=-5
(2) Solve for m.
m = [-5-(-5)]/(-6-2)
m = 0/-8

192
m=0

17. P(1,2), Q(2,3)


a. 0
b. -1
c. 1
d. undefined
Answer:
(1) Label the points.
x1= 1 y1=2
x2= 2 y2=3
(2) Solve for m.
m = (3-2)/(2-1)
m = 1/1
m=1

18. P(2,1), Q(2,3)


a. 0
b. -1
c. 2
d. Undefined
Answer:
(1) Label the points.
x1= 2 y1=1
x2= 2 y2=3
(2) Solve for m.
m = (3-1)/(2-2)
m = 2/0
m = undefined

19. If the slope is negative, the graph is ____.


a. Inclined to the left
b. Inclined to the right
c. A horizontal line
d. A vertical line
Answer:
The graph of a line with a negative slope is inclined to the left.

20. Which geometric interpretation represents a line with a positive slope?


a. b. c. d.

Answer:
The geometric interpretation of a line with a positive slope is a line inclined to the
right.

193
6.4 Parallel and Perpendicular Lines
Parallel lines and their slopes are easy. Since slope is a measure of the angle of a line from the
horizontal, and since parallel lines must have the same angle, then parallel lines have the same
slope — and lines with the same slope are parallel.
Perpendicular lines are a bit more complicated. If you visualize a line with positive slope (so it's
an increasing line), then the perpendicular line must have negative slope (because it will be a
decreasing line). So perpendicular slopes have opposite signs. The other "opposite" thing with
perpendicular slopes is that their values are reciprocals; that is, you take the one slope value, and
flip it upside down. Put this together with the sign change, and you get that the slope of the
perpendicular line is the "negative reciprocal" of the slope of the original line — and two lines with
slopes that are negative reciprocals of each other are perpendicular to each other. In numbers, if
the one line's slope is m = 4/5, then the perpendicular line's slope will be m = –5/4. If the one line's
slope is m = –2, then the perpendicular line's slope will be m = 1/2.
 One line passes through the
points (–1, –2) and (1, 2); another line passes through the points
(–2, 0) and (0, 4).

Are these lines parallel, perpendicular, or neither?


To answer this question, I'll find the slopes.

Since these two lines have identical slopes, then these lines are parallel.
 One line passes through the points (0, –4) and (–1, –7); another line passes
through the points (3, 0) and (–3, 2). Are these lines parallel, perpendicular, or
neither?
I'll find the values of the slopes. Copyright © Elizabeth Stapel 2000-2011 All Rights
Reserved

If I were to flip the "3" and then change its sign, I would get " –1/3". In other words, these
slopes are negative reciprocals, so the lines through the points are perpendicular.
 One line passes through the points (–4, 2) and (0, 3); another line passes through
the points (–3, –2) and (3, 2). Are these lines parallel, perpendicular, or neither?
I'll find the slopes.

194
These slope values are not the same, so the lines are not parallel. The slope values are
not negative reciprocals either, so the lines are not perpendicular. Then the answer is
"neither".
6.5 Equation of Straight Lines
From the preceding modules, we had learned how to graph given a linear equation in
two variables. Now, we learn how to make an equation given some details such as intercepts,
some points and slope.
Learning Objectives:
 To be able to form an equation given the slope and the y-intercept.
 To be able to form an equation given the intercepts.
 To be able to form an equation given the slope and a point on the line.
 To be able to form an equation given two points on the line.

Below are some forms used to make a linear equation describing the behavior of the line.
1. Slope – intercept form
y = mx + b
where m is the slope and b is the y-intercept

2. Point – slope Form


y – y1 = m (x – x1)
where m is the slope, x1 is the abscissa of the ordered pair
and y1 is the ordinate of the ordered pair

3. Two – point form


( 𝒚𝟐 – 𝒚𝟏 )
y – y1 = (x – x1)
(𝐱𝟐 – 𝐱𝟏)

where ( x1 , y1) and ( x2 , y2 ) are the ordered pairs

4. Two-intercept form
𝒙 𝒚
+ =1
𝒃𝒂
where a is the x-intercept and b is the y-intercept
Examples

195
Find the equation of the line in general formgiven the following information:
1. Slope is 6 , y-intercept is 2
2. Slope is 1, passing through the origin
3. Line passing through P( -2, 1) and Q(5,-4)
Answers:
1. Using the slope – intercept form y=mx + b, we can already substitute the
given. Therefore, the equation of the line in slope-intercept form is y = 6x + 2.
The equation of the line in general form is 6x – y + 2 = 0.
2. Using the point-slope form,
y – 0 = 1(x-0)
y = x ( in slope-intercept form)
In general form, the equation of the line x-y = 0.
3. (1) Label the points.
x1= - 2 , y1 =1
x2= 5 , y2 =-4
(2) Substitute the given.
Using the two-point form,
y-1 = [(-4-1)/(5-(-2)](x-(-2))
y-1 = (-5/7)(x + 2)
y-1 = (-5/7)x + (-10/7)
y = (-5/7)x – (3/7) (in slope-intercept form)
(5/7)x + y + 3/7 = 0 (in general form)

EXERCISE
1. What is the general equation of a line?
a. Ax + By + C = 0
b. Ax + By = C
c. y = mx + b
d. y-y1 = m(x-x1)
Answer:
The graph of every linear equations Ax + By + C = 0 is a line. Ax + By + C= 0 is
the general equation of a line.
2. What is the slope-intercept form?
a. Ax + By + C = 0
b. Ax + By = C
c. y = mx + b
d. y-y(sub1) = m(x-xsub1)
Answer:

196
The slope-intercept form is y = mx + b.

3. What is the two-intercept form?


a. Ax + By + C = 0
b. y = mx + b
c. y-y(sub1) = m(x-xsub1)
d. (x/a) + (y/b) =1
Answer:
Given the x- and y-intercept, a and b, the two-intercept form is (x/a) + (y/b) = 1.

4. Find an equation for the line whose graph is sketched.


a. x – 2y = 2
b. x + 2y = 2
c. 2x – y = 2
d. 2x + y = 2
Answer:
Notice that the graph crosses the y-axis at (0,1) and the x-axis at (2,0).
Using the two-intercept form,
(x/2) + (y/1) = 1
x + 2y = 2

5-12. Find an equation of the line that satisfies the given conditions.
5. The line passing through ( -4, -6) and has the slope 5.
a. 5x – y = -14
b. -5x + y = 20
c. 5x + y = 14
d. 5x + y = -6
Answer:
Using the point-slope form,
y - (-6) = 5 (x - (-4))
y + 6 = 5 (x + 4)
y + 6 = 5x + 20
5x – y = -14

197
6. The line passing through points (2,1) and (1,6).
a. 5x + y = 11
b. 5x – y = 9
c. x – 5y = 10
d. x + 5y = -11
Answer:
Using the two-point form,
y – 1 = [(6-1)/(1-2)](x – 2)
y – 1 = -5(x – 2)
y – 1 = -5x + 10
5x + y = 11

7. The line that has the slope 2/5 and y-intercept 4.


a. (5/2)x – y = -4
b. (2/5)x – y = -4
c. 5x – y = -4
d. x – (2/5)y = -4
Answer:
Using the slope-intercept form,
y = (2/5) x + 4
(2/5)x – y = -4

8. The line passing through (-1,2) and parallel to the line x=5.
a. x = 2
b. y = 2
c. 5x – y = -7
d. 5x + y = 7
Answer:
Parallel lines have the same slope. Line x = 5 is a horizontal line and horizontal
lines have a slope 0. Therefore, the line parallel to the line x = 5 is 0.
Using the point-slope form,
y – 2 = 0 (x - (-1))
y–2=0
y=2

198
9. The line that has x-intercept 1 and y-intercept -2.
a. 2x – y = 2
b. 2x – y = -2
c. 2x + y = -2
d. 2x – y = -2
Answer:
Using the two-intercept form,
(x/1) + (y/-2) = 1
-2x + y = -2
-1
2x – y = 2

10. The line passing through (1,5) and perpendicular to the line 2x + 5y + 8 = 0.
a. (-2/5)x + y = -3/5
b. (-2/5)x – y = -3/5
c. 5x – 2y = 8
d. (5/2)x – y = -5/2
Answer:
First, we solve for the slope of the line 2x + 5y + 8 by transforming it into slope-
intercept form.
2x + 5y + 8 = 0
5y = -2x - 8
5
y = (-2/5)x – 8/5 (y = mx + b)
Therefore, the slope of line 2x + 5y + 8 = 0 is -2/5.
If Line 2 is perpendicular to Line 1, the slope of Line 2 is the negative reciprocal
of the slope of Line 1. Therefore, the new slope would be 5/2.
Using the point-slope form,
y – 5 = 5/2(x – 1)
y – 5 = (5/2)x – 5/2
5/2x – y = -5/2

11. The line passing through the origin and (-1,-2).


a. y = 0.5x
b. y = 0.25x

199
c. y = 2x
d. 2y = x
Answer:
Using the two-point form,
y – 0 = [(-2-0)/(-1-0)](x-0)
y = 2x

12. The line that has y-intercept -2 and is perpendicular to the line passing through (1,1) and
(5, -2).
a. (3/4)x + y = 7/4
b. (4/3)x – y = 2
c. (3/4)x + y = 2
d. (4/3)x + y = -2
Answer:
First, we must find the slope of the line containing (1,1) and (5, -2).
m = (-2-1)/(5-1)
m = -3/4
If Line 2 is perpendicular to Line 1, the slope of Line 2 is the negative reciprocal of
the slope of Line 1.
Therefore, the new slope would be 4/3.

Using the slope-intercept form,


y = (4/3)x – 2
(4/3)x – y = 2

13-14. Find the slope of the line.


13. x + y = 2
a. -1
b. 0
c. 1
d. undefined
Answer:
Transform the equation into slope-intercept form.
x+y=2
y = -x + 2 (y = mx + b)

200
m = -1
14. 4x + 5y = 10
a. -5/4
b. 5/4
c. -4/5
d. 4/5
Answer:
Transform the equation into slope-intercept form.
4x + 5y = 10
5y = -4x + 10
5
y = (-4/5)x + 2 (y = mx + b)
m = -4/5

15. Find the slope of the line perpendicular to line 3x + 2y - 1 = 0.


a. 3/2
b. 2/3
c. 1/3
d. -2/3
Answer:
Transform the equation into slope-intercept form.
3x + 2y – 1 = 0
2y = -3x + 1
2
y = (-3/2)x + ½
m = -3/2
If Line 2 is perpendicular to Line 1, the slope of Line 2 is the negative reciprocal
of the slope of Line 1.
Therefore, the new slope would be 2/3.

16. Fine the slope of the line parallel to line 2x – 5y = 0


a. 2/5
b. 5/2
c. -5/2
d. -2/5
Answer:

201
Transform the equation into slope-intercept form.
2x – 5y = 0
5y = 2x
5
y = (2/5)x
m = 2/5
Parallel lines have the same slope. Therefore, the slope parallel to line 2x – 5y = 0 is
2/5.

17-19. Find the y-intercept of the line.


17. -2x + 5y – 30 =0
a. 2/5
b. -30
c. 6
d. 5
Answer:
Transform the equation into slope-intercept form.
-2x + 5y – 30 = 0
5y = 2x + 30
5
y = (2/5)x + 6 (y = mx + b)
The y-intercept is (0,6).

18. 4y + 8 = 0
a. 8
b. 4
c. -2
d. 2
Answer:
Transform the equation into slope-intercept form.
4y = -8
4
y = -2 (y = mx + b)
The y-intercept is (0,-2).

19. x – y = 3
a. 2
b. 0
c. 3
d. 1
Answer:
Transform the equation into slope-intercept form.
x–y=3
y=x+3 (y = mx + b)
The y-intercept is (0,3).

202
20. Find an equation of the perpendicular bisector of the line segment joining the points A(1,
3) and B(5, -1).
a. x + y = 2
b. x – y = 2
c. x + y = 1
d. x – y = 1
Answer:
Recall that a perpendicular bisector is a line segment that forms a right
angle with aline and divides that line into two equal parts. Therefore, it passes
through the midpoint of that line.
M = ( (1+5)/2 , (3 -1)/2)
M = (3,1)
The midpoint of the line is (3,1).

Since the perpendicular bisector is perpendicular to that line, then its slope is
the negative reciprocal of the slope of that line. So we must first find the slope of
that line.
m = (-1-3)/ (5-1)
m = -1
Therefore, the slope of the perpendicular bisector is 1.
Given that the perpendicular bisector passes through point (3,1) and has the
slope 1, we can already use the point-slope form.
y – 1 = 1(x – 3)
y–1=x–3
x–y=2

6.6 Graphing Linear Inequalities


Graphing Linear Inequalities: y > mx + b, etc
Think about how you've done linear inequalites on the number line. For instance, they'd ask you
to graph something like x > 2. How did you do it? You would draw your number line, find the
"equals" part (in this case, x = 2), mark this point with the appropriate notation (an open dot or a
parenthesis, indicating that the point x = 2 wasn't included in the solution), and then you'd shade
everything to the right, because "greater than" meant "everything off to the right". The steps for
graphing two-variable linear inequalities are very much the same.
 Graph the solution to y < 2x + 3.
Just as for number-line inequalities, my first step is to find the "equals" part. For two-
variable linear inequalities, the "equals" part is the graph of the straight line; in this case,
that means the "equals" part is the line y = 2x + 3:

203
Now we're at the point where your book probably gets complicated, with talk of "test points" and
such. But when you did those one-variable inequalities (like x < 3), you didn't bother with "test
points"; you just shaded one side or the other. We can do the same here. Ignore the "test point"
stuff, and look at the original inequality: y < 2x + 3.

I've already graphed the "or equal to" part (it's just the line); now I'm ready to do the
"y less than" part. In other words, this is where I need to shade one side of the line or the
other. Now think about it: If I need y LESS THAN the line, do I want ABOVE the line, or
BELOW? Naturally, I want below the line. So I shade it in:

And that's all there is to it: the side I shaded is the "solution region" they want.

This technique worked because we had y alone on one side of the inequality. Just as with plain
old lines, you always want to "solve" the inequality for y on one side.
 Graph the solution to 2x– 3y < 6.
First, I'll solve for y:
2x – 3y < 6
–3y < –2x + 6
y > ( 2/3 )x – 2

204
[Note the flipped inequality sign in the last line. I mustn't forget to flip the inequality if I
multiply or divide through by a negative!] Copyright © Elizabeth Stapel 2000-2011 All
Rights Reserved
Now I need to find the "equals" part, which is the line y = ( 2/3 )x – 2. It looks like this:

But this exercise is what is called a "strict" inequality. That is, it isn't an "or equals to"
inequality; it's only "y greater than". When I had strict inequalities on the number line
(such as x< 3), I denote this by using a parenthesis (instead of a square bracket) or an
open [unfilled] dot (instead of a closed [filled] dot). In the case of these linear
inequalities, the notation for a strict inequality is a dashed line. So the border of my
solution region actually looks like this:

By using a dashed line, I still know where the border is, but I also know that the border
isn't included in the solution. Since this is a "y greater than" inequality, I want to
shade above the line, so my solution looks like this:

205
7 Polynomials
7.1 Properties of Exponents

Lesson Proper

Properties of Exponents:

1. Product Rule – when multiplying powers with the same base, just add the
exponents and keep the common base.

𝑎𝑛 ∙ 𝑎𝑚 = 𝑎𝑚+𝑛 , for positive integers m and n.

Example 4.1.1: x 2 ∙ x 3 = x 2+3 = x 5

2. Quotient Rule – when dividing powers with the same base, just subtract the
exponents and keep the common base.
𝑎𝑚
= 𝑎𝑚−𝑛 , if a ≠ 0 and if m>n
𝑎𝑛
𝑥 10 10−5
Example 4.1.2: = 𝑥 = 𝑥5
𝑥5

3. Power Rule - when raising power to a power, just multiply the exponents and keep
the base.

(𝑎𝑚 )𝑛 = 𝑎𝑚𝑛 , for positive integers m and n

Example 4.1.3: (x 2 )3 = x 2 ∙3 = x 6

4. Product to a Power Rule - when raising product to a power, just raise each factor to
the power.
(𝑎𝑏)𝑚 =𝑎𝑚 𝑏 𝑚 , for positive integers m

Example 4.1.4: (x 4 y 2 )2 = x 4∙2 y 2∙2 = x 8 y 4

5. Quotient to a Power Rule – when raising a quotient to a power, raise the numerator
and the denominator to the power.
𝑎 𝑛 𝑎𝑛
( ) = , if b ≠ 0
𝑏 𝑏𝑛
3
𝑥3 𝑥 3∙3 𝑥9
Example 4.1.5: ( 4 ) = =
𝑦 𝑦 4∙3 𝑦 12
6. Zero Exponents – any number raised to zero power is equal to “1”.

𝑎0 = 1
Example 4.1.6: (5𝑥 3 )0 = 1

7. Exponent of One - any number raised to the power of “1” is equal to itself.
𝑎1 = a
2 1 2
Example 4.1.7: (x ) = 𝑥

206
8. Negative Exponent – When raising a number to negative power, get the reciprocal
of that number and change the sign of the exponent from negative to positive.
1 1
𝑎−𝑛 = 𝑛 or −𝑛 = 𝑎𝑛
𝑎 𝑎
1 1
Example 4.1.8: 5−2 = =
52 25
3
Example 4.1.9: = 3𝑥 2
𝑥 −2

Lesson Exercise

Simplify the following expressions using the different properties of exponents.


1. (𝑥𝑦)5 = _____________
𝑎) x 8 y 4
𝑏) xy 5
𝑐) x 5 y 5
𝑑) x 25 y 25
Correct Answer: c)
Solution: (𝑥𝑦)5 = x1∙5 y1∙5 = x 5 y 5

𝑥6
2. = ______________
𝑥𝑦
𝑎) x 5 y
𝑥7
𝑏)
𝑦
𝑦7
𝑐)
𝑥
𝑥5
𝑑)
𝑦
Correct Answer: d)
𝑥6 𝑥 6−1 𝑥5
Solution: = =
𝑥𝑦 𝑦 𝑦

3. xy 2 ∙ x 3 y 3 = ______________
𝑎) x 3 y 6
𝑏) xy 5
𝑐) x 4 y 5
𝑑) x 3 y 9
Correct Answer: c)
Solution: xy 2 ∙ x 3 y 3 = x1+3 y 2+3 = x 4 y 5
2
2𝑥𝑦 2
4. ( ) = ______________
𝑧2
2𝑥𝑦 2
𝑎) 4
𝑧
4𝑥 2 𝑦 4
𝑏)
𝑧4
𝑐) 4x 2 y 4 𝑧 4
4𝑦 4
𝑑)
𝑧4

207
Correct Answer: 𝑏)
2
2𝑥𝑦 2 21∙2 𝑥 1∙2 𝑦 2∙2 2∙2 𝑥 2 𝑦 4 4𝑥 2 𝑦 4
Solution: ( ) = = =
𝑧2 𝑧 2∙2 𝑧4 𝑧4

5. (2x 2 y 2 )3 = ______________
𝑎) 8x 6 y 6
𝑏) 8x 8 y 8
𝑐) 2x 5 y 5
𝑑) 8x 5 y 5
Correct Answer: 𝑎)
Solution: (2x 2 y 2 )3 = 21∙3 x 2∙3 y 2∙3 = 23 x 6 y 6 = 8x 6 y 6

6. 𝑎−1 𝑥 −2 = ______________
1
𝑎) 2
𝑎𝑥
𝑏) 𝑎𝑥 2
1
𝑐) −1 −2
𝑎 𝑥
𝑎
𝑑) 2
𝑥
Correct Answer: 𝑎)
1 1
Solution: 𝑎−1 𝑥 −2 = 𝑎−1 reciprocal= , 𝑥 −2 reciprocal =
𝑎 𝑥2
1 1 1
= ∙ =
𝑎 𝑥2 𝑎𝑥 2

7. (4x 2 y 4 𝑧 4 )0 = ______________
𝑎) 1
𝑏) 0
𝑐) 4x 2 y 4 𝑧 4
𝑑) Undefined
Correct Answer: 𝑏)
Solution: (4x 2 y 4 𝑧 4 )0 = 0; Zero Exponent Rule

8. (4x 2 y 4 𝑧 4 )1 =______________
𝑎) 1
𝑏) 0
𝑐) 4x 2 y 4 𝑧 4
𝑑) Undefined
Correct Answer: 𝑐)
Solution: (4x 2 y 4 𝑧 4 )1 = 4x 2 y 4 𝑧 4 ; Exponent of One

10(𝑥+𝑦)4
9. = ______________
5(𝑥+𝑦)3
𝑎) 2𝑥 + 2𝑦
𝑏) 10𝑥 + 𝑦
𝑐) 5𝑥 + 𝑦
𝑑) 𝑥 + 2𝑦
Correct Answer: 𝑎)

208
10(𝑥+𝑦)4 10(𝑥 1∙4 +𝑦 1∙4 ) 10𝑥 1∙4 +10𝑦 1∙4 10𝑥 4 +10𝑦 4
Solution: = = = = 2𝑥 4−3 + 2𝑦 4−3
5(𝑥+𝑦)3 5(𝑥 1∙3 +𝑦 1∙3 ) 5𝑥 1∙3 +5𝑦 1∙3 5𝑥 3 +5𝑦 3
=2𝑥 + 2𝑦

10. (−3𝑦)4 = ______________


𝑎) −81−𝑦 −4
𝑏) − 81𝑦 −4
𝑐) − 81𝑦 4
𝑑) 81𝑦 4
Correct Answer: 𝑑)
Solution: (−3𝑦)4 = −31∙4 𝑦 1∙4 = −34 𝑦 4 =−3 ∙ −3 ∙ −3 ∙ −3 ∙ 𝑦 4 = 81𝑦 4

5𝑥 2
11. = ______________
25𝑥 5
𝑎) 5𝑥 3
𝑥3
𝑏)
5
1
𝑐) 3
5𝑥
𝑑) −5𝑥 3
Correct Answer: 𝑐)
5𝑥 2 1 𝑥 −3 1 1 1
Solution: = 𝑥 2−5 = = ∙ =
25𝑥 5 5 5 5 𝑥3 5𝑥 3

1 −2
12. ( ) = ______________
4
1
𝑎)
8
1
𝑏)
16
1
𝑐)
−16
𝑑) 16
Correct Answer: 𝑑)
1 −2 11∙(−2) 1−2 42
Solution: ( ) = = = = 16
4 41∙(−2) 4 −2 12

13.(3𝑧)2 (6𝑧 2 )−3 = ______________


1
𝑎)
24𝑧 2
1
𝑏)
24𝑧 4
𝑐) 18𝑧 2
𝑑) −9𝑧 2
Correct Answer: 𝑏)
1 1
Solution: (3𝑧)2 (6𝑧 2 )−3 = (31∙2 𝑧 1∙2 )(61∙(−3) 𝑧 2∙(−3) ) = (32 𝑧 2 )(6−3 𝑧 −6 ) = (9𝑧 2 ) ( 3 ∙ ) =
6 𝑧6
9𝑧 2
63 ∙𝑧 6
9𝑧 2 1 1
= = 𝑧 −4 =
216𝑧 6 24 24𝑧 4

209
3
14. = ______________
3−2
1
𝑎)
3
𝑏) 27
1
𝑐)
27
𝑑) −3
Correct Answer: 𝑏)
3 3
Solution: −2 = 1 = 3 ∙ 32 = 3 ∙ 9 =27
3
32

𝑥 9 (2𝑥)4
15. = ______________
𝑥3

𝑎) 16𝑥 10
1
𝑏)
16𝑥 10
𝑐) 2𝑥 13
𝑑) −16𝑥 10
Correct Answer: 𝑎)
𝑥 9 (2𝑥)4 𝑥 9 (21∙4 𝑥 1∙4 ) 𝑥 9 (24 𝑥 4 ) 𝑥 9 (16𝑥 4 ) 16𝑥 9+4 16𝑥 13
Solution: = = = = = = 16𝑥 13−3 = 16𝑥 10
𝑥3 𝑥3 𝑥3 𝑥3 𝑥3 𝑥3

16. 𝑎9 𝑎−5 = ______________


1
𝑎) 4
𝑎
𝑏) 𝑎4
𝑐) −𝑎4
1
𝑑) 4
−𝑎
Correct Answer: 𝑏)
Solution: 𝑎9 𝑎−5 = 𝑎9+(−5) = 𝑎4
32 ∙4−2 ∙5
17. = ______________
2−4 ∙33 ∙25
1
𝑎)
15
𝑏) 15
𝑐) − 15
𝑑) 108
Correct Answer: 𝑎)
32 ∙4−2 ∙5 32 ∙24 5 9∙16∙5 720 1
Solution: = = = =
2−4 ∙33 ∙25 42 ∙33 ∙25 16∙27∙25 10,800 15

18. −33 = ______________


1
𝑎)
27
𝑏) − 27
𝑐) − 9
𝑑) 27
Correct Answer: 𝑏)
Solution: −33 = −3 ∙ −3 ∙ −3 = −27

210
19. −60 = ______________
𝑎) 1
𝑏) 0
𝑐) 6
𝑑) −6
Correct Answer: 𝑏)
Solution: −60 = 0

20. 3−1 3−3 = ______________


1
𝑎)
27
𝑏) − 27
𝑐) − 9
𝑑) 27
Correct Answer: 𝑎)
1 1 1
Solution: 3−1 3−3 = 1 ∙ =
3 33 27

7.2 Addition and Subtraction on Polynomials


Lesson Proper

Adding Polynomials
To add polynomials, combine their like terms (terms with the same variables
raised to the same powers) using the Distributive Property.
Distributive Property: 𝑎𝑐 + 𝑏𝑐 = (𝑎 + 𝑏)c

Example 7.2.1: Find the sum


1) 5𝑥 2 + 2𝑥 2 = (5 + 3)𝑥 2 = 8𝑥 2
2) (𝑥 3 − 6𝑥 2 + 2𝑥 + 4 ) + (𝑥 3 + 5𝑥 2 − 7𝑥)
= (𝑥 3 + 𝑥 3 ) + (−6𝑥 2 + 5𝑥 2 ) + (2𝑥 − 7𝑥 ) + 4 Group like terms
= 2𝑥 3 − 𝑥 2 − 5𝑥 + 4 Combine like
terms

Subtracting Polynomials
To subtract polynomials, if a minus sign precedes an expression in parentheses,
then the sign of every term within the parentheses is changed when we remove the
parentheses:
−(𝑎 + 𝑏) = −𝑎 − 𝑏
Distributive Property is used to distribute the sign outside the parentheses.

Example 7.2.2: Find the difference


(𝑥 3 − 6𝑥 2 + 2𝑥 + 4 ) - (𝑥 3 + 5𝑥 2 − 7𝑥)
= 𝑥 3 − 6𝑥 2 + 2𝑥 + 4 − 𝑥 3 − 5𝑥 2 + 7𝑥 Distributive Property
= (𝑥 3 − 𝑥 3 ) + (−6𝑥 2 − 5𝑥 2 ) + (2𝑥 + 7𝑥 ) + 4 Group like terms
=11𝑥 2 + 9𝑥 + 4 Combine like terms

Lesson Exercise
Perform the indicated operations and simplify

211
1. (12𝑥 − 7 )– (5𝑥 − 12) =_____________
𝑎) 7x + 5
𝑏) 7x − 5
𝑐) 5x − 5
𝑑) −7x + 5
Correct Answer: 𝑎)
Solution: (12𝑥 − 7 )– (5𝑥 − 12)
= 12𝑥 − 7 − 5𝑥 + 12
= (12𝑥 − 5𝑥) + (−7 + 12)
= 7x + 5

2. (3𝑥 2 + 𝑥 + 1 ) + (2𝑥 2 − 3𝑥 − 5) =_____________


𝑎) 5x 3 − 2x − 4
𝑏) 5x 2 − 2x − 4
𝑐) 5x − 5
𝑑) 5x 2 + 2x + 4
Correct Answer: 𝑏)
Solution: (3𝑥 2 + 𝑥 + 1 ) + (2𝑥 2 − 3𝑥 − 5)
= 3𝑥 2 + 𝑥 + 1 + 2𝑥 2 − 3𝑥 − 5
= (3𝑥 2 + 2𝑥 2 ) + (𝑥 − 3𝑥) + (1 − 5)
= 5𝑥 2 – 2𝑥 − 4

3. (𝑥 3 + 6𝑥 2 − 4𝑥 + 7 )– (3𝑥 2 + 2𝑥 − 4) =_____________
𝑎) x 3 − 3x 2 + 6x − 11
𝑏) x 3 − 3x 2 − 6x − 11
𝑐) x 3 + 3x 2 + 6x + 11
𝑑) x 3 + 3x 2 − 6x + 11
Correct Answer: 𝑑)
Solution: (𝑥 3 + 6𝑥 2 − 4𝑥 + 7 )– (3𝑥 2 + 2𝑥 − 4)
= 𝑥 3 + 6𝑥 2 − 4𝑥 + 7 - 3𝑥 2 − 2𝑥 + 4
= 𝑥 3 + (6𝑥 2 − 3𝑥 2 ) + (−4𝑥 − 2𝑥) + (7 + 4)
= x 3 + 3x 2 − 6x + 11

4. 8(2𝑥 + 5 )– 7(𝑥 − 9) =_____________


𝑎) −9x − 103
𝑏) − 9x + 103
𝑐) 9x + 103
𝑑) 9x − 103
Correct Answer: 𝑐)
Solution: 8(2𝑥 + 5 )– 7(𝑥 − 9)
= 16𝑥 + 40 − 7𝑥 + 63
= (16𝑥 − 7𝑥 ) + (40 + 63) = 9x + 103

5. 2(2 − 5𝑡 ) + 𝑡 2 (𝑡 − 1) − (𝑡 4 − 1)=_____________
𝑎) t 4 + t 3 − t 2 + 10t + 5
𝑏 − t 4 − t 3 – t 2 − 10t − 5

212
𝑐)t 4 + t 3 + t 2 + 10t + 5
𝑑) −t 4 + t 3 − t 2 − 10t + 5
Correct Answer: 𝑑)
Solution: 2(2 − 5𝑡 ) + 𝑡 2 (𝑡 − 1) − (𝑡 4 − 1)
= 4 − 10𝑡 + t 3 − t 2 − 𝑡 4 + 1
=−𝑡 4 + t 3 − t 2 − 10𝑡 + (4 + 1)
= −t 4 + t 3 − t 2 − 10t + 5

6. (5 − 3𝑥 ) + (2𝑥 − 8) =_____________
𝑎) −𝑥 − 3
𝑏𝑥−3
𝑐) 𝑥 + 3
𝑑) −𝑥 + 3
Correct Answer: 𝑎)
Solution: (5 − 3𝑥 ) + (2𝑥 − 8)
= 5 − 3𝑥 + 2𝑥 − 8
= (−3𝑥 + 2𝑥) + (5 − 8)
= −𝑥 − 3

7. (3𝑥 2 + 𝑥 + 1 )– (2𝑥 2 − 3𝑥 − 5) =_____________


𝑎) −𝑥 2 + 4𝑥 + 6
𝑏 𝑥 2 + 4𝑥 + 6
𝑐) 𝑥 2 − 4𝑥 − 6
𝑑) 𝑥 2 − 4𝑥 + 6
Correct Answer: 𝑏)
Solution: (3𝑥 2 + 𝑥 + 1 )– (2𝑥 2 − 3𝑥 − 5)
= 3𝑥 2 + 𝑥 + 1 − 2𝑥 2 + 3𝑥 + 5
= (3𝑥 2 − 2𝑥 2 ) + (𝑥 + 3𝑥) + (1 + 5)
= 𝑥 2 + 4𝑥 + 6

8. 3(𝑥 − 1 ) + 4(𝑥 + 2) =_____________


𝑎) 7𝑥 + 5
𝑏 7𝑥 − 5
𝑐) − 7𝑥 − 5
𝑑) 7𝑥 - 9
Correct Answer: 𝑎)
Solution: 3(𝑥 − 1 ) + 4(𝑥 + 2
= 3𝑥 − 3 + 4𝑥 + 8
= (3𝑥 + 4𝑥) + (−3 + 8)
= 7𝑥 + 5

9. 4(𝑥 2 − 3𝑥 + 5 ) − 3(𝑥 2 − 2𝑥 + 1) =_____________


𝑎) 𝑥 2 + 6𝑥 + 17
𝑏) 𝑥 2 − 6𝑥 + 17
𝑐) − 𝑥 2 − 6𝑥 + 17
𝑑) 𝑥 2 − 6𝑥 − 17

213
Correct Answer: 𝑏)
Solution: 4(𝑥 2 − 3𝑥 + 5 ) − 3(𝑥 2 − 2𝑥 + 1)
= 4𝑥 2 − 12𝑥 + 20 − 3𝑥 2 + 6𝑥 − 3
= (4𝑥 2 − 3𝑥 2 ) + (−12𝑥 + 6𝑥) + (20 − 3)
= 𝑥 2 − 6𝑥 + 17

10. (4x 5 + x 4 -12x 3 + x - 6) + (3x 4 +8x 3 +6x 2 - x) =_____________


𝑎) 4x 5 + 4𝑥 4 - 4x 3 +6x 2 - 6
𝑏) 4x 5 - 4𝑥 4 - 4x 3 -6x 2 - 6
𝑐) 4x 5 + 4𝑥 4 + 4x 3 +6x 2 - 6
𝑑) 4x 5 + 4𝑥 4 - 4x 3 +6x 2 +6
Correct Answer: 𝑎)
Solution: (4x 5 + x 4 -12x 3 + x - 6) + (3x 4 +8x 3 +6x 2 - x)
= 4x 5 + x 4 -12x 3 + x – 6 + 3x 4 +8x 3+6x 2 – x
=4x 5 +(x 4+ 3x 4) + (-12x 3+8x 3) + 6x 2 + (x – x) – 6
= 4x 5 + 4𝑥 4 - 4x 3 +6x 2 – 6

11. (- 3x 4 -2x 2 + x + 17) + (14x 6 +12x 5 -2x 4 +7x 3 +2x 2 - 4x - 12) =_____________
𝑎) 14x 6 +12x 5 -5x 4 +7x 3 - 3x - 5
𝑏) 14x 6 +12x 5 -5x 4 +7x 3 + 3x + 5
𝑐) 14x 6 -12x 5 -5x 4 -7x 3 - 3x + 5
𝑑) 14x 6 +12x 5 -5x 4 +7x 3 - 3x + 5
Correct Answer: 𝑑)
Solution: (- 3x 4 -2x 2 + x + 17) + (14x 6 +12x 5 -2x 4 +7x 3 +2x 2 - 4x - 12)
= 14x 6 +12x 5 + (- 3 - 2) x 4 +7x 3 + (- 2 + 2) x 2 + (1 - 4) x + (17 - 12)
= 14x 6 +12x 5 -5x 4 +7x 3 +0x 2 - 3x + 5
= 14x 6 +12x 5 -5x 4 +7x 3 - 3x + 5

12. (3x 4 +4x 3 -5x 2 +18) - (7x 5 +3x 4 -12x 3 -3x 2 + 7x - 9) =_____________
a) - 7x 5 +16x 3 -2x 2 - 7x + 27
b) - 7x 5 - 16x 3 -2x 2 - 7x + 27
c) 7x 5 +16x 3 -2x 2 - 7x + 27
d) 7x 5 +16x 3 -2x 2 - 7x - 27
Correct Answer: 𝑎)
Solution: (3x 4 +4x 3 -5x 2 +18) - (7x 5 +3x 4 -12x 3 -3x 2 + 7x - 9)
= (3x 4 +4x 3 -5x 2 +18) + (- 7x 5 -3x 4 +12x 3 +3x 2 - 7x + 9)
= - 7x 5 + (3 - 3)x 4 + (4 + 12)x 3 + (- 5 + 3)x 2 - 7x + (18 + 9)
= - 7x 5 +16x 3 -2x 2 - 7x + 27

13. (- 5x 6 -3x 4 +18x 3 -7x 2 +9x + 11) - (- x 4 + x 3 -12x 2 + 10x + 5) =_____________


a) -5x 6 -2x 4 +17x 3 +5x 2 - x - 6
b) - 5x 6 -2x 4 +17x 3 +5x 2 - x + 6
c) 5x 6 +2x 4 +17x 3 +5x 2 + x + 6
d) - 5x 6 +2x 4 +17x 3 +5x 2 - x + 6

Correct Answer: 𝑏)

214
Solution: (- 5x 6 -3x 4 +18x 3 -7x 2 +9x + 11) - (- x 4 + x 3 -12x 2 + 10x + 5)
= (- 5x 6 -3x 4 +18x 3 -7x 2 +9x + 11) + (x 4 - x 3 +12x 2 - 10x - 5)
= - 5x 6 + (- 3 + 1)x 4 + (18 - 1)x 3 + (- 7 + 12)x 2 + (9 - 10)x + (11 - 5)
= - 5x 6 -2x 4 +17x 3 +5x 2 - x + 6

14. (2𝑥 + 1) − 5(4𝑥 − 1) =_____________


𝑎) −18𝑥 − 6
𝑏 18𝑥 − 6
𝑐) 18𝑥 + 6
𝑑) −18𝑥 + 6

Correct Answer: 𝑑)
Solution: (2𝑥 + 1) − 5(4𝑥 − 1)
= 2𝑥 + 1 − 20𝑥 + 5
= (2𝑥 − 20𝑥) + (1 + 5)
= −18𝑥 + 6

15. (2𝑦 − 7) + (2𝑥 − 1)=_____________


𝑎) −2𝑦 − 2𝑥 − 8
𝑏 2𝑦 + 2𝑥 − 8
𝑐) 2𝑦 − 2𝑥 − 8
𝑑) 2𝑦 + 2𝑥 + 8

Correct Answer: 𝑏)
Solution: (2𝑦 − 7) + (2𝑥 − 1)
= 2𝑦 − 7 + 2𝑥 − 1
= 2𝑦 + 2𝑥 + (−7 − 1)
= 2𝑦 + 2𝑥 − 8

16. 3(𝑥 + 6) + 4(2𝑥 − 5)=_____________


𝑎) −11𝑥 + 2
𝑏 − 11𝑥 − 2
𝑐) 11𝑥 − 2
𝑑) 11𝑥 + 2

Correct Answer: 𝑐)
Solution: 3(𝑥 + 6) + 4(2𝑥 − 5)
= 3𝑥 + 18 + 8𝑥 − 20
= (3𝑥 + 8𝑥) + (18 − 20)
= 11𝑥 -2

17. (2𝑥 + 5) − 2(𝑥 2 + 𝑥 + 3) =_____________


𝑎) −2𝑥 2 − 1
𝑏) − 2𝑥 2 + 1
𝑐) 2𝑥 2 − 1
𝑑) 2𝑥 2 + 1

Correct Answer: 𝑎)
Solution: (2𝑥 + 5) − 2(𝑥 2 + 𝑥 + 3)

215
= 2𝑥 + 5 − 2𝑥 2 − 2𝑥 − 6
= −2𝑥 2 + (2𝑥 − 2𝑥) + (5 − 6)
= −2𝑥 2 + 0𝑥 − 1
= −2𝑥 2 − 1

18. (𝑥 3 + 𝑥 − 5) − 1(3𝑥) =_____________


𝑎) 𝑥 3 − 2𝑥 + 5
𝑏) − 𝑥 3 − 2𝑥 − 5
𝑐) 𝑥 3 + 2𝑥 − 5
𝑑) 𝑥 3 − 2𝑥 − 5

Correct Answer: 𝑎)
Solution: (𝑥 3 + 𝑥 − 5) − 1(3𝑥)
= 𝑥 3 + 𝑥 − 5 − 3𝑥
= 𝑥 3 + (𝑥 − 3𝑥) − 5
= 𝑥 3 − 2𝑥 − 5

19. (3𝑥 + 5 − 4) + (𝑥 + 2)
𝑎) 4𝑥 + 3
𝑏) 4𝑥 − 3
𝑐) − 4𝑥 + 3
𝑑) −4𝑥 − 3

Correct Answer: 𝑎)
Solution: (3𝑥 + 5 − 4) + (𝑥 + 2)
= 3𝑥 + 5 − 4 + 𝑥 + 2
= (3𝑥 + 𝑥) + (5 − 4 + 2)
= 4𝑥 + 3

20. 3(𝑥 3 + 𝑥 2 ) − (𝑥 2 − 𝑥 + 2)
𝑎) 3𝑥 3 − 2𝑥 2 + 𝑥 − 2
𝑏) − 3𝑥 3 + 2𝑥 2 + 𝑥 − 2
𝑐) 3𝑥 3 + 2𝑥 2 + 𝑥 + 2
𝑑) 3𝑥 3 + 2𝑥 2 + 𝑥 − 2

Correct Answer: 𝑎)
Solution: 3(𝑥 3 + 𝑥 2 ) − (𝑥 2 − 𝑥 + 2)
= 3𝑥 3 + 3𝑥 2 − 𝑥 2 + 𝑥 − 2
= 3𝑥 3 + (3𝑥 2 − 𝑥 2 ) + 𝑥 − 2
= 3𝑥 3 + 2𝑥 2 + 𝑥 − 2

7.3 Multiplication of Polynomials


Multiplying Polynomials

To find the product of the polynomials use Distributive Property repeatedly.


Recall that:
Distributive Property: 𝑎𝑐 + 𝑏𝑐 = (𝑎 + 𝑏)c
Example:
Using the distributive property in finding the product of two binomials:

216
(𝑎 + 𝑏)(𝑐 + 𝑑) = 𝑎(𝑐 + 𝑑) + 𝑏(𝑐 + 𝑑) = 𝑎𝑐 + 𝑎𝑑 + 𝑏𝑐 + 𝑏𝑑

FOIL method helps you remember that the product of two binomials is the sum of
the products of the First terms, Outer terms, Inner terms, and the Last terms.
Example:
Using the FOIL method in finding the product of two binomials after applying the
Distributive Property:

(𝑎 + 𝑏)(𝑐 + 𝑑) = 𝑎𝑐 + 𝑎𝑑 + 𝑏𝑐 + 𝑏𝑑

F O I L

Examples: Find the product of the following.


1. (2𝑥 + 1)(3𝑥 − 5)
= 6𝑥 2 − 10𝑥 + 3𝑥 − 5 Distributive Property

F O I L
= 6𝑥 2 − 7𝑥 − 5 Combine like terms

2. (𝑥 2 − 3)(𝑥 3 + 2𝑥 + 1)
= 𝑥 2 (𝑥 3 + 2𝑥 + 1) − 3(𝑥 3 + 2𝑥 + 1) Distributive Property
= 𝑥 5 + 2𝑥 3 + 𝑥 2 − 3𝑥 3 − 6𝑥 − 3 Distributive Property
= 𝑥 5 − 𝑥 3 + 𝑥 2 − 6𝑥 − 3 Combine like terms

Also, you can use the table form to multiply trinomials or other polynomials:
Example: Find the product.
(𝑥 2 + 2𝑥 − 1)(2𝑥 2 − 4𝑥 + 3)
Solution:
𝑥 2 + 2𝑥 − 1
__2x 2 − 4x + 3
2𝑥 4 + 4𝑥 3 − 2𝑥 2
−4𝑥 3 − 8𝑥 2 + 4𝑥
________+ 3𝑥 2 + 6𝑥 − 3
2𝑥 4 − 7𝑥 2 + 10𝑥 − 3
4 2
Product: 2𝑥 − 7𝑥 + 10𝑥 − 3

Lesson Exercise
Give the product of the following:

1. 𝑥 2 (2𝑥 2 − 𝑥 + 1)= _____________


𝑎) 2𝑥 4 + 𝑥 3 + 𝑥 2
𝑏) − 2𝑥 4 − 𝑥 3 + 𝑥 2
𝑐) − 2𝑥 4 − 𝑥 3 − 𝑥 2
𝑑) 2𝑥 4 − 𝑥 3 + 𝑥 2
Correct Answer: 𝑑)

217
Solution: 𝑥 2 (2𝑥 2 − 𝑥 + 1)
=2𝑥 4 − 𝑥 3 + 𝑥 2

2. (3𝑡 − 2)(7𝑡 − 5) = _____________


𝑎) 21𝑡 2 − 29𝑡 + 10
𝑏) 21𝑡 2 − 29𝑡 − 10
𝑐) 21𝑡 2 + 29𝑡 − 10
𝑑) −21𝑡 2 − 29𝑡 − 10
Correct Answer: 𝑎)
Solution: (3𝑡 − 2)(7𝑡 − 5)
= 3𝑡(7𝑡 − 5) + −2(7𝑡 − 5)
= 21𝑡 2 − 15𝑡 + (−14𝑡 + 10)
= 21𝑡 2 − 15𝑡 − 14𝑡 + 10
=21𝑡 2 + (−15𝑡 − 14𝑡) + 10
= 21𝑡 2 − 29𝑡 + 10

3. (𝑥 + 2𝑦)(3𝑥 − 𝑦) = _____________
𝑎) 3𝑥 2 − 5𝑥𝑦 − 2𝑦 2
𝑏) 3𝑥 2 + 5𝑥𝑦 − 2𝑦 2
𝑐) 3𝑥 2 + 5𝑥𝑦 + 2𝑦 2
𝑑) −3𝑥 2 + 5𝑥𝑦 + 2𝑦 2
Correct Answer: 𝑏)
Solution: (𝑥 + 2𝑦)(3𝑥 − 𝑦)
= 𝑥(3𝑥 − 𝑦) + 2𝑦(3𝑥 − 𝑦)
= 3𝑥 2 − 𝑥𝑦 + 6𝑥𝑦 − 2𝑦 2
= 3𝑥 2 + (−𝑥𝑦 + 6𝑥𝑦) − 2𝑦 2
= 3𝑥 2 + 5𝑥𝑦 − 2𝑦 2

4. (4𝑥 − 1)(3𝑥 + 7) = _____________


𝑎) −12𝑥 2 + 25𝑥 − 7
𝑏) 12𝑥 2 − 25𝑥 − 7
𝑐) 12𝑥 2 + 25𝑥 + 7
𝑑) 12𝑥 2 + 25𝑥 − 7
Correct Answer: 𝑑)
Solution: (4𝑥 − 1)(3𝑥 + 7)
= 4𝑥(3𝑥 + 7) + −1(3𝑥 + 7)
= 12𝑥 2 + 28𝑥 + (−3𝑥 − 7)
= 12𝑥 2 + 28𝑥 − 3𝑥 − 7
= 12𝑥 2 + (28𝑥 − 3𝑥) − 7)
= 12𝑥 2 + 25𝑥 − 7

5. (4𝑥 − 3)(2𝑥 + 5𝑦) = _____________


𝑎) −8𝑥 2 + 20𝑥𝑦 − 6𝑥 − 15𝑦
𝑏) 8𝑥 2 + 20𝑥𝑦 − 6𝑥 + 15𝑦
𝑐) 8𝑥 2 + 20𝑥𝑦 − 6𝑥 − 15𝑦
𝑑) 8𝑥 2 + 20𝑥𝑦 + 6𝑥 − 15𝑦

218
Correct Answer: 𝑐)
Solution: (4𝑥 − 3)(2𝑥 + 5𝑦)
= 4𝑥(2𝑥 + 5𝑦) − 3(2𝑥 + 5𝑦)
= 8𝑥 2 + 20𝑥𝑦 − 6𝑥 − 15𝑦

6. 3𝑥 3 (𝑥 4 − 4𝑥 2 + 5) = _____________
𝑎) 3𝑥 7 − 12𝑥 5 + 15𝑥 3
𝑏) −3𝑥 7 + 12𝑥 5 + 15𝑥 3
𝑐) 3𝑥 7 + 12𝑥 5 + 15𝑥 3
𝑑) 3𝑥 7 − 12𝑥 5 − 15𝑥 3

Correct Answer: 𝑎)
Solution: 3𝑥 3 (𝑥 4 − 4𝑥 2 + 5)
= 3𝑥 3+4 − 12𝑥 3+2 + 15𝑥 3
= 3𝑥 7 − 12𝑥 5 + 15𝑥 3

7. (2𝑦 − 7)(2𝑦 + 7) = _____________


𝑎) 4𝑦 2 + 28𝑦 − 49
𝑏) 4𝑦 2 − 49
𝑐) − 4𝑦 2 − 49
𝑑) 4𝑦 2 + 49
Correct Answer: 𝑏)
Solution: (2𝑦 − 7)(2𝑦 + 7)
= 2𝑦(2𝑦 + 7) − 7(2𝑦 + 7)
= 4𝑦 2 + 14𝑦 − 14𝑦 − 49
= 4𝑦 2 − 49

8. 5(2𝑥𝑦 + 1)(2𝑦 + 3) = _____________


𝑎) 20𝑥𝑦 + 30𝑥 − 10𝑦 2 + 25𝑦 − 15
𝑏) - 20𝑥𝑦 − 30𝑥 + 10𝑦 2 + 25𝑦 + 15
𝑐)20𝑥𝑦 − 30𝑥 + 10𝑦 2 + 25𝑦 − 15
𝑑) 20𝑥𝑦 + 30𝑥 + 10𝑦 2 + 25𝑦 + 15
Correct Answer: 𝑑)
Solution: 5(2𝑥𝑦 + 1)(2𝑦 + 3)
= (10𝑥5𝑦 + 5)(2𝑦 + 3)
= 10𝑥(2𝑦 + 3) + 5𝑦(2𝑦 + 3) + 5(2𝑦 + 3)
= 20𝑥𝑦 + 30𝑥 + 10𝑦 2 + 15𝑦 + 10𝑦 + 15
= 20𝑥𝑦 + 30𝑥 + 10𝑦 2 + (15𝑦 + 10𝑦) +15
= 20𝑥𝑦 + 30𝑥 + 10𝑦 2 + 25𝑦 + 15

9. 2(𝑦 + 𝑥)(𝑦 + 3𝑥) = _____________


𝑎) 12𝑥𝑦 + 4𝑦 2
𝑏) - 12𝑥𝑦 + 4𝑦 2
𝑐) 12𝑥𝑦 − 4𝑦 2
𝑑) 12𝑥 + 4𝑦 2
Correct Answer: 𝑑)

219
Solution: 2(𝑦 + 𝑥)(𝑦 + 3𝑥)
= (2𝑦 + 2𝑥)(𝑦 + 3𝑥)
= 2𝑦(𝑦 + 3𝑥) + 2𝑦(𝑦 + 3𝑥)
= 2𝑦 2 + 6𝑥𝑦 + 2𝑦 2 + 6𝑥𝑦
= 12𝑥𝑦 + 4𝑦 2

10. 4(5𝑥 − 1)(𝑥 + 𝑦) = _____________


𝑎) 20𝑥 2 − 20𝑥𝑦 + 4𝑥 − 4𝑦
𝑏) 20𝑥 2 + 20𝑥𝑦 − 4𝑥 − 4𝑦
𝑐) 20𝑥 2 − 20𝑥𝑦 − 4𝑥 + 4𝑦
𝑑) − 20𝑥 2 − 20𝑥𝑦 − 4𝑥 − 4𝑦
Correct Answer: 𝑏)
Solution: 4(5𝑥 − 1)(𝑥 + 𝑦)
= (20𝑥 − 4)(𝑥 + 𝑦)
= 20𝑥(𝑥 + 𝑦) − 4(𝑥 + 𝑦)
= 20𝑥 2 + 20𝑥𝑦 − 4𝑥 − 4𝑦

11. (𝑥 − 𝑦 − 𝑧)4(2𝑥 − 𝑦) = _____________


𝑎) 8𝑥 2 − 12𝑥𝑦 + 4𝑥𝑦 2 − 8𝑥𝑧 + 4𝑦𝑧
𝑏) - 8𝑥 2 − 12𝑥𝑦 + 4𝑥𝑦 2 − 8𝑥𝑧 + 4𝑦𝑧
𝑐) 8𝑥 2 + 12𝑥𝑦 + 4𝑥𝑦 2 − 8𝑥𝑧 + 4𝑦𝑧
𝑑)8𝑥 2 − 12𝑥𝑦 + 4𝑥𝑦 2 + 8𝑥𝑧 + 4𝑦𝑧
Correct Answer: 𝑎)
Solution: (𝑥 − 𝑦 − 𝑧)4(2𝑥 − 𝑦)
= (𝑥 − 𝑦 − 𝑧)(8𝑥 − 4𝑦)
= 𝑥(8𝑥 − 4𝑦) − 𝑦(8𝑥 − 4𝑦) − 𝑧(8𝑥 − 4𝑦)
= 8𝑥 2 − 4𝑥𝑦 − 8𝑥𝑦 + 4𝑥𝑦 2 − 8𝑥𝑧 + 4𝑦𝑧
= 8𝑥 2 + (−4𝑥𝑦 − 8𝑥𝑦) + 4𝑥𝑦 2 − 8𝑥𝑧 + 4𝑦𝑧
= 8𝑥 2 − 12𝑥𝑦 + 4𝑥𝑦 2 − 8𝑥𝑧 + 4𝑦𝑧

12. 8(3𝑥 + 2𝑦)(𝑦 − 𝑧) = _____________


𝑎) 24𝑥𝑦 − 24𝑥𝑧 + 16𝑦 2 − 16𝑦𝑧
𝑏) - 24𝑥𝑦 − 24𝑥𝑧 + 16𝑦 2 − 16𝑦𝑧
𝑐) 24𝑥𝑦 + 24𝑥𝑧 + 16𝑦 2 − 16𝑦𝑧
𝑑)24𝑥𝑦 − 24𝑥𝑧 + 16𝑦 2 + 16𝑦𝑧
Correct Answer: 𝑎)
Solution: 8(3𝑥 + 2𝑦)(𝑦 − 𝑧)
= (24𝑥 + 16𝑦)(𝑦 − 𝑧)
= 24𝑥(𝑦 − 𝑧) + 16𝑦(𝑦 − 𝑧)
= 24𝑥𝑦 − 24𝑥𝑧 + 16𝑦 2 − 16𝑦𝑧

13. −2(5𝑥 − 𝑦 + 2𝑧)(𝑦 − 3) = _____________


𝑎) − 10𝑥𝑦 − 30𝑥 + 2𝑦 2 − 6𝑦 − 4𝑧𝑦 + 12𝑧
𝑏) 10𝑥𝑦 + 30𝑥 + 2𝑦 2 − 6𝑦 − 4𝑧𝑦 + 12𝑧
𝑐) − 10𝑥𝑦 + 30𝑥 + 2𝑦 2 − 6𝑦 − 4𝑧𝑦 + 12𝑧
𝑑) − 10𝑥𝑦 + 30𝑥 − 2𝑦 2 − 6𝑦 − 4𝑧𝑦 + 12𝑧

220
Correct Answer: 𝑐)
Solution: −2(5𝑥 − 𝑦 + 2𝑧)(𝑦 − 3)
= (−10𝑥 + 2𝑦 − 4𝑧)(𝑦 − 3)
= −10𝑥(𝑦 − 3) + 2𝑦(𝑦 − 3) + −4𝑧(𝑦 − 3)
= −10𝑥𝑦 + 30𝑥 + 2𝑦 2 − 6𝑦 − 4𝑧𝑦 + 12𝑧

14. (𝑥 + 4)(𝑥 − 2) = _____________


𝑎) 𝑥 2 + 2𝑥 − 8
𝑏) 𝑥 2 − 2𝑥 − 8
𝑐) 𝑥 2 + 2𝑥 − 8
𝑑) − 𝑥 2 + 2𝑥 + 8
Correct Answer: 𝑎)
Solution: (𝑥 + 4)(𝑥 − 2)
= 𝑥(𝑥 − 2)4(𝑥 − 2)
= 𝑥 2 − 2𝑥 + 4𝑥 − 8
= 𝑥 2 + 2𝑥 − 8

15. (𝑥 − 2)(𝑦 − 5) = _____________


𝑎) − 𝑥𝑦 + 5𝑥 − 2𝑦 + 10
𝑏) −𝑥𝑦 − 5𝑥 − 2𝑦 + 10
𝑐) 𝑥𝑦 + 5𝑥 + 2𝑦 + 10
𝑑) 𝑥𝑦 − 5𝑥 − 2𝑦 + 10
Correct Answer: 𝑑)
Solution: (𝑥 − 2)(𝑦 − 5)
= 𝑥(𝑦 − 5) − 2(𝑦 − 5)
= 𝑥𝑦 − 5𝑥 − 2𝑦 + 10

16. (5𝑥 − 5𝑦)(𝑦 + 1) = _____________


𝑎) 5𝑥𝑦 − 5𝑥 − 5𝑦 2 − 5𝑦
𝑏) 5𝑥𝑦 + 5𝑥 + 5𝑦 2 − 5𝑦
𝑐) 5𝑥𝑦 + 5𝑥 − 5𝑦 2 − 5𝑦
𝑑) 5𝑥𝑦 + 5𝑥 − 5𝑦 2 + 5𝑦
Correct Answer: 𝑐)
Solution: (5𝑥 − 5𝑦)(𝑦 + 1)
= 5𝑥(𝑦 + 1) − 5𝑦(𝑦 + 1)
= 5𝑥𝑦 + 5𝑥 − 5𝑦 2 − 5𝑦

17. −𝑥(𝑥 − 𝑦 − 𝑧) = _____________


𝑎) −𝑥 2 + 𝑥𝑦 + 𝑥𝑧
𝑏) −𝑥 2 + 𝑥𝑦 − 𝑥𝑧
𝑐) 𝑥 2 + 𝑥𝑦 − 𝑥𝑧
𝑑) 𝑥 2 − 𝑥𝑦 + 𝑥𝑧
Correct Answer: 𝑎)
Solution: −𝑥(𝑥 − 𝑦 − 𝑧)
= −𝑥 2 + 𝑥𝑦 + 𝑥𝑧

221
18. (2𝑥 − 3𝑧)(𝑦 − 8) = _____________
𝑎) 2𝑥𝑦 − 16𝑥 + 3𝑧𝑦 + 24𝑧
𝑏) 2𝑥𝑦 − 16𝑥 − 3𝑧𝑦 + 24𝑧
𝑐) − 2𝑥𝑦 − 16𝑥 − 3𝑧𝑦 + 24𝑧
𝑑) 2𝑥𝑦 + 16𝑥 + 3𝑧𝑦 + 24𝑧

Correct Answer: 𝑏)
Solution: (2𝑥 − 3𝑧)(𝑦 − 8)
= 2𝑥(𝑦 − 8) − 3𝑧(𝑦 − 8)
= 2𝑥𝑦 − 16𝑥 − 3𝑧𝑦 + 24𝑧
1
19. (𝑥 − 𝑦)(𝑦 − 2) = _____________
2
1 1
𝑎) 𝑥𝑦 + 𝑥 + 𝑦 2 + 𝑦
2 2
1 1
𝑏) 𝑥𝑦 − 𝑥 + 𝑦 2 + 𝑦
2 2
1 1
𝑐) 𝑥𝑦 + 𝑥 − 𝑦 2 + 𝑦
2 2
1 1
𝑑) 𝑥𝑦 − 𝑥 − 𝑦 2 + 𝑦
2 2

Correct Answer: 𝑑)
1
Solution: (𝑥 − 𝑦)(𝑦 − 2)
2
1 1
= ( 𝑥 − 𝑦) (𝑦 − 2)
2 2
1 1
= 𝑥(𝑦 − 2) − 𝑦(𝑦 − 2)
2 2
1 1
= 𝑥𝑦 − 𝑥 − 𝑦 2 + 𝑦
2 2

20. (2𝑥 + 𝑦)(2𝑥 + 𝑦 − 𝑧) = _____________


𝑎) 4𝑥 2 + 4𝑥𝑦 − 2𝑥𝑧 − 𝑦𝑧
𝑏) 4𝑥 2 + 4𝑥𝑦 − 2𝑥𝑧 + 𝑦𝑧
𝑐) 4𝑥 2 + 4𝑥𝑦 + 2𝑥𝑧 − 𝑦𝑧
𝑑) 4𝑥 2 − 4𝑥𝑦 − 2𝑥𝑧 − 𝑦𝑧
Correct Answer: 𝑎)
Solution: (2𝑥 + 𝑦)(2𝑥 + 𝑦 − 𝑧)
= 2𝑥(2𝑥 + 𝑦 − 𝑧) + 𝑦(2𝑥 + 𝑦 − 𝑧)
= 4𝑥 2 + 2𝑥𝑦 − 2𝑥𝑧 + 2𝑥𝑦 + 𝑦 2 – 𝑦𝑧
= 4𝑥 2 + (2𝑥𝑦 + 2𝑥𝑦) − 2𝑥𝑧 − 𝑦𝑧
= 4𝑥 2 + 4𝑥𝑦 − 2𝑥𝑧 − 𝑦𝑧

7.4 Special Products

Lesson Proper

Special Product Formulas

If A and Bare any real numbers or algebraic expressions, then:

222
(𝐴 + 𝐵)(𝐴 − 𝐵) = 𝐴2 − 𝐵2 SUM AND PRODUCT OF THE SAME
TERMS
(𝐴 + 𝐵)2 = 𝐴2 + 2𝐴𝐵 + 𝐵2 SQUARE OF A SUM
(𝐴 − 𝐵)2 = 𝐴2 − 2𝐴𝐵 + 𝐵2 SQUARE OF A DIFFERENCE
(𝐴 + 𝐵)3 = 𝐴3 + 3𝐴2 𝐵 + 3𝐴𝐵2 + 𝐵3 CUBE OF A SUM
(𝐴 − 𝐵)3 = 𝐴3 − 3𝐴2 𝐵 + 3𝐴𝐵2 − 𝐵3 CUBE OF A DIFFERENCE
𝐴3 + 𝐵3 = (𝐴 + 𝐵)(𝐴2 − 𝐴𝐵 + 𝐵2 ) SUM OF A CUBE
𝐴3 − 𝐵3 = (𝐴 − 𝐵)(𝐴2 + 𝐴𝐵 + 𝐵2 ) DIFFERENCE OF A CUBE

To use these formulas, just apply the Principle of Substitution in which you can
substitute any algebraic expression for any letter in a formula.

Examples 7.4.1 Find the product.

(3𝑥 + 5)2
Solution: 1. Use Formula 2
2. Substitute 3𝑥 for A and 5 for B then;
(3𝑥 + 5)2 = 3𝑥 2 + 2(3𝑥)(5) + 52
3. Simplify
3𝑥 2 + 30𝑥 + 25
Examples 7.4.1 Find the product.
(𝑥 2 − 2)3
Solution: 1. Use Formula 5
2. Substitute 𝑥 2 for A and 5 for B then;
(𝑥 − 2) = (𝑥 2 )3 − 3(𝑥 2 )2 (2) + 3(𝑥 2 )(2)2 − 23
2 3

3. Simplify
𝑥 6 − 6𝑥 4 + 12𝑥 2 − 8

Lesson Exercise
Use Special Product Formulas to find each product:
1. (𝑥 + 2𝑦)(𝑥 − 2𝑦) = _____________
𝑎)𝑥 2 − 4𝑦 2
𝑏) − 2𝑥 4 − 𝑥 3 + 𝑥 2
𝑐) − 2𝑥 4 − 𝑥 3 − 𝑥 2
𝑑)𝑥 2 + 4𝑦 2
Correct Answer: 𝑎)
Solution: (𝑥 + 2𝑦)(𝑥 − 2𝑦)
= (𝑥 2 −(2𝑦)2
= 𝑥 2 − 4𝑦 2

2. (2𝑥 + 2𝑦)2 = _____________


𝑎)𝑥 2 − 4𝑦 2
𝑏) − 2𝑥 4 − 𝑥 3 + 𝑥 2
𝑐) 2𝑥 2 + 8𝑥𝑦 + 4𝑦 2

223
𝑑)2𝑥 2 − 8𝑥𝑦 + 4𝑦 2
Correct Answer: 𝑐)
Solution: (2𝑥 + 2𝑦)2
= (2𝑥 2 + 2(2𝑥)(2𝑦) +(2𝑦)2
= 2𝑥 2 + 8𝑥𝑦 + 4𝑦 2

3. (1 − 2𝑦)2 =_____________
𝑎)1 − 4𝑦 − 4𝑦 2
𝑏) − 1 − 4𝑦 + 4𝑦 2
𝑐) 1 + 4𝑦 + 4𝑦 2
𝑑)1 − 4𝑦 + 4𝑦 2
Correct Answer: 𝑑)
Solution:(1 − 2𝑦)2
= 12 − 2(1)(2𝑦) + (2𝑦)2
= 1 − 4𝑦 + 4𝑦 2

4. (3𝑥 + 4)2 =_____________


𝑎)9𝑥 2 + 24𝑥 − 16
𝑏)9𝑥 2 − 24𝑥 + 16
𝑐) 9𝑥 2 + 24𝑥 + 16
𝑑)9𝑥 2 − 24𝑥 − 16
Correct Answer: 𝑐)
Solution:(3𝑥 + 4)2
= (3𝑥)2 + 2(3𝑥)(4) + (4)2
= 9𝑥 2 + 24𝑥 + 16

5. (2𝑥 2 + 3𝑦 2 )2 =_____________
𝑎)9𝑥 2 + 24𝑥 − 16
𝑏)4𝑥 4 + 12𝑥 2 𝑦 2 + 9𝑦 4
𝑐) 4𝑥 4 − 12𝑥 2 𝑦 2 + 9𝑦 4
𝑑)9𝑥 2 − 24𝑥 − 16
Correct Answer: 𝑏)
Solution:(2𝑥 2 + 3𝑦 2 )2
= (2𝑥 2 )2 + 2(2𝑥 2 )(3𝑦 2 ) + (3𝑦 2 )2
= 4𝑥 4 + 12𝑥 2 𝑦 2 + 9𝑦 4

1 2
6. (𝐶 + ) =_____________
𝐶
1
𝑎)− 𝐶 2 + 1 + 2
𝐶
2
1
𝑏)𝐶 + 1 + 2
𝐶
2
1
𝑐) 𝐶 + 1 − 2
𝐶
2
1
𝑑)𝐶 − 1 + 2
𝐶

224
Correct Answer: 𝑏)
1 2
Solution:(𝐶 + )
𝐶
1 1 2
= 𝐶2 + 2 ( ) + ( )
2 𝐶
1
= 𝐶2 + 1 +
𝐶2

7. (1 + 𝑏) + (1 − 𝑏)=_____________
𝑎)12 − 𝑏 2
𝑏)12 ∓
𝑐) −12 − 𝑏 2
𝑑)−12 + 𝑏 2
Correct Answer: 𝑎)
Solution: (1 + 𝑏) + (1 − 𝑏)
=12 − 𝑏 2

8. (1 + 𝑎3 )3 =_____________
𝑎)1 + 3𝑎3 + 3𝑎6 − 𝑎9
𝑏)1 + 3𝑎3 + 3𝑎6 + 𝑎9
𝑐) 1 + 3𝑎3 − 3𝑎6 + 𝑎9
𝑑)1 − 3𝑎3 + 3𝑎6 + 𝑎9
Correct Answer: 𝑏)
Solution:(1 + 𝑎3 )3
= 13 + 3(12 )(𝑎3 ) + 3(1)(𝑎3 )2 + (𝑎3 )3
= 1 + 3𝑎3 + 3𝑎6 + 𝑎9

9. (1 − 2𝑦)3 =_____________
𝑎)1 + 3𝑎3 + 3𝑎6 − 𝑎9
𝑏)1 − 6𝑦 + 12𝑦 2 + 8𝑦 3
𝑐) 1 + 3𝑎3 − 3𝑎6 + 𝑎9
𝑑)1 + 6𝑦 + 12𝑦 2 + 8𝑦 3
Correct Answer: 𝑑)
Solution:(1 − 2𝑦)3
= 13 − 3(12 )(−2𝑦) + 3(1)(−2𝑦)2 − (−2𝑦)3
= 1 + 6𝑦 + 12𝑦 2 + 8𝑦 3

10. (2𝑥 3 + 𝑦 3 ) =_____________


𝑎)(2𝑥 + 𝑦)(4𝑥 2 − 2𝑥𝑦 + 𝑦 2 )
𝑏)(2𝑥 − 𝑦)(4𝑥 2 + 2𝑥𝑦 + 𝑦 2 )
𝑐) (2𝑥 + 𝑦)(4𝑥 2 + 2𝑥𝑦 + 𝑦 2 )
𝑑)(2𝑥 − 𝑦)(4𝑥 2 − 2𝑥𝑦 + 𝑦 2 )
Correct Answer: A
Solution: (2𝑥 3 + 𝑦 3 )
= (2𝑥 + 𝑦)((2𝑥)2 − 2𝑥𝑦 + 𝑦 2 )
= (2𝑥 + 𝑦)(4𝑥 2 − 2𝑥𝑦 + 𝑦 2 )

11. 3𝑥 3 − 2𝑦 3 =_____________
𝑎) (3𝑥 + 2𝑦)(9𝑥 2 − 6𝑥𝑦 − 4𝑦 2 )

225
𝑏)(3𝑥 + 2𝑦)(9𝑥 2 + 6𝑥𝑦 + 4𝑦 2 )
𝑐) (3𝑥 − 2𝑦)(9𝑥 2 − 6𝑥𝑦 + 4𝑦 2 )
𝑑) (3𝑥 + 2𝑦)(9𝑥 2 − 6𝑥𝑦 + 4𝑦 2 )
Correct Answer: C
Solution: 3𝑥 3 − 2𝑦 3
= (3𝑥 − 2𝑦)((3𝑥)2 + 3𝑥(−2𝑦) + (−2𝑦)2 )
= (3𝑥 − 2𝑦)(9𝑥 2 − 6𝑥𝑦 + 4𝑦 2 )

12. (𝑥 2 − 𝑎2 )(𝑥 2 + 𝑎2 )=_____________


𝑎) −𝑥 4 + 𝑎4
𝑏) −𝑥 4 − 𝑎4
𝑐) 𝑥 4 + 𝑎4
𝑑) 𝑥 4 − 𝑎4 )
Correct Answer: 𝑑)
Solution: (𝑥 2 − 𝑎2 )(𝑥 2 + 𝑎2 )
= (𝑥 2 )2 − (𝑎2 )2
= 𝑥 4 − 𝑎4
1 1 1 1
13. (𝑥 2 + 𝑦 2 )(𝑥 2 − 𝑦 2 )=_____________
𝑎) 𝑥 − 𝑦
1 1
𝑏) 𝑥 2 + 𝑦 2
𝑐) 𝑥 + 𝑦
1 1
𝑑) 𝑥 2 − 𝑦 2
Correct Answer: 𝑎)
1 1 1 1
Solution: (𝑥 2 + 𝑦 2)(𝑥 2 − 𝑦 2 )
1 2 1 2
= (𝑥 2 ) − (𝑦 2 )
=𝑥−𝑦

14. (2𝑥 2 + 3𝑦 2 )3 =_____________


𝑎) 8𝑥 6 + 36𝑥 4 𝑦 2 + 54𝑥 2 𝑦 4 + 27𝑦 6
𝑏)8𝑥 6 + 36𝑥 4 𝑦 2 + 54𝑥 2 𝑦 4 − 27𝑦 6
𝑐) 8𝑥 6 + 36𝑥 4 𝑦 2 − 54𝑥 2 𝑦 4 + 27𝑦 6
𝑑) 8𝑥 6 − 36𝑥 4 𝑦 2 + 54𝑥 2 𝑦 4 + 27𝑦 6
Correct Answer: 𝑎)
Solution: (2𝑥 2 + 3𝑦 2 )3
= (2𝑥 2 )3 + 3(2𝑥 2 )2 (3𝑦 2 ) + 3(2𝑥 2 )(3𝑦 2 )2 + (3𝑦 2 )3
= 8𝑥 6 + 36𝑥 4 𝑦 2 + 54𝑥 2 𝑦 4 + 27𝑦 6

15. (𝑥 + 2)3 =_____________


𝑎) −𝑥 3 + 6𝑥 2 + 12𝑥 + 8
𝑏)𝑥 3 + 6𝑥 2 − 12𝑥 + 8
𝑐) 𝑥 3 − 6𝑥 2 + 12𝑥 + 8
𝑑) 𝑥 3 + 6𝑥 2 + 12𝑥 + 8
Correct Answer: 𝑑)
Solution: (𝑥 + 2)3
= 𝑥 3 + 3(𝑥)2 (2) + 3(𝑥)(2)2 + 23

226
= 𝑥 3 + 6𝑥 2 + 12𝑥 + 8

16. (2𝑥 + 3)2 =_____________


𝑎) 4𝑥 2 − 12𝑥 − 9
𝑏) 4𝑥 2 + 12𝑥 + 9
𝑐) −4𝑥 2 + 12𝑥 + 9
𝑑) 4𝑥 2 + 12𝑥 − 9
Correct Answer: 𝑏)
Solution: (2𝑥 + 3)2
= (2𝑥)2 + 2(2𝑥)(3) + (3)2
= 4𝑥 2 + 12𝑥 + 9
3
17. (𝑥 − 2𝑦) =_____________
𝑎) 𝑥 3 + 6𝑥 2 𝑦 + 12𝑥𝑦 2 + 8𝑦 3
𝑏) 𝑥 3 + 6𝑥 2 𝑦 − 12𝑥𝑦 2 + 8𝑦 3
𝑐) 𝑥 3 + 6𝑥 2 𝑦 + 12𝑥𝑦 2 − 8𝑦 3
𝑑) 𝑥 3 − 6𝑥 2 𝑦 + 12𝑥𝑦 2 + 8𝑦 3
Correct Answer: 𝑎)
Solution: (𝑥 − 2𝑦)3
= 𝑥 3 − 3(𝑥)2 (−2𝑦) + 3(𝑥)(−2𝑦)2 − (−2𝑦)3 )
= 𝑥 3 + 6𝑥 2 𝑦 + 12𝑥𝑦 2 + 8𝑦 3

18. 𝑥 3 − 5𝑥 3 =_____________
𝑎) (𝑥 + 5𝑥)(𝑥 2 − 5𝑥 2 − 25𝑥 2 )
𝑏)(𝑥 − 5𝑥)(𝑥 2 + 5𝑥 2 + 25𝑥 2 )
𝑐) (𝑥 + 5𝑥)(𝑥 2 − 5𝑥 2 + 25𝑥 2 )
𝑑) (𝑥 − 5𝑥)(𝑥 2 − 5𝑥 2 + 25𝑥 2 )
Correct Answer: 𝑑)
Solution: 𝑥 3 − 5𝑥 3
= (𝑥 − 5𝑥)(𝑥 2 + (𝑥)(−5𝑥) + (−5𝑥)2
= (𝑥 − 5𝑥)(𝑥 2 − 5𝑥 2 + 25𝑥 2 )

19. 𝑥 3 + 5𝑥 3 =_____________
𝑎) (𝑥 + 5𝑥)(𝑥 2 − 5𝑥 − 25𝑥 2 )
𝑏)(𝑥 − 5𝑥)(𝑥 2 + 5𝑥 + 25𝑥 2 )
𝑐) (𝑥 + 5𝑥)(𝑥 2 − 5𝑥 + 25𝑥 2 )
𝑑) (𝑥 − 5𝑥)(𝑥 2 − 5𝑥 + 25𝑥 2 )
Correct Answer: 𝑐)
Solution: 𝑥 3 + 5𝑥 3
= (𝑥 + 5𝑥)(𝑥 2 − (𝑥)(5𝑥) + (5𝑥)2
=(𝑥 + 5𝑥)(𝑥 2 − 5𝑥 2 + 25𝑥 2 )

20. (2𝑥 + 2𝑦)2 =_____________


𝑎) (𝑥 + 5𝑥)(𝑥 2 − 5𝑥 − 25𝑥 2 )
𝑏)(𝑥 − 5𝑥)(𝑥 2 + 5𝑥 + 25𝑥 2 )
𝑐) (𝑥 + 5𝑥)(𝑥 2 − 5𝑥 + 25𝑥 2 )
𝑑) (𝑥 − 5𝑥)(𝑥 2 − 5𝑥 + 25𝑥 2 )
Correct Answer:
Solution: (2𝑥 + 2𝑦)2
= (2𝑥)2 + 2(2𝑥)(2𝑦) + (2𝑦)2

227
= 4𝑥 2 + 8𝑥𝑦 + 4𝑦 2

7.5 Division of Polynomials


Division of Polynomial by a Monomial

To divide polynomial by a monomial, just multiply the dividend with the reciprocal
of the divisor and use the distributive property after.
Example: Find the quotient:
1 6𝑥 2 +3𝑥−2 6𝑥 2 3𝑥 2 2
6𝑥 2 + 3𝑥 − 2/3 = 6𝑥 2 + 3𝑥 − 2 ∙ = = + − = 2𝑥 2 + 𝑥 −
3 3 3 3 3 3

Division of Polynomial by Polynomial


When the divisor is not a monomial, use Long Division.

Example 7.5.1

Find the quotient:


(𝑥 2 + 5𝑥 + 6) ÷ (𝑥 + 2)
Solution:
𝑥
2
𝑥 + 2)𝑥 + 5𝑥 + 6 Dividing 1st term by 1st term to get x
2
𝑥 + 2𝑥 Multiplying x by divisor 𝑥 + 2
3𝑥 Subtracting
“Bring down” the next term of the dividend (6).
𝑥+3
2
𝑥 + 2)𝑥 + 5𝑥 + 6
𝑥 2 + 2𝑥
3𝑥 + 6
3𝑥 + 6 Multiplying 3 by divisor 𝑥 + 2
0 Subtracting
Quotient: 𝒙 + 𝟑
What if like this, if we have remainder other than 0?
Examples 7.5.2
(24𝑥 3 − 9𝑥 2 − 3𝑥 − 9) ÷ (3𝑥 − 2)
We do the same as the previous example.

8𝑥 2 + 9𝑥 + 5
3𝑥 − 2)24𝑥 + 9𝑥 2 − 3𝑥 − 9
3

24𝑥 3 − 18𝑥 2
27𝑥 2 − 3𝑥
27𝑥 2 − 18𝑥
15𝑥 − 9
15𝑥 − 10
1
Now, we have a remainder which will be divided by (3𝑥 − 2)
1
Quotient: 8𝑥 2 + 9𝑥 + 5 +
3𝑥−2

228
Lesson Exercise
Give the quotient of the following:
1. 24𝑥 4 − 4𝑥 3 + 𝑥 2 /8 = _____________
1 1 1 1
𝑎) 2𝑥 4 − 𝑥 3 − 𝑥 2 𝑏) 3𝑥 4 + 𝑥 3 + 𝑥 2
2 8 2 8
4
1 3 1 2
𝑐) 2𝑥 + 𝑥 + 𝑥
2 8
1 3 1
𝑑) 3𝑥 − 𝑥 + 𝑥 2
4
2 8
Correct Answer: D
Solution: 24𝑥 4 − 4𝑥 3 + 𝑥 2 /8
24𝑥 4 4𝑥 3 𝑥2
= − +
8 8 8
4 1 3 1 2
= 3𝑥 − 𝑥 + 𝑥
2 8

2. 12𝑎4 − 3𝑎2 + 𝑎 − 6/6= _____________


1 𝑎
𝑎) 2𝑎4 − 𝑎2 + − 1
2 6
1 𝑎
𝑏) 2𝑎4 − 𝑎2 + + 1
2 6
1 𝑎
𝑐) 2𝑎4 − 𝑎2 − − 1
2 6
1 𝑎
𝑑)2𝑎4 + 𝑎2 + −1
2 6
Correct Answer: A
Solution: 12𝑎4 − 3𝑎2 + 𝑎 − 6/6
12𝑎4 3𝑎2 𝑎 6
= − + −
6 6 6 6
1 𝑎
= 2𝑎4 − 𝑎2 + − 1
2 6

3. 𝑢 − 2𝑢2 − 𝑢5 /𝑢= _____________


𝑎) 1 + 2𝑢 − 𝑢4
𝑏) 1 − 2𝑢 + 𝑢4
𝑐) 1 − 2𝑢 − 𝑢4
𝑑) − 1 − 2𝑢 − 𝑢4
Correct Answer: C
Solution: 𝑢 − 2𝑢2 − 𝑢5 /𝑢
𝑢 2𝑢2 𝑢5
= − −
𝑢 𝑢 𝑢
= 1 − 2𝑢 − 𝑢4

4. 50𝑥 5 − 7𝑥 4 + 𝑥 2 /𝑥 = _____________
𝑎) 50𝑥 4 − 7𝑥 3 + 𝑥
𝑏) − 50𝑥 4 − 7𝑥 3 + 𝑥
𝑐) 50𝑥 4 + 7𝑥 3 + 𝑥
𝑑)50𝑥 4 − 7𝑥 3 – 𝑥
Correct Answer: A
Solution: 50𝑥 5 − 7𝑥 4 + 𝑥 2 /𝑥

229
50𝑥 5 7𝑥 4 𝑥2
= − +
𝑥 𝑥 𝑥
4 3
= 50𝑥 − 7𝑥 + 𝑥

5. 15𝑥 5 − 7𝑥 4 + 𝑥 2 /𝑥 = _____________
𝑎) − 15𝑥 4 − 7𝑥 3 + 𝑥
𝑏) 15𝑥 4 − 7𝑥 3 + 𝑥
𝑐) 15𝑥 4 − 7𝑥 3 − 𝑥
𝑑)15𝑥 4 + 7𝑥 3 + 𝑥
Correct Answer: B
Solution: 15𝑥 5 − 7𝑥 4 + 𝑥 2 /𝑥
15𝑥 5 7𝑥 4 𝑥2
= − +
𝑥 𝑥 𝑥
4 3
= 15𝑥 − 7𝑥 + 𝑥

6. (𝑥 4 + 3𝑥 3 − 𝑥 2 − 𝑥 + 6) ÷ (𝑥 + 3) = _____________
𝑎) 𝑥 3 − 𝑥 − 2
𝑏) 𝑥 3 − 𝑥 + 2
𝑐) − 𝑥 3 − 𝑥 + 2
𝑑) 𝑥 3 + 𝑥 + 2
Correct Answer: B
Solution: (𝑥 4 + 3𝑥 3 − 𝑥 2 − 𝑥 + 6) ÷ (𝑥 + 3)
= x3 −x −4 +6
𝑥 + 3)𝑥 + 3𝑥 3 − 𝑥 2 − 𝑥 + 6
4

𝑥 4 + 3𝑥 3
0 −𝑥 2 − 𝑥
−𝑥 2 − 3𝑥________
−4𝑥 + 6
−4𝑥 − 12______
18
18
0
= x3 − x − 4 + 6 = 𝑥3 − 𝑥 + 2

7. (7𝑥 4 − 10𝑥 3 + 3𝑥 2 + 3𝑥 − 3) ÷ (𝑥 − 1) = _____________


𝑎)7𝑥 3 − 3𝑥 2 − 3
𝑏)7𝑥 3 + 3𝑥 2 + 3
𝑐) − 7𝑥 3 − 3𝑥 2 + 3
𝑑) 7𝑥 3 − 3𝑥 2 + 3
Correct Answer: D
Solution: (7𝑥 4 − 10𝑥 3 + 3𝑥 2 + 3𝑥 − 3) ÷ (𝑥 − 1)
= 7𝑥 3 − 3𝑥 2 +3
𝑥 − 1)7𝑥 − 10𝑥 + 3𝑥 2 + 3𝑥 − 3
4 3

7𝑥 4 − 7𝑥 3
−3𝑥 3 + 3𝑥 2
_ − 3𝑥 3 + 3𝑥 2 _____
0 + 3𝑥 − 3
0 + 3𝑥 − 3

230
0

8. (2𝑥 4 − 8𝑥 3 − 5𝑥 2 − 4𝑥 + 2) ÷ (𝑥 2 + 4𝑥 − 2) = _____________
𝑎) − 2𝑥 2 − 1
𝑏)2𝑥 2 − 5
𝑐) 2𝑥 2 − 1
𝑑) 2𝑥 2 + 1
Correct Answer: C
Solution: (2𝑥 4 − 8𝑥 3 − 5𝑥 2 − 4𝑥 + 2) ÷ (𝑥 2 + 4𝑥 − 2)
2𝑥 2 −1
2 4 3 2
𝑥 + 4𝑥 − 2)2𝑥 − 8𝑥 − 5𝑥 − 4𝑥 + 2
2𝑥 4 + 8𝑥 3 − 4𝑥 2 _____________
−𝑥 2 −4𝑥 + 2
−𝑥 2 −4𝑥 + 2

9. (3𝑥 4 − 𝑥 3 + 8𝑥 2 + 5𝑥 + 3) ÷ (𝑥 2 − 𝑥 + 3) = _____________
𝑎)3𝑥 2 + 2𝑥 + 1
𝑏)3𝑥 2 − 2𝑥 + 1
𝑐) 3𝑥 2 + 2𝑥 − 1
𝑑) − 3𝑥 2 + 2𝑥 + 1
Correct Answer: A
Solution:(3𝑥 4 − 𝑥 3 + 8𝑥 2 + 5𝑥 + 3) ÷ (𝑥 2 − 𝑥 + 3)
3𝑥 2 + 2𝑥 +1
𝑥 − 𝑥 + 3)3𝑥 4 − 𝑥 3 + 8𝑥 2 + 4𝑥 + 3
2

3𝑥 4 − 3𝑥 3 + 9𝑥 2
2 𝑥 3 − 𝑥 2 + 4𝑥
3
𝑥 − 2𝑥 2 + 6𝑥
𝑥 2 − 2𝑥 + 3
𝑥2 – 𝑥 + 3
−𝑥 +0
−𝑥
0

10. (21 𝑥 2 − 14𝑥) ÷ 7𝑥= _____________


𝑎)3𝑥 + 1
𝑏)3𝑥 − 2
𝑐) − 3𝑥 − 2
𝑑)3𝑥 + 2
Correct Answer: B
Solution: (21 𝑥 2 − 14𝑥) ÷ 7𝑥
21 𝑥 2 14𝑥
= −
7𝑥 7𝑥
= 3𝑥 − 2

11. (30𝑥 2 + 20𝑥 + 15) ÷ 5= _____________


𝑎)6𝑥 2 + 4𝑥 + 3
𝑏)6𝑥 2 + 4𝑥 − 3
𝑐)6𝑥 2 + 4𝑥 + 3
𝑑)6𝑥 2 − 4𝑥 − 3

231
Correct Answer: C
Solution: (30𝑥 2 + 20𝑥 + 15) ÷ 5
30𝑥 2 20𝑥 15
= + +
5 5 5
= 6𝑥 2 + 4𝑥 + 3

12. (18𝑥 2 𝑦 3 + 12𝑥 4 𝑦 2 ) ÷ 6𝑥


𝑎)2𝑥𝑦 3 + 2𝑥 3 𝑦 2
𝑏)3𝑥𝑦 3 + 2𝑥 3 𝑦 2
𝑐)3𝑥𝑦 3 − 2𝑥 3 𝑦 2
𝑑) − 3𝑥𝑦 3 + 2𝑥 3 𝑦 2
Correct Answer: B
Solution: (18𝑥 2 𝑦 3 + 12𝑥 4 𝑦 2 ) ÷ 6𝑥
18𝑥 2 𝑦 3 12𝑥 4 𝑦 2
= +
6𝑥 6𝑥
= 3𝑥𝑦 3 + 2𝑥 3 𝑦 2

13. (𝑎3 𝑏 2 + 2𝑎4 𝑏 − 5𝑎2 𝑏 2 ) ÷ 𝑎𝑏


𝑎)𝑎2 + 2𝑎3 − 5𝑎𝑏
𝑏)𝑎2 𝑏 + 2𝑎3 − 5𝑎𝑏
𝑐)𝑎2 𝑏 + 2𝑎3 + 5𝑎𝑏
𝑑) − 𝑎2 𝑏 + 2𝑎3 − 5𝑎𝑏
Correct Answer: B
Solution: (𝑎3 𝑏 2 + 2𝑎4 𝑏 − 5𝑎2 𝑏 2 ) ÷ 𝑎𝑏
𝑎3 𝑏2 2𝑎4 𝑏 5𝑎2 𝑏2
= + −
𝑎𝑏 𝑎𝑏 𝑎𝑏
= 𝑎2 𝑏 + 2𝑎3 − 5𝑎𝑏

14. (8𝑥 3 𝑦 2 𝑧 − 12𝑥 4 𝑦𝑧 2 )÷(−4𝑥𝑦𝑧)


𝑎) − 3𝑥 2 𝑦 + 3𝑥 3 𝑧
𝑏)2𝑥 2 𝑦 + 3𝑥 3 𝑧
𝑐) − 2𝑥 2 𝑦 − 3𝑥 3 𝑧
𝑑) − 2𝑥 2 𝑦 + 3𝑥 3 𝑧
Correct Answer: D
Solution: (8𝑥 3 𝑦 2 𝑧 − 12𝑥 4 𝑦𝑧 2 ) ÷(−4𝑥𝑦𝑧)
8𝑥 3 𝑦 2 𝑧 12𝑥 4 𝑦𝑧 2
= −
−4𝑥𝑦𝑧 −4𝑥𝑦𝑧
= −2𝑥 2 𝑦 + 3𝑥 3 𝑧

15. (3𝑥 4 + 9𝑥 3 − 5𝑥 2 − 6𝑥 + 2) ÷ (3𝑥 2 − 2) = _____________


𝑎) 𝑥 2 + 3𝑥 − 2
𝑏) − 𝑥 2 + 3𝑥 − 1
𝑐) 𝑥 2 − 3𝑥 − 1
𝑑) 𝑥 2 + 3𝑥 − 1
Correct Answer: D
Solution:(3𝑥 4 + 9𝑥 3 − 5𝑥 2 − 6𝑥 + 2) ÷ (3𝑥 2 − 2)
𝑥 2 + 3𝑥 − 1
3𝑥 − 2)3𝑥 + 9𝑥 − 5𝑥 2 − 6𝑥 + 2
2 4 3

3𝑥 4 − 2𝑥 2
9𝑥 3 −3𝑥 2 − 6𝑥
3
9𝑥 − 6𝑥

232
−3𝑥 2 +2
−3𝑥 2 +2
0

16. (𝑥 2 − 4) ÷ (𝑥 − 2) = _____________
𝑎)𝑥 + 2
𝑏)𝑥 − 3
𝑐) 𝑥 + 3
𝑑)𝑥 − 2
Correct Answer: A
Solution:(𝑥 3 − 2𝑥 2 − 4) ÷ (𝑥 − 2)
𝑥 +2
𝑥 − 2)𝑥 2 −4
𝑥 2 − 2𝑥
−2𝑥 − 4
−2𝑥 − 4
0
17. (𝑥 3 − 4𝑥 2 + 9) ÷ (𝑥 − 3) = _____________
𝑎) 𝑥 2 − 5𝑥 − 3
𝑏) 𝑥 2 − 2𝑥 − 3
𝑐) −𝑥 2 − 𝑥 − 3
𝑑) 𝑥 2 − 𝑥 − 3
Correct Answer: D
Solution:(𝑥 3 − 4𝑥 2 + 9) ÷ (𝑥 − 3)
𝑥2 −𝑥−3
𝑥 − 3)𝑥 − 4𝑥 2 + 9
3

𝑥 3 − 3𝑥 2
−𝑥 2 +9
−𝑥 2 + 3𝑥
−3𝑥 + 9
−3𝑥 + 9
0

18. (𝑥 4 − 13𝑥 − 42) ÷ (𝑥 2 − 𝑥 − 6) = _____________


𝑎) 𝑥 2 − 5𝑥 − 3
𝑏) 𝑥 2 − 2𝑥 − 3
𝑐) − 𝑥 2 + 𝑥 + 7
𝑑) 𝑥 2 + 𝑥 + 7
Correct Answer: D
Solution:(𝑥 4 − 13𝑥 − 42) ÷ (𝑥 2 − 𝑥 − 6)
𝑥2 +𝑥 +7
2 4
𝑥 − 𝑥 − 6)𝑥 − 13𝑥 − 42
𝑥 4 − 𝑥 3 − 6𝑥 2
−𝑥 3 − 6𝑥 2 − 13𝑥
𝑥 3 − 𝑥 2 − 6𝑥
−2𝑥 3 − 5𝑥 2 − 7𝑥 − 42
7𝑥 2 − 7𝑥 − 42
0

233
19. (100 𝑥 2 − 10𝑥) ÷ 5𝑥= _____________
𝑎) 𝑥 + 2
𝑏) 20𝑥 + 2
𝑐) 20𝑥 − 2
𝑑) 𝑥 − 2
Correct Answer: C
Solution:(100 𝑥 2 − 10𝑥) ÷ 5𝑥
100 𝑥 2 10𝑥
= −
5𝑥 5𝑥
= 20𝑥 − 2

20. (50 𝑥 2 − 60𝑥) ÷ 10𝑥= _____________


𝑎) 5𝑥 − 6𝑥 + 1
𝑏) − 5𝑥 − 6𝑥
𝑐) 5𝑥 − 6𝑥
𝑑) 5𝑥 + 6𝑥
Correct Answer: C
Solution:(50 𝑥 2 − 60𝑥) ÷ 10𝑥
50 𝑥 2 60𝑥
= −
10𝑥 10𝑥
= 5𝑥 − 6𝑥

8 Factoring
8.1 Common Factors
The common factor of an integer is an integer that is a factor of two or more integers.
Example: 5
25 5 is the common factor of integers 25 and 30.
5

5
30
6

Greatest common factor (GCF): it is the largest common factor of a list of integers.
Example:

5
15
3
5 is the greatest common factor
5 of all three integers.
5
1

234
5
25
5

When faced with large integers, it is important to follow these simplified steps in finding the
greatest common factor (GCF).
Step 1 Write the integers in prime factored form.
Example: 40 = 2 . 2 . 2. 2 . 5
20 = 2 . 2 . 5
Step 2 Consider the least number of time a prime appears in all the factored forms.
Example: 40 = 2 ∙ 2 ∙ 2 ∙ 2 ∙ 5
20 = 2 ∙ 2 ∙ 5
In this example, the least number of times the prime 2 appeared as a common
factor is twice (2) while the least number of times 5 appeared as a common factor is
only once (1).

Step 3 Either you multiply directly the primes or choose the smallest exponents then
multpily.
Example: 40 = 2 ∙ 2 ∙ 2 ∙ 2 ∙ 5
20 = 2 ∙ 2 ∙ 5

(2) (2) (5) = 20


OR
2
2 . 5 = 20
Note: If in the case that there are no primes common among a list of integers, then the
GCF is 1.
Example: 7=7.1
12 = 2 . 2 . 3
9=3.3

Factoring out the greatest common factor (GCF) may not only be done on a set of integers but
also on a list of variable terms.
Example: x2, x5, x8
x2 = 1 . x2
x5 = x3 . x2
x8 = x6 . x2
The exponent on a variable in the GCF is the smallest exponent that appears in the factors. In
this case, note that each term can be written with x2 as a factor. Therefore, x2 is the greatest
common factor.
More examples on factoring out GCF of variables.

235
1. 2a3, 14a7, 28a5

2a3 = 2 . a3
14a7 = 2 . 7 . a7
28a5 = 2 . 2 . 7 . a5
The greatest common factor of the coefficients 2, 14 and 28 is 2. Meanwhile the smallest
exponent of variable a is 3. Hence, the GCF of the terms is 2a3.
2. 7xy4, 42x3y2

7xy4 = 7 . x . y4
42x3y2 = 7 . 2 . 3 . x3 . y2
The greatest common factor of coefficients 7 and 42 is 7. Meanwhile the smallest
exponent on x is 1 and y is 2. Hence, the GCF of the terms is 7xy2
Factoring the GCF of a polynomial:

Example: 5b + 30 = (5 ∙ b) + (5 ∙ 6)

5b and 30 are two terms and the greatest common factor for these is 5 so that each of these
terms is a product with 5 as the factor.
5b + 30 = 5 (b + 6)
The factored form of 5b+30 is 5(b + 6).

More examples on factoring out GCF of polynomials:


1. 3y4 + 27y2 + 9y5
3y4 + 27y2 + 9y5 = 3y2 (y2) + 3y2 (3) + 3y2 (3y3)
= 3y2 (y2+ 2 + 3y3)

The GCF for the terms of this polynomial is 3y2. To know if you’ve factored it out
right, check it by multiplying 3y2 and y2+ 2 + 3y3. It should result to the original
polynomial.
2. xy2 + 4x2y2 + 10xy5
xy2 + 4x2y2 + 10xy5 = xy2 (1+4x+10y3)

3. z(y+5) + 4(y+5)
z (y+5) + 4(y+5) = (y+5) (z+4)

The greatest common factor is the binomial y + 5.


Factoring by Grouping
The goal in factoring by grouping is to get a common factor among the terms.
Example:
ab + 2a + 3b + 6

236
2a and ab have a common factor of a; and 3y and b have a common factor of 2.
ab + 2a + 3b + 6 = a (b + 2) + 3 (b + 2)
Though the expression has already been factored, it isn’t entirely factored out yet since b
+ 2 can still be factored out and the expression is still the sum of two terms.
ab + 2a + 3b + 6 = a (b + 2) + 3 (b + 2) = (b + 2) (a + 3)
Now the final result is a product and in its factored form.

It is important to be careful in using negative signs when grouping. To aid in factoring four
terms, keep in mind the steps below.
Step 1 Group terms into two so that in each group, there is a common factor.
Step 2 Factor out the GCF from each group.
Step 3 If you can factor out a common binomial, then do so.
Step 4 Rearrange terms if in Step 2 doesn’t yield a common binomial factor.

Example:
10x2 – 12y2 + 15xy – 8xy
We can factor this expression in this manner.
2 (5x2 – 6y2) + xy (15 – 8)
But in this way, it did not result to a common factor. So we try rearranging the terms.

10x2 – 8xy – 12y2 + 15xy


2x (5x – 4y) + 3y (-4y +5x)
Note that in -12y2 + 15xy, 3y has been factored out to get a common binomial factor.
10x – 8xy – 12y + 15xy
2 2

= 2x (5x – 4y) + 3y (5x – 4y)


= (5x – 4y) (2x + 3y)
Lesson Exercise for Common Factors
Find the greatest common factor for each list of terms:
1. 15, 27, 30
a. 5

237
b. 2
c. 3
Sol. 15 = 5 x 3
27 = 3 x 3 x 3
30 = 5 x 2 x 3

2. 2, 36, 8
a. 2
b. 3
c. 4
Sol. 2=2x1
36 = 2 x 2 x 3 x 3
8=2x2x2

3. zy2, y, z2y2
a. zy
b. z2
c. y
Sol. zy2 = z ∙ 𝑦 ∙ 𝒚
y=y.1
z2y2 = 𝑧 . 𝑧 . 𝑦 . 𝒚

4. 15x3 – 30x2
a. 5x(3x2 – 6x)
b. 15x2(x – 2)
c. 15x(x2 -2x)
Sol. 15x3 – 30x2 = 15x2(x – 2)
Use the FOIL method to check.

5. 45c3d, 75c, 90d, 105cd


a. 15cd
b. 15
c. 15d
Sol. 45c3d = 5 · 3 · 3 ·c · c · c · d
75c = 5 · 5 · 3 · c
90d = 5 · 2 · 3 · 3 · d
105cd = 5 · 3 · 7 · c · d

6. P2 + 4p + 3p + 12
a. (p + 4) (p+ 3)
b. (p + 2) (p + 1)

238
c. (p + 3) (p + 2)
Sol. p2 + 4p + 3p + 12 = p2 + 7p + 12 = (p + 4) (P + 3)

7. c(x + 2) – d(x + 2)
a. (c + d) (x + 2)
b. cd (x + 2)
c. (x + 2) (c – d)

Sol. c(x + 2) – d(x + 2) = (x + 2) (c – d)

8. 25p5r7, 30p7r8, 50p5r3


a. 10p5r3
b. 5pr7
c. 5p5r3
Sol. 25p5r7 = 5 · 7 · p · p · p · p · p · r · r · r · r · r · r · r
30p7r8 = 5 · 3 · 2 · p · p · p · p · p · p · p · r · r · r · r · r · r · r · r
50p5r3 = 5 · 5 · 2 · p · p · p · p · p · r · r · r

8.2 Factoring Trinomials


To factor out trinomials, we must do the opposite of the FOIL method.
Example:
FOIL Factoring
(b-3) (b+1) = b2 – 2b – 3 b2 – 2b – 3 = (b – 3) (b +1)

Example: x2 + 7x + 12
We use a and b in finding the integers.
x2 + 7x + 12 = (x + a) (x + b)

Factored form
x + 7x + 12 = x2 + (a +b)x + ab
2

Sum of a and b is 7.

Product of a and b is 12.


Think of pairs of integers that have a sum of 7 and when multiplied result to 12. Both 7 and 12
are positive so both integers are positive as well.

239
x2 + 7x + 12 = (x + 4) (x + 3)

4+3=7
4 ∙ 3 = 12
Integers 4 and 3 have a sum of 7 and a product of 12. To check, we can multiply the binomials
using the FOIL method.

Example: a2 – 2a – 8
Find two integers whose sum is -2 and whose product is -8.
Multiply. -8

a2 – 2a – 8 = (a – 4) (a + 2 )
-4a
2a Add.
-2a
Tips in using negative and positive signs in factoring trinomials:
1. A positive sign is assigned to both integers if a trinomial has all positive terms.
2. A negative and a positive sign are assigned to integers, either a or b, when a trinomial
has two negative terms.
3. A negative sign is assigned to both integers when a trinomial’s middle term has a
negative sign.

Example: y2 – 5y + 6
Multiply. 6

y2 – 5y + 6 = (y – 3) (y – 2)
-3y
-2y Add.
-5y
Prime polynomial: it is a polynomial that cannot be factored out using only integer factors.
Factoring a Trinomial with Two Variables
Example: z2 – 5bz + 6b2
Look for two expressions whose sum is -5b and whose product is +6b2.

240
Multiply. 3b2

z2 – 5bz + 6b2 = (z – 3b) (z – 2b)


-3bz
-2bz Add.
-5bz
Example: 5x4 + 30x3 + 40x2
Factor out the GCF first which is 5x2.
5x4 + 30x3 + 40x2 = 5x2 (x2 + 6x + 8)
See that the polynomials x2 + 6x + 8 can still be factored out. Then find a pair of integers
whose sum is 6x and whose product is 8.
5x4 + 30x3 + 40x2 = 5x2 (x2 + 6x + 8) = 5x2 (x + 4) (x + 2)
When your coefficient of the squared term is not 1, use the grouping method to factor.

Lesson Exercise
Factor completely.
1. x2 – x – 42
a. (x – 21) (x – 2)
b. (x – 14) (x – 3)
c. (x – 7) (x + 6)
Sol. x2 – x – 42 = (x + 7) (x + 6)
Use the FOIL method to check.

2. y3z + 3y2z2 – 54yz3


a. yz(y – 6z) (y + 9z)
b. y2z2(y – 6) (y + 9)
c. yz(y – 6z) (y – 9z)
Sol. y3z + 3y2z2 – 54yz3 = yz(y2 + 3yz – 54z2) = yz(y – 6z) (y + 9z)

3. b2 + 4b + 5
a. (b – 5) ( b – 1)
b. (b + 5) (b + 1)
c. Prime

4. 5y2 – 5y – 30
a. 5(y + 2) (y – 3)
b. (y + 2) (y – 3)
c. Prime
Sol. 5y2 – 5y – 30 = 5(y2 – y – 6) = 5(y + 2) (y – 3)

5. k7 – 2k6m - 15k5m2
a. 5k6(k – m)

241
b. k5(k – 5m) (k + 3m)
c. k5(k –m) (k + m)
Sol. k7 – 2k6m – 15k5m2

6. –x2 – 4x + 21
a. 1(x + 7) (x – 3)
b. -1(x + 7) (x – 3)
c. Prime
Sol. –x2 – 4x + 21 = -1(x2 + 4x – 21) = -1(x + 7) (x – 3)

7. 6m6n + 7m5n2 + 2m4n3


a. m4n(3m + 2n) (2m + n)
b. m4n3(3m + 2) (2m + 1)
c. prime
Sol. 6m6n + 7m5n2 + 2m4n3 = m4n(6m2 + 7mn + 2n2) = m4n(3m + 2n) (2m + n)

8.3 Factoring Techniques


Factoring the Difference of Two Squares
x2 – y2 = (x + y) (x – y)

x + y and x – y are a pair of expressions called conjugates.


Example:
a. x2 – 36 = x2 – 62 = (x + 6) (x – 6)
9 3 3
b. a2 – = a2 - (35)2 = ( a + ) ( a - )
25 5 5

c. p2 – 8
8 is not the square of an integer. Thus, this is a prime polynomial.
d. 16z – 9y2 =
2
(4z)2 – (3y)2 = (4z + 3y) (4z – 3y)
e. b4 – 16 = (b2)2 - 42
= (b2 + 4) (b2 – 4)
= (b2 + 4) (b + 2) (b – 2)
f. 36t2 – 16 = 4(9t2 – 4)
Factor out the common factor of 4 first.
4(9t – 4) = 4(3t + 2) (3t – 2)
2

Perfect Square Trinomial: is a trinomial that is the square of a binomial.


x2 + 2xy + y2 = (x + y) 2
x2 – 2xy + y2 = (x – y) 2

242
For a trinomial to be a perfect square, two of its terms must be perfect squares.

Example: x2 + 10x + 25 = (x + 5)2

Difference of Two Cubes


x3 – y3 = (x – y) (x2 + xy + y2)

The binomial factor (x – y) has the difference of the cube roots of the given terms. All
terms in the trinomial factor are positive. Note that x3 – y3 is not the same with (x – y)3.
Example:
a. z3 – 64 = z3 – 43
= (z – 4) (z2 + 4z + 42)
= (z – 4) (z2 + 4z + 16)

b. 27s3 – 125 = (3s)3 - 53


= (3s – 5) [(3s)2 2 + (3s)(5) + 52 ]
= (3s – 5) (9s2 + 15s + 25)
Take note, it is very unusual to further factor an expression of the form x2 + xy + y2.
Sum of Two Cubes
x3 + y3 = (x + y) (x2 – xy + y2)
The only difference between the sum of two cubes and the difference of two cubes is the
positive and negative sign.
Example:
a. y3 + 125 = y3 + 53
= (y + 5) (y2 – 5y + 25)
b. 64b3 + 27 = (4b)3 + 33
= (4b + 3) [(4b)2 + (4b)(3) + 32 ]
= (4b + 3) (16b2 – 12b + 9)

Lesson Exercises.
Complete the following list of squares.
1. 182
a. 289
b. 324
c. 361
Sol. 18 x 18 = 324

243
2. 92
a. 81
b. 100
c. 64
Sol. 9 x 9 = 81

Complete the following list of cubes.


1. 93
a. 1, 000
b. 729
c. 512
Sol. 9 x 9 x 9 = 729

2. 43
a. 64
b. 216
c. 27
Sol. 4 x 4 x 4 = 64
Factor completely.
1. y4 – 16
a. (y – 4) (y + 2) (y + 2)
b. (y2 + 4) (y + 2) (y – 2)
c. Prime
Sol. y4 – 16 = (y2 + 4) (y2 – 4) = (y2 + 4) (y + 2) (y – 2)

16
2. 36m2 -
25
4 4
a. (6m + ) (6m + )
5 5
2 8
b. (6m + ) (6m + )
5 5
𝟒 𝟒
c. (6m + ) (6m – )
𝟓 𝟓

16 4 4
Sol. 36m2 - = (6m + ) (6m – )
25 5 5
Use the FOIL method to check.

3. 8p3 + 729q3
a. (2p – 9q) (4p2 – 18pq – 81q2)
b. (2p + 9q) (4p2 – 18pq + 81q2)
c. (2p + 9q) (4p2 + 18pq + 81q2)
Sol. 8p3 + 729q3 = (2p + 9q) (4p2 – 18pq + 81q2)
Use the FOIL method to check.

244
8.4 Solving equations by factoring
Quadratic equations is an equation in the form
ax2 + bx + c = 0,
where a, b and c are real numbers with a ≠ 0.
This is the standard form of a quadratic equation.
Exercise:
Identify if the equations are (a) quadratic equation and (b) if so, are they in the standard
form.
1. x2 = 3 (a) yes (b) no
2. 3x2 – 5x = 2 (a) yes (b) no
2
3. x + 7x + 12 = 0 (a) yes (b) yes
In factoring a quadratic equation, use the zero-factor property. That is, if a and b are real
numbers and if ab = 0, then a =0 or b = 0.
Step 1 Write in standard form.
If the equation is not in its standard form, rewrite it with all the terms on one side.
Example: x2 + 5x = -6
X2 + 5x + 6 = 0 Transpose -6 or add 6 to both sides of the
equation.
Step 2 Factor completely.
Example: X2 + 5x + 6 = 0
(x + 3) (x + 2) = 0 Factored form of the polynomial.
Step 3 Use the zero-factor property.
Example: x+3=0 or x+2=0
x = -3 or x = -2
Both of these equations have solutions that are -3 and -2. Therefore, the solution
set is {−3, −2} .
Step 4 Check each solution in the original equation.
To check these answers, substitute -3 or -2 for x in the original equation.
Example: If x = -3, then
(-3 + 3) (-3 + 2) = 0
0 (-1) = 0
0 =0 True.
If x = -2, then
(-2 + 3) (-2 + 2) = 0
(1) (0) = 0
0 =0

245
When a quadratic equation has a common factor, it is important to extract this first before
factoring it out completely.
Example: 12s2 – 2s = 4
Subtract 4 from each side to rewrite the equation in its standard form.
12s2 – 2s – 4= 0
2 (6s2 – s – 2) = 0 Factor out 2.
2 (3s – 2) (2s + 1) = 0 Factor the trinomial.
2=0 or 3s – 2 = 0 or 2s + 1 = 0 Zero-factor property.
The equation 2 = 0 has no solution. For 3s – 2 and 2s + 1, solve for s each.
3s – 2 = 0 2s + 1 = 0
3s = 2 2s = -1
3𝑠 2 2𝑠 −1
= =
3 3 2 2
2 −1
s= s=
3 2
2 −1
The solutions of 12s2 – 2s = 4 are and . Check these by substituting the solutions to the
3 2
original equation.

Lesson Exercise:
Solve.
1. t(t + 4) = 0
a. {𝑡}
b. {−4}
c. {𝟎, −𝟒}
Sol. t(t + 4) = 0
t=0 t+4=0
t = -4

2. The quadratic form: ____ = 0


a. x2 + bx + c
b. ax2 + bx + c
c. ax2 + bx – c

3. 2y2 – 18 = 0
a. {−4, 4}
b. {−𝟑 , 𝟑}

246
c. {−3, 2}
Sol. (2y – 6) (y + 3) = 0
2y – 6 = 0 y+3=0
2y = 6 y = -3
y=3

4. n2 = 121
a. {11}
b. {−11, 10}
c. {−𝟏𝟏, 𝟏𝟏}
Sol. √𝑛2 = √121
n = 11 or n = -11

5. z2 = -2 – 3z
a. {−1, 2}
b. {1, 2}
c. {−𝟏, −𝟐}
2
Sol. z + 3z + 2 =0
(z + 2) (z + 1) = 0
z = -2 z = -1

6. 4b(2b + 3) = 36
𝟑
a. {−𝟑, }
𝟐
33
b. {9, }
2

c. {3, 2}
Sol. 4b(2b + 3) – 36 = 0
8b2 + 12b – 36 = 0
(4b – 6) (2b + 6) = 0
4b – 6 = 0 2b + 6 = 0
4b = 6 2b = -6
3
b= b = -3
2

7. (3m + 4) (6m2 + m – 2) = 0

247
−4 1 2
a. { , , }
3 2 3
−𝟒 −𝟐 𝟏
b. { , , }
𝟑 𝟑 𝟐
−2 1 4
c. { , , }
3 2 3

Sol. 3m + 4 = 0 6m2 + m – 2 = 0
3m = -4 (3m + 2) (2m – 1) = 0
−4
m= 3m + 2 = 0 2m – 1 = 0
3
−2 1
m= m=
3 2

8. 4x2 = 4x
a. {𝟎, 𝟏}
b. {0}
c. {4}
Sol. 4x – 4x = 0
2

4x(x – 1) = 0
4x = 0 x – 1 = 0
x=0 x=1

9. y3 – 6y2 + 18y = 0
a. {2, 4}
b. {𝟎, 𝟐, 𝟒}
c. {0, 1, 8}
Sol. y(y2 – 6y + 8) = 0
y=0 y2 – 6y + 8 = 0
(y – 4) (y – 2) = 0
y–4=0 y–2=0
y=4 y=2

10. a(a – 5) = -6
a. {0, 4}
b. {1, 3}
c. {𝟐, 𝟑}
Sol. a(a – 5) + 6 = 0
a2 – 5a + 6 = 0

248
(a – 3) (a -2) = 0
a–3=0 a–2=0
a=3 a=2

9 Rational Algebraic Expressions and Equations


9.1 Simplifying Rational Algebraic Expressions
Lowest Terms:

A rational expression (Q ≠ 0) is in lowest terms if the greatest common factor of its


numerator and denominator is 1.
Fundamental Property of Rational Expressions

If is a rational expression and if k represents any polynomial, where k ≠ 0, then

= since = · = ·1=
Example:

1. =

2. =

The rational expression equals for all values of x, where x ≠ -2.

3.
In solving for factors that are opposites, either we multiply the numerator or denominator with -1
to even them out.

= = = = -1
A fraction in which the numerator and denominator show subtraction of the same terms but in
opposite order is equal to -1. Moreover, if a factor is the opposite of a factor in the denominator,
the quotient of those two factors is -1.

249
Here’s a video link courtesy of YouTube that will help you understand further how to simplify
rational expressions.
A tip you can learn from the video is when you have to simplify rational algebraic expressions,
always factor out first before you can simplify so that you can identify whether the expressions
have like terms you can cancel out.
Lesson Exercise:
Simplify the following expressions.
𝑚2 − 𝑛 2
1.
𝑚+𝑛

a. m + n
b. m –n
c. (m + n) (m –n)
(,𝑚+𝑛) (𝑚−𝑛)
Sol. = 𝑚−𝑛
𝑚+𝑛

6− 𝑡
2.
𝑡− 6

a. -1
b. 1
c. 0
6−𝑡
Sol. = -1
−1(6−𝑡)

2𝑥 2 − 3𝑥−5
3.
2𝑥 2 − 7𝑥+5
𝒙+𝟏
a.
𝒙−𝟏

b. x – 1
c. x + 1
(2𝑥−5)(𝑥+1) 𝑥+1
Sol. (2𝑥−5)(𝑥−1) =
𝑥−1

15(𝑚−1)
4.
9(𝑚−1)
15
a.
9
5
b.-
3
𝟓
c.
𝟑
15(𝑚−1) 15 5
Sol. = =
9(𝑚−1) 9 3

250
12𝑚2 − 3
5.
8𝑚−4
3(4𝑚2 − 1)
a.
4(2𝑚−1)
𝟑(𝟐𝒎+ 𝟏)
b.
𝟒
3
c.
4
3(4𝑚2 − 1) 3(2𝑚−1)(2𝑚+1) 3(2𝑚+ 1)
Sol. = =
4(2𝑚−1) 4(2𝑚−1) 4

9𝑟 2 − 4𝑠 2
6.
9𝑟+6𝑠

a. r – 2s
3𝑟+2𝑠
b.
3
𝟑𝒓 − 𝟐𝒔
c.
𝟑
(3𝑟 − 2𝑠)(3𝑟+2𝑠) 3𝑟 − 2𝑠
Sol. =
3(3𝑟+2𝑠) 3

5−𝑥
7.
5+𝑥

a. -1
𝟓−𝒙
b.
𝟓+𝒙
−5+𝑥
c.
5+𝑥

Sol. It’s already in its lowest term.

𝑥+4
8. – is equivalent to ___.
𝑥−3
–(𝑥+4)
a.
𝑥−3
𝑥+4
b.
−(𝑥−3)

c. both a and b

𝑥 2 𝑦+ 𝑦+ 𝑥 2 + 𝑧
9.
𝑥𝑦+𝑥𝑧

a. x2 + 1
b. x(y + z)
𝒙𝟐 + 𝟏
c.
𝒙

251
𝑥 2 𝑦+ 𝑦+ 𝑥 2 + 𝑧
Sol.
𝑥𝑦+𝑥𝑧

𝑦( 𝑥 2 + 1)+ 𝑧 ( 𝑥 2 + 1)
𝑥(𝑦+𝑧)

(𝑦+𝑧)( 𝑥 2 + 1) 𝑥 2+ 1
=
𝑥(𝑦+𝑧) 𝑥

𝑚2 − 𝑛2 − 4𝑚−4𝑛
10.
2𝑚−2𝑛−8

a. m – 4 – n
1
b.
m–4–n
𝒎+𝒏
c.
𝟐
𝑚2 − 𝑛2 − 4𝑚−4𝑛 (𝑚−𝑛)(𝑚+𝑛)− 4(𝑚+𝑛)
Sol. =
2𝑚−2𝑛−8 2(𝑚−𝑛−4)
(𝑚+𝑛) 𝑚−𝑛 − 4 𝑚+𝑛
=
2(𝑚−𝑛−4) 2

9.2 Multiplications and Division of Rational Algebraic Expressions


From the video itself, it is important to always factor out common factors before you can
operate.
Multiplying Rational Expressions

The product of the rational expressions · =

Example:

1. · =

2. · = ·

Dividing Rational Expressions

If and are any two rational expressions, with 0, then

252
= · =
Example:

1. =

3. = · =

Lesson Exercise:
Perform the operation and simplify.
2(𝑐+𝑑) 18
1. ∙
3 6(𝑐+𝑑)2
2
a.
3(𝑐+𝑑)
𝟐
b.
𝒄+ 𝒅
1
c.
𝑐+ 𝑑
2(𝑐+𝑑) 18 2
Sol. ∙ =
3 6 ( 𝑐+𝑑)(𝑐+𝑑) 𝑐+ 𝑑

35𝑞8 25𝑞6
2. ÷
9𝑞5 10𝑞5

𝟏𝟒𝒒𝟐
a.
𝟗
175𝑞4
b.
18
3𝑞2
c.
4
35𝑞8 25𝑞6 35𝑞8 10𝑞5 7𝑞2 14𝑞2
Sol. ÷ = ∙ = ∙2=
9𝑞5 10𝑞5 9𝑞5 25𝑞6 9 9

4 16
3. ÷
𝑚−2 2−𝑚
−𝟏
a.
𝟒

253
2−𝑚
b.
4𝑚−8
1
c.
4
4 2−𝑚 4 2−𝑚 −1
Sol. ∙ = ∙ =
𝑚−2 16 −1(2−𝑚) 16 4

8𝑟+16 6𝑟− 6
4. ∙
24𝑟−24 3𝑟+6

a. 3
𝟐
b.
𝟑

c. 2
8(𝑟+2) 6(𝑟−1) 2
Sol. ∙ =
24(𝑟−1) 3(𝑟+2) 3

(𝑥+4)3 (𝑥−3) 𝑥 2 + 8𝑥+16


5. ÷
𝑥 2− 9 𝑥 2 + 6𝑥+9

a. (x + 4) (x + 3)
b. (x + 4)2 (x + 3)
c. x + 3
(𝑥+4)3 (𝑥−3) 𝑥 2 + 6𝑥+9 (𝑥+4)(𝑥+4)(𝑥+4) (𝑥−3) (𝑥+3)(𝑥+3)
Sol. ∙ = ∙ = (x + 4) (x + 3)
𝑥 2− 9 𝑥 2 + 8𝑥+16 (𝑥− 3)(𝑥+3) (𝑥+4)(𝑥+4)

𝑧 2 − 3𝑥+2 𝑧−1
6. ÷
𝑧 2 + 4𝑧+3 𝑧+1

a. z – 2
b. z + 3
𝒛−𝟐
c.
𝒛+𝟑
(𝑧−1)( 𝑧−2) 𝑧+1 𝑧−2
Sol. (𝑧+1)(𝑧+3)
∙ =
𝑧−1 𝑧+3

9.3 Least Common Multiple of Algebraic Expressions


Least Common Denominator (LCD): the least expression that all denominators divide into
without remainder.
In finding the LCD
Step 1 Factor each denominator into prime factors

Example: and

 x2 – 6x + 9 = (x-3)2
 5x2 – 15x = 5x (x- 3)

254
Step 2 List each different factor the greatest number of times it appears.
Example: x2 – 6x + 9 = (x-3)2
5x2 – 15x = 5x (x- 3)
In this example, x -3 has the greatest number of times.
Step 3 Multiply the factors from Step 2 to get the LCD.

Example: and

X2 – 6x + 9 = (x-3)2
5x2 – 15x = 5x (x- 3)
LCD = (x-3)2 · 5x
= 5x(x-3)2

Lesson Exercise:
Find the LCD for each fraction.
2𝑞 18
1. ,
𝑝−𝑞 𝑞−𝑝

a. p – q
b. p2 – q2
c. q + p
The expression q – p can be written as -1(p – q).

7 −23
2. ,
8𝑘 12𝑘−24

a. k - 2
b. 24k(k -2)
c. 12(k – 2)
Sol. LCD : 12k – 24 = 12(k – 2)
24k = 8 ∙ 𝟑𝑘

−7 −22
3. ,
3𝑟 4 𝑠 5 9𝑟 6 𝑠8

a. 9r6s8
b. 27r6s8
c. 3r4s5

255
19 16 6
4. , ,
𝑧 2 + 4𝑧−12 𝑧 2 + 𝑧−30 𝑧 2 + 2𝑧−24

a. (z + 6) (z – 2) (z – 4)
b. (z + 6) (z – 2) (z – 5) (z – 4)
c. (z – 2) (z – 5) (z – 4)
Sol. 𝑧2 + 4𝑧 − 12 = (z – 2) (z + 6)
𝑧 2 + 𝑧 − 30 = (z – 5) (z + 6)
𝑧 2 + 2𝑧 − 24 = (z + 6) (z – 4)

−14 5
5. ,
5𝑝−30 6𝑝−36

a. 30(p – 6)
b. p – 6
c. 6(p – 6)
Sol. 5p – 30 = 5(p – 6)
6p – 36 = 6(p – 6)
LCD = 5 ∙ 𝟔(p – 6)
9.4 Addition and Subtraction of Rational Algebraic Expressions
Adding Rational Expressions

If and are rational expressions, then

.
In adding rational expressions with different denominators, here are the steps.
Step 1 Find the LCD.

Example:
10x = 5 · 2 · x
7x = 7 · x
LCD = 5 · 2 · 7 · x
= 70x
Step 2 Rewrite the fractions with the LCD as the denominator.

Example:
Step 3 Add.

256
Example: = =
Step 4 Simplifying it to its lowest term.

Example: =

=
Subtracting Rational Expressions

If and are rational expressions, then

.
The same process in adding rational expressions applies in subtracting but only use and
operate the subtract sign.

Example: Fundamental Property.

=
Source: Lial. , & Hornsby, (2000). Beginning and intermediate algebra. (Second ed.).
Lesson Exercise:
Perform the operations and simplify.
𝑡2 −3𝑡
1. +
𝑡−3 𝑡−3

a. t
b. t(t + 3)
c. t – 3
𝑡 2 + (−3𝑡) 𝑡 2 − 3𝑡 𝑡(𝑡−3)
10. Sol. = = =t
𝑡−3 𝑡−3 𝑡−3

5 𝑟
2. −
7 2
5−7𝑟
a.
7

257
b. 7r
𝟏𝟎−𝟕𝒓
c.
𝟏𝟒
(2)5−(7)𝑟 10−7𝑟
Sol. =
14 14

2𝑥 3 4
3. + − 2
𝑥−1 𝑥+1 𝑥 −1
1
a.
𝑥+1

b. 2x – 1
𝟐𝒙+𝟕
c.
𝒙+𝟏
2𝑥(𝑥+1)+ 3(𝑥−1)− 4 2𝑥 2 + 2𝑥+3𝑥−3− 4 2𝑥 2 + 5𝑥−7 (2𝑥+7)(𝑥−1) 2𝑥+7
Sol. (𝑥−1)(𝑥+1)
= (𝑥−1)(𝑥+1)
= (𝑥−1)(𝑥+1)
= (𝑥−1)(𝑥+1)
=
𝑥+1

2𝑥−6 𝑥+5
4. +
4 6
4(2𝑥−2)
a.
12
𝟐𝒙−𝟐
b.
𝟑
8𝑥−8
c.
12
3(2𝑥−6)+ 2(𝑥+5) 6𝑥−18+2𝑥+10 8𝑥−8 4(2𝑥−2) 2𝑥−2
Sol. = = = =
12 12 12 12 3

4 6 1
5. + 2 −
𝑟2− 𝑟 𝑟 + 2𝑟 𝑟 2 + 𝑟−2
𝟗𝒓+𝟐
a.
𝒓(𝒓+𝟐)(𝒓−𝟏)
10𝑟−3
b.
𝑟(𝑟+2)(𝑟−1)

10𝑟 2 −3
c. (𝑟+2)(𝑟−1)
4 6 1 4(𝑟+2)+ 6(𝑟−1)− 𝑟 4𝑟 + 8 + 6𝑟−6 − 𝑟 9𝑟+2
Sol. + − (𝑟−1)(𝑟+2)
= = =
𝑟(𝑟−1) 𝑟(𝑟+2) 𝑟(𝑟+2)(𝑟−1) 𝑟(𝑟+2)(𝑟−1) 𝑟(𝑟+2)(𝑟−1)

12 5
6. −
5𝑚2 𝑚
12−5𝑚
a.
𝑚2
𝟏𝟐−𝟐𝟓𝒎
b.
𝟓𝒎𝟐
12−5(𝑚)
c.
𝑚2
12−5(5𝑚) 12−25𝑚
Sol. =
5𝑚2 5𝑚2

258
𝑚 𝑚−1
7. +
𝑚2 − 1 𝑚2 + 2𝑚+1
2𝑚2 − 𝑚+1
a. (𝑚−1)(𝑚+1)(𝑚+1)
2𝑚2 − 𝑚+1
b.
(𝑚+1)2 (𝑚−1)

c. both a and b
𝑚 𝑚−1 𝑚(𝑚+1)+ (𝑚−1)(𝑚−1) 𝑚2 + 𝑚+ 𝑚2 − 𝑚−𝑚+1 2𝑚2 − 𝑚+1
Sol. (𝑚−1)(𝑚+1) + (𝑚+1)(𝑚+1)
= (𝑚−1)(𝑚+1)(𝑚+1)
= (𝑚−1)(𝑚+1)(𝑚+1)
= (𝑚−1)(𝑚+1)(𝑚+1)
𝑜𝑟
2𝑚2 − 𝑚+1
(𝑚+1)2 (𝑚−1)

5𝑝 3𝑝+1
8. −
𝑝−𝑞 4𝑝−4𝑞
𝟏𝟕𝒑−𝟏
a.
𝟒(𝒑−𝒒)
17𝑝+1
b.
4(𝑝−𝑞)
17𝑝+1
c.
4𝑝−4𝑞)
5𝑝(4)− 3𝑝−1 20𝑝− 3𝑝−1 17𝑝−1
Sol. = =
4(𝑝−𝑞) 4(𝑝−𝑞) 4(𝑝−𝑞)

9.5 Simplifying Complex Rational Algebraic Expressions


Complex Fraction: is a rational expression with fractions in the numerator, denominator or
both.
There are 2 methods in simplifying complex fractions.
Method 1
Step 1 Write both the numerator and denominator as single fractions.
𝑥𝑦2
𝑧3
Example: 𝑦3
𝑥𝑧3

Step 2 Make the complex fraction to a division problem.


𝑥𝑦2
𝑧3 𝑥𝑦 2 𝑥𝑧 3
Example: 𝑦3
= ∙
𝑧3 𝑦3
𝑥𝑧3

Step 3 Simplify the expression.


𝑥𝑦2
𝑧3 𝑥𝑦 2 𝑥𝑧 3
Example: 𝑦3
= ∙
𝑧3 𝑦3
𝑥𝑧3

𝑥𝑦 2 𝑥𝑧 3
= ∙
𝑧3 𝑦3

259
𝑥2
=
𝑦

Method 2
Step 1 Find all the LCD of all fractions within the complex fraction.
2 1
+
3𝑏 6𝑏
Example: 5 3

2𝑏 3𝑏

The LCD for 3b, 6b and 2b is 6b.


Step 2 Multiply the LCD to both the numerator and denominator of the complex fraction.
2 1 2 1
+ 6𝑏(3𝑏+ 6𝑏)
3𝑏 6𝑏
Example: 5 3 = 5 3
− 6𝑏(2𝑏 − 3𝑏)
2𝑏 3𝑏
2 1
6𝑏(3𝑏)+6𝑏( 6𝑏)
= 5 3
6𝑏(2𝑏) − 6𝑏(3𝑏)

2(2)+ 1
=
3(5)− 2(3)
4+1
=
15−6
5
=
9

Lesson Exercise:
Simplify.
𝑥
𝑦2
1. 𝑥2
𝑦
𝟏
a.
𝒙𝒚
1
b.
𝑥
1
c.
𝑦
𝑥
(𝑦 2 ) 2 𝑥 1
𝑦
Sol. 𝑥2
= =
(𝑦 2 ) 𝑥 2𝑦 𝑥𝑦
𝑦

𝑝4
𝑟
2. 𝑝2
𝑟2
𝑝4 𝑟
a.
𝑝2
b. 𝒑𝟐 𝒓
𝑝4 𝑟
c.
𝑝
𝑝4
𝑝4 𝑟
Sol. (r ) 2 𝑟
𝑝2
= = 𝑝2𝑟
𝑝2
𝑟2

260
𝑞−5
𝑞
3. 𝑞+5
3
𝟑(𝒒−𝟓)
a.
𝒒(𝒒+𝟓)
b. 3𝑞 − 5
3𝑞−15
c.
𝑞+5
𝑞−5
𝑞 3(𝑞−5) 3(𝑞−5)
Sol. (3q) 𝑞+5 = =
𝑞(𝑞+5) 𝑞(𝑞+5)
3

4𝑎4𝑏3
3𝑎
4. 2𝑎𝑏4
𝑏2
𝑎2 𝑏
a.
3
2𝑎2
b.
3𝑏
𝟐𝒂𝟐 𝒃
c.
𝟑
4𝑎4𝑏3
2 3𝑎 𝑏2 (4𝑎4 𝑏3 ) 4𝑎4 𝑏5 2𝑎2 𝑏
Sol. (3ab ) 2𝑎𝑏4
= = =
3𝑎(2𝑎𝑏4 ) 6𝑎2 𝑏4 3
𝑏2

5
𝑎− 𝑎
5. 1
𝑎+ 𝑎
𝑎−5
a.
𝑎+1
b. a
𝒂𝟐 − 𝟓
c.
𝒂𝟐 + 𝟏
5 𝑎2 − 5
𝑎− 𝑎 𝑎2 − 5 𝑎 𝑎2 − 5
𝑎
Sol. (a) 1 = 𝑎2 + 1
= ∙ =
𝑎+ 𝑎 𝑎2 + 1 𝑎2 + 1
𝑎 𝑎

1 1

4 𝑎2
6. 1 1
+
2 𝑎
𝑎2 − 4
a.
2𝑎(𝑎+2)
𝑎2 − 4
b.
2𝑎
𝒂−𝟐
c.
𝟐𝒂
1 1
− 2 𝑎2 − 4 (𝑎−2)(𝑎+2) 𝑎−2
2 4 𝑎
Sol. (4a ) 1 1 = = =
+ 2𝑎(𝑎+2) 2𝑎 (𝑎+2) 2𝑎
2 𝑎

2
+2
𝑥−1
7. 2
−2
𝑥−1
a. 4
𝒙
b.
𝟐−𝒙

261
2𝑥−1
c.
2(2−𝑥)
2
+2 2+2(𝑥−1) 2+2𝑥−2 2𝑥 2𝑥 𝑥
Sol. (x – 1) 𝑥−1
2 = = = = =
−2 2−2(𝑥−1) 2−2𝑥+2 −2𝑥+4 2(2−𝑥) 2−𝑥
𝑥−1

1
𝑎+1
8. 2
𝑎2 − 1
𝒂−𝟏
a.
𝟐
1
b.
2
𝑎+1
c.
2
1
𝑎+1 𝑎−1
Sol. (a- 1) (a + 1) 2 =
2
𝑎2− 1

9.6 Solving Rational Equations


What we mean when we say equations involving rational expression is that it has to be
solved. That is the difference between an expression and an equation since an expression
needs only to be simplified.

Step 1 Multiply the LCD to both sides of the equation.


𝑧+2 𝑧
Example: − =2
3 4

LCD = 12
𝑧+2 𝑧
12( − ) = 12 · 2 Distributive Property.
3 4

Step 2 Solve.
𝑧+2 𝑧
Example: − =2
3 4
𝑧+2 𝑧
12( − ) = 12 · 2
3 4

4(z + 2) – 3(z) = 24
4z + 8 – 3z = 24
z = 24 – 8
z = 16
Step 3 Check.
In the next example, we will be solving for a specified variable But we will still use the
same method above in solving the rational equation.
3 1 1
Step 1 = + for y
𝑥 𝑧 𝑦

LCD = xyz
3 1 1
𝑥𝑦𝑧 ( ) = 𝑥𝑦𝑧 ( ) + 𝑥𝑦𝑧 ( ) Distributive Property.
𝑥 𝑧 𝑦

3yz = xy + xz

262
3 1 1
Step 2 = +
𝑥 𝑧 𝑦
3 1 1
𝑥𝑦𝑧 ( ) = 𝑥𝑦𝑧 ( ) + 𝑥𝑦𝑧 ( )
𝑥 𝑧 𝑦

3yz = xy + xz
3yz – xy = xz Subtract xy since we are solving for y.
y(3z – x) = xz Factor out y.
𝑥𝑧
y= Divide both side by the coefficient of y.
3𝑧−𝑥

Step 3 Don’t forget to check your answer by substituting the value to the original
equation.

Lesson Exercise:
Find all the values for which at least one denominator is equal to 0.
3 5
1. − =1
𝑥+2 𝑥
a. -2
b. -2, 1
c. -2, 0
3 5
Sol. − =1
−2+2 0

8 7
2. (𝑥−8)(𝑥+2)
=
3𝑥−10
𝟏𝟎
a. 8, -2,
𝟑
3
b. 8, -2,
10
c. 8, 2, 10
8 7
Sol. (8−8)(−2+2) = 10
3( 3 )−10

4 1
3. + 2 =0
𝑥 2 + 8𝑥−9 𝑥 −4
a. 1, -2, 2
b. -9, 1, -2, 2
c. 1, 0, -2, 2
4 1
Sol. (𝑥+9)(𝑥−1) + =0
𝑥 2− 4

4 1
+ =0
(−9+9)(1−1) (±2)2 − 4

Solve and Simplify.


5𝑥 1
1. =
14𝑥−3 𝑥
𝟏
a. {− , 𝟑}
𝟓
b. {0, 1}
1
c. {− , 0}
5

263
Sol. 5x(x) = 14x + 3
5x2 = 14x + 3
5x2 –14x – 3 = 0
(5x + 1) (x – 3) = 0
5x + 1 = 0 x–3=0
1
x=− x=3
5

4 1
2. + = 2
𝑦 𝑦
2
a. { }
5
𝟓
b. { }
𝟐
c. {5}
5
Sol. =2
𝑦
2y = 5
5
y=
2

𝑡 4 𝑡−2
3. + =
6 3 3
a. {𝟏𝟐}
𝑡
b. { }
6
𝑡
c. {− }
6
𝑡+2(4) 2(𝑡−2) 𝑡+8 2𝑡−4
Sol. = = = = 6(t + 8) = 6(2t – 4) = 6t – 12t = -24 – 48
6 6 6 6
-6t = -2
t = 12

𝑚 1
4. =
8𝑚+3 3𝑚
1
a. { , 3}
3
b. {−1, 3}
𝟏
c. {− , 𝟑}
𝟑
Sol. 3m(m) = 8m + 3
3m2 – 8m – 3 = 0
(3m + 1) (m – 3) = 0
3m + 1 = 0 m–3=0
1
m=− m=3
3

10 Roots and Radical Expressions and Equations


10.1 Rational Exponents
𝑝 𝑞 𝑞
If 𝑝 𝑎𝑛𝑑 𝑞 are integers with q > 0 and x is a positive real number, then 𝑥𝑞 = ( √𝑥 )p = √𝑥 p.
Rational exponents are also called fractional exponents.
𝑛 𝑛
If 𝑛 is a positive integer greater than one and √𝑥 is a real number then 𝑥𝑛1 = √𝑥.

264
Notice that the denominator of the fractional exponents of x is the index of the radical.

Converting radicals into exponential notation is also another way to solve it. But rational
exponents are subject to all of the same rules as other exponents when they appear in algebraic
expressions.

Laws of Exponents
a. 𝑥 1 = 𝑥
b. 𝑥 0 = 𝑥
1
c. 𝑥 −𝑛 = n
𝑥
1
d. 𝑥 −1 =
𝑥

e. 𝑥 𝑚 𝑥 𝑛 = 𝑥 𝑚+𝑛
𝑥𝑚 𝑥 𝑚−𝑛 𝑥𝑚 1
f. = = 𝑥 𝑚−𝑛 or =
𝑥𝑛 1 𝑥𝑛 𝑥 𝑛−𝑚
𝑦5 𝑦 5−3 𝑦2
 = = = 𝑦2
𝑦3 1 1
𝑦3 1 1
 = = 2
𝑦5 𝑦 5−3 𝑦

g. (𝑥𝑦)𝑚 = 𝑥 𝑚 𝑦 𝑚
 (𝑎𝑏)3 = 𝑎3 𝑏 3
𝑥 𝑥𝑛
h. ( )n =
𝑦 𝑦𝑛

𝑚 𝑚5
 ( )5 =
𝑛 𝑛5

i. (𝑥 𝑚 )n = 𝑥 𝑚.𝑛 = 𝑥 𝑚𝑛
 (𝑦 2 )3 = 𝑦 2 .3 = 𝑦 6
For expressions with a negative base:
 If root is odd, retain the negative.
1 1
(−64)3 = [(−4)3 ]3 = −4

 If root is even, it is not a real number.


1 1
(−16)4 = [(−2)4 ]4 = 2 but 24 ≠ −16

Examples:
1
2 2 2
1. 642 = ( √64)1 = √64 = √82 = ± 8
−3
1 1 1 1
2. 25 2 = 3 = 3= =
( 2√25) (5)3 125
252
5
2
3. 92 = ( √9)5 = (3)5 = 243

265
2 2
32𝑥 2 2 32 2 325 (25 )5 22 4
4. ( 12 )5 = ( 10)5 = 20 = = =
𝑥 𝑥 𝑥4 𝑥4 𝑥4
𝑥5

5 5 3
5 −2 5 2
𝑎3 ( 3√𝑎 ) √𝑎 5 𝑎3 5
5. 𝑎3 𝑏 3 = 𝑎3 𝑏 −3 = 2 = 3 2 = 3 =√ 2
𝑏3 ( √𝑏 ) √𝑏2 𝑏

Lesson Exercise:
Simplify the following expressions.
5
1. 1
𝑏2

a. √5𝑏
5
b. 2
√𝑏
1
c. 2
√5𝑏
𝑏
d. 2
√5

Solution:
5 5
1 = 2
𝑏2 √𝑏
1 1
2. (3𝑦)3 𝑦 3
3
a. √3𝑦 2
2
b. √3𝑦 3
2
c. y √3
2
d. √3𝑦 3
Solution:
1 1 1 1
3 3
(3𝑦)3 𝑦 3 = (3𝑦 . 𝑦)3 = (3𝑦 2 )3 = ( √3𝑦 2 )1 = √3𝑦 2

5 1
𝑎3 𝑏3
3. 2
𝑐3

𝑎3 𝑏 5
a. √ 2 ( 3 2)
𝑐 √𝑐

𝑎3 5
b. √ 2 𝑏
𝑐

2
𝑎 𝑏 5
c. √ 3
𝑐

266
3
𝑎 𝑏 5
d. √ 2
𝑐

Solution:
5 1
𝑎3 𝑏3 𝑎5 𝑏1 1 3
𝑎 𝑏 5
2 =( )3 =√ 2
𝑐2 𝑐
𝑐3
2
4. Rewrite √36𝑥 2 𝑦 4 into its exponential notation and simplify.
a. 6𝑥 2 y
b. (36𝑥 2 𝑦 4 )2
c. 6𝑥𝑦 2
3
d. (36𝑥 2 𝑦 4 )2
Solution:
1 1 1 1
2
√36𝑥 2 𝑦 4 = (36𝑥 2 𝑦 4 )2 = (36)2 (𝑥 2 )2 (𝑦 4 )2 = 6𝑥𝑦 2

2
5. (𝑥 − 4)5 − 3 = 1 Find x.
1
a. 𝑥 =
36

b. 𝑥 = 36
c. 𝑥 = 28
1
d. 𝑥 = −
28

Solution:
2
(𝑥 − 4)5 - 3 = 1
2
(𝑥 − 4)5 = 4
2 5
[(𝑥 − 4)5 = 4]2
25 5
(𝑥 − 4)5.2 = 42
2 5
𝑥 − 4 = ( √4)
𝑥 − 4 = 25
𝑥 − 4 = 32
𝑥 = 32 + 4
𝑥 = 36

1
6. Rewrite 4 into its exponential notation and simplify.
√𝑝4 𝑞−8

267
1
a.
𝑝𝑞2

𝑞2
b.
𝑝
𝑞
c.
𝑝2

𝑞2
d. −
𝑝

Solution:
1 1 1 1 𝑞2
4 = 1 = 4 −8 = =
√𝑝4 𝑞 −8 𝑝𝑞−2 𝑝
(𝑝4 𝑞−8 )4 𝑝4 𝑞 4

3
7. Rewrite √8𝑏 6 𝑐 −4 into its exponential notation.
2𝑏2
a. 3
𝑐4
2𝑏3
b. . 4
𝑐3
𝑏2
c. 4
2𝑐 3
2𝑏2
d. 4
𝑐3

Solution:
3 3 3
√8𝑏 6 𝑐 −4 = √8 . √𝑏 6 𝑐 −4
1 6 −4
2𝑏2
(𝑏 6 𝑐 −4 )3 = 2(𝑏 3𝑐 3 ) = 4
𝑐3

1 2
8. Evaluate (− )3 when m = 27.
𝑚
1
a.
9
1
b. −
9

c. 9
d. no solution
Solution:
2 2
−1 2
3
1 2 3 1 √−1 1
(− )3 = ( √− ) = (3 ) =( ) =
27 27 √27 3 9

1 1
9. Evaluate 𝑎 + 𝑎2 − 𝑎4 when 𝑎 = 16.
a. 17

268
b. 18
c. 19
d. 20
Solution:
1 1 2 4
(16) + 162 − (16)4 = (16) + √16 − √16 = 16 + 4 − 2 = 18

2
10. If (𝑥) = 𝑥 3 , find 𝑓(8).
a. 2√2
1
b.
2√2
1
c.
4

d. 4
Solution:
2 2 2
3
If𝑓(𝑥) = 𝑥 3 , then 𝑓(8) = 83 = ( √8) = (2)2 = 4

10.2 Simplifying Rational Expressions


10.3 Operations on Radicals
Just like algebraic expression, we can also perform arithmetic operations with radical
expressions.
 Addition and Subtraction
You can only add and subtract only if they are like radical terms, which means same
radicand and same index.
𝑛 𝑛 𝑛
𝑎 √𝑥 + 𝑏 √𝑥 = (𝑎 + 𝑏) √𝑥
Example: 2√3 + 5√3 = 7√3
Steps in Adding or Subtracting Radicals:
Example: 5√3 + 2√75
 First you must try to simplify all radicals to its simplest form.
5√3 + 2√75 = 5√3 + 2√25 ∙ 3 = 5√3 + 2√25 ∙ √3 = 5√3 + (2 ∙ 5) ∙ √3 = 5√3 + 10√3
 Then you can combine any like radicals by combining the coefficients of the radical
terms. The coefficient is the factor that sits outside the radical to the left.
5√3 + 10√3 = 15√3
Example: 5√8 − 3√18 + √3
 Simplify:
5√8 − 3√18 + √3 = 5√4 ∙ 2 − 3√9 ∙ 2 + √3 = 5(2)√2 − 3(3)√2 + √3 = 10√2 − 9√2 + √3
 Combine like terms.
10√2 − 9√2 + √3 = √2 + √3
 Multiplication
Remember the product rule for radicals:

269
𝑛 𝑛 𝑛
√𝑎𝑏 = √𝑎 ∙ √𝑏
A. If indices are the same:
You need to multiply the coefficient and the radicands themselves and simplify the
radical if possible.

Example: 2√3𝑎2 ∙ 3√6𝑎


Multiply. 2√3𝑎2 ∙ 3√6𝑎 = 2 ∙ 3 ∙ √3𝑎2 ∙ √6𝑎 = 6√3𝑎2 ∙ 6𝑎 = 6√18𝑎3
Simplify. 6√18𝑎3 = 6√9 ∙ 2 ∙ 𝑎2 ∙ 𝑎 = 6(3𝑎)√2𝑎 = 18𝑎√2𝑎

4 𝑥3 4 𝑥4
Example: √ ∙√
2 4
4 𝑥3 4 𝑥4 𝑥4 5
Multiply. √ ∙√ = √
2 4 8
4 𝑥4
4 𝑥5 √𝑥 4 ∙ 4√𝑥 √𝑥
Simplify. √ = 4 = 4
8 √8 √8
In this case, we need to rationalize denominator.
𝑥4 𝑥4 𝑥4 4 𝑥4 𝑥4 𝑥4
√𝑥 √2 ( √𝑥 ) ∙ ( √2)
4
√𝑥 √2𝑥 √2𝑥 √2𝑥
4 = 4 ∙4 = 4 4 =4 = 4 =
√8 √8 √2 ( √8) ∙ ( √2) √8 ∙ 2 √16 2
B. If the indices are not the same:
In multiplying radical with different indices, you need to convert the radicals to its
exponential notation.
3
Example: √2𝑥 ∙ 4√3𝑦
1 1 4 3 4 3 7
Exponential Notation. √2 ∙ √2 = (2)3 ∙ (2)4 = (2)12 ∙ (2)12 = (2)12+12 = (2)12
2 4

7
12 12
Simplify. (2)12 = √27 = √128
C. Conjugates:
Recall that(𝑎 + 𝑏)(𝑎 − 𝑏) = 𝑎2 − 𝑏 2.
2
Example: (4 + √3)(4 − √3) = (4)2 − (√3) = 16 − 3 = 13
In this example, 4 + √3 and 4 − √3 are called conjugates to one another.
 Division
We can also use the quotient rule for radicals to write a square root of a quotient as a
quotient of square roots.
𝑛
𝑛 𝑎 √𝑎
√ =𝑛
𝑏 √𝑏
4 4 4
4 9 √9 √9 √9
Example: √ = 4 = 4 4
=
16 √16 √2 2
However if the denominator is a binomial in which one or both terms contain a square
root, multiply numerator and denominator by the conjugate. Oftentimes, this is where
rationalization takes place.

5 5 √3−2 5(√3−2) 5√3−2(5) 5√3−10 5√3 10


Example: = ∙ = 2 = = = = = −5√3 + 10
√3+2 √3+2 √3−2 (√3) −22 3−4 −1 −1 −1

270
Lesson Exercise:
Solve the following expressions:
1. 4√5 + 3√5 − 8√5
a. √5
b. 15√5
c. -√5
d. −15√5
Solution:
4√5 + 3√5 − 8√5 = 7√5 − 8√5 = −√5
2. 5√7 − √𝑎 + 3√7 − 5√𝑎
a. 2√7𝑎
b. 8√7 − 6√7𝑎
c. 8√𝑎 − 6√7
d. 2√𝑎
Solution:
5√7 − √𝑎 + 3√7 − 5√𝑎 = 5√7 + 3√7 − √𝑎 − 5√𝑎 = 8√7 − 6√𝑎

3. 3𝑥 √𝑦 − √𝑎 + 2𝑥 √𝑦 + 3√𝑎

a. 2𝑥 √𝑦 + 5√𝑎

b. 5𝑥 − 4√𝑎𝑦

c. . 5𝑥 √𝑦 + 2√𝑎

d. 5𝑥 √𝑎 + 2√𝑦
Solution:
3𝑥 √𝑦 − √𝑎 + 2𝑥 √𝑦 + 3√𝑎 = 3𝑥 √𝑦 + 2𝑥 √𝑦 − √𝑎 + 3√𝑎 = 5𝑥 √𝑦 + 2√𝑎

4. 2√27𝑥 2 − 4√75𝑦

a. 6𝑥√3 − 20√3𝑦

b. 6𝑥√3 + 20√3𝑦
c. 6𝑥 √3𝑦 − 20√3

d. 20√3 − 6𝑥 √3𝑦
Solution:

2√27𝑥 2 − 4√75𝑦 = 2√9 ∙ 3 ∙ 𝑥 2 − 4√25 ∙ 3 ∙ 𝑦 = 2(3𝑥)√3 − 4(5)√3𝑦 = 6𝑥√3 − 20√3𝑦

271
5. (√3 − √6)(√3 + √6)
a. 3
b. 9
c. -3
d. cannot be simplified
Solution:
2 2
(√3 − √6)(√3 + √6) = (√3) − (√6) = 3 − 6 = −3

3 3
6. √100𝑚4 ∙ √10𝑚2
a. 100𝑚2
3
b. 𝑚 √100
c. 10𝑚2
3
d. 10𝑚 √𝑚2
Solution:
3 3 3 3
√100𝑚4 ∙ √10𝑚2 = 𝑚 3√100𝑚 ∙ √10𝑚2 = 𝑚 3√(100𝑚)(10𝑚2 ) = 𝑚 √10003 = 10𝑚2

7. 2√5(√5 − 3√10)
a. −4√50
b. 10 − 6√50
c. 6 − 10√50
d. 4√50
2√5(√5 − 3√10) = (2√5 ∙ √5) − (2√5 ∙ 3√10) = 2√25 − 6√50 = 2(5) − 6√50 = 10 − 6√50

8. (2√5 + 1)(3√5 − 2)
a. 28 − √5
b. 27√5
c. 28 + √5
d. -27√5

Solution:
(2√5 + 1)(3√5 − 2) = (2√5 ∙ 3√5) + (2√5 ∙ (−2)) + (1 ∙ 3√5) + (1 ∙ (−2))

272
= 6(5) + (−4√5) + 3√5 − 2
= 30 − 4√5 + 3√5 − 2
= 28 − √5

15√6
9.
3√2
5√8
a.
3√2
5√6
b.
√2

c. 5√4
5√12
d.
2

Solution:
15√6 5√6 5√6 √2 (5√6)(√2) 5√12
= = ∙ = =
3√2 √2 √2 √2 2 2

4−√20
10.
4
2+√5
a.
2

b. 1 − √5
2−√5
c.
2

d. . 1 − √20
Solution:
4 − √20 4 − √4 ∙ 5 4 − 2√5 2(2 − √5) 2 − √5
= = = =
4 4 4 2∙2 2

10.4 Radical Equations


A radical equation is an equation with at least one radical expression.
A radical equation is an equation in which the variable is inside a radical symbol (in the
radicand).
√𝑥 + 3 = 10 𝑥 + √3 = 10
This is a radial equation. This is not a radical equation.

 Steps in solving radical equations:


Example: √2𝑥 + 9 − 5 = 0

273
1. Isolate the radical (or one of the radicals) to one side of the equal sign.
√2𝑥 + 9 − 5 = 0
√2𝑥 + 9 = 5
2. If the radical is a square root, square each side of the equation. (If the radical is not a square
root, raise each side to a power equal to the index of the root.)
√2𝑥 + 9 = 5
2
(√2𝑥 + 9) = (52 )
Take note that you should square the side and NOT the terms. Consider the following example:
2+3= 5 𝑏𝑢𝑡 2+3=5
(2 + 3)2 = (5)2 22 + 32 = 52
25 = 25 4 + 9 = 25
13 ≠ 25
3. Solve the resulting equation.
2
(√2𝑥 + 9) = (5)2
2𝑥 + 9 = 25
2𝑥 = 25 − 9
2𝑥 = 16
2𝑥 16
=
2 2
𝑥=8
4. Check your answer(s) to avoid extraneous roots.
√2(8) + 9 − 5 = 0
√16 + 9 − 5 = 0
√25 − 5 = 0
5−5= 0
0=0
Examples:
1. √7𝑥 + 2 = 3
Solution:
√7𝑥 + 2 = 3
2
(√7𝑥 + 2) = 32
7𝑥 + 2 = 9
7𝑥 = 7

274
7𝑥 7
=
7 7
𝑥=1

Check:
√7(1) + 2 = 3
√7 + 2 = 3
√9 = 3
3=3

2. 𝑥 − 3 = √30 − 2𝑥
Solution:
𝑥 − 3 = √30 − 2𝑥
2
(𝑥 − 3)2 = (√30 − 20𝑥)
𝑥 2 − 6𝑥 + 9 = 30 − 2𝑥
𝑥 2 − 6𝑥 + 2𝑥 + 9 − 30 = 0
𝑥 2 − 4𝑥 − 21 = 0
(𝑥 − 7)(𝑥 + 3) = 0
𝑥 − 7 = 0 𝑜𝑟 𝑥 + 3 = 0
𝑥 = 7 = 0 𝑜𝑟 𝑥 = −3

Check:
(7) − 3 = √30 − 2(7) (−3) − 3 = √30 − 2(−3)
4 = √30 − 14 − 6 = √30 + 6
4 = √16 − 6 = √36
4=4 −6 ≠ 6

3. √3𝑥 + 1 − √𝑥 + 8 = 1
Solution:
√3𝑥 + 1 − √𝑥 + 8 = 1
√3𝑥 + 1 = 1 + √𝑥 + 8
2 2
(√3𝑥 + 1) = (1 + √𝑥 + 8)
3𝑥 + 1 = 1 + 2√𝑥 + 8 + 𝑥 + 8

275
3𝑥 + 1 = 𝑥 + 9 + 2√𝑥 + 8
3𝑥 + 1 − 𝑥 − 9 = 2√𝑥 + 8
2𝑥 − 8 = 2√𝑥 + 8
2𝑥 − 8 2√𝑥 + 8
=
2 2
𝑥 − 4 = √𝑥 + 8
2
(𝑥 − 4)2 = (√𝑥 + 8)
𝑥 2 − 8𝑥 + 16 = 𝑥 + 8
𝑥 2 − 8𝑥 − 𝑥 + 16 − 8 = 0
𝑥 2 − 9𝑥 + 8 = 0
(𝑥 − 8)(𝑥 − 1) = 0
𝑥 − 8 = 0 𝑜𝑟 𝑥 − 1 = 0
𝑥 = 8 𝑜𝑟 𝑥 = 1
Check:
√3(8) + 1 − √(8) + 8 = 1 √3(1) + 1 − √(1) + 8 = 1
√24 + 1 − √16 = 1 √3 + 1 − √9 = 1
√25 − √16 = 1 √4 − √9 = 1
5−4= 1 2−3 =1
1=1 −1 ≠1
Lesson Exercise:
1. √2𝑥 − 1 + 5 = 2
a. 𝑥 = 5
b. 𝑥 = −5
c. 𝑥 = 0
d. no solution
Solution:
√2𝑥 − 1 + 5 = 2
√2𝑥 − 1 = 2 − 5
√2𝑥 − 1 = −3
2
(√2𝑥 − 1) = (−3)2
2𝑥 − 1 = 9
2𝑥 = 9 + 1
2𝑥 = 10

276
2𝑥 10
=
2 2
𝑥=5
Check:
√2(5) − 1 + 5 = 2
√10 − 1 + 5 = 2
√9 + 5 = 2
3+5= 2
8≠2
∴ 𝑛𝑜 𝑠𝑜𝑙𝑢𝑡𝑖𝑜𝑛

2. 𝑥 − 1 = √5𝑥 − 9
a. 𝑥 = 4 𝑜𝑟 𝑥 = 1
b. 𝑥 = 5 𝑜𝑟 𝑥 = 2
c. 𝑥 = 3 𝑜𝑟 𝑥 = 0
d. no solution
Solution:
𝑥 − 1 = √5𝑥 − 9
2
(𝑥 − 1)2 = (√5𝑥 − 9)
𝑥 2 − 2𝑥 + 1 = 5𝑥 − 9
𝑥 2 − 2𝑥 − 5𝑥 + 1 + 9 = 0
𝑥 2 − 7𝑥 + 10 = 0
(𝑥 − 5)(𝑥 − 2) = 0
𝑥 − 5 = 0 𝑜𝑟 𝑥 − 2 = 0
𝑥 = 5 𝑜𝑟 𝑥 = 2
Check:
(5) − 1 = √5(5) − 9 (2) − 1 = √5(2) − 9

4 = √25 − 9 1 = √10 − 9
4 = √16 1 = √1
4=4 1=1

3. 1 = 𝑡 + √2𝑡 − 3
a. 𝑡 = 2 𝑜𝑟 𝑡 = −2

277
b. 𝑡 = 2
c. 𝑡 = −2
d. no solution
Solution:
1 = 𝑟 + √2𝑡 − 3
1 − 𝑡 = √2𝑡 − 3
2
(1 − 𝑡)2 = (√2𝑡 − 3)
1 − 2𝑡 + 𝑡 2 = 2𝑡 − 3
𝑡 2 − 4𝑡 + 4 = 0
(𝑡 − 2)(𝑡 − 2) = 0
𝑡−2=0
𝑡=2
Check:
1 = (2) + √2(2) − 3
1 = 2 + √4 − 3
1 = 2 + √1
1≠3

4. √5𝑥 + 3 = √3𝑥 + 7
a. 𝑥 = −4
b. 𝑥 = 4
c. 𝑥 = −2
d. 𝑥 = 2
Solution:
√5𝑥 + 3 = √3𝑥 + 7
2 2
(√5𝑥 + 3) = (√3𝑥 + 7)
5𝑥 + 3 = 3𝑥 + 7
5𝑥 − 3𝑥 = 7 − 3
2𝑥 = 4
2𝑥 4
=
2 2
𝑥=2
Check:

278
√5(2) + 3 = √3(2) + 7
√10 + 3 = √6 + 7
√13 = √13

5. 2√𝑥 + 8 = 3√𝑥 − 2
a. 𝑥 = 10
b. 𝑥 = −10
c. 𝑥 = 0
d. no solution.
2√𝑥 + 8 = 3√𝑥 − 2
2 2
(2√𝑥 + 8) = (3√𝑥 − 2)
22 (𝑥 + 8) = 32 (𝑥 − 2)
4(𝑥 + 8) = 9(𝑥 − 2)
4𝑥 + 32 = 9𝑥 − 18
32 + 18 = 9𝑥 − 4𝑥
50 = 5𝑥
50 50𝑥
=
5 5
𝑥 = 10
Check:
2√(10) + 8 = 3√(10) − 2
2√18 = 3√8

2√32 ∙ 2 = 32√22 ∙ 2
2(3)√2 = 3(2)√2
6√2 = 6√2

6. √𝑥 + 5 = √𝑥 2 − 15
a. 𝑥 = 5 𝑜𝑟 𝑥 = 4
b. 𝑥 = −5 𝑜𝑟 𝑥 = −4
c. 𝑥 = −5 𝑜𝑟 𝑥 = 4
d. 𝑥 = 5 𝑜𝑟 𝑥 = −4
Solution:

√𝑥 + 5 = √𝑥 2 − 15

279
2 2
(√𝑥 + 5) = (√𝑥 2 − 15)

𝑥 + 5 = 𝑥 2 − 15
0 = 𝑥 2 − 𝑥 − 15 − 5
0 = 𝑥 2 − 𝑥 − 20
(𝑥 − 5)(𝑥 + 4) = 0
𝑥 = 5 𝑜𝑟 𝑥 + 4 = 0
𝑥 = 5 𝑜𝑟 𝑥 = −4
Check:
√(5) + 5 = √(5)2 − 15 √(−4) + 5 = √(4)2 − 15

√10 = √25 − 15 √1 = √16 − 15


√10 = √10 √1 = √1

7. 𝑥 + 3 = √𝑥 + 5
a. 𝑥 = −1 𝑜𝑟 𝑥 = −4
b. 𝑥 = −1
c. 𝑥 = −4
d. no solution
Solution:
𝑥 + 3 = √𝑥 + 5
(𝑥 + 3)2 = (𝑥 + 5)2
𝑥 2 + 6𝑥 + 9 = 𝑥 + 5
𝑥 2 + 6𝑥 − 𝑥 + 9 − 5 = 0
𝑥 2 + 5𝑥 + 4 = 0
(𝑥 + 1)(𝑥 + 4) = 0
𝑥 + 1 = 0 𝑜𝑟 𝑥 + 4 = 0
𝑥 = −1 𝑜𝑟 𝑥 = −4
Check:
(−1) + 3 = √(−1) + 5 (−4) + 3 = √(−4) + 5
2 = √4 − 1 = √1
2=2 −1≠ 1

8. 𝑥 = √𝑥 + 6
a. 𝑥 = 3 𝑜𝑟 𝑥 = −2

280
b. 𝑥 = −2
c. 𝑥 = 3
d. no solution
Solution:
𝑥 = √𝑥 + 6
2
𝑥 2 = √𝑥 + 6
𝑥2 = 𝑥 + 6
𝑥2 − 𝑥 − 6 = 0
(𝑥 − 3)(𝑥 + 2) = 0
𝑥 − 3 = 0 𝑜𝑟 𝑥 + 2 = 0
𝑥 = 3 𝑜𝑟 𝑥 = −2
Check:
(3) = √(3) + 6 (−2) = √(−2) + 6
3 = √9 − 2 = √4
3=3 −2 ≠2

9. √𝑦 − 4 = 4 − 𝑦
a. 𝑦 = 4
b. 𝑦 = 5
c. 𝑦 = 5 𝑜𝑟 𝑦 = 4
d. no solution
Solution:
√𝑦 − 4 = 4 − 𝑦
2
(√𝑦 − 4) = (4 − 𝑦)2
𝑦 − 4 = 16 − 8𝑦 + 𝑦 2
0 = 𝑦 2 − 9𝑦 + 20
0 = (𝑦 − 5)(𝑦 − 4)
𝑦 − 5 = 0 𝑜𝑟 𝑦 − 4 = 0
𝑦 = 5 𝑜𝑟 𝑦 = 4
Check:
√(5) − 4 = 4 − (5) √(4) − 4 = 4 − (4)
√1 = −1 0=0
1 ≠ −1

281
10. √𝑥 + 7 = 1 + √𝑥 + 2
a. 𝑥 = 6
b. 𝑥 = −2
c. 𝑥 = 2
d. no solution
Solution:
√𝑥 + 7 = 1 + √𝑥 + 2
2 2
(√𝑥 + 7) = (1 + √𝑥 + 2)
𝑥 + 7 = (1 ∙ 1) + (1 ∙ √𝑥 + 2) + (√𝑥 + 2 ∙ 1) + (√𝑥 + 2) ∙ √𝑥 + 2
2
𝑥 + 7 = 1 + √𝑥 + 2 + √𝑥 + 2 + (√𝑥 + 2)
𝑥 + 7 = 1 + 2√𝑥 + 2 + (𝑥 − 2)
𝑥 + 7 = 𝑥 + 3 + 2√𝑥 + 2
𝑥 + 7 − (𝑥 + 3) = 𝑥 + 3 − (𝑥 + 3) + 2√𝑥 + 2
4 = 2√𝑥 + 2
2
42 = 2√𝑥 + 2
16 = 22 (𝑥 + 2)
16 = 4(𝑥 + 2)
16 = 4𝑥 + 8
16 − 8 = 4𝑥
4𝑥 = 8
4𝑥 8
=
4 4
𝑥=2
Check:
√(2) + 7 = 1 + √(2) + 2
√9 = 1 + √4
3= 1+2
3=3

11 Solving Quadratic Equations


11.1 by Square root Property
Solving Quadratic Equations by Square Root Property

282
There are several ways to find the roots of the quadratic equations. One of these ways
involves the taking the square root of both sides of the equation. Using this method is
very straightforward and can be calculated using fundamental math principles.
We all know that a quadratic equation always contains 2 nd degree variable. For us to solve for
its roots, we have different methods available to use. However, quadratic equation is classified
as complete and incomplete.
 Complete Quadratic Equation – has both 1st and 2nd degrees variables
 Incomplete Quadratic Equation – has only 2nd degree variable

For the case of the incomplete quadratic equation, square root property would be the most
appropriate method to apply.
Examples of Incomplete Quadratic Equation:

a) 9x2 = 225
b) 2x2 – 18 = 0

To solve the equations shown above, consider the following solutions:

a) 9x2 = 225
9𝑥 2 225
= Divide both sides by 9 since x2 must always have a numerical
9 9
coefficient of 1.
√𝑥 2 = ±√25 Extract square roots of both sides and prefix the ± sign to the right
number.
X= ±5

b) 2x2 – 18 = 0
2x2 = 18 Transpose -18 to the right side.
2𝑥 2 18
= Divide both sides by 2.
2 2

√𝑥 2 = ±√9 Extract square roots of both sides and put the ± sign before the right
number.
X= ±3
Take note:
If x2 = y2, then x =± 𝑦; y must always be any real number in which y ≥ 0.

Lesson Exercise:

1. 𝑐 2 - 169 = 0

283
√𝑐 2 = ± √169
c = ± 13

2. 11𝑏 2 - 891 = 0
11𝑏 2 = 891
√𝑏 2 = ± √81
b=±9

𝑥 4
3. =
16 𝑥
√𝑥 2 = ± √64
x= ±8

4. 7𝑑 2 = 343
√𝑑 2 = ± √49
d= ±7

1
5. 1 = 𝑥 2
9
1
𝑥2 = 1
9
√𝑥 2 = ± √9
x= ±3

36 𝑥
6. =
𝑥 4
√𝑥 2 = ± √144
x = ± 12

7. 𝑝 2 - 5 = 59
𝑝 2 = 59 + 5
√𝑝 2 = ± √64
p= ±8

8. 6𝑠2 - 150 = 0
6𝑠2 = 150
√𝑠 2 = ± √25
s= ±5

9. 𝑦 2 - 15 = 10
𝑦 2 = 15 + 10

284
√𝑦 2 = ± √25
y= ±5

10. 8𝑥 2 = 288
𝑥 2 = 36
√𝑥 2 = ± √36
x=±6

11.2 by Completing the Square


Since not all quadratic polynomials are factorable, so we have to use another method
that will enable us to find solutions to every quadratic equation that has real coefficient. This
method is by completing the square. Completing the square is referring to the method of
creating a perfect trinomial square, i.e. (x + 𝑎)2.
With completing the square, we must see to it that the left side of the equation is a perfect
square trinomial.

To execute this, consider the following steps:

1. Change the quadratic equation form ax2 + bx + c = 0 into the form ax2 + bx = c, where a
≠ 0.

2. Divide the coefficient of x by 2 and then square it. The result must be added to each side
of the equation.

Examples:

Quadratic Equation Number Added

𝑥 2 + 3x + 9⁄4 (3⁄2)2 = 9⁄4

𝑥 2 + 18x + 81 (18⁄2)2 = 81

3. Convert the left side into a square of a binomial.

4. Extract the square root of both sides and then put ± sign before the right side.

5. Solve the linear equation and check for the solutions with the original equation.

285
Application:

a. 𝑦 2 + 4y = 32
𝑦 2 + 4y + 4 = 32 + 4
√(𝑦 + 2)2 = √36
y+2=±6
y = -2 ± 6
y = -2 + 6 y = -2 - 6
=4 = -8

b. 5𝑥 2 + 3x - 2 = 0
5𝑥 2 3𝑥 2
+ - =0
5 5 5
2 3𝑥 2
𝑥 + =
5 5
3𝑥 9 2 9
𝑥2 + + = +
5 100 5 100
3 2 40+9
(x + ) =
10
100
3 2 49
√(𝑥 + ) = √
10 100
3 7
x+ =±
10 10
−3 7
x= ±
10 10
−3 7 −3 7
x= + x= -
10 10 10 10
4 2 −10
= or = or -1
10 5 10

Lesson Exercise:

1. 𝑚2 - 3m + ____ Solution: (−3⁄2)2 = 9⁄4


2. 𝑐 2 – 12c + ____ Solution: (−12⁄2)2 = 36
3. 𝑑 2 + 1⁄3 𝑑 + ____ Solution: (1⁄3)2 = 1⁄36
4. 𝑎2 – 7a + ____ Solution: (−7⁄2)2 = 49⁄4
5. 𝑠2 + 30𝑠 + ____ Solution: (30⁄2)2 = 900⁄4

6. 3𝑦 2 - 7y – 6 = 0
3𝑦 2 7𝑦 6
- - =0
3 3 3

286
7𝑦
𝑦2 + =2
3
2 7𝑦 49 49
𝑦 - + =2+
3 36 36
7 121
(y - )2 =
6 36
7 2 121
√(𝑦 − ) = √
6 36
7 11
y- =±
6 6
7 11
y= ±
6 6
7 11 7 11
x= + x= -
6 6 6 6
18 −5
= or 3 =
6 6

7. 𝑥 2 + 4x = 96
𝑥 2 + 4x + 4 = 96 + 4
√(𝑥 + 2)2 = √100
x + 2 = ± 10
x = -2 ± 10
x = -2 + 10 x = -2 - 10
=8 = -12

8. 𝑚2 + 8m = -7
𝑚2 + 8m + 16 = -7 + 16
√(𝑚 + 4)2 = √9
m+4=±3
m = -4 ± 3
m = -4 + 3 m = -4 - 3
= -1 = -7

9. 𝑑 2 - 4d – 45 = 0
𝑑 2 - 4d = 45
𝑑 2 - 4d + 4 = 45 + 4
√(𝑑 − 2)2 = √49
d-2=±7
d=2±7
d=2+7 x=2-7
=9 = -5

10. 4𝑑 2 – 21d + 5 = 0
4𝑑 2 21𝑑 5
- + =0
4 4 4
21𝑑 5
𝑑2 - =−
4 4
21𝑑 441 5 441
𝑑2 - + =− +
4 64 4 64
21 2 −80+441
(d - ) =
8 64

287
21 2 361
√(𝑑 − ) =√
8 64
21 19
d- ± =
8 8
21 19
d= ±
8 8
21 19 21 19
x= + x= -
8 8 8 8
40 2 1
= or 5 = or
8 8 4

11.3 by Quadratic Formula


We can derive the quadratic formula from the general quadratic equation ax2 + bx + c = 0
where a, b, and c are real numbers, and a ≠ 0. This is solved using the completing the square.

Learning Objectives:

1. At the end of this lesson, students must be able to understand the process how
the quadratic formula is derived.
2. The student should be able to solve quadratic equations using the quadratic
formula.

Using the completing the square method, the quadratic formula is derived as executed below.

𝑎𝑥 2 + bx + c = 0
𝑎𝑥 2 + bx = -c
𝑏𝑥 −𝑐
𝑥2+ =
𝑎 𝑎
𝑏𝑥 𝑏 −𝑐 𝑏
𝑥2 + + ( )2 = + ( )2 Complete the square.
𝑎 2𝑎 𝑎 2𝑎
𝑏 2 −𝑐 𝑏2
(x + ) = +
2𝑎 𝑎 4𝑎2
𝑏 2 𝑏2 − 4𝑎𝑐
(x + ) =
2𝑎 4𝑎2
𝑏 2 𝑏2 − 4𝑎𝑐
√(𝑥 + ) =±√ If 𝑏 2 − 4𝑎𝑐 ≥ 0.
2𝑎 4𝑎2
𝑏 𝑏2 − 4𝑎𝑐
x+ =±√
2𝑎 4𝑎2

𝑏 𝑏2 − 4𝑎𝑐
𝑥=- ±√
2𝑎 4𝑎2

𝑏 √𝑏2 − 4𝑎𝑐
𝑥=- ±
2𝑎 2𝑎

−𝑏±√𝑏2 −4𝑎𝑐
𝑥=
2𝑎

288
The last equation above is the derived quadratic formula. This is used to find for the roots of the
quadratic equation, ax2 + bx + c = 0 in which the numerical coefficients of a, b, and c are
considered.

For the application of the quadratic formula, consider the following equations below:
Example 1:
3x2 + 5x - 2 = 0 where a = 3, b = 5, and c = - 2

Solution:
−5 ± √52 −4(3)(−2)
𝑥=
2(3)

−5±√25+ 24
𝑥=
6

−5 ±√49
𝑥=
6

−5 ± 7
𝑥=
6

−5 + 7 −5 − 7
𝑥= 𝑥=
6 6
2 1 −12
= or = or - 2
6 3 6

It is shown that the solution set is (1⁄3 , −2).


Example 2:
5x2 + 9x = 2
Convert the equation first into its standard form, which is 5x2 + 9x - 2 = 0, where a = 5, b = 9,
and c = - 2.
Solution:
−9 ± √92 −4(5)(−2)
𝑥=
2(5)

−9±√81+ 40
𝑥=
10

−9 ±√121
𝑥=
10

289
−9 ± 11
𝑥=
10

−9 + 11 −9 − 11
𝑥= 𝑥=
10 10
2 1 −20
= or = or - 2
10 5 10

It is shown that the solution set is (1⁄5 , −2).

Lesson Exercise:

1. 6x + 1 = 2x2
Convert the equation first into its standard form, which is -2x2 + 6x + 1 = 0, where a = - 2,
b = 6, and c = 1.

Solution:
−6 ± √(6)2 −4(1)(−2)
𝑥=
2(−2)

−6±√36+8
𝑥=
−4

−6 ±√44
𝑥=
−4

−6 ±√4∗11
𝑥=
−4

−6 ± 2√11
𝑥=
−4

−6 + 2√11 −6 − 2√11
𝑥= 𝑥=
−4 −4
3 √11 3 √11
= − = +
2 2 2 2
3 √11 3 √11
It is shown that the solution set is ( − , + ).
2 2 2 2

2. 3x2 = 4x
Convert the equation first into its standard form, which is 3x2 - 4x = 0, where a = 3, b = -
4, and c = 0.

Solution:
−(−4) ± √(−4)2 −4(3)(0)
𝑥=
2(3)

4 ±√16
𝑥=
6

290
4 ±4
𝑥=
6

4+4 4−4
𝑥= 𝑥=
6 6
8 4
= or =0
6 3
4
It is shown that the solution set is ( , 0).
3

3. 2r = 5 – 4r2
Convert the equation first into its standard form, which is 4r 2 + 2r – 5 = 0, where a = 4, b =
2, and c = - 5.

Solution:
−2 ± √22 −4(4)(−5)
𝑟=
2(4)

−2±√4+80
𝑟=
8

−2 ±√84
𝑟=
8

−2 ±√4∗21
𝑟=
8

−2 ± 2√21
𝑟=
8

−2 + 2√21 −2 − 2√21
𝑟= 𝑟=
8 8
−1 √21 −1 √21
= + = −
4 4 4 4
−1 √21 −1 √21
It is shown that the solution set is ( + , − ).
4 4 4 4

4. 2x2 + 4x - 8 = 0
a = 2, b = 4, and c = - 8

Solution:
−4 ± √42 −4(−8)(2)
𝑥=
2(2)

−4±√16+ 64
𝑥=
4

−4 ±√80
𝑥=
4

−4 ± √16∗5
𝑥=
4

−4 ± 4√5
𝑥=
4

291
−4 + 4√5 −4 −4√5
𝑥= 𝑥=
4 4
= −1 + √5 = −1 − √5

It is shown that the solution set is (−1 + √5, −1 − √5).

5. 3x = -3x2 + 2
Convert the equation first into its standard form, which is 3x 2 + 3x – 2 = 0, where a = 3, b
= 3, and c = - 2.

Solution:
−3 ± √32 −4(3)(−2)
𝑥=
2(3)

−3±√9+24
𝑥=
6

−3 ±√33
𝑥=
6

−3 ±√33
𝑥=
6

−3 ± √33
𝑥=
6

−3 + √33 −3 − √33
𝑥= 𝑥=
6 6
−1 √33 −1 √33
= + = −
2 6 2 6
−1 √33 −1 √33
It is shown that the solution set is ( + , − ).
2 6 2 6

6. -2 = 2x2 - 6x
Convert the equation first into its standard form, which is -2x2 + 6x – 2 = 0, where a = -2,
b = 6, and c = - 2.
Solution:
−6 ± √62 −4(−2)(−2)
𝑥=
2(−2)

−6±√36−16
𝑥=
−4

−6 ±√20
𝑥=
−4

−6 ±√4∗5
𝑥=
−4

−6 ± 2√5
𝑥=
−4

292
−6 + 2√5 −6 − 2√5
𝑥= 𝑥=
−4 −4
3 √5 3 √5
= − = +
2 2 2 2
3 √5 3 √5
It is shown that the solution set is ( − , + ).
2 2 2 2

7. x2 + 3x - 10 = 0
a = 1, b = 3, and c = - 10.
Solution:
−3 ± √32 −4(1)(−10)
𝑥=
2(1)

−3±√9+40
𝑥=
2

−3 ±√49
𝑥=
2

−3 ±7
𝑥=
2

−3 + 7 −3 − 7
𝑥= 𝑥=
2 2
4 −10
= 𝑜𝑟 2 = −5 𝑜𝑟
2 2
It is shown that the solution set is (2, −5 ).

8. 2y2 - 6y - 8 = 0
a = 2, b = - 6, and c = - 8.
Solution:
−(−6) ± √(−6)2 −4(2)(−8)
𝑦=
2(2)

6±√36+64
𝑦=
4

6 ±√100
𝑦=
4

6 ±10
𝑦=
4

6 +10 6 − 10
𝑦= 𝑦=
4 4
16 −4
= 4𝑜𝑟 −1
= 𝑜𝑟
4 4
It is shown that the solution set is (4, −1).

9. x2 - 6x - 11 = 0
a = 1, b = - 6, and c = - 11.
Solution:
−(−6) ± √(−6)2 −4(1)(−11)
𝑥=
2(1)

293
6±√36+44
𝑥=
2

6 ±√80
𝑥=
2

6 ±√16∗5
𝑥=
2

6 ±4√5
𝑥=
2

6+4√5 6−4√5
𝑥= 𝑥=
2 2

= 3 + 2√5 = 3 − 2√5
It is shown that the solution set is (3 + 2√5, 3 − 2√5).

10. 2s2 – 3s - 2 = 0
a = 2, b = - 3, and c = - 2.
Solution:
−(−3) ± √(−3)2 −4(2)(−2)
𝑠=
2(2)

3±√9+16
𝑠=
4

3 ±√25
𝑠=
4

3 ±5
𝑠=
4

3+5 3−5
𝑠= 𝑠=
4 4

−1
=2 =
2
It is shown that the solution set is (2, −1⁄2).

11. z2 – 5z - 6 = 0
a = 1, b = - 5, and c = - 6.
Solution:
−(−5) ± √(−5)2 −4(1)(−6)
𝑧=
2(1)

5±√25+24
𝑧=
2

5 ±√49
𝑧=
2

5 ±7
𝑧=
2

294
5+7 5−7
𝑧= 𝑧=
2 2

=6 = -1
It is shown that the solution set is (6, −1).

12. k2 – 3k - 1 = 0
a = 1, b = - 3, and c = - 1.
Solution:
−(−3) ± √(−3)2 −4(1)(−1)
𝑘=
2(1)

3±√9+4
𝑘=
2

3 ±√13
𝑘=
2

3+√13 3−√13
𝑘= 𝑘=
2 2

3+√13 3−√13
It is shown that the solution set is ( , ).
2 2

13. 5z2 – 11z + 2 = 0


a = 5, b = - 11, and c = 2
Solution:
−(−11) ± √(−11)2 −4(5)(2)
𝑧=
2(5)

11±√121−40
𝑧=
10

11 ±√81
𝑧=
10

11 ±9
𝑧=
10

11+9 11−9
𝑧= 𝑧=
10 10
1
=2 =
5
It is shown that the solution set is (2, 1⁄5).

14. m2 + 8m + 7 = 0
a = 1, b = 8, and c = 7
Solution:
−8 ± √82 −4(1)(7)
𝑚=
2(1)

295
−8±√64−28
𝑚=
2

−8 ±√36
𝑚=
2

−8 ±6
𝑚=
2

−8+6 −8−6
𝑚= 𝑚=
2 2
= -1 = -7
It is shown that the solution set is (−1, −7).

15. r2 + 8r + 5 = 0
a = 1, b = 8, and c = 5
Solution:
−8 ± √82 −4(1)(5)
𝑟=
2(1)

−8±√64−20
𝑟=
2

−8 ±√44
𝑟=
2

−8 ±√4∗11
𝑟=
2

−8 ±2√11
𝑟=
2

−8+2√11 −8−2√11
𝑟= 𝑟=
2 2
= -4 + √11 = -4 - √11
It is shown that the solution set is (−4 + √11, −4 − √11).

16. n2 – 6n - 1 = 0
a = 1, b = -6, and c = -1
Solution:
−(−6) ± √(−6)2 −4(1)(−1)
𝑛=
2(1)

6±√36+4
𝑛=
2

6 ±√40
𝑛=
2

6 ±√4∗10
𝑛=
2

6 ±2√10
𝑛=
2

296
6+2√10 6−2√10
𝑛= 𝑛=
2 2
= 3 + √10 = 3 - √10
It is shown that the solution set is (3 + √10, 3 − √10).

17. 7x2 + 2x - 2 = 0
a = 7, b = 2, and c = -2
Solution:
−2 ± √22 −4(7)(−2)
𝑥=
2(7)

−2±√4+56
𝑥=
14

−2 ±√60
𝑥=
14

−2 ±√4∗15
𝑥=
14

−2 ±2√15
𝑥=
14

−2+2√15 −2−2√15
𝑥= 𝑥=
14 14
−1 √15 −1 √15
= + = -
7 7 7 7
−1 √15 −1 √15
It is shown that the solution set is ( + , - ).
7 7 7 7

18. -2z2 + 8z + 5 = 0
a = -2, b = 8, and c = 5
Solution:
−8 ± √82 −4(−2)(5)
𝑧=
2(−2)

−8±√64−40
𝑧=
−4

−8 ±√24
𝑧=
−4

−8 ±√4∗6
𝑧=
−4

−8 ± 2√6
𝑧=
−4

−8 + 2√6 −8 − 2√6
𝑧= 𝑧=
−4 −4
√6 √6
=2- =2+
2 2
√6 √6
It is shown that the solution set is (2 − ,2 + ).
2 2

19. 2s2 – 3s - 2 = 0

297
a = 2, b = -3, and c = -2
Solution:
−(−3) ± √(−3)2 −4(2)(−2)
𝑠=
2(2)

3±√9+16
𝑠=
4

3 ±√25
𝑠=
4

3±5
𝑠=
4

3+5 3−5
𝑠= 𝑠=
4 4
−1
=2 =
2
−1
It is shown that the solution set is (2, ).
2

20. j2 – 6j = 13
Convert the equation first into its standard form, which is j2 - 6j – 13 = 0, where a = 1, b =
-6, and c = - 13.
Solution:
−(−6) ± √(−6)2 −4(1)(−13)
𝑥=
2(1)

6±√36+52
𝑥=
2

6 ±√88
𝑥=
2

6 ±√4∗22
𝑥=
2

6 ± 2√22
𝑥=
2

6 + 2√22 6 − 2√22
𝑥= 𝑥=
2 2
= 3 + √22 = 3 − √22
It is shown that the solution set is (3 + √22, 3 − √22).

11.4 Graphing Quadratic Equations


The general technique for graphing quadratics is the same as for graphing linear equations.
However, since quadratics graph as curvy lines (called "parabolas"), rather than the straight
lines generated by linear equations, there are some additional considerations.
The most basic quadratic is y = x2. When you graphed straight lines, you only needed two points
to graph your line, though you generally plotted three or more points just to be on the safe side.
However, three points will almost certainly not be enough points for graphing a quadratic, at

298
least not until you are very experienced. For example, suppose a student computes these three
points:

incorrect graph
Then, based only on his
experience with linear graphs,
he tries to put a straight line
through the points.

He got the graph wrong. You, on the other hand, are more careful.

You find many points:

That last point has a rather large y-value, so you decide that you won't bother drawing your
graph large enough to plot it.

299
But you plot all the other points:

Even if you'd forgotten that quadratics graph as curvy parabolas, these points will remind you of
this fact.
correct graph of y = x2

You draw a nicely smooth


curving line passing neatly
through the plotted points:

Copyright © Elizabeth
Stapel 2002-2011 All Rights
Reserved

Unlike the careless student, you just got the graph right.
incorrect "segment" graph

Some students will plot the points


correctly, but will then connect the
points with straight line segments, like
this:

300
This is not correct. You do still need a ruler for doing your graphing, but only for drawing the
axes, not for drawing the parabolas. Parabolas graph as smoothly curved lines, not as jointed
segments.
The general form of a quadratic is "y = ax2 + bx+ c". For graphing, the leading coefficient "a"
indicates how "fat" or how "skinny" the parabola will be.
For | a | > 1 (such as a = 3 or a = –4), the parabola will be "skinny", because it grows more
quickly (three times as fast or four times as fast, respectively, in the case of our sample values
of a).
For | a | < 1 (such as a = 1/3 or a = –1/4 ), the parabola will be "fat", because it grows more slowly
(one-third as fast or one-fourth as fast, respectively, in the examples). Also, if a is negative, then
the parabola is upside-down.
As you can see, as the leading coefficient goes from very negative to slightly negative to zero
(not really a quadratic) to slightly positive to very positive, the parabola goes from skinny upside-
down to fat upside-down to a straight line (called a "degenerate" parabola) to a fat right-side-up
to a skinny right-side-up. Copyright © Elizabeth Stapel 2002-2011 All Rights Reserved
There is a simple, if slightly "dumb", way to remember the difference between right-side-up
parabolas and upside-down parabolas:
positive quadratic y = x2 negative quadratic y = –x2

301
This can be useful information: If, for instance, you have an equation where a is negative, but
you're somehow coming up with plot points that make it look like the quadratic is right-side-up,
then you will know that you need to go back and check your work, because something is wrong.

Parabolas always have a lowest point (or a highest point, if the parabola is upside-down). This
point, where the parabola changes direction, is called the "vertex".
If the quadratic is written in the form y = a(x –h)2 + k, then the vertex is the point (h, k). This
makes sense, if you think about it. The squared part is always positive (for a right-side-up
parabola), unless it's zero. So you'll always have that fixed value k, and then you'll always be
adding something to it to make y bigger, unless of course the squared part is zero. So the
smallest y can possibly be is y = k, and this smallest value will happen when the squared
part, x – h, equals zero. And the squared part is zero when x – h = 0, or when x= h. The same
reasoning works, with k being the largest value and the squared part always subtracting from it,
for upside-down parabolas.
(Note: The "a" in the vertex form "y = a(x – h)2 + k" of the quadratic is the same as the "a" in the
common form of the quadratic equation, "y = ax2 + bx + c".)

Since the vertex is a useful point, and since you can "read off" the coordinates for the vertex
from the vertex form of the quadratic, you can see where the vertex form of the quadratic can be
helpful, especially if the vertex isn't one of your T-chart values. However, quadratics are not
usually written in vertex form. You can complete the square to convert ax2 + bx + c to vertex
form, but, for finding the vertex, it's simpler to just use a formula. (The vertex formula is derived
from the completing-the-square process, just as is the Quadratic Formula. In each case,
memorization is probably simpler than completing the square.)
For a given quadratic y = ax2 + bx + c, the vertex (h, k) is found by computing h = –b/2a, and then
evaluating y at h to find k. If you've already learned the Quadratic Formula, you may find it easy
to memorize the formula for k, since it is related to both the formula for h and the discriminant in
the Quadratic Formula: k = (4ac – b2) / 4a.
 Find the vertex of y = 3x2 + x – 2 and graph the parabola.
To find the vertex, I look at the coefficients a, b, and c. The formula for the vertex gives
me:
h = –b/2a = –(1)/2(3) = –1/6
Then I can find k by evaluating y at h = –1/6:
k = 3( –1/6 )2 + ( –1/6 ) – 2
= 3/36 – 1/6 – 2

302
= 1/12 – 2/12 – 24
/12
–25
= /12
So now I know that the vertex is at ( –1/6 , –25/12 ). Using the formula was helpful,
because this point is not one that I was likely to get on my T-chart.

I need additional points for my


graph:

Now I can do my graph,


and I will label the vertex:

When you write down the vertex in your homework, write down the exact coordinates: "( –1/6 , –
25
/12)". But for graphing purposes, the decimal approximation of "(–0.2, –2.1)" may be more
helpful, since it's easier to locate on the axes.
The only other consideration regarding the vertex is the "axis of symmetry". If you look at a
parabola, you'll notice that you could draw a vertical line right up through the middle which
would split the parabola into two mirrored halves. This vertical line, right through the vertex, is
called the axis of symmetry. If you're asked for the axis, write down the line "x = h", where h is
just the x-coordinate of the vertex. So in the example above, then the axis would be the vertical
line x = h = –1/6.
Helpful note: If your quadratic's x-intercepts happen to be nice neat numbers (so they're
relatively easy to work with), a shortcut for finding the axis of symmetry is to note that this
vertical line is always exactly between the two x-intercepts. So you can just average the two
intercepts to get the location of the axis of symmetry and the x-coordinate of the vertex.
However, if you have messy x-intercepts (as in the example above) or if the quadratic doesn't

303
actually cross the x-axis (as you'll see on the next page), then you'll need to use the formula to
find the vertex.
 Find the vertex and intercepts of y = 3x2 + x – 2 and graph; remember to label the
vertex and the axis of symmetry.
This is the same quadratic as in the last example. I already found the vertex when I
worked the problem above. This time, I also need to find the intercepts before I do my
graph. To find the y-intercept, I set xequal to zero and solve:
y = 3(0)2 + (0) – 2
= 0 + 0 – 2 = –2
Then the y-intercept is the point (0, –2). To find the x-intercept, I set y equal to zero, and
solve:
0 = 3x2 + x – 2
0 = (3x – 2)(x + 1)
3x – 2 = 0 or x + 1 = 0
x = 2/3 or x = – 1
Then the x-intercepts are at the points (–1, 0) and ( 2/3, 0).
The axis of symmetry is halfway between
the two x-intercepts at (–1, 0) and at ( 2/3 , 0);
using this, I can confirm the answer from the
previous page:
(–1 + 2/3) / 2 = (–1/3) / 2 = –1/6
The complete answer is a listing of the vertex, the
axis of symmetry, and all three intercepts, along
with a nice neat graph:

The vertex is at ( –1/6 , –25/12 ), the axis of


symmetry is the line x = –1/6 , and the
intercepts are at (0, –2), (–1, 0), and ( 2/3, 0).

 Find the intercepts, the axis of symmetry, and vertex of y = x2 – x – 12.


To find the y-intercept, I set x equal to 0 and solve:
y = (0)2 – (0) – 12 = 0 – 0 – 12 = –12
To find the x-intercept, I set y equal to 0 and solve:
0 = x2 – x – 12
0 = (x – 4)(x + 3)
x = 4 or x = –3

304
To find the vertex, I look at the coefficients: a = 1 and b = –1. Plugging into the formula, I
get:
h = –(–1)/2(1) = 1/2 = 0.5
To find k, I plug h = 1/2 in for x inside y = x2 – x – 12, and simplify:
k = (1/2)2 – (1/2) – 12 = 1/4 – 1/2 – 12 = –12.25
Once I have the vertex, it's easy to write down the axis of symmetry: x = 0.5. Now I'll find
some additional plot points, to fill in the graph:

For my own convenience, I picked x-values that were centered around the x-coordinate
of the vertex. Now I can plot the parabola: Copyright © Elizabeth Stapel 2002-2011 All
Rights Reserved

305
The vertex is at the point (0.5, –12.25),
the axis of symmetry is the line x = 0.5,
and the intercepts are at the points (0, –12), (–3, 0), and (4, 0).
 Find the x-intercepts and vertex of y = –x2 – 4x + 2.
Since it is so simple to find the y-intercept (and it will probably be a point in my T-chart
anyway), they are only asking for the x-intercepts this time. To find thex-intercept, I
set y equal 0 and solve:
0 = –x2 – 4x + 2 ADVERTISEMENT
x + 4x – 2 = 0
2

For graphing purposes, the intercepts


are at about (–4.4, 0) and (0.4, 0). (When I write down the answer, I will of course use
the "exact" form, with the square roots; my calculator's decimal approximations are just
for helping me graph.)

To find the vertex, I look at the coefficients: a = –1 and b = –4. Then:


h = –(–4)/2(–1) = –2
To find k, I plug h = –2 in for x in y = –x2 – 4x + 2, and simplify:
k = –(–2)2 – 4(–2) + 2 = –4 + 8 + 2 = 10 – 4 = 6
Now I'll find some additional plot points, to help me fill in my graph:

306
Note that I picked x-values that were centered around the x-coordinate of the vertex.
Now I'll plot the parabola:

The vertex is at (–2, 6), and the intercepts are at the following points:

(0, 2), , and


 Find the x-intercepts and vertex of y = –x2 + 2x – 4.
To find the vertex, I look at the coefficients: a = –1 and b = 2. Then:
h = –(2)/2(–1) = 1
To find k, I'll plug h in for x and simplify:
k = –(1)2 + 2(1) – 4 = –1 + 2 – 4 = 2 – 5 = –3
The vertex is below the x-axis, and, since this is a negative quadratic, I know that the
parabola is going to be upside-down. So can my line possibly cross the x-axis? Can
there possibly be any x-intercepts? Of course not! So I expect to get "no (real) solution"

307
when I try to find the x-intercepts, but I need to show my work anyway. To find the x-
intercept, I set y equal 0 and solve:
0 = –x2 + 2x – 4
x2 – 2x + 4 = 0

As soon as I get a negative inside the square root, I know that I can't get a graphable
solution. So, as expected, there are no x-intercepts. Now I'll find some additional plot
points, to fill in my graph:

Note that I picked x-values that were centered around the x-coordinate of the vertex.
Now I'll plot the parabola: Copyright © Elizabeth Stapel 2002-2011 All Rights Reserved

308
The vertex is at (1, –3), and the only intercept is at (0, –4).
This last exercise illustrates one way you can cut down a bit on your work. If you solve for the
vertex first, then you can easily tell if you need to continue on and look for the x-intercepts, or if
you can go straight on to plotting some points and drawing the graph. If the vertex is below
the x-axis (that is, if the y-value is negative) and the quadratic is negative (so the parabola
opens downward), then there will be no x-intercepts. Similarly, if the vertex is above the x-axis
(that is, if the y-value is positive) and the quadratic is positive (so the parabola opens upward),
then there will be no x-intercepts.
In most of the graphs that I did (though not the first one), it just so happened that the points on
the T-chart were symmetric about the vertex; that is, that the points "matched" on either side of
the vertex. While a parabola is always symmetric about the vertical line through the vertex (the
parabola's "axis"), the T-chart points might not be symmetric. In particular, the T-chart points will
not "match" if the x-coordinate of the vertex is something other than a whole number or a half-
number (such as "3.5"). Warning: Don't expect the plot-points always to "match up" on either
side of the vertex; in particular, don't do half the points on your T-chart and then "fill in" the rest
of your T-chart by assuming a symmetry that might not exist.
Other tips for graphing: If the parabola is going to be "skinny", then expect that you will get
some very large values in your T-chart. You will either end up with a really tall graph or else a
rather short T-chart. If the parabola is going to be "fat", then expect that you will probably have
to plot points with fractions as coordinates. In either case, when you go to connect the dots to
draw the parabola, you might find it helpful to turn the paper sideways and first draw the really
curvy part through the vertex, making sure that it looks nice and round. Then turn the paper
back right-side-up and draw the "sides" of the parabola.
Warning: Draw your graphs big enough to be clearly seen by your instructor. If you're fitting
more than two or maybe three graphs on one side of a standard sheet of paper, then you're
drawing your graphs way too small.

309
Source: http://www.purplemath.com/modules/grphquad4.htm

12 Variations
12.1 Direct Variation
The statement " y varies directly as x ," means that when x increases, y increases by the same
factor. In other words, y and x always have the same ratio:

=k

where k is the constant of variation.


We can also express the relationship between x and y as:

y = kx

where k is the constant of variation.


Since k is constant (the same for every point), we can find k when given any point by dividing
the y-coordinate by the x-coordinate. For example, if y varies directly as x, and y = 6 when x= 2 ,
the constant of variation is k = = 3 . Thus, the equation describing this direct variation is y =
3x .
Example 1: If y varies directly as x , and x = 12 when y = 9 , what is the equation that describes
this direct variation?

k= =
y= x

Example 2: If y varies directly as x , and the constant of variation is k = , what is y when x =


9?

y= x= (9) = 15
As previously stated, k is constant for every point; i.e., the ratio between the y -coordinate of a
point and the x -coordinate of a point is constant. Thus, given any two
points (x 1, y 1) and (x 2, y 2) that satisfy the equation, = k and =k.
Consequently, = for any two points that satisfy the equation.
Example 3: If y varies directly as x , and y = 15 when x = 10 , then what is ywhen x = 6 ?

=
=
6( ) = y
y=9

310
Graphing Direct Variation
An equation of the form y = kx can be thought of as an equation of the
form y = mx + bwhere m = k andb = 0 . Thus, a direct variation equation is an equation in slope-
intercept form which passes through (0, 0) and has a slope equal to the constant of variation.
Therefore, to graph a direct variation equation, start at (0, 0) and then proceed as you would in
graphing a slope. Or, if you know one point, draw a straight line between (0, 0) and that point,
and extend the line on both sides.

Example 4: y varies directly as x . If the constant of variation is , graph the line which
represents the variation, and write an equation that describes the variation.

y= x
To calculate the constant of variation, given a graph of direct variation, simply calculate the
slope.
Problem : If y varies directly as x , and y = 3 when x = 15 , write an equation describing the
variation.
Problem : If r varies directly as q , and r = 10 when q = 6 , write an equation describing the
variation.
Problem : If y varies directly as x , and the constant of variation is 3 , what is ywhen x = 7 ?

Problem : If y varies directly as x , and the constant of variation is , what isx when y = 5 ?
Problem : If y varies directly as x , and y = 6 when x = 8 , what is y when x = 14?
Problem : If y varies directly as x , and y = 5 when x = 4 , what is x when y = 15?

311
Problem : y varies directly as x , and the constant of variation is . Graph the line which
represents the variation, and write an equation that describes the variation.
Problem : y varies directly as x , and y = 3 when x = 2 . Graph the line which represents the
variation, and write an equation that describes the variation.

12.2 Direct Square Variation


A quantity a varies as the square of a quantity b, if, when b changes, achanges by the
square of that ratio. Thus, if b changes by a factor of 4, then a will change by a factor of 42 =
16. If b changes to one third of its value, then a will change to one ninth of its value.

Problem 1. a varies as the square of b. When b = 7, a = 4. What is the value of a when b =


35?
In going from 7 to 35, b has changed by a factor of 5.a therefore will change by a
factor of 52 = 25. a = 25· 4 = 100.
Problem 10. a varies as the square of b. When b = 20, a = 32. What is the value
of a when b = 15?
9
× 32 = 18
16

Theorem. If a varies directly as b, then a2 will vary as b2.


This is easily proved if we write the ratios in fractional form.
a varies directly as b means:

Therefore, on squaring both sides:

This implies

This means that a2 varies as b2; which is what we wanted to prove.


Problem 2. The area A of a circle varies directly as the area of the circumscribed
square. That is, as the area of the square changes, the

312
area of the circle changes proportionally.
a) Show that this implies that the area A of the circle varies as the
a) square of the radius r.
The side of the circumscribed square is equal to the diameter D of the circle.
Therefore the area of the circumscribed square is equal to D2. Hence the area A of the
circle varies as D2.
But D varies directly as r -- D = 2r -- and therefore, according to
the theorem, D2 varies as r2. Therefore, since A varies as D2, and D2 varies as r2,
then transitively, A varies asr2. The area of the circle varies as the square of the radius.

b) If the radius of a circle changes from 6 cm to 12 cm, how will the area change?
In going from 6 cm to 12 cm, the radius has doubled, that is, it has changed by a factor of 2.
The area therefore will change by a factor of 22 = 4. It will be four times larger.
c.) What is the constant of proportionality that relates the area A to r2?
π. A = πr2.
Example 4. The surface area of a sphere.

The surface area of a sphere is proportional to the surface area of the circumscribed
cube.
Now, each face of the cube is a square whose side is equal to the diameter D of the
sphere. And a cube has 6 faces. Therefore, the surface area of the cube is equal to 6D2.
In other words, the surface area A of a sphere is proportional to the square of its
diameter.
Do you know what the constant of proportionality is?
π. A = πD2
Problem 3. Show that the surface area of a sphere varies as the square of its radius. Write
the equation that relates the surface area A to the radius r.
Since A = πD2, and D = 2r, then A = π(2r)2 = 4πr2.

313
12.3 Inverse Variation
The statement " y varies inversely as x means that when x increases, ydecreases by the same
factor. In other words, the expression xy is constant:
xy = k

where k is the constant of variation.


We can also express the relationship between x and y as:

y=

where k is the constant of variation.


Since k is constant, we can find k given any point by multiplying the x-coordinate by the y-
coordinate. For example, if yvaries inversely asx , and x = 5 wheny = 2 , then the constant of
variation is k = xy= 5(2) = 10 . Thus, the equation describing this inverse variation is xy =
10 or y = .

Example 1: If y varies inversely as x , and y = 6 when x = , write an equation describing this


inverse variation.

k= (6) = 8
xy = 8 or y =

Example 2: If y varies inversely as x , and the constant of variation is k = , what is y when x =


10 ?

xy =
10y =
y= × = × =
k is constant. Thus, given any two points ( x 1, y 1 ) and ( x 2, y 2 ) which satisfy the inverse
variation, x 1 y 1 = k and x 2 y 2 = k . Consequently, x 1 y 1 =x 2 y 2 for any two points that satisfy
the inverse variation.
Example 3: If y varies inversely as x , and y = 10 when x = 6 , then what is ywhen x = 15 ?

x1y1=x2y2
6(10) = 15y
60 = 15y
y=4
Thus, when x = 6 , y = 4 .

Graphing Inverse Variation

314
Unlike the graph of direct variation, the graph of inverse variation is not linear. Rather, it is a
hyperbola:

xy = 1
Note that the lines never cross the axes -- they get closer and closer to x = 0and y = 0 ,
but x and y never equal zero.
To graph an inverse variation, make a data table and plot points. Then connect the points with a
smooth (not straight) curve. There should be two curves -- one in the first quadrant (where
both x and y are positive) and one in the third quadrant (where both x and y are negative). The
result should be qualitatively similar to the graph of xy = 1 above.
To calculate the constant of variation from a graph of inverse variation, simply pick a point and
multiply its two coordinates.
Problem : If y varies inversely as x , and y = 6 when x = 5, write an equation describing the
variation.
Problem : If y varies inversely as x , and the constant of variation is 75, what isy when x =
12.5 ?

Problem : If h varies inversely as k , and the constant of variation is , what is k when h = 6 ?


Problem : If y varies inversely as x , and y = 7 when x = 6 , what is y when x = 3 ? What
is x when y = 3 ?
Problem : If y varies inversely as x , and y = 6 when x = 3a , what is y when x =a ?

315
Problem : If y varies inversely as x , and y = 4b when x = 6a , what is y in terms
of a and b when x = 8 ?

12.4 Joint Variation


Definition of Joint Variation
Joint variation is the same as direct variation with two or more quantities.
That is:
Joint variation is a variation where a quantity varies directly as the product of two or more other
quantities.

Let’s first understand direct variation.


Direct variation occurs when two quantities change in the same manner.

That is:
Increase in one quantity causes an increase in the other quantity.
Decrease in one quantity causes a decrease in the other quantity.
For example:

The cost of a pencil and the number of pencils you buy.


Buy more pay more….Buy less pay less.

Direct variation between variables x and y can be expressed as:


y = kx, where ‘k’ is the constant of variation and k ≠ 0.

y = kxz represents joint variation. Here, y varies jointly as x and z.

More Examples on Joint Variation


 y = 7xz, here y varies jointly as x and z.
 y = 7x2z3, here y varies jointly as x2 and z3.

 Area of a triangle = is an example of joint variation. Here the constant is ½. Area of a


triangle varies jointly with base ‘b’ and height ‘h’.
 Area of a rectangle = l × w represents joint variation. Here the constant is 1. Area of a rectangle
varies jointly with length ‘l’ and width ‘w’.
Solved Example on Joint Variation
Assume a varies jointly with b and c. If b = 2 and c = 3, find the value of a. Given that a =
12 when b = 1 and c = 6.

316
Solution:
Step 1: First set up the equation. a varies jointly with b and c
a = kbc
Step 2: Find the value of the constant, k. sdw
Given that a = 12 when b = 1 and c = 6
a = kbc
12 = k × 1 × 6
k=2
Step 3: Rewrite the equation using the value of the constant ‘k’.
a = 2bc
Step 4: Using the new equation, find the missing value.
If b = 2 and c = 3, then a = 2 × 2 × 3 = 12.

Step 5: So, when a varies jointly with b and c and If b = 2 and c = 3, then the value of a is 12.

13 Fundamental Counting Principle


The Basic Counting Principle
When there are m ways to do one thing,
and n ways to do another,
then there are m×n ways of doing both.

Example: you have 3 shirts and 4 pants.


That means 3×4=12 different outfits.

Example: There are 6 flavors of ice-cream, and 3 different cones.


That means 6×3=18 different single-scoop ice-creams you could order.

It also works when you have more than 2 choices:


Example: You are buying a new car.
There are 2 body styles:

sedan or hatchback

There are 5 colors available:

317
There are 3 models:  GL (standard model),
 SS (sports model with bigger engine)
 SL (luxury model with leather seats)
How many total choices?
You can see in this "tree" diagram:

You can count the choices, or just do the simple calculation:


Total Choices = 2 × 5 × 3 = 30
Independent or Dependent?
But it only works when all choices are independent of each other.
If one choice affects another choice (i.e. depends on another choice), then a simple
multiplication is not right.
Example: You are buying a new car ... but ...
the salesman says "You can't choose black for the hatchback" ... well then things
change!

318
You now have only 27 choices.
Because your choices are not independent of each other.
But you can still make your life easier with this calculation:
Choices = 5×3 + 4×3 = 15 + 12 = 27

Ben can take any one of three routes from school (S) to the town center (T), and can then take
five possible routes from the town center to his home (H). He doesn't retrace his steps.
How many different possible ways can Ben walk home from school?
A. 7
B. 8
C. 15
D. 35

Sarah goes to her local pizza parlor and orders a pizza. She can choose either a large or a
medium pizza, has a choice of seven different toppings, and can have three different choices of
crust.
How many different pizzas could Sarah order?
A. 12
B. 20
C. 27
D. 42

319
Derek must choose a four-digit PIN number. Each digit can be chosen from 0 to 9.
How many different possible PIN numbers can Derek choose?
A. 5040
B. 6561
C. 9000
D. 10000

For her literature course, Rachel has to choose one novel to study from a list of four, one poem
from a list of six and one short story from a list of five.
How many different choices does Rachel have?
A. 15
B. 120
C. 225
D. 240

Simon visits his local fast-food restaurant and orders a burger, a side, a drink and an ice cream.
He can choose either a hamburger, a cheeseburger or an eggburger; he can choose either fries
or salad as a side; he can choose one drink from coke, lemonade, orange or raspberry; and for
his ice cream he can choose either a cone or a sundae.
How many different possible meals could Simon choose?
A. 11
B. 24
C. 40
D. 48
How many even five digit whole numbers are there?
A. 5000
B. 45000
C. 50000
D. 100000

The ten thousands digit cannot be zero, so there are 9 choices


There are 10 possibilities for the thousands digit
There are 10 possibilities for the hundreds digit
There are 10 possibilities for the tens digit
The units digit can be 0, 2, 4, 6 or 8, so there are 5 choices

By the Basic Counting Principle, the number of even five digit whole numbers is
9 × 10 × 10 × 10 × 5 = 45,000

320
How many 'words' with five letters are there that start with a vowel and end with an S?
A. 3,380
B. 87,880
C. 105,456
D. 2,284,880

14 Series and Sequence


14.1 Meaning of Sequence
A sequence, in mathematics, is a string of objects, like numbers, that follow a particular pattern.
The individual elements in a sequence are called terms. Some of the simplest sequences can be
found in multiplication tables:
 3, 6, 9, 12, 15, 18, 21, …
Pattern: “add 3 to the previous number to get the next number”
 0, 12, 24, 36, 48, 60, 72, …
Pattern: “add 12 to the previous number to get the next number”
Of course we can come up with much more complicated sequences:
 10,–2 8,×2 16,–2 14,×2 28,–2 26,×2 52, …
Pattern: “alternatingly subtract 2 and multiply by 2 to get the next number”
 0,+2 2,+4 6,+6 12,+8 20,+10 30,+12 42, …
Pattern: “add increasing even numbers to get the next number”
We can also create sequences based on geometric objects:

Triangle Numbers
Pattern: “add increasing
integers to get the next
number”

1 3 6 10 15

Square Numbers
Pattern: “add increasing odd
numbers to get the next number”

1 4 9 16 25
Note that the sequences of triangle and square numbers also have numerical patterns like the
ones we saw at the beginning. To find the following triangle numbers we have to add increasing
integers to the last term of the sequence (+2, +3, +4, …). To find the following square numbers
we have to add increasing odd numbers (+3, +5, +7, …).

321
Patterns and Equations
There are different ways in which we can describe the pattern underlying a sequence
mathematically. First let us represent every term in a sequence by a variable: let us call the nth
term in the sequence xn. The first term of the sequence is represented by x1, the second term
by x2, and so on. The x’s are simply place holders until we have calculated their actual value.
Let us think about the sequence of multiples of 3 above: 3, 6, 9, 12, 15, and so on. If we know
any term in the sequence, we can work out the next term by adding 3. With the notation
above, xn = xn–1 + 3. Every term xn is the previous one, xn–1 plus 3. If we know the first term, x1 =
3, we work out the following terms step by step:
x2 = x1 + 3 = 3 + 3 = 6
x3 = x2 + 3 = 6 + 3 = 9

An equation that expresses xn in terms of previous values is called a recurrence relation, and
these are very useful for calculating the terms of the sequence step by step. However if we were
only interested in the 100th term of the sequence, we would have to calculate all terms up to 100.
It would be much easier if we had an equation that tells us any term of the sequence, without
calculating all the previous ones.
Let us think again about the multiples of 3. Above we found the recurrence relation xn = xn–1 + 3.
But it is clear that the nth term of the sequence has to be 3 × n. For example, the 4th term is 3 ×
4 = 12. An expression for xn = 3n which only depends on n and not any other x’s is called a closed
form solution. It has the advantage that we can quickly find very large terms of the sequence
without having to calculate all previous terms. The 55th term, for example, is 3 × 55 = 165.
Here is one more interesting sequence:
2, 4, 8, 16, 32, 64, …
The pattern is very simple: we multiply terms by two to get the next term. In this case there is also
a closed form solution. By the time we are at term n, we have multiplied the initial 2 by 2 n – 1
times. Therefore xn = 2n. The sequence is therefore called the powers of 2. (Often we start with
20 = 1.)
Source: http://world.mathigon.org/Sequences

14.2 Arithmetic Sequence


REVIEW OF ARITHMETIC SEQUENCES

The formula for the n th term an of an arithmetic sequence with a common difference d and a
first term a1 is given by
an = a1 + (n - 1 )d

The sum sn of the first n terms of an arithmetic sequence is defined by

sn = a1 + a2 + a3 + ... + a n

322
and is a1 is given by

sn = n (a1 + an) / 2
Problem 1:
The first term of an arithmetic sequence is equal to 6 and the common difference is equal to 3.
Find a formula for the n th term and the value of the 50 th term
Solution to Problem 1:
 Use the value of the common difference d = 3 and the first term a 1 = 6 in the formula for
the n th term given above

an = a1 + (n - 1 )d

= 6 + 3 (n - 1)

=3n+3
 The 50 th term is found by setting n = 50 in the above formula.
a50 = 3 (50) + 3 = 153

Problem 2:
The first term of an arithmetic sequence is equal to 200 and the common difference is equal to
-10. Find the value of the 20 th term
Solution to Problem 2:
 Use the value of the common difference d = -10 and the first term a1 = 200 in the formula
for the n th term given above and then apply it to the 20 th term

a20 = 200 + (-10) (20 - 1 ) = 10

Problem 3:
An arithmetic sequence has a common difference equal to 10 and its 6 th term is equal to 52.
Find its 15 th term.
Solution to Problem 3:
 We use the n th term formula for the 6 th term, which is known, to write

a6 = 52 = a1 + 10 (6 - 1 )
 The above equation allows us to calculate a 1.

a1 = 2
 Now that we know the first term and the common difference, we use the n th term
formula to find the 15 th term as follows.

323
a15 = 2 + 10 (15 - 1) = 142

Problem 4:
An arithmetic sequence has a its 5 th term equal to 22 and its 15 th term equal to 62. Find its
100 th term.
Solution to Problem 4:
 We use the n th term formula for the 5 th and 15 th terms to write

a5 = a1 + (5 - 1 ) d = 22

a15 = a1 + (15 - 1 ) d = 62
 We obtain a system of 2 linear equations where the unknown are a 1 and d. Subtract the
right and left term of the two equations to obtain

62 - 22 = 14 d - 4 d
 Solve for d.

d=4
 Now use the value of d in one of the equations to find a 1.

a1 + (5 - 1 ) 4 = 22
 Solve for a1 to obtain.

a1 = 6
 Now that we have calculated a1 and d we use them in the n th term formula to find the
100 th formula.

a100 = 6 + 4 (100 - 1 )= 402

Problem 5:
Find the sum of all the integers from 1 to 1000.
Solution to Problem 5:
 The sequence of integers starting from 1 to 1000 is given by

1 , 2 , 3 , 4 , ... , 1000
 The above sequence has 1000 terms. The first term is 1 and the last term is 1000 and
the common difference is equal to 1. We have the formula that gives the sum of the first
n terms of an arithmetic sequence knowing the first and last term of the sequence and
the number of terms (see formula above).

s1000 = 1000 (1 + 1000) / 2 = 500500

324
Problem 6:
Find the sum of the first 50 even positive integers.
Solution to Problem 6:
 The sequence of the first 50 even positive integers is given by

2 , 4 , 6 , ...
 The above sequence has a first term equal to 2 and a common difference d = 2. We use
the n th term formula to find the 50 th term

a50 = 2 + 2 (50 - 1) = 100


 We now the first term and last term and the number of terms in the sequence, we now
find the sum of the first 50 terms

s50 = 50 (2 + 100) / 2 = 2550

Problem 7:
Find the sum of all positive integers, from 5 to 1555 inclusive, that are divisible by 5.
Solution to Problem 7:
 The first few terms of a sequence of positive integers divisible by 5 is given by

5 , 10 , 15 , ...
 The above sequence has a first term equal to 5 and a common difference d = 5. We
need to know the rank of the term 1555. We use the formula for the n th term as follows

1555 = a1 + (n - 1 )d
 Substitute a1 and d by their values

1555 = 5 + 5(n - 1 )
 Solve for n to obtain

n = 311
 We now know that 1555 is the 311 th term, we can use the formula for the sum as
follows

s311 = 311 (5 + 1555) / 2 = 242580

Problem 8:
Find the sum S defined by
10
S = ∑ (2n + 1 / 2)
n=1

325
Solution to Problem 8:
 Let us first decompose this sum as follows
10
S = ∑ (2n + 1 / 2)
n=1

10 10
= 2 ∑ n + ∑ (1 / 2)
i=1 n=1

 The term ∑ n is the sum of the first 10 positive integers. The 10 first positive integers
make an arirhmetic sequence with first term equal to 1, it has n = 10 terms and its 10 th
term is equal to 10. This sum is obtained using the formula sn = n (a1 + an) / 2 as follows

10(1+10)/2 = 55
 The term ∑ (1 / 2) is the addition of a constant term 10 times and is given by

10(1/2) = 5
 The sum S is given by

S = 2(55) + 5 = 115

Exercises:
Answer the following questions related to arithmetic sequences:

a) Find a20 given that a3 = 9 and a8 = 24

b) Find a30 given that the first few terms of an arithmetic sequence are given by 6,12,18,...

c) Find d given that a1 = 10 and a20 = 466

d) Find s30 given that a10 = 28 and a20 = 58

e) Find the sum S defined by


20
S = ∑ (3n - 1 / 2)
n=1

f) Find the sum S defined by

326
20 40
S = ∑ 0.2 n + ∑ 0.4 j
n=1 j=21
Solutions to Above Exercises:

a) a20 = 60

b) a30 = 180

c) d = 24

d) s30 = 1335

e) 1380

f) 286

14.3 Sum of a Finite Arithmetic Series


Finite arithmetic series
An arithmetic sequence is a sequence of numbers, such that the difference between any term
and the previous term is a constant number called the common difference (d):
Tn=a+(n−1)d
where
o Tn is the n term of the sequence;
o a is the first term;
o d is the common difference.
When we sum a finite number of terms in an arithmetic sequence, we get a finite arithmetic
series.
The sum of the first one hundred integers
A simple arithmetic sequence is when a=1 and d=1, which is the sequence of positive integers:
Tn∴{Tn}=a+(n−1)d=1+(n−1)(1)=n=1;2;3;4;5;…
If we wish to sum this sequence from n=1 to any positive integer, for example 100, we would
write
∑n=1100n=1+2+3+⋯+100
This gives the answer to the sum of the first 100 positive integers.
The mathematician, Karl Friedrich Gauss, discovered the following proof when he was only 8
years old. His teacher had decided to give his class a problem which would distract them for the
entire day by asking them to add all the numbers from 1 to 100. Young Karl quickly realised how
to do this and shocked the teacher with the correct answer, 5050. This is the method that he
used:

327
o Write the numbers in ascending order.
o Write the numbers in descending order.
o Add the corresponding pairs of terms together.
o Simplify the equation by making Sn the subject of the equation.
S100+S100−−−−∴2S100∴2S100∴S100=1+2+3+⋯+98+99+100=100+99+98+⋯+3+2+1−−−−−−−
−−−−−−−−−−−−−−−−−−−−−=101+101+101+⋯+101+101+101=101×100=10 100=10 1002=5050
General formula for a finite arithmetic series
If we sum an arithmetic sequence, it takes a long time to work it out term-by-term. We therefore
derive the general formula for evaluating a finite arithmetic series. We start with the general
formula for an arithmetic sequence of n terms and sum it from the first term (a) to the last term
in the sequence (\l):
∑n=1ℓTnSn+Sn−−∴2Sn∴2Sn∴Sn=Sn=a+(a+d)+(a+2d)+⋯+(ℓ−2d)+(ℓ−d)+ℓ=ℓ+(ℓ−d)+(ℓ−2d)+⋯+(a
+2d)+(a+d)+a−−−−−−−−−−−−−−−−−−−−−−−−−−−−−−−−−−−−−−−−=(a+ℓ)+(a+ℓ)+(a+ℓ)+⋯+(a+ℓ)+
(a+ℓ)+(a+ℓ)=n×(a+ℓ)=n2(a+ℓ)
This general formula is useful if the last term in the series is known.
We substitute ℓ=a+(n−1)d into the above formula and simplify:
Sn∴Sn=n2(a+[a+(n−1)d])=n2[2a+(n−1)d]
Identity 1
The general formula for determining the sum of an arithmetic series is given by:
Sn=n2[2a+(n−1)d]
or
Sn=n2(a+ℓ)
For example, we can calculate the sum S20 for the arithmetic sequence Tn=3+7(n−1) by
summing all the individual terms:
S20=∑n=120[3+7(n−1)]=3+10+17+24+31+38+45+52+59+66+73+80+87+94+101+108+115+12
2+129+136=1390
or, more sensibly, we could use the general formula for determining an arithmetic series by
substituting a=3, d=7 and n=20:
SnS20=n2(2a+(n−1)d)=202[2(3)+7(20−1)]=1390
Example 1: General formula for the sum of an arithmetic sequence
Question
Find the sum of the first 30 terms of an arithmetic series with Tn=7n−5 by using the formula.
Answer
Use the general formula to generate terms of the sequence and write down the known
variables
Tn∴T1T2T3=7n−5=7(1)−5=2=7(2)−5=9=7(3)−5=16
This gives the sequence: 2;9;16…
a=2;d=7;n=30

328
Write down the general formula and substitute the known values
SnS30=n2[2a+(n−1)d]=302[2(2)+(30−1)(7)]=15(4+203)=15(207)=3105
Write the final answer
S30=3105
Example 2: Sum of an arithmetic sequence if first and last terms are known
Question
Find the sum of the series −5−3−1+⋯⋯+123
Answer
Identify the type of series and write down the known variables
dd=T2−T1=−3−(−5)=2=T3−T2=−1−(−3)=2
a=−5;d=2;ℓ=123
Determine the value of n
Tn∴123∴130∴n=a+(n−1)d=−5+(n−1)(2)=−5+2n−2=2n=65
Use the general formula to find the sum of the series
SnS65=n2(a+ℓ)=652(−5+123)=652(118)=3835
Write the final answer
S65=3835
Example 3: Finding n given the sum of an arithmetic sequence
Question
Given an arithmetic sequence with T2=7 and d=3, determine how many terms must be added
together to give a sum of 2146.
Answer
Write down the known variables
d∴3∴a=T2−T1=7−a=4
a=4;d=3;Sn=2146
Use the general formula to determine the value of n
Sn21464292∴0∴n=−1163=n2(2a+(n−1)d)=n2(2(4)+(n−1)(3))=n(8+3n−3)=3n2+5n−4292=(3n+11
6)(n−37) or n=37
but n must be a positive integer, therefore n=37.
We could have solved for n using the quadratic formula but factorising by inspection is usually
the quickest method.
Write the final answer
S37=2146

329
Example 4: Finding n given the sum of an arithmetic sequence
Question
The sum of the second and third terms of an arithmetic sequence is equal to zero and the sum
of the first 36 terms of the series is equal to 1152. Find the first three terms in the series.
Answer
Write down the given information
T2+T3So(a+d)+(a+2d)∴2a+3d=0=0=0……(1)
SnS361152∴64=n2(2a+(n−1)d)=362(2a+(36−1)d)=18(2a+35d)=2a+35d……(2)
Solve the two equations simultaneously
2a+3d2a+35dEqn (2)−(1):32d∴dAnd 2a+3(2)2a∴a=0……(1)=64……(2)=64=2=0=−6=−3
Write the final answer
The first three terms of the series are:
T1T2T3=a=−3=a+d=−3+2=−1=a+2d=−3+2(2)=1
−3−1+1
Calculating the value of a term given the sum of n terms:
If the first term in a series is T1, then S1=T1.
We also know the sum of the first two terms S2=T1+T2, which we rearrange to make T2 the
subject of the equation:
T2Substitute S1∴T2=S2−T1=T1=S2−S1
Similarly, we could determine the third and fourth term in a series:
T3And T4=S3−S2=S4−S3
Identity 2
Tn=Sn−Sn−1, for n∈{2;3;4;…} and T1=S1

Source: http://everythingmaths.co.za/
14.4 Geometric Sequence

Introduction
In this section, you will learn how to identify geometric sequences, calculate the nth term of
geometric sequences, find the number of terms in an geometric sequence and find the sum
of geometric sequences. Soon, you will be invited to try our quiz masters at the end of the
lesson.
Identifying a Geometric Sequence
Sequences of numbers that follow a pattern of multiplying a fixed number from one term to
the next are called geometric sequences. The following sequences are geometric
sequences:

330
Sequence A: 1 , 2 , 4 , 8 , 16 , ...
Sequence B: 0.01 , 0.06 , 0.36 , 2.16 , 12.96 , ...
Sequence C: 16 , -8 , 4 , -2 , 1 , ...
For sequence A, if we multiply by 2 to the first number we will get the second number. This
works for any pair of consecutive numbers. The second number times 2 is the third number:
2 × 2 = 4, and so on.

For sequence B, if we multiply by 6 to the first number we will get the second number. This
also works for any pair of consecutive numbers. The third number times 6 is the fourth
number: 0.36 × 6 = 2.16, which will work throughout the entire sequence.
Sequence C is a little different because it seems that we are dividing; yet to stay consistent
with the theme of geometric sequences, we must think in terms of multiplication. We need
to multiply by -1/2 to the first number to get the second number. This too works for any pair
of consecutive numbers. The fourth number times -1/2 is the fifth number: -2 × -1/2 = 1.
Because these sequences behave according to this simple rule of multiplying a constant
number to one term to get to another, they are called geometric sequences. So that we can
examine these sequences to greater depth, we must know that the fixed numbers that bind
each sequence together are called the common ratios. Mathematicians use the
letter r when referring to these types of sequences.
Mathematicians also refer to generic sequences using the letter a along with subscripts that
correspond to the term numbers as follows:
Generic Sequence: a1, a2, a3, a4, ...
This means that if we refer to the tenth term of a certain sequence, we will label it a 10. a14 is
the 14th term. This notation is necessary for calculating nth terms, or a n, of sequences.
r can be calculated by dividing any two consecutive terms in a geometric sequence. The
formula for calculating r is...

...where n is any positive integer greater than 1.


Calculating the nth Term
In order for us to know how to obtain terms that are far down these lists of numbers, we
need to develop a formula that can be used to calculate these terms. If we were to try and
find the 20th term, or worse the 2000th term, it would take a long time if we were to simply
multiply a number -- one at a time -- to find our terms.
If we had to find the 400th term of sequence A above, we would undertake a tedious task
had we decided to multiply by two each step of the way all the way to the 400th term.

331
Luckily, there is a way to arrive at the 400th term without the need for calculating terms 1
through 399.
The formula for the general term for each geometric sequence is...

1. Let's examine sequence A so that we can find a formula to express its nth term.
If we match each term with it's corresponding term number, we get:

n 1 2 3 4 5 ...

Term 1 2 4 8 16 . . .

The fixed number, called the common ratio (r), is 2; so, the formula will be a n = a12n -
1
or an = (1)2n - 1 or...
an = 2n - 1
For sequence A above, the rule an = 2n - 1 would give the values...
21 - 1 = 20 = 1
22 - 1 = 21 = 2
23 - 1 = 22 = 4
24 - 1 = 23 = 8
25 - 1 = 24 = 16
Now if we were asked to find the 12th term in this sequence, we would calculate for
a12 or 212 - 1 which is equal to 211 = 2048. So, a11 = 2048, or the 11th term is 2048.
Likewise, the 20th term would be a20 = 219 = 524288.
2. Let's take a look at sequence B.

n 1 2 3 4 5 ...

Term 0.01 0.06 0.36 2.16 12.96 . . .

3. The fixed number, r, is 6. So the formula will be...


4. an = (0.01)6n - 1
5. For the sequence above, the rule an = (0.01)6n - 1 would give the values...
6. (0.01)61 - 1 = (0.01)60 = (0.01)(1) = 0.01
(0.01)62 - 1 = (0.01)61 = (0.01)(6) = 0.06
(0.01)63 - 1 = (0.01)62 = (0.01)(36) = 0.36
(0.01)64 - 1 = (0.01)63 = (0.01)(216) = 2.16
(0.01)65 - 1 = (0.01)64 = (0.01)(1296) = 12.96
7. If we wanted to calculate the 8th term, we would calculate for
8. a8 = (0.01)68 - 1 = (0.01)67 = (0.01)(279936) = 2799.36. If we needed the 12th term,
we would calculate a12 = (0.01)612 - 1 = (0.01)611 = (0.01)(362797056) = 3627970.56.
As one can see, the general formula is very handy when n is large.

332
9. Now let's do the third and final example with sequence C....

n 1 2 3 4 5 ...

Term 16 -8 4 -2 1 . . .

10. The common ratio is -1/2. So the formula will be ...


11. an = (16)(-1/2)n - 1
12. For sequence C, the rule an = (16)(-1/2)n - 1 would give the values...
13. (16)(-1/2)1 - 1 = (16)(-1/2)0 = (16)(1) = 16
(16)(-1/2)2 - 1 = (16)(-1/2)1 = (16)(-1/2) = -8
(16)(-1/2)3 - 1 = (16)(-1/2)2 = (16)(1/4) = 4
(16)(-1/2)4 - 1 = (16)(-1/2)3 = (16)(-1/8) = -2
(16)(-1/2)5 - 1 = (16)(-1/2)4 = (16)(1/16) = 1
14. If for some reason we needed the 11th term, we would calculate for a11 = (16)(-1/2)11
-1
= (16)(-1/2)10 (16)(0.0009765625) = 0.015625. Similarly, a16 = (16)(-1/2)16 - 1 =
(16)(-1/2)15 = (16)(-0.000030517578125) = -0.00048828125.
Finding the Number of Terms
It may be necessary to calculate the number of terms in a certain geometric sequence. To
do so, we would need to know two things.
We would need to know a few terms so that we could calculate the common ratio and
ultimately the formula for the general term. We would also need to know the last number in
the sequence.
Once we know the formula for the general term in a sequence and the last term, the
procedure is relatively uncomplicated, but the technique needed to complete the procedure
is complicated.
Set the last term equal to the formula for the general term. Since the formula uses the
variable n to calculate terms, we can also use it to determine the term number for any given
term.
If we again look at sequence A above, let's use the formula that was found to calculate term
values, an = a1rn - 1. If we knew that 256 was a number in the sequence (1, 2, 4, 8, 16, ...,
256 ) we would set the number 256 equal to the formula a n = a1rn - 1and get 256 = 2n - 1.
Solving this equation using proper techniques requires the use of logarithms and would
yield n = 9. [We could use the method of guessing and checking to arrive at the same
value.] This means that there are 9 terms in the mentioned sequence and that the 9th term,
a9, is equal to 256.
Let's look at a portion of sequence C. If the sequence went from 16 to -1/8, we would have:
16, -8, 4, ..., -1/8. We would use the formula for the general term...
an = (16)(-1/2)n - 1
...and set it equal to the last term, -1/8. We would get -1/8 = (16)(-1/2)n - 1. Solving this
equation [with the use of either logarithms or the method of guessing and checking] allows
us to arrive at n = 8. This means that there are 8 terms in the sequence and that a 8 = -1/8.

333
Finding the Sum of a Series
Given our generic arithmetic sequence a1, a2, a3, a4, ..., an, we can look at it as a series:
a1 + a2 + a3 + a4 + ... + an. There exists a formula that can add a finite list of these numbers
and a formula for an infinite list of these numbers. Here are the formulas...

...where Sn is the sum of the first n numbers, a1 is the first number in the sequence, r is the
common ratio of the sequence, and -1 < r < 1 for infinite series. Let's use examples to
investigate both formulas.
Formula One: Finite Sum
i. If we wanted to add the first 7 terms of sequence A, we would know that a 1 = 1, r = 2
and n = 7. We would then plug those numbers into the formula and get S n= [a1(1 -
rn)]/(1 - r) = [(1)(1 - 27)]/(1 - 2) = (1 - 128)/(-1) = (-127)/(-1) = 127. So, S7 = 127.
ii. If we wanted to add the first 10 terms of sequence B, we would know that a 1 = 0.01,
r = 6 and n = 10. We would then plug those numbers into the formula and get S n =
[a1(1 - rn)]/(1 - r) = [(0.01)(1 - 610)]/(1 - 60466176) = (0.01)(-60466175)/(-5) = (-
604661.75)/(-5) = 120932.35. So, S10 = 120932.35.
Formula Two: Infinite Sum
i. If we had to add an infinite number of terms of sequence A, we would first see if it
met the requirement of the formula for the common ratio, r. If r is between -1 and 1,
an infinite term sum exists. Since sequence A has an r-value of 2 and 2 falls outside
the acceptable range, there is no finite sum to that infinite list of numbers.
ii. Let's take an infinite number of terms from sequence C. If we had 16 + (-8) + 4 + (-2)
+ 1 + ..., then we would know that a1 = 16 and r = -1/2. Since r lies within the
acceptable limits, a finite sum exists. Using the infinite term sum formula, we get
S∞ = a1/(1 - r) = 16/(1 - (-1/2)) = 16/1.5 = 10 2/3.
iii. If we had to add the infinite sum 27 + 18 + 12 + 8 + ..., we would first identify the first
term a1 and the common ratio r, which are 27 and 2/3 respectively. Since rlies
between 1 and -1, the formula will work and a finite sum exists for the infinite series.
The formula yields S∞ = 27/(1 - (2/3)) = 27/(1/3) = 81.

14.5 Sum of Geometric Series

334
You can take the sum of a finite number of terms of a geometric sequence. And, for reasons
you'll study in calculus, you can take the sum of aninfinite geometric sequence, but only in the
special circumstance that the common ratio r is between –1 and 1; that is, you have to have | r |
< 1.
For a geometric sequence with first term a1 = aand common ratio r, the sum of the first n terms
is given by:

Note: Your book may have a slightly different form of the partial-sum formula above. For
instance, the "a" may be multiplied through the numerator, the factors in the fraction might be
reversed, or the summation may start at i = 0 and have a power of n + 1 on the numerator. All of
these forms are equivalent, and the formulation above may be derived from polynomial long
division.
In the special case that | r | < 1, the infinite sum exists and has the following value:

Copyright © Elizabeth Stapel 2006-2011 All Rights Reserved

 Evaluate the following:

The first few terms are –6, 12, –24, so this is a geometric series with common ratio r = –
2. (I can also tell that this must be a geometric series because of the form given for each
term: as the index increases, each term will be multiplied by an additional factor of –2.)
The first term of the sequence is a = –6. Plugging into the summation formula, I get:

So the value of the summation is 2 097 150


 Evaluate S10 for 250, 100, 40, 16,....
The notation "S10" means that I need to find the sum of the first ten terms. The first term
is a = 250. Dividing pairs of terms, I get 100 ÷ 250 = 2/5, 40 ÷ 100 = 2/5, etc, so the
terms being added form a geometric sequence with common ratio r = 2/5. When I plug in
the values of the first term and the common ratio, the summation formula gives me:

335
Note: If you try to do the above computations in your
calculator, it may very well return the decimal
approximation of 416.62297...instead of the fractional (and
exact) answer. As you can see in the screen-capture to the
right, entering the values in fractional form and using the
"convert to fraction" command still results in just a decimal
approximation to the answer. But (warning!) the decimal
approximation will almost certain be regarded as a "wrong"
answer! Take the time to find the fractional form!
 Find an if S4 = 26/27 and r = 1/3.
They've given me the sum of the first four terms, S4, and the value of the common ratio r.
Since there is a common ratio, I know this must be a geometric series. Plugging into the
geometric-series-sum formula, I get:

336
Multiplying on both sides by 27/40 to solve for the first term a = a1, I get:

Then:

 Show, by use of a geometric series, that 0.3333... is equal to 1/3.


There's a trick to this. I first have to break the repeating decimal into separate terms:
0.333... = 0.3 + 0.03 + 0.003 + 0.0003 + ...
This shows the repeating pattern of the non-terminating (never-ending) decimal
explicitly: For each term, I have a decimal point, followed by a steadily-increasing
number of zeroes, and then ending with a "3". This can be written in fractional form, and
then converted into geometric-series form:

Then 0.333... is an infinite geometric series with a = 3/10 and r = 1/10. Since | r | < 1, I
can use the formula for summing infinite geometric series:

337
Using the summation formula to show that the geometric series "expansion" of 0.333... has a
value of one-third is the "showing" that the exercise asked for. You can use this method to
convert any repeating decimal to its fractional form:
 By use of a geometric series, convert 1.363636... to fractional form.
First I'll break this into its constituent parts, so I can find the pattern:
1.363636.. = 1 + 0.36 + 0.0036 + 0.000036 + ...
There are two digits that repeat, so the fractions are a little bit different. But this is still a
geometric series:

Then this is the leading "1", plus a geometric series having a = 9/25 and r = 1/100. Then
the sum is:

Note: This technique can also be used to convert any repeating decimal into fractional form, and
also can be used to prove that 0.999... = 1.

338
15 Binomial Theorem
The Binomial Theorem is a quick way (okay, it's a less slow way) of expanding (or multiplying
out) a binomial expression that has been raised to some (generally inconveniently large) power.
For instance, the expression (3x – 2)10 would be very painful to multiply out by hand. Thankfully,
somebody figured out a formula for this expansion, and we can plug the binomial 3x – 2and the
power 10 into that formula to get that expanded (multiplied-out) form.
The formal expression of the Binomial Theorem is as follows:

Copyright © Elizabeth Stapel 1999-2009 All Rights Reserved


Yeah, I know; that formula never helped me much, either. And it doesn't help that different texts
use different notations to mean the same thing. The parenthetical bit above has these
equivalents:

Recall that the factorial notation "n!" means " the product of all the whole numbers
between 1 and n", so, for instance, 6! = 1×2×3×4×5×6. Then the notation "10C7" (often
pronounced as "ten, choose seven") means:

Many calculators can evaluate this "n choose m"


notation for you. Just look for a key that looks like
"nCm" or "nCr", or for a similar item on the "Prob"
or "Math" menu, or check your owner's manual
under "probability" or "combinations".

The evaluation will probably look something like


this:

339
There is another way to find the value of "nCr", and it's called "Pascal's
Triangle". To make the triangle, you start with a pyramid of three 1's, like
this:

Then you get the next row of numbers by adding the pairs of numbers
from above. (Where there is only one number above, you just carry down
the 1.)

Keep going, always adding pairs of


numbers from the previous row..

To find, say, 6C4, you go down to the


row where there is a "6" after the
initial "1", and then go over to the 5th
(not the 4th) entry, to find that 6C4 =
15.

As you might imagine, drawing Pascal's Triangle every time you have to expand a binomial
would be a rather long process, especially if the binomial has a large exponent on it. People
have done a lot of studies on Pascal's Triangle, but in practical terms, it's probably best to just
use your calculator to findnCr, rather than using the Triangle. The Triangle is cute, I suppose, but
it's not terribly helpful in this context, being more time-consuming than anything else. For
instance, on a test, do you want to evaluate "10C7" by calculating eleven rows of the Triangle, or
by pushing four buttons on your calculator?

I could never remember the formula for the Binomial Theorem, so instead, I just learned how it
worked. I noticed that the powers on each term in the expansion always added up to
whatever n was, and that the terms counted up from zero to n. Returning to our intial example
of (3x – 2)10, the powers on every term of the expansion will add up to 10, and the powers on
the terms will increment by counting up from zero to 10:
(3x – 2)10 = 10C0 (3x)10–0(–2)0 + 10C1 (3x)10–1(–2)1 + 10C2 (3x)10–2(–2)2
+ 10C3 (3x)10–3(–2)3 + 10C4 (3x)10–4(–2)4 + 10C5 (3x)10–5(–2)5
+ 10C6 (3x)10–6(–2)6 + 10C7 (3x)10–7(–2)7 + 10C8 (3x)10–8(–2)8
+ 10C9 (3x)10–9(–2)9 + 10C10 (3x)10–10(–2)10
Note how the highlighted counter number counts up from zero to 10, with the factors on the
ends of each term having the counter number, and the factor in the middle having the counter
number subtracted from 10. This pattern is all you really need to know about the Binomial
Theorem; this pattern is how it works.

340
Your first step, given a binomial to expand, should be to plug it into the Theorem, just like I did
above. Don't try to do too many steps at once. Only after you've set up your binomial in the
Theorem's pattern should you start to simplify the terms. The Binomial Theorem works best as a
"plug-n-chug" process, but you should plug in first; chug later. I've done my "plugging" above;
now "chugging" gives me:
(1)(59049)x10(1) + (10)(19683)x9(–2) + (45)(6561)x8(4) + (120)(2187)x7(–8)
+ (210)(729)x6(16) + (252)(243)x5(–32) + (210)(81)x4(64)
+ (120)(27)x3(–128) + (45)(9)x2(256) + (10)(3)x(–512) + (1)(1)(1)(1024)
= 59049x10 – 393660x9 + 1180980x8 – 2099520x7 + 2449440x6 – 1959552x5
+ 1088640x4 – 414720x3 + 103680x2 – 15360x + 1024
As painful as the Binomial-Theorem process is, it's still easier than trying to multiply this stuff out
by hand. So don't let the Formula put you off. It's just another thing to memorize, so memorize it,
at least for the next test. The biggest source of errors in the Binomial Theorem (other than
forgetting the Theorem) is the simplification process. Don't try to do it in your head, or try to do
too many steps at once. Write things out nice and clearly, as I did above, so you have a better
chance of getting the right answer. (And it would be good to do a bunch of practice problems, so
the process is fairly automatic by the time you hit the next test.)
 Expand (x2 + 3)6
Students trying to do this expansion in their heads tend to mess up the powers. But this
isn't the time to worry about that square on the x. I need to start my answer by plugging
the terms and power into the Theorem. The first term in the binomial is "x2", the second
term in "3", and the power n is 6, so, counting from0 to 6, the Binomial Theorem gives
me:
(x2 + 3)6 = 6C0 (x2)6(3)0 + 6C1(x2)5(3)1 + 6C2 (x2)4(3)2 + 6C3 (x2)3(3)3
+ 6C4 (x2)2(3)4 + 6C5 (x2)1(3)5 + 6C6 (x2)0(3)6
Then simplifying gives me
(1)(x12)(1) + (6)(x10)(3) + (15)(x8)(9) + (20)(x6)(27)
+ (15)(x4)(81) + (6)(x2)(243) + (1)(1)(729)
= x12 + 18x10 + 135x8 + 540x6 + 1215x4 + 1458x2 + 729
 Expand (2x – 5y)7
I'll plug "2x", "–5y", and "7" into the Binomial Theorem, counting up from zero to seven to
get each term. (I mustn't forget the "minus" sign that goes with the second term in the
binomial.)
(2x – 5y)7 = 7C0 (2x)7(–5y)0 + 7C1 (2x)6(–5y)1 + 7C2 (2x)5(–5y)2
+ 7C3 (2x)4(–5y)3 + 7C4 (2x)3(–5y)4 + 7C5 (2x)2(–5y)5
+ 7C6 (2x)1(–5y)6 + 7C7 (2x)0(–5y)7
Then simplifying gives me: Copyright © Elizabeth Stapel 1999-2009 All Rights
Reserved
(1)(128x7)(1) + (7)(64x6)(–5y) + (21)(32x5)(25y2) + (35)(16x4)(–125y3)
+ (35)(8x3)(625y4) + (21)(4x2)(–3125y5) + (7)(2x)(15625y6)

341
+ (1)(1)(–78125y7)
= 128x7 – 2240x6y + 16800x5y2 – 70000x4y3 + 175000x3y4 – 262500x2y5
+ 218750xy6 – 78125y7
You may be asked to find a certain term in an expansion, the idea being that the exercise will be
way easy if you've memorized the Theorem, but will be difficult or impossible if you haven't. So
memorize the Theorem and get the easy points.
 What is the fourth term in the expansion of (3x – 2)10?
I've already expanded this binomial, so let's take a look:
(3x – 2)10 = 10C0 (3x)10–0(–2)0 + 10C1 (3x)10–1(–2)1 + 10C2 (3x)10–2(–2)2
+ 10C3 (3x)10–3(–2)3 + 10C4 (3x)10–4(–2)4 + 10C5 (3x)10–5(–2)5
+ 10C6 (3x)10–6(–2)6 + 10C7 (3x)10–7(–2)7 + 10C8 (3x)10–8(–2)8
+ 10C9 (3x)10–9(–2)9 + 10C10 (3x)10–10(–2)10
So the fourth term is not the one where I've counted up to 4, but the one where I've
counted up just to 3. (This is because, just as with Javascript, the counting starts with 0,
not 1.)
Note that, in any expansion, there is one more term than the number in the power. For
instance:
(x + y)2 = x2 + 2xy + y2 (second power: three terms)
(x + y)3 = x3 + 3x2y + 3xy2 + y3 (third power: four terms)
(x + y)4 = x4 + 4x3y + 6x2y2 + 4xy3 + y4 (fourth power: five terms)
The expansion in this exercise, (3x – 2)10, has power of n = 10, so the expansion will
have eleven terms, and the terms will count up, not from 1 to 10 or from 1 to 11, but
from 0 to 10. This is why the fourth term will not the one where I'm using "4" as my
counter, but will be the one where I'm using "3".
10C3 (3x)10–3(–2)3 = (120)(2187)(x7)(–8) = –2099520x7
 Find the tenth term in the expansion of (x + 3)12.
To find the tenth term, I plug x, 3, and 12 into the Binomial Theorem, using the
number 10 – 1 = 9 as my counter:
12C9 (x)12–9(3)9 = (220)x3(19683) = 4330260x3
 Find the middle term in the expansion of (4x – y)8.
Since this binomial is to the power 8, there will be nine terms in the expansion, which
makes the fifth term the middle one. So I'll plug 4x, –y, and 8 into the Binomial Theorem,
using the number5 – 1 = 4 as my counter.
8C4 (4x)8–4(–y)4 = (70)(256x4)(y4) = 17920x4y4
You might be asked to work backwards.
 Express 1296x12 – 4320x9y2 + 5400x6y4 – 3000x3y6 + 625y8 in the form (a + b)n.
I know that the first term is of the form an, because, for whatever n is, the first term
is nC0(which always equals 1) times an times b0 (which also equals 1). So 1296x12 = an.
By the same reasoning, the last term is bn, so 625y8 = bn. And since there are alternating

342
"plus" and "minus" signs, I know from experience that the sign in the middle has to be a
"minus". (If all the signs had been "plusses", then the middle sign would have been a
"plus" also. But in this case, I'm really looking for "(a – b)n".)
I know that, for any power n, the expansion has n + 1 terms. Since this has 5 terms, this
tells me that n = 4. So to find a and b, I only have to take the 4th root of the first and last
terms of the expanded polynomial:

Then a = 6x3, b = 5y2, there is a "minus" sign in the middle, and:


1296x12 – 4320x9y2 + 5400x6y4 – 3000x3y6 + 625y8 = (6x3 – 5y2)4
Don't let the Binomial Theorem scare you. It's just another formula to memorize. A really
complicated and annoying formula, I'll grant you, but just a formula, nonetheless. Don't overthink
the Theorem; there is nothing deep or meaningful here. Just memorize it, and move on.

Source: http://www.purplemath.com/

343

You might also like